100% found this document useful (3 votes)
523 views443 pages

Examguru Mathematics Class 12 Question Bank 2023

Uploaded by

fuck
Copyright
© © All Rights Reserved
We take content rights seriously. If you suspect this is your content, claim it here.
Available Formats
Download as PDF, TXT or read online on Scribd
100% found this document useful (3 votes)
523 views443 pages

Examguru Mathematics Class 12 Question Bank 2023

Uploaded by

fuck
Copyright
© © All Rights Reserved
We take content rights seriously. If you suspect this is your content, claim it here.
Available Formats
Download as PDF, TXT or read online on Scribd
You are on page 1/ 443

12

CBSE

MATHEMATICS

Ram Awatar Gupta


BSc (Hons) (Physics)
MSc (Mathematics)
Retd. Senior Lecturer
DTEA Sr. Sec. School, Delhi
(More than 40 Years of Teaching Experience)

Full Marks Pvt Ltd


(Progressive Educational Publishers)
New Delhi-110002
Published by:

9, Daryaganj, New Delhi-110002


Phone: 011- 40556600 (100 Lines)

Website: www.fullmarks.org
E-mail: [email protected]

© Author

All rights reserved. No part of this publication may be reproduced or transmitted, in any form or by any means, without permission. Any person
who does any unauthorised act in relation to this publication may be liable to criminal prosecution and civil claims for damages.

Branches:
• Chennai • Guwahati

Marketing Offices:
• Ahmedabad • Bengaluru • Bhopal • Dehradun • Hyderabad • Jaipur • Jalandhar • Kochi • Kolkata • Lucknow
• Mumbai • Patna • Ranchi

NEW EDITION

“This book is meant for educational and learning purposes. The author(s) of the book has/have taken all reasonable care to ensure that the contents
of the book do not violate any existing copyright or other intellectual property rights of any person in any manner whatsoever. In the event the
author(s) has/have been unable to track any source and if any copyright has been inadvertently infringed, please notify the publisher in writing for
corrective action.”
Preface

Mathe atics-12 is based on the latest curriculum guidelines specified by the CBSE.
m
It will certainly prove to be a torch-bearer for those who toil hard to achieve their goal.
This All-in-one Question Bank has been developed keeping in mind all the requirement of the students for Board
Examinations preparations like learning, practicing, revising and assessing.
alient Features o the Book:
S
f
● Each chapter is designed in ‘Topic wise’ manner where each topic is briefly explained with sufficient Examples and

Exercise. Exercise which covers all the possible variety of Questions.
● Answers with hints are provided separately after the exercise.

● Previous years’ Board Questions have been covered in every chapter.

● 1 Solved and 2 Unsolved Sample Papers are given with hints & answers for self assessment.

● Common Errors by the students are provided to make students aware what errors are usually done unknowingly.

● The book has been well prepared to build confidence in students.

Suggestions for further improvement of the book, pointing out printing errors/mistakes which might have crept in spite

of all efforts, will be thankfully received and incorporated in the next edition.
–Author

(iii)
Mathe m atics 12
GLIMPSE OF A CHAPTER

1
EXERCISE 7.9
Relations and Functions I. Multiple Choice Questions (MCQs)
9.
 sin 4x − 4 
∫ 1 − cos 4x  e
x
dx [Delhi 2010]
1. ∫e
x
(1 − cot x + cot 2 x ) dx =
(a) ex cot x + C (b) –ex cot x + C
Topics covered   x
10. ∫  (2 − sin x)  e 2 dx [V. Imp.]
(c) ex cosec x + C (d) –ex cosec x + C  1 − cos x 
1.1 Relations 1.2 Types of Relations
( x + 3) e x
1.3 Functions 2. ∫ ( x + 4) 2
dx = [NCERT Exemplar]
11. ∫
(x − sin x) dx
1 − cos x
ex ex

C hapter map
(a) +C (b) +C  x + sin x 
x+4 x+3 12. ∫  1 + cos x  dx
1 ex
(c) +C (d) +C
( x + 4) 2 ( x + 4) 2  (2 − 5 sin 2x)  e5xdx
RELATIONS
2
13. ∫  1 − cos 2x 
 log x − 1 
Type of Relations
3. ∫  1 + ( log x )
2 

dx is equal to
 1 
14. ∫ log (log x) + (log x)2  dx [V. Imp.] [Delhi 2010 (C)]
x xe x
(a) +C (b) +C
( log x ) 2 + 1 1 + x2  1 + sin 2x  − x
15. ∫   e dx [V. Imp.]
Universal Empty Reflexive Symmetric Transitive Equivalence x log x
(c) +C (d) +C  1 − cos 2x 
x2 + 1 ( log x ) 2 + 1
1 − sin x  −2x 
Equivalence Relation Evaluate each of the following Integrals 16. ∫ e  dx [V. Imp.] [A.I. 2013 (C)]
1 + cos x  
II. Short Answer Type Questions-I
∫ (sin x) a
x
Functions (x − 3) e x dx 17. dx
1. ∫ (x + 1)3 [Delhi 2009]
2 x
III. Short Answer Type Questions-II
2. ∫ (x + 1) e 2 dx
Types of Functions
(x + 1)
[Delhi 2005, 2006 (C)
( x cos x) dx
−1

(x − 4) e x dx
1. ∫ 1 − x2
[A.I. 2014 (C)]
One-one/Injective Function Onto/Surjective Function Domain and Co-domain 3. ∫ [Delhi 2008]
(x − 2) 2
of the Function
(1 − x) 2 e x dx 2.  (
x + 1 log x 2 + 1 − 2 log x  dx
2
)
 [A.I. 2011, 2014 (C)]
4. ∫ ∫ x4
(x 2 + 1) 2
 1 − sin x  x
Bijective Function 2 3. ∫  e dx
 x + 2  1 − cos x 
5. ∫  x + 4  e x dx
IV. Long Answer Type Questions
Cartesian Product of Sets (Recap) (5x + 14) e5x dx
Let A and B be two non-empty sets. A set of all the ordered pairs (a, b) such that a ∈ A and b ∈ B is called cartesian product of set
6. ∫ (x + 3)2 (sin −1
x − cos −1 x dx )
1. ∫ (sin −1
x + cos −1 x )
A with set B and denoted as (A × B). 7. ∫ [sin (log x) + cos (log x)] dx [V. Imp.]
x dx 2
Or A × B = {(a, b) : a ∈ A and b ∈ B)} 2. ∫ ( x sin x + cos x)2 [Delhi 2012]
∫ 3x tan x + x sec x dx
2 3 2
8.
The number of ordered pairs in (A × B) = n(A × B) = n(A) × n(B)
= (number of elements in set A) × (number of elements in set B). Answers 7.9
Example 1. Given set A = {1, 2, 3} and set B = {3, 5}. Value of A × B and n(B × A) respectively are given by
∫e ( x + 3) e x ( x + 4 − 1) x
x
I. 1. (b) I = (1 − cot x + cot 2 x) dx
2. (a) I = ∫ ( x + 4) 2
dx = ∫
( x + 4) 2
e dx
(a) {(1, 3), (1, 5), (2, 3), (2, 5), (3, 3), (5, 3)} and 6
∫e
x
= {(1+ cot 2 x ) − cot x} dx
(b) {(1, 3), (1, 5) (2, 3), (5, 2), (3, 3), (5, 3)} and 5  1 1  x 1 x
= ∫ − 2
e dx = e +C

. (d)(c)E{(1,{(1,ach chapter is divided into topics and  x + 4 ( x + 4)  x+4


∫ e {cosec
x 2
3), (1, 5), (2, 3), (2, 5), (3, 3), (3, 5) and 6 = x + ( − cot x )}dx
 e x [ f ( x) + f ′ ( x)] dx = e x f ( x) 
 ∫


3), (5, 1), (2, 3), (2, 5), (3, 3), (5, 3)} and 6 
{ ( )}
explained We are given two sets: A = {1, separately.
= − ∫ e cot x + −cosec x dx
x 2

Solution. 2, 3} and B = {3, 5} 2


 log x − 1  (log x − 1) 2
 e x { f ( x) + f ′ ( x)} dx = e x f ( x)  3. (a) I = ∫  2 
dx = ∫ dx
A × B = {1, 2, 3} × {3, 5} = {(1, 3), (1, 5), (2, 3), (2, 5), (3, 3), (3, 5)} = –ex cot x + C  ∫
. Exercise is segregated according  1 + (log x )  {1 + (log x ) 2 }2
.We areChapter
given: n(A) = 3 and n(B) Map representation of the chapter.

 =2  

216 MatheMatics–12
\ n(B × A) = n(B) × n(A) = 2 × 3 = 6
Hence, option (c) is the correct answer.
to different variety of objective
(ii) B × A = {(2, 1), (2, 3), (2, 5), (4, 1), (4, 3), (v) n(A) = 3
11
(4, 5), (6, 1), (6, 3), (6, 5), (8, 1), (8, 3), (8, 5)} (vi) n(B) = 4
and subjective questions. Also
(iii) A × A = {(1, 1), (1, 3), (1, 5), (3, 1), (3, 3), (vii) n(A × B) = 12 (viii) n(B × A) = 12 includes Previous year board
(3, 5), (5, 1), (5, 3), (5, 5)}
(iv) B × B = {(2, 2), (2, 4), (2, 6), (2, 8), (4, 2),
(ix) n(A × A) = 9 (x) n(B × B) = 16 questions and HOTS questions.
(xi) A × B ≠ B × A
(4, 4), (4, 6), (4, 8), (6, 2), (6, 4), (6, 6), (6, 8),
(xii) n(A × B) = n(B × A)
(8, 2), (8, 4), (8, 6), (8, 8)}

1. relations
Relation from Set A to Set B
Let A and B be two non-empty sets, then the relation from set A to set B is a subset of set A × B. For
example, if set A = {1, 2, 3} and set B = {3, 4, 5}, then there can be many relations from set A to set B.
R1 = {1, 3), (2, 5)}; R2 = {(1, 5), (2, 4), (2, 3); R3 = {(1, 3), (1, 4), (1, 5), (2, 5)}; etc. are some of the
relations from set A to set B.
Number of all Possible Relations from Set A to Set B
Number of elements in set A = n(A). Number of elements in set B = n(B)
\ Number of ordered pairs in (A × B) = n(A × B) = n(A) × n(B) . Topic wise concepts are
\ Number of all possible relations from set A to set B = 2n(A × B) = 2n(A) × n(B).
 

Example 1. How many relations are possible from set A = {1, 2, 3} to set B = {6, 7, 8, 9}?
presented to remember them
Solution. We have A = {1, 2, 3}, B = {6, 7, 8, 9} easily.
⇒ n(A) = 3, n(B) = 4 ⇒ n(A × B) = 3 × 4 = 12
\ Number of possible relations from set A to set B = 212.
Domain of a Relation R
Relation R from set A to set B is a subset of set of (A × B) and A × B is a set of all possible ordered pairs
(a, b), where a ∈ A and b ∈ B. Now, a set of all the first coordinates of all the ordered pairs of relation
R is called the domain of relation R.
Or D = {a : (a, b) ∈ R}
In relation: R = {(2, 1), (3, 7), (5, 8)}, Domain = {2, 3, 5} .   Each topic is well explained
Range of a Relation R with relevant example for better
Range of a relation R is a set of all the second coordinates of all the ordered pairs of relation R.
Or Range = {b : (a, b) ∈ R}
understanding.
In relation: R = {(3, 1), (7, 8), (–9, 5)}, Range = {1, 8, 5}
Example 2. If set A = {2, 3, 4, 7} and set B = {3, 4, 5, 6}. A relation R from set A to set B is defined as:
R : {(a, b): b is divisible by a and a ∈ A and b ∈ B}. Write the relation in roster form and with arrow
diagram. Also write the domain and range of relation R.
Solution. The given sets are: A = {2, 3, 4, 7} and B = {3, 4, 5, 6}
(i) R = {(2, 4), (2, 6), (3, 3), (3, 6), (4, 4)}
This is the roster form of the relation R.

(ii) Arrow diagram of relation R

(iii) Domain of R = {2, 3, 4} (iv) Range of R = {3, 4, 6}

Relations and Functions 13

(iv)
then a ⊗ e = a and e ⊗ a = a.
2. Inverse of an element a of ⊗ : A × A → A a binary operation: If a and b ∈ A and are inverse of
each other under the ⊗ operation then
a ⊗ b = e and b ⊗ a = e. COMMON ERRORS
3. Number of all possible relations from set A to set B = 2n(A × B) = 2n(A) × n(B).
ERRORS CORRECTIONS
4. (i) Commutative binary operations for ⊗ : A × A → A,
(i) In a set A = {1, 2, 3}, a relation R = {(1, 1), (2, 2), (3, (i) In reflexive relation, every element of set A must be related
a ⊗ b = b ⊗ a ∀ a, b ∈ A 2)}. Here R is not reflexive relation because (3, 3) is not to itself.
(ii) Associative binary operations for ⊗ : A × A → A, present in R. Students make a mistake that if only (1, 1)
(a ⊗ b) ⊗ c = a ⊗ (b ⊗ c) ∀ a, b, c ∈ A. or (1, 1), (2, 2) are present R is reflexive.
(ii) To check the symmetric relation R. (ii) Find R–1, if R = R–1 only then R is symmetric.
(iii) To check the transitivity of a relation R. (iii) Its every ordered pair must be tested.
(iv) If x ∈ N and x12 = x22 ⇒ x1 = ± x2 (iv) Here x1 = x2, because x1 = –x2 is not possible because x is
a natural number.
. Quite effective for a quick(v) revision
If x ∈ R and x before
=x ⇒ x exams.
=x 1
2
2
2
1 2 (v) Here x1 = ± x2, because x is a real number, here negative
number is possible.



Have the complete essence (vi) Whileof theinjectivity,
checking chapter. surjectivity of a function, (vi) The first step while checking injectivity, surjectivity, take
students ignore the set in which function is defined, i.e., f care of set under consideration.
: N → N or
f : Z → Z or f : R → R etc
Common errors (vii) Identification of co-domain, range (vii) Co-domain is the set of all second elements, i.e., if
f : A → B, then B is the co-domain.
errors CorreCtions Range is the set of all second entries of the ordered pairs.
(i) In a set A = {1, 2, 3}, a relation R = {(1, 1), (i) In reflexive relation, every element of set A
(2, 2), (3, 2)}. Here R is not reflexive relation must be related to itself.
because (3, 3) is not present in R. Students
make a mistake that if only (1, 1) or (1, 1), REVISION CHART—RELATIONS
(2, 2) are present R is reflexive.
–1 –1
empty relation reflexive relation
(ii) To check the symmetric relation R. (ii) Find R , if R = R only then R is symmetric. A relation R in set A is called empty A relation R in set A is called reflexive
(iii) To check the transitivity of a relation R. (iii) Its every ordered pair must be tested. relation if no element of set A related if (a, a) ∈ R for all a ∈ A or R is reflexive
to any element of set A, i.e, R = f. in set A if (a, a) ∈ R ∀ a ∈ A.
(iv) If x ∈ N and x12 = x22 ⇒ x1 = ± x2 (iv) Here x1 = x2, because x1 = –x2 is not possible
because x is a natural number.
(v) If x ∈ R and x12 = x22 ⇒ x1 = xuthor 2 A ’ C
Here x1 = ± x2, because x is a real number,
s (v)omments
here negative number is possible. relationS
(vi) Questionsof
While checking injectivity, surjectivity based
(vi)on following
The first steptypes
whileare very important
checking for Exams. So, students
injectivity, are advised
(i) Relation R from set A toto
setrevise
B: Relationthem
R from setthoroughly.
A to set B is a subset of Cartesian product (A × B)
or Relation from set A to set B = {(a, b) : a ∈ A and b ∈ B}
a function, students ignore the set 1. inTowhich
express a square matrix take
surjectivity, intocare
sumofofset
symmetric and skew-symmetric matrices.
under considered. If A = {1, 2} and B = {3, 4} then R = {(1, 3), (2, 4)}
function is defined, i.e., f : N → N 2. orQuestions based on property |adj A| = |A|n – 1. (ii) Relation R in set A: Relation R in set A is a subset of Cartesian product (A × A).
f : Z → Z or f : R → R etc 3. To find inverse of a square matrix. (Most Important) If A = {1, 2, 3} then Relation R = {(1, 1), (1, 2), (2, 3)}

(vii) Identification of co-domain, range (vii) Co-domain is the set of all second elements,
i.e., if f : A → B, then B is the co-domain.
Range is the set of all second entries of the Symmetric relation equivalence relation tranSitive relation
ordered pairs. COMMON ERRORS A relation R in set A is said to be If a relation R in set A is reflexive,
symmetric if (a, b) ∈ R and (b, a) ∈ R,
A relation R in set A is said to be
symmetric and transitive, then transitive if (a, b) ∈ R and (b, c) ∈ R
a, b ∈ A or if relation R = its inverse relation R is called an equivalence ⇒ (a, c) ∈ R for a, b, c ∈ A.
Errors and Functions 43
Relations
symmetric relation.
Corrections
relation R–1, then relation R is called relation.

(i) Wrong notation for matrices. (i) Matrices: ( ) or [ ].


(ii) Solution of equations: (ii) If AX = B, then X = A–1B.
If AX = B, then X = BA–1 is wrong.
. Common errors have been tagged to clear confusions
with cautions (iii) Not multiplying the first row elements with the second (iii) By memorising tips like Run and Jump remember while


andmatrix corresponding column elements and add
answers for productive learning. 32 MatheMatics–12
multiplying two matrices. More practice on various order
matrices for multiplication.

IMPORTANT FORMULAE
1. Multiplication of a Matrix by a Scalar Number
If A = [aij]m×n is a matrix and k is any scalar number, then kA = k[aij]m×n = [kaij]m×n.
2. Addition of Matrices
If matrix A = [aij]m×n and matrix B = [bij]m×n, then
A + B = [aij]m×n + [bij]m×n = [aij + bij]m×n.
3. Subtraction of Matrices
If matrix A = [aij]m×n and matrix B = [bij]m×n then
(A – B) = [aij]m×n – [bij]m×n = [aij – bij]m×n
4. Multiplication of a matrix with another matrix
[aij]m×n × [bjk]n×p = [cik]m×p
5. If A–1 is the inverse matrix of matrix A then AA–1 = A–1A = I.
6. If A′ is the transpose of matrix A, then
(i) (A′)′ = A (ii) (A + B)′ = A′ + B′
(iii) (A – B)′ = A′ – B′ (iv) (AB)′ = B′A′
(v) (ABC)′ = C′B′A′.
7. If A is a square matrix, then
(i) (A + A′) is a symmetric matrix. (ii) (A – A′) is a skew-symmetric matrix.
(iii) (AA′) and (A′A) are symmetric matrix.
8. Any square matrix A can be expressed as a sum of symmetric and skew-symmetric matrix:

A= 1
( A + A′ ) +
2 

1
(A − A′
2 

)
Symmetric matrix Skew-symmetric matrix

Errors 73
. Important Formulae are given at the


end of the each chapter.

(v)
Dedicated to the everlasting memory

of my mother Shanti Devi and Grandmother Gopali Devi

who were a perpetual source of inspiration for me.


Syllabus

One Paper Max. Marks: 80



Units No. of Periods Marks
I. Relations and Functions 30 08
II. Algebra 50 10
III. Calculus 80 35
IV. Vectors and Three - Dimensional Geometry 30 14
V. Linear Programming 20 05
VI. Probability 30 08
Total 240 80
Internal Assessment 20

UNIT I: RELATIONS AND FUNCTIONS


1. Relations and Functions (15 Periods)



Types of relations: reflexive, symmetric, transitive and equivalence relations. One to one and onto functions.

2. Inverse Trigonometric Functions (15 Periods)



Definition, range, domain, principal value branch. Graphs of inverse trigonometric functions.

UNIT II: ALGEBRA
1. Matrices (25 Periods)



Concept, notation, order, equality, types of matrices, zero and identity matrix, transpose of a matrix, symmetric and skew symmetric

matrices. Operation on matrices: Addition and multiplication and multiplication with a scalar. Simple properties of addition,
multiplication and scalar multiplication. On-commutativity of multiplication of matrices and existence of non-zero matrices whose
product is the zero matrix (restrict to square matrices of order 2). Invertible matrices and proof of the uniqueness of inverse, if it
exists; (Here all matrices will have real entries).
2. Determinants (25 Periods)



Determinant of a square matrix (up to 3 × 3 matrices), minors, co-factors and applications of determinants in finding the area

of a triangle. Adjoint and inverse of a square matrix. Consistency, inconsistency and number of solutions of system of linear
equations by examples, solving system of linear equations in two or three variables (having unique solution) using inverse
of a matrix.

UNIT III: CALCULUS


1. Continuity and Differentiability (20 Periods)



Continuity and differentiability, chain rule, derivatives of inverse trigonometric functions, like sin x, cos x and tan–1 x,
–1 –1


derivative of implicit functions. Concept of exponential and logarithmic functions.
Derivatives of logarithmic and exponential functions. Logarithmic differentiation, derivative of functions expressed in parametric

forms. Second order derivatives.
2. Applications of Derivatives (10 Periods)



Applications of derivatives: rate of change of bodies, increasing/decreasing functions, maxima and minima (first derivative test

motivated geometrically and second derivative test given as a provable tool). Simple problems (that illustrate basic principles
and understanding of the subject as well as real-life situations).

(vii)
3. Integrals (20 Periods)



Integration as inverse process of differentiation. Integration of a variety of functions by substitution, by partial fractions and

by parts, Evaluation of simple integrals of the following types and problems based on them.
dx dx dx dx dx
∫ x2 ± a2 , ∫ x ±a 2 2
,∫
2
a −x 2
,∫
ax 2 + bx + c ∫ ax 2 + bx + c
,

px + q px + q
∫ ax 2 + bx + c dx, ∫ 2
ax + bx + c
dx, ∫ a 2 ± x 2 dx,∫ x 2 − a 2 dx

∫ ax 2 + bx + c dx

Fundamental Theorem of Calculus (without proof). Basic properties of definite integrals and evaluation of definite integrals.

4. Applications of the Integrals (15 Periods)



Applications in finding the area under simple curves, especially lines, circles/parabolas/ellipses (in standard form only).

5. Differential Equations (15 Periods)



Definition, order and degree, general and particular solutions of a differential equation. Solution of differential equations by

method of separation of variables, solutions of homogeneous differential equations of first order and first degree. Solutions
of linear differential equation of the type:
dy
+ py = q, where p and q are functions of x or constants.
dx

dx + px = q, where p and q are functions of y or constants.
dy

UNIT IV: VECTORS AND THREE-DIMENSIONAL GEOMETRY
1. Vectors (15 Periods)



Vectors and scalars, magnitude and direction of a vector. Direction cosines and direction ratios of a vector. Types of vectors

(equal, unit, zero, parallel and collinear vectors), position vector of a point, negative of a vector, components of a vector,
addition of vectors, multiplication of a vector by a scalar, position vector of a point dividing a line segment in a given ratio.
Definition, Geometrical Interpretation, properties and application of scalar (dot) product of vectors, vector (cross) product of
vectors.
2. Three - dimensional Geometry (15 Periods)



Direction cosines and direction ratios of a line joining two points. Cartesian equation and vector equation of a line, skew

lines, shortest distance between two lines. Angle between two lines.

UNIT V: LINEAR PROGRAMMING


1. Linear Programming (20 Periods)



Introduction, related terminology such as constraints, objective function, optimization, graphical method of solution for

problems in two variables, feasible and infeasible regions (bounded and unbounded), feasible and infeasible solutions, optimal
feasible solutions (up to three non-trivial constraints).

UNIT VI: PROBABILITY


1. Probability (30 Periods)



Conditional probability, multiplication theorem on probability, independent events, total probability, Bayes’ theorem, Random

variable and its probability distribution, mean of random variable.

(viii)
Question Paper Design for Mathematics (Class XII)
Time : 3 Hours Max. Marks : 80


Total
S. No. Typology of Questions % Weightage
Marks

1. Remembering: Exhibit memory of previously learned material
by recalling facts, terms, basic concepts, and answers.
Understanding: Demonstrate understanding of facts and ide- 44 55
as by organizing, comparing, translating, interpreting, giving
descriptions, and stating main ideas

2. Applying: Solve problems to new situations by applying ac-


20 25
quired knowledge, facts, techniques and rules in a different way.

3. Analysing :
Examine and break information into parts by identifying mo-
tives or causes. Make inferences and find evidence to support
generalizations
Evaluating:
Present and defend opinions by making judgments about in- 16 20
formation, validity of ideas, or quality of work based on a set
of criteria.
Creating:
Compile information together in a different way by combining
elements in a new pattern or proposing alternative solutions

TOTAL 80 100

1. No chapter wise weightage. Care to be taken to cover all the chapters


2. Suitable internal variations may be made for generating various templates keeping the overall weightage to different
form of questions and typology of questions same.
Choice(s):
There will be no overall choice in the question paper.
However, 33% internal choices will be given in all the sections

(ix)
CONTENTS

1. Relations and Functions ..........................................................................................................................................11




.
2. Inverse Trigonometric Functions ........................................................................................................................... 34


.
3. Matrices .................................................................................................................................................................. 44


.
4. Determinants .......................................................................................................................................................... 75


.
5. Continuity And Differentiability ............................................................................................................................ 98


.
6. Application of Derivatives ................................................................................................................................... 137


.
7. Integrals ................................................................................................................................................................ 178


.
8. Area Between Curves ........................................................................................................................................... 246


.
9. Differential Equations .......................................................................................................................................... 277


.
10. Vector Algebra ...................................................................................................................................................... 309


.
11. Three Dimensional Geometry .............................................................................................................................. 335


.
12. Linear Programming ............................................................................................................................................ 355


.
13. Probability ............................................................................................................................................................ 377


.
• Sample Paper-1 (Solved) ................................................................................................................................. 410

.
• Sample Paper-2 (Unsolved) ............................................................................................................................. 427

.
• Sample Paper-3 (Unsolved) ............................................................................................................................. 434

.
(x)
EXAM BITES

This Pdf Is
Downloaded From
www.exambites.in

Visit www.exambites.in for


More Premium Stuffs,Latest
Books,Test Papers,Lectures etc.
jeeneetadda
jna_official
jeeneetadda

VISIT NOW !!
E:\AMIT_WORKS\Exam_Guru\EG_Mathematics-12_(working_02-06-2022)\EG_Mathematics-12_working\Open_Files\Chap_1\Chap_1
\ 16-Aug-2022  Amit   Proof-5 Reader’s Sign _______________________ Date __________


Topics
C
overed
1 Relations and Functions

1.1 Relations 1.2 Types of Relations






1.3 Functions


C hapter map
RELATIONS

Type of Relations

Universal Empty Reflexive Symmetric Transitive Equivalence

Equivalence Relation

Functions

Types of Functions

One-one/Injective Function Onto/Surjective Function Domain and Co-domain


of the Function

Bijective Function

Cartesian Product of Sets (Recap)


Let A and B be two non-empty sets. A set of all the ordered pairs (a, b) such that a ∈ A and b ∈ B is called cartesian product of set
A with set B and denoted as (A × B).
Or A × B = {(a, b) : a ∈ A and b ∈ B)}

The number of ordered pairs in (A × B) = n(A × B) = n(A) × n(B)


= (number of elements in set A) × (number of elements in set B).


Example 1. Given set A = {1, 2, 3} and set B = {3, 5}. Value of A × B and n(B × A) respectively are given by
(a) {(1, 3), (1, 5), (2, 3), (2, 5), (3, 3), (5, 3)} and 6


(b) {(1, 3), (1, 5) (2, 3), (5, 2), (3, 3), (5, 3)} and 5


(c) {(1, 3), (1, 5), (2, 3), (2, 5), (3, 3), (3, 5) and 6


(d) {(1, 3), (5, 1), (2, 3), (2, 5), (3, 3), (5, 3)} and 6


Solution. We are given two sets: A = {1, 2, 3} and B = {3, 5}
A × B = {1, 2, 3} × {3, 5} = {(1, 3), (1, 5), (2, 3), (2, 5), (3, 3), (3, 5)}

We are given: n(A) = 3 and n(B) = 2
\ n(B × A) = n(B) × n(A) = 2 × 3 = 6

Hence, option (c) is the correct answer.

11
E:\AMIT_WORKS\Exam_Guru\EG_Mathematics-12_(working_02-06-2022)\EG_Mathematics-12_working\Open_Files\Chap_1\Chap_1
\ 16-Aug-2022  Amit   Proof-5 Reader’s Sign _______________________ Date __________


RECAP EXERCISE
I. Multiple Choice Questions (MCQs)


Choose the correct answer from the given options.

Given for Q1 to Q6: Set A = {–1, 0, 1} and Set B = {3, 7, 8}

1. The value of A × B will be


(a) {(–1, 0), (–1, –1), (–1, 3), (0, –1), (1, –1)}


(b) {(3, –1), (7, –1), (8, –1), (3, 0), (7, 0), (8, 0), (3, 1), (7, 1), (8, 1)}


(c) {(–1, 3), (–1, 7), (–1, 8), (3, 0), (7, 0), (8, 0), (3, 1), (7, 1), (8, 1)}


(d) {(–1, 3), (–1, 7), (–1, 8), (0, 3), (0, 7), (0, 8), (1, 3), (1, 7), (1, 8)}


2. The value of (B × A) is


(a) {(3, –1), (3, 0), (3, 1), (7, –1), (7, 0), (7, 1), (8, –1), (8, 0), (8, 1)}


(b) {(–1, 3), (0, 3), (1, 7), (–1, 7), (0, 7), (1, 7), (–1, 8), (0, 8), (1, 8)}


(c) {(3, 0), (3, –1), (7, 0), (–1, 7), (8, –1), (1, 7), (1, 8), (0, 8)}


(d) None of these


3. The value of A × A equals


(a) {(1, 1), (1, 0), (1, 1), (0, 1), (0, 0), (0, –1), (–1, 1), (–1, 0), (1, –1)}


(b) {(–1, 1), (1, 0), (1, –1), (0, 1), (0, 0), (0, –1), (1, 1), (1, 0), (1, 1)}


(c) {(–1, –1), (–1, 0), (–1, 1), (0, –1), (0, 0), (0, 1), (1, –1), (1, 0), (1, 1)}


(d) None of these


4. n(A) is equal to


(a) 1 (b) 2 (c) 3 (d) 6








5. n(A × B) is equal to


(a) 3 (b) 6 (c) 9 (d) None of these








6. n(A × A) is equal to


(a) 3 (b) 6 (c) 9 (d) None of these








Given for Q7 to Q8: Set A = {2, 3, 4}
7. (A × A ) is equal to


(a) {(2, 2), (2, 3), (2, 4), (3, 2), (3, 3), (3, 4), (4, 2), (4, 3), (4, 4)}


(b) {(2, 2), (2, 3), (2, 4), (2, 3), (3, 3), (4, 3), (2, 4), (3, 4), (4, 4)}


(c) {(2, 2), (3, 2), (4, 2), (2, 3), (3, 3), (3, 4), (4, 2), (4, 3), (4, 4)}


(d) None of these


8. n(A) is equal to


(a) 1 (b) 2 (c) 3 (d) 6








Answers and Hints
1. (d) {(–1, 3), (–1, 7), (–1, 8), (0, 3), (0, 7), (0, 8), (1, 3), 3. (c) {(–1, –1), (–1, 0), (–1, 1), (0, –1), (0, 0), (0, 1), (1, –1),






(1, 7), (1, 8)} (1, 0), (1, 1)}


4. (c) 3 5. (c) 9 6. (c) 9
2. (a) {(3, –1), (3, 0), (3, 1), (7, –1), (7, 0), (7, 1), (8, –1),







7. (a) {(2, 2), (2, 3), (2, 4), (3, 2), (3, 3), (3, 4), (4, 2), (4, 3), (4, 4)}



(8, 0), (8, 1)}



8. (c) 3

Topic 1. Relations



Relation from Set A to Set B
Let A and B be two non-empty sets, then the relation from set A to set B is a subset of set (A × B). For example, if set A = {1, 2, 3}
and set B = {3, 4, 5}, then there can be many relations from set A to set B. R1 = {1, 3), (2, 5)}; R2 = {(1, 5), (2, 4), (2, 3)}; R3 = {(1, 3),

(1, 4), (1, 5), (2, 5)}; etc. are some of the relations from set A to set B.

12 Mathe atics–12
m
E:\AMIT_WORKS\Exam_Guru\EG_Mathematics-12_(working_02-06-2022)\EG_Mathematics-12_working\Open_Files\Chap_1\Chap_1
\ 16-Aug-2022  Amit   Proof-5 Reader’s Sign _______________________ Date __________


Number of all Possible Relations from Set A to Range of a Relation R
Set B Range of a relation R is a set of all the second coordinates of all


Number of elements in set A = n(A). Number of elements in set the ordered pairs of relation R.
B = n(B) Or Range = {b : (a, b) ∈ R}



In relation: R = {(3, 1), (7, 8), (–9, 5)}, Range = {1, 8, 5}
\ Number of ordered pairs in (A × B) = n(A × B)


Example 2. Given: set A = {2, 3, 4, 7} and set B = {3,


= n(A) × n(B) 4, 5, 6}. A relation R from set A to set B is defined as:

\ Number of all possible relations from set A to set R : {(a, b): b is divisible by a and a ∈ A and b ∈ B}. Domain of
relation R will be given by


B = 2n(A × B) = 2n(A) × n(B).
(a) {2, 4, 5} (b) {2, 3, 5} (c) {3, 5, 6} (d) {2, 3, 4}

Example 1. Number of relations possible from set A = {1, 2, 3}









Solution. The given sets are: A = {2, 3, 4, 7} and B = {3, 4, 5, 6}
to set B = {6, 7, 8, 9} is given by R = {(2, 4), (2, 6), (3, 3), (3, 6), (4, 4)}
(a) 210 (b) 28 (c) 212 (d) 211


Domain of R = set of all first coordinates of all the ordered









Solution. We have A = {1, 2, 3}, B = {6, 7, 8, 9} pairs of relation R = {2, 3, 4}
⇒ n(A) = 3, n(B) = 4 ⇒ n(A × B) = n(A) × n(B) = 3 × 4 = 12 Hence, option (d) is the correct answer.

\ Number of possible relations from set A to set B = 212. Example 3. A relation R in set A = {1, 2, 3, 4, 5} is defined as:


R = {(a, b) : a is an odd number and b is an even number and a
Hence, option (c) is the correct answer.
∈ A and b ∈ A}. Range of R will be
Domain of a Relation R (a) {2, 3} (b) {2, 4} (c) {5, 4} (d) {1, 5}









Relation R from set A to set B is a subset of set (A × B) and Solution. The given set is A = {1, 2, 3, 4, 5}
A × B is a set of all possible ordered pairs (a, b), where a ∈ A In roster form R = {(1, 2), (1, 4), (3, 2), (3, 4), (5, 2),


and b ∈ B. Now, a set of all the first coordinates of all the ordered (5, 4)}
pairs of relation R is called the domain of relation R. \ Range of R = set of all second coordinates of all the ordered


Or D = {a : (a, b) ∈ R} pairs of relation R = {2, 4}.


In relation: R = {(2, 1), (3, 7), (5, 8)}, Domain = {2, 3, 5} Hence, option (b) is the correct answer.

EXERCISE 1.1
I. Multiple Choice Questions (MCQs) 4. A relation R is defined in set A = {1, 3, 5, 7, 9} such that




Choose the correct answer from the given options. R = {(a, b) : a + b ≤ 10} a ∈ A, b ∈ A}. Domain of the

relation R is given by
1. Number of relations possible in set A having 3 elements


will be (a) {1, 3, 5, 8, 9} (b) {1, 3, 5, 7, 9}




(a) 26 (b) 29 (c) 28 (d) 23 (c) {1, 3, 6, 7, 9} (d) {1, 6, 7, 9, 5}












2. Number of relations possible in set A = {2, 4, 7, 9} is given by II. Short Answer Type Questions-I


(a) 2 10
(b) 2 8
(c) 2 12
(d) 2 16 1. A relation R is defined from set A = {1, 3, 4, 5} to










3. A relation R is defined from set A = {1, 2, 3} to set B = {4, set B = {2, 4, 6} as R = {(a, b) : b is divisible by a}. Here,


a ∈ A and b ∈ B. Write the relation R in


5, 6, 7}. Number of relations possible from set A to set B
will be (i) Roster form (ii) domain of R and




(a) 2 7
(b) 2 8
(c) 2 12
(d) 2 10 (iii) Range of the relation R










Answers 1.1
I. 1. (b) 29 4. (b) {1, 3, 5, 7, 9}







2n(A) × n(A) = 23 × 3 = 29 II. 1. (i) Relation R in roster form





2. (d) 216 R = {(1, 2), (1, 4), (1, 6), (3, 6), (4, 4)}



2n(A) × n(A) = 24 × 4 = 216


(ii) Domain of relation R = {1, 3, 4}
3. (c) 212



(iii) Range of relation R = {2, 4, 6}



2n(A) × n(B) = 23 × 4 = 212



Topic 2. Types of Relations
Here, we shall discuss following types of relations: (i) Empty relation, (ii) Universal relation, (iii) Identity relation, (iv) Reflexive
relation, (v) Symmetric relation, (vi) Transitive relation and (vii) Equivalence relation.
(i) (a) Empty Relation from Set A to Set B: Let A and B be two non-empty sets. Then the relation R from set A to set B is called



an empty relation if no element of set A is related to any element of set B.

Relations and Functions 13



E:\AMIT_WORKS\Exam_Guru\EG_Mathematics-12_(working_02-06-2022)\EG_Mathematics-12_working\Open_Files\Chap_1\Chap_1
\ 16-Aug-2022  Amit   Proof-5 Reader’s Sign _______________________ Date __________


For example, set A= {1, 2, 3, 4} and set B = {5, 6, 7}. is symmetric because (2, 3), (3, 2) and (2, 4), (4, 2) are


Now, consider a relation R from set A to set B defined present whereas relation
as: R = {(a, b) : a + b = 15, a ∈ A and b ∈ B}. R2 = {(2, 3), (2, 4), (3, 2), (4, 2), (3, 4)}


This is an empty relation as no pair (a, b) satisfies the is not a symmetric relation because for ordered pair (3, 4),

condition: a + b = 15.


ordered pair (4, 3) is not present.
R = φ ⊂ (A× B). (vi) Transitive Relation: A relation R in set A is called





(b) Empty Relation in Set A: Relation R in a non-empty transitive if for ordered pairs (a, b) and (b, c) present in



set A is called empty relation if no element of set A is relation R, there must be a third ordered pair (a, c) also
related to any element of set A i.e., R = φ ⊂ (A × A). present in relation R. Here a, b, c ∈ A
Consider a set A={1, 2, 3, 4}. Now, define a relation or A relation R in set A is called transitive if (a, b) ∈ R and




R in set A as: (b, c) ∈ R
R = {(a, b) : a – b = 10, a, b ∈ A}. ⇒ (a, c) ∈ R for a, b, c ∈ A




This is an empty set as no pair (a, b) satisfies the Take a set A = {1, 2, 3} and take a relation R1 in set A



condition a – b = 10. defined as R = {(1, 2), (2, 3), (1, 3), (3, 3), (3, 2)} is


(ii) (a) Universal Relation from Set A to Set B: Let A and transitive because
(1, 2) ∈ R and (2, 3) ∈ R ⇒ (1, 3) ∈ R



B be two non-empty sets, then the relation R from




set A to set B is called universal relation if each element (2, 3) ∈ R and (3, 3) ∈ R ⇒ (2, 3) ∈ R




of set A is related to every element of set B, i.e., (1, 3) ∈ R and (3, 3) ∈ R ⇒ (1, 3) ∈ R




R = A × B. (3, 3) ∈ R and (3, 2) ∈ R ⇒ (3, 2) ∈ R




Take two sets, A = {1, 2, 3, 4} and B = {5, 6, 7}. Now, (3, 2) ∈ R and (2, 3) ∈ R ⇒ (3, 3) ∈ R


define a relation R from set A to set B as:



(vii) Equivalence Relation: A relation R in set A is said to be


R = {(a, b) : |a – b| ≥ 0, a ∈ A and b ∈ B} is the universal


an equivalence relation if R is reflexive, symmetric and

relation as each element of set A is related to every transitive.
element of set B. For example, a relation R defined in set Z of integers.

(b) Universal Relation in Set A: A relation R in set A is R = {(x, y) : x + y is an integer x ∈ Z and y ∈ Z} is an


called a universal relation if each element of set A is

equivalence relation.

related to every element of set A i.e., R = A × A.
Take a set A = {2, 3, 4}. Relation R in set A is defined Reason
(i) Reflexive: Take any integer x. Since, sum of two integers

as: R = {(a, b : a + b < 10, a, b ∈ A}, is a universal


relation as each element of set A is related to every is an integer, therefore x + x is an integer.
element of set A. \ (x, x) ∈ R ⇒ R is reflexive.




(iii) Identity Relation: Let A be a set, then relation IA on A is (ii) Symmetric: Take any two integers x and y. Since, sum of




called identity relation if every element of set A is related any two integers is an integer, therefore (x + y) and (y + x)
to itself only are integers.
or IA = {(a, a) : a ∈ A} \ (x, y) ∈ R and (y, x) ∈ R ⇒ R is symmetric.






For example: Take A = {1, 2, 3} then relation R = {(1, 1), (iii) Transitive: Take any three integers x, y and z. Since, sum


of any two integers is an integer, therefore x + y, y + z and

(2, 2), (3, 3)} is an identity relation on set A.


x + z are integers.


(iv) Reflexive Relation: A relation R on set A is said to be
\ (x, y) ∈ R and (y, z) ∈ R


reflexive if every element of set A is related to itself. Thus,


R on a set A is not reflexive if there exists an element a ∈ ⇒ (x, z) ∈ R. So, relation R is transitive.


A such that (a, a) ∉ R. Above discussion proves that R is reflexive, symmetric and
Note: Every identity relation is reflexive but every reflexive transitive, therefore R is an equivalence relation.

relation is not identity relation. In set A = {2, 3} Example 1. A relation R in the set of natural number N, let R
(a) R1 = {(2, 2), (3, 3)} is identity and reflexive relation be defined as:


and R = {(x, y) : x + y = a natural number, x, y ∈ N}. R is

(b) R2 = {(2, 2), (3, 3), (2, 3)} is reflexive relation but not (a) Equivalence relation




identity relation due to presence of (2, 3). (b) Transitive, symmetric, not reflexive



(v) Symmetric Relation: A relation R on set A is said to be (c) Reflexive, symmetric, not transitive




symmetric relation when for every ordered pair (a, b) (d) None of these


present in relation R, there must present an ordered pair Solution. We have N = set of all natural numbers and R = {(x, y)
(b, a) also in relation R, here a, b ∈ A. : x + y = a natural number, x ∈ N and y ∈ N}.


Take a set A = {2, 3, 4} and a relation R1 in set A defined (i) R is reflexive relation: Let x be any natural number, then



as (x + x) is also a natural number.
R1 = {(2, 3), (2, 4), (3, 2), (4, 2)} \ (x, x) ∈ R ⇒ R is reflexive relation.




14 Mathe atics–12
m
E:\AMIT_WORKS\Exam_Guru\EG_Mathematics-12_(working_02-06-2022)\EG_Mathematics-12_working\Open_Files\Chap_1\Chap_1
\ 16-Aug-2022  Amit   Proof-5 Reader’s Sign _______________________ Date __________


(ii) R is symmetric relation: Take any two natural numbers x Example 3. A relation R in the set of integers Z is defined as:


and y. Since, sum of any two natural numbers is a natural R = {(x, y) : x ≥ y and x, y ∈ Z}. The relation R is
number, therefore (x + y) and (y + x) are natural numbers. (a) reflexive and transitive but not symmetric



(x, y) ∈ R and (y, x) ∈ R (b) reflexive, symmetric but not transitive



⇒ R is symmetric relation. (c) equivalence




(iii) R is transitive relation: Take any three natural numbers x, (d) None of these.





y and z. Since, sum of any two natural numbers is a natural Solution. We have R = {(x, y) : x ≥ y and x, y ∈ Z}
number, therefore, (x + y), (y + z) and (x + z) are natural (i) Relation R is reflexive: Relation R is : x ≥ y, x, y ∈ Z. If
numbers.



we put y = x in relation x ≥ y and it remains true, then R is
Hence, (x, y) ∈ R and (y, z) ∈ R ⇒ (x, z) ∈ R. reflexive. So, put y = x in x ≥ y. It gives:

This shows that R is transitive relation. x ≥ x ⇒ x = x. It is true for any integers.



Above discussion proves that R is reflexive, symmetric \ R is reflexive.




and transitive and hence R is an equivalence relation. (ii) Relation R is transitive: Relation R is: x ≥ y, x, y ∈ Z.



Hence, option (a) is the correct answer. Take any three integers, x, y, z such that x ≥ y, y ≥ z. This




Example 2. A relation R in the set of natural numbers N is gives: x ≥ z.
defined as: Thus, x ≥ y and y ≥ z ⇒ x≥z




R = {(x, y) : x2 – 4xy + 3y2 = 0, x, y ∈ N} \ (x, y) ∈ R and (y, z) ∈ R ⇒ (x, z) ∈ R.






The relation R is Thus, relation R is transitive.



(a) reflexive, not symmetric and not transitive (iii) Relation R is not symmetric: Relation R is: x ≥ y, x, y ∈



Z. If x ≥ y, then it is not possible: y ≥ x. It can be true only


(b) equivalence
for equality and not for all numbers.


(c) symmetric
Thus, R is not symmetric.


(d) None of these

Hence, option (a) is the correct answer.


Solution. We have R = {(x, y) : x2 – 4xy + 3y2 = 0, x, y ∈ N} and

N = set of all natural numbers Equivalence Classes
(i) R is reflexive relation: To prove a relation is reflexive, Assume that R is an equivalence relation in a non-empty set A.


put y = x in x2 – 4xy + 3y2 = 0. If the relation remains true, Let a ∈ A, then the set of all those elements of set A, which are
then the relation is reflexive. related to a, is called equivalence class determined by a and is
x2 – 4xy + 3y2 = 0 ⇒ x2 – 4x(x) + 3x2 = 0 denoted by [a].



⇒ x2 – 4x2 + 3x2 = 0 ⇒ 0 = 0 (True) Example 4. Take a relation in Z, i.e., in integers defined as R =




⇒ R is reflexive relation. {(x, y) : (x – y) is divisible by 2 ∀ x ∈ Z and y ∈ Z}. Relation R




(ii) R is not symmetric relation: If we put x for y and y for x is an equivalence relation. Find the equivalence classes of [0], [1].


in the relation: x2 – 4xy + 3y2 = 0 and this relation remains Solution. [0] = {x ∈ Z : (0, x) ∈ R or (x, 0) ∈ R} = {x ∈ Z :


true, then the relation R will be symmetric otherwise not. (x – 0) or (0 – x) is divisible by 2}

Let us put x for y and y for x in: x2 – 4xy + 3y2 = 0, we get = {x ∈ Z, and x is divisible by 2}



y2 – 4yx + 3x2 = 0 ⇒ 3x2 – 4xy + y2 = 0. = {x = ..., .., –4, –2, 0, 2, 4, ...,}





It is not the same as x – 4xy + 3y2 = 0.
2
\ [0] = {..., ..., –4, –2, 0, 2, 4, ...,} is equivalence class




\ Thus, relation R is not symmetric. of 0.


(iii) R is not transitive relation: Relation R : x2 – 4xy + 3y2 = [1] = {x ∈ Z : (1, x) ∈ R or (x, 1) ∈ R} = {x ∈ Z :





0 is true for (9, 3) and (3, 1) but not true for (9, 1) (1, x) or (x, 1) is divisible by 2}

\ (9, 3) ∈ R and (3, 1) ∈ R ⇒ (9, 1) ∈ R = {x ∈ Z : (x – 1) or (1 – x) is divisible by 2}






Thus, R is not transitive relation. = {x ∈ Z : ..., –3, –1, 1, 3, ....,}



Hence, option (a) is the correct answer. \ [1] = {..., –3, –1, 1, 3, ...,} is equivalence class of 1.




EXERCISE 1.2
I. Multiple Choice Questions (MCQs) 2. Let R be a relation over the set of straight lines in a plane




Choose the correct answer from the given options. such that l1 R l2 such that l1 ^ l2, then R is

1. R is a relation over the set N × N and it is defined by (a) symmetric (b) reflexive






(a, b) R(c, d) ⇒ a + d = b + c, then R is (c) transitive (d) equivalence relation






(a) reflexive only (b) symmetric only 3. A relation R is defined on plane A of triangles in a given




(c) transitive only (d) equivalence relation


planes defined as R = {(T1, T2) : T1 @ T2 and T1, T2 ∈ A},




Relations and Functions 15

E:\AMIT_WORKS\Exam_Guru\EG_Mathematics-12_(working_02-06-2022)\EG_Mathematics-12_working\Open_Files\Chap_1\Chap_1
\ 16-Aug-2022  Amit   Proof-5 Reader’s Sign _______________________ Date __________


then R is 4. Let R be the equivalence relation in the set



(a) reflexive (b) symmetric A = {0, 1, 2, 3, 4, 5} given by R = {(a, b) : 2 divides (a – b);




(c) transitive (d) equivalence a, b ∈ A}. Write the equivalence class of [0].




4. Consider the non-empty set consisting of children in a [Delhi 2014]




family and a relation R defined as a Rb if a is a brother of 5. Let A = {1, 2, 3, 4} and R be a relation in set A given by



b. Then R is R = {(1, 1), (2, 2), (3, 3), (4, 4), (1, 2), (2, 1), (3, 1), (1, 3)}


(a) symmetric but not transitive then show that the relation R is reflexive and symmetric.


(b) transitive but not symmetric


6. Show that the relation R less than in the set of natural


(c) neither symmetric nor transitive



numbers is transitive.


(d) both symmetric and transitive. 7. For real numbers x and y, we write xRy ⇒ x – y + 2


5. Let R be the relation on the set R of all real numbers defined



is an irrational number, then show that the relation R is


by a R b iff |a – b| ≤ 1. Then R is reflexive.
(a) reflexive and symmetric 8. State the reason why a relation R = {(a, b) : a > b, a, b ∈ real


(b) symmetric only



numbers} defined in the set of real numbers, is transitive.


(c) transitive only 9. A relation R = {(x, y) : x + y < 10; x, y ∈ A} defined in the


(d) anti symmetric only



set A = {1, 2, 3, 4}. Is relation R symmetric? Give reason.


6. A relation R = {(a, b) : a divides b} on the set N of all 10. Take a relation R = {(a, b) : a = b, a, b ∈ A} defined on a


natural numbers is



set A of real numbers. Is R a reflexive relation? Give reason.
(a) reflexive (b) symmetric
III. Short Answer Type Questions-I




(c) reflexive and transitive (d) symmetric and transitive


1. If set A = {1, 2, 3, 4}, set B = {a, b, c, d} and relation




7. Let R be a relation on the set N defined by {(x, y) :


R = {(1, a), (2, b), (3, c), (4, d)} is a relation defined from


x, y ∈ N, 2x + y = 41}. Then R is
set A to set B. Write relation R–1.
(a) reflexive (b) symmetric
2. A relation R defined in set A = {1, 2, 3, ...., 10} as




(c) transitive (d) None of these.


R = {(x, y) : 3x – y = 0, x, y ∈ A}. Show that the relation




8. Let S be the set of all real numbers. Then the relation
is not reflexive.


R = {(a, b) : 1 + ab > 0} on S is
3. Let A = {a, b, c} and a relation R is defined in set A as:
(a) reflexive and symmetric but not transitive.


R = {(a, a), (b, c), (a, b)}. Write the minimum number of


(b) reflexive and transitive but not symmetric.
ordered pairs be included in relation R to make it reflexive


(c) symmetric and transitive but not reflexive. and symmetric. [HOTS]


(d) reflexive, symmetric and transitive.

4. Check whether the relation R defined in the set A = {1, 2, 3,


9. Let W denote the words in the English dictionary. The


4, 5, 6} as R = {(x, y) : y = x + 1, x, y ∈ A} is reflexive or


relation R is defined by R = {(x, y) ∈ W × W : the words


symmetric.
x and y have at least one letter in common}.
5. Show that the relation: R = {(1, 2), (2, 3), (3, 2), (1, 3),
Then R is


(2, 1), (2, 2), (3, 3), (3, 1), (1, 1)} is reflexive, symmetric

(a) not reflexive, symmetric and transitive and transitive.


(b) reflexive, symmetric and not transitive 6. Show that the relation: R ={(1, 1), (1, 2), (2, 1), (2, 2)} is


(c) reflexive, symmetric and transitive


reflexive, symmetric and transitive.


(d) reflexive, transitive and not symmetric 7. Show that the relation: R = {(1, 1), (3, 3), (5, 5), (1, 3),


10. On the set N of all natural numbers define the relation R


(3, 1), (3, 5)} is reflexive and is neither symmetric nor


by aRb if and only if the G.C.D. of a and b is 2, then R is transitive.
(a) reflexive but not symmetric 8. Take a set A={1, 3, 7} and find a relation R = A × A, then


(b) symmetric only


show that it is reflexive, symmetric and transitive.


(c) reflexive and transitive 9. Relation R defined in a set of natural numbers N as: R =


(d) reflexive, symmetric and transitive.


{(x, y) : x > y ∀ x, y ∈ N}. Show that R is transitive and


II. Very Short Answer Type Questions it is neither reflexive nor symmetric.


1. State the reason why the relation R = {(a, b) : a ≥ b2} 10. Relation R defined in a set of natural numbers N as:




defined in the set of real numbers is not reflexive. R = {(x, y) : x ≥ y ∀ x, y ∈ N}. Show that relation R is
2. State the reason of the relation R in the set {1, 2, 3} given reflexive and transitive but not symmetric.


by: R = {(1, 2), (2, 1)} not to be transitive. [A.I. 2011] 11. Relation R defined in a set of natural numbers N as:



3. If R = {(x, y) : x + 2y = 8} is a relation on N. Write the R = {(a, b) : (a + b) is an even natural number, where a, b


domain of R. [A.I. 2011] ∈ N}. Show that R is reflexive, symmetric and transitive.

16 Mathe atics–12
m
E:\AMIT_WORKS\Exam_Guru\EG_Mathematics-12_(working_02-06-2022)\EG_Mathematics-12_working\Open_Files\Chap_1\Chap_1
\ 16-Aug-2022  Amit   Proof-5 Reader’s Sign _______________________ Date __________


12. Relation R defined in a set of natural numbers N as: elements in A related to the right angled triangle T with


R = {(a, b) : (a – b) is an even natural number and a, b sides 3, 4 and 5? [NCERT]


∈ N}. Show that R is neither reflexive nor symmetric but 7. Let L be the set of all lines in the XY plane and R be the
transitive.



relation in L defined as: R = {(L1, L2) : L1 is parallel to L2
13. Relation R defined in a set of real numbers R as: and L1, L2 ∈ L}. Show that R is an equivalence relation.


R ={(x, y) : (x + y) is divisible by 5 ∀ x, y ∈ R}. Show Find the set of all lines related to line y = 2x + 4.

that R is neither reflexive nor transitive but symmetric. 8. Prove that the relation R in the set A ={1, 2, 3, 4, 5} given



Note: In a set of real numbers R, if relation R = {(a, b) : by R ={(a, b) : |a – b| and a, b ∈ A is an even number} is

(a + b) is a multiple of 3, 4, 5, 6, …, n ∀ a, b ∈ R} then R is an equivalence relation.




neither reflexive nor transitive but symmetric. 9. Show that the relation S in the set R of real numbers defined
14. Show that the relation R in set A ={1, 2, 3} given by



as: S = {(a, b) : a, b ∈ R and a ≤ b3} is neither reflexive


R = {(1, 1), (2, 2), (3, 3), (1, 2), (2, 3)} is reflexive but nor symmetric nor transitive. [HOTS]
neither symmetric nor transitive.


10. Let R be a relation defined on the set of natural numbers
15. Let L be the set of lines in a plane and R be the relation in



N as follows:


L defined as: R = {(L1, L2) : L1 is perpendicular to L2, L1
and L2 ∈ L}. Show that R is symmetric but neither reflexive R = {(x, y) : x, y ∈ N and 2x + y = 24}


nor transitive. Find the domain and range of the relation R. Also find R is


an equivalence relation or not. [Delhi 2014 (C)]
IV. Short Answer Type Questions-II


1. Let T be the set of all the triangles in a plane with R a V. Long Answer Type Questions





relation in T given by: R = {(T1, T2) : T1 is congruent to 1. Let N denote the set of all natural numbers and R be the



T2, T1 and T2 ∈ T}. Show that R is an equivalence relation. relation on N × N defined by (a, b) R (c, d) if ad(b + c) =
2. Show that the relation R in the set A of all the books in a bc(a + d). Show that R is an equivalence relation.


library of a college given by R = {(x, y) : x and y have the [Delhi 2015] [HOTS]

same number of pages and x, y ∈ A} is an equivalence

2. Let n be a fixed positive integer. Define a relation R in Z as
relation.


follows ∀ a, b ∈ Z, aRb if and only if (a – b) is divisible
3. Show that each of the relation R in the set A = {x ∈ Z : 0 by n. Show that R is an equivalence relation. [HOTS]


≤ x ≤ 12} given by


3. If R and S are equivalence relation in set A, show that
(A) R = {(a, b) : |a – b| is a multiple of 4 and a, b ∈ A}


R ∩ S is also an equivalence relation. [HOTS]


(B) R = {(a, b) : a = b and a, b ∈ A}


4. Let R be a relation defined as


are equivalence relations.


R = {(x, y) : x, y ∈ N and 2x + y = 41}

Find the set of all elements related to 1 in each case.

Find the domain and range of R also verify that R is neither

[NCERT]

reflexive, nor symmetric nor transitive.

4. Show that the relation R in the set A of points in a plane
5. Given the relation R = {(1, 2), (2, 3)} in set A = {1, 2, 3}.


given by: R = {(p, q) : distance of the point p from the


Find the minimum number of ordered pairs which when

origin is same as the distance of the point q from the origin}
is an equivalence relation. Further, show that the set of added to R makes it an equivalence relation.
all points related to a point p ≠ (0, 0) is the circle passing 6. Give an example to show that union of two equivalence


through p with origin as centre. [NCERT] relation on set A is not necessarily equivalence on A.

5. Show that the relation R defined in the set A of all triangles [HOTS]



as: R = {(T1, T2) : T1 is similar to T2 and T1, T2 ∈ T} is an 7. Let R be a relation defined in N × N defined by:
equivalence relation. Consider three right angled triangles


R = {(a, b) R (c, d) if and only if ad = bc and a, b, c,
T1 with sides 3, 4, 5, T2 with sides 5, 12, 13 and T3 with

sides 6, 8, 10. Which triangles among T1, T2 and T3 are d ∈ N}
related? [NCERT] Prove that R is an equivalence relation.[V. Imp.][HOTS]



6. Show that the relation R defined in the set A of all polygons 8. Let R be a relation on the set of integers given by aRb ⇒


a = 2Kb for some integer K. Show that R is an equivalence


as: R = {(P1, P2) : P1 and P2 have the same number of
sides} is an equivalence relation. What is the set of all the relation.

Answers 1.2
I. 1. (d) an equivalence relation R is not symmetric because if a is a brother of b, then b may not






2. (a) symmetric be a brother of a, b can be a sister of a.



3. (d) equivalence ∀ a, b ∈ A, (a, b) ∈ R does not imply (b, a) ∈ R. Therefore, R




4. (b) transitive but not symmetric is not symmetric.




Let A be the non-empty set consisting of children in a family Again, ∀ a, b, c ∈ A, (a, b) ∈ R and (b, c) ∈R


and a relation R in the set A given by aRb, if a is a brother of b. ⇒ (a, c) ∈ R

Relations and Functions 17

E:\AMIT_WORKS\Exam_Guru\EG_Mathematics-12_(working_02-06-2022)\EG_Mathematics-12_working\Open_Files\Chap_1\Chap_1
\ 16-Aug-2022  Amit   Proof-5 Reader’s Sign _______________________ Date __________


For if a is a brother of b and b is a brother of c, then a is brother 1 4
 1

of c. Therefore, R is transitive. and 1 +  −  (– 2) = 1 + = >0
 6 3 3


Hence, R is transitive but not symmetric.
But 1 + 3 (–2) = 1 – 6 = – 5 >/ 0

5. (a) reflexive and symmetric



R is not transitive.



∀ a ∈ R , |a – a| = 0 < 1, aRa



Hence, the relation R is reflexive and symmetric but not




⇒ R is reflexive.


transitive.

Now, aRb ⇒ |a – b| ≤ 1 9. (b) reflexive, symmetric and not transitive


  




⇒ |b – a| ≤ 1 [ |a – b| = |b – a|] W denote the words in the English dictionary.




⇒ bRa R = {(x, y) ∈ W × W : the words x and y have at least one letter


R is symmetric. in common}


Again, 1R2 and 2R1 but 1 ≠ 2 ∀ x ∈ W, (x, x) ∈ R


R is not anti-symmetric [ Every letter of a word is common with itself]



[Note that a relation R in a set A is said to be an anti-symmetric R is reflexive.



relation iff (a, b) ∈ R and (b, a) ∈ R ⇒ a = b, ∀ a, b ∈ R] Again, (x, y) ∈ R ⇒ (y, x) ∈ R ∀ x, y ∈ W.





Now, 1R2, 2R3 but 1 R 3 [ |1 – 3| = |– 2| = 2 > 1] [  If the words x and y have at least one letter in common,


R is not transitive. then words y and x have at least one letter in common]


Therefore, R is reflexive and symmetric. ⇒ R is symmetric.



6. (c) reflexive and transitive (x, y) ∈ R, (y, z) ∈ R but (x, z) ∉ R





As every natural number divides itself, i.e., ∀ a ∈ N, [The words ‘every’ and ‘word’ have ‘r’ common and ‘word’



(a, a) ∈ R. and ‘with’ have ‘w’ common, but ‘every’ and ‘with’ do not have
So, R is reflexive. any letter common].
⇒ R is not transitive.

2 divides 6 but 6 does not divide 2. Therefore a divides b does

Therefore, R is reflexive, symmetric and not transitive.

not imply b divides a, i.e., (a, b) ∈ R need not imply (b, a) ∈ R.


Hence, R is not symmetric. 10. (b) symmetric only



II. 1. R is not reflexive because a ≥ b2 does not imply a ≥ a2 ∀ a.

Now, 2 divides 4 and 4 divides 8 ⇒ 2 divides 8








(a, b) ∈ R and (b, c) ∈ R ⇒ (a, c) ∈ R 2. Relation R is not transitive because: (1, 2), (2, 1) ∈ R but




(1, 1) ∉ R.



So, R is transitive.
3. Here R = {(2, 3), (4, 2), (6, 1)},

Hence, R is reflexive and transitive.


Domain of R = {2, 4, 6}

7. (d) None of these.

4. Equivalence class of [0] = {0, 2, 4}



On the set N of natural numbers, R is a relation defined by {(x,


5. Relation R is reflexive and symmetric because (1, 1),

y) : x, y ∈ N, 2x + y = 41}


(2, 2), (3, 3) ∈ R
R = {(x, y) : x, y ∈ N, 2x + y = 41} 1 ∈ N
⇒ R is reflexive and (1, 2), (2, 1), (3, 1), (1, 3) ∈ R

but (1, 1) ∉ R as 2(1) + 1 = 3 ≠ 41


⇒ R is symmetric.

So, R is not reflexive.


6. Reason: Take a, b, c ∈ N such that a < b and b < c

Again, (1, 39) ∈ R but (39, 1) ∉ R.




⇒ a<c \ (a, b) ∈ R and (b, c) ∈ R

R is not symmetric.




⇒ (a, c) ∈ R.


Now, (20, 1) ∈ R and (1, 39) ∈ R but (20, 39) ∉ R.


Hence, relation R is transitive.

So, R is not transitive.

7. Relation R = x – y + 2 is reflexive because

R is neither reflexive, nor symmetric nor transitive.


x R x = x –x + 2 = 2 .
∴ 
8. (a) reflexive and symmetric but not transitive.


8. Same reason as given in solution of Q.No. 6.



R = {(a, b) : 1 + ab > 0 ∀ a ∈ S}


9. Relation R is symmetric because

1 + a ⋅ a = 1 + a2 > 0


x + y < 10 ⇒ y + x < 10
Therefore, aRa, i.e. the relation R is reflexive.



⇒ (x, y) ∈ R ⇒ (y, x) ∈ R

Again, ∀ a, b ∈ S, 1 + ab > 0 ⇒ 1 + ba > 0




10. Relation R is reflexive because a = a




[ ab = ba ∀ a, b ∈ S]


⇒ (a, a) ∈ R for all a ∈ A.


i.e. aRb = bRa


III. 1. R–1 = {(a, 1), (b, 2), (c, 3), (d, 4)}.

The relation R is symmetric.



2. Given 3x – y = 0. Put y = x in 3x –y = 0

1
Now, we consider three real numbers 3, − and – 2.


6 It gives 3x – x = 0 ⇒ 2x = 0.




 1 1 1 It is not true.
We have1 + 3  −  = 1 – = >0

 6 2 2 \ R is not reflexive.



18 Mathe atics–12
m
E:\AMIT_WORKS\Exam_Guru\EG_Mathematics-12_(working_02-06-2022)\EG_Mathematics-12_working\Open_Files\Chap_1\Chap_1
\ 16-Aug-2022  Amit   Proof-5 Reader’s Sign _______________________ Date __________


3. To make relation R reflexive, include (b, b) and (c, c). i.e., (x, y) ∈ R and (y, z) ∈ R







Now, (a, b) ∈ R and (b, c) ∈ R but (b, a) and (c, b) are not ⇒ (x, z) ∈ R \ R is transitive.






included in R. Therefore, include (b, a) and (c, b) to make R 11. R = {(a, b) : (a + b) is an even natural number and a, b ∈ N}






symmetric. (i) R is reflexive: Take a ∈ N. Since (a + a) is an even number



Hence, to make R reflexive and symmetric include (b, b), (c, c), ∀ a ∈ N.

(b, a) and (c, b). (a, a) ∈ R ⇒ R is reflexive.




4. (i) Relation: y = x + 1. (ii) Symmetric: Let a, b ∈ N.





If y = x makes it true, R will be reflexive If (a + b) is an even number then (b + a) is also an even



\ Put y = x in y = x + 1. number.
\ (a, b) ∈ R ⇒ (b, a) ∈ R


It gives x = x + 1





⇒ R is symmetric.

⇒ 0 = 1 (False)



(iii) Transitive: Since sum of two even numbers is even or sum


\ R is not reflexive.



of two odd numbers is even.


(ii) Relation: y = x + 1.
\ Let a, b, c ∈ N are either all even numbers or odd numbers.


Put x = y and y = x in y = x + 1.



\ (a + b) is even and (b + c) is even

It gives x = y + 1.



⇒ (a + c) is also even



It is not same as y = x + 1. \ (a, b) ∈ R and (b, c) ∈ R ⇒ (a, c) ∈ R



Hence, R is no symmetric. ⇒ R is transitive.



5. (i) Reflexive: R is reflexive because 12. R = {(a, b) : (a – b) is an even natural number and a, b ∈ N}








{(1, 1), (2, 2), (3, 3)} ∈ R (i) Reflexive: Since (a – a) = 0 (not a natural no.)



(ii) Symmetric: R is symmetric because ⇒ (a, a) ∉ R \ R is not reflexive.






{(1, 2), (2, 1), (2, 3), (3, 2), (1, 3), (3, 1)} ∈ R
(ii) Symmetric: If (a – b) is even natural number then (b – a)




(iii) Transitive: R is transitive because (1, 2) ∈ R and is not an even natural number.
\ (a, b) ∈ R whereas (b, a) ∉ R.



(2, 3) ∈ R


⇒ (1, 3) ∈ R and so on. Hence, R is not symmetric.



6. (i) Reflexive: R is reflexive because (iii) Transitive: a, b, c ∈ N such that (a – b) and (b – c) are even




natural numbers then (a – c) will also be an even natural



{(1, 1), (2, 2)} ∈ R numbers.

(ii) Symmetric: R is symmetric because \ (a, b) ∈ R and (b, c) ∈ R




{(1, 2), (2, 1)} ∈ R ⇒ (a, c) ∈ R.



(iii) Transitive: R is transitive because {1, 2) ∈ R and Hence, R is transitive.



(2, 1) ∈ R

13. R = {(x, y) : (x + y) is divisible by 5 ∀ x, y ∈ R}


⇒ (1, 1) ∈ R and so on. (i) Reflexive: Take x ∈ R.




7. (i) Reflexive: R is reflexive because Since (x + x) is not divisible by 5 for any real number x.




{(1, 1), (3, 3), (5, 5)} ∈ R ⇒ R is not reflexive.



(ii) Symmetric: R is not symmetric because (3, 5) ∈ R but (ii) Symmetric: Let x, y ∈ R be such that (x + y) is divisible by









(5, 3) ∉ R. 5, then (y + x) is also divisible by 5.

(iii) Transitive: R is not transitive because (1, 3) ∈ R and (3, 5) \ (x, y) ∈ R ⇒ (y, x) ∈ R








∈ R does not imply (1, 5) ∈ R. Hence, R is symmetric.


8. A = {1, 3, 7} (iii) Transitive: Take x, y, z ∈ R such that (x + y) and (y + z) are






R = A × A = {(1, 1), (1, 3), (1, 7), (3, 1), (3, 3), (3, 7), (7, 1), divisible by 5, then (x + z) may or may not be divisible by


(7, 3), (7, 7)} 5.
(i) R is reflexive because {(1, 1), (3, 3), (7, 7)} ∈ R \ (a, b) ∈ R and (b, c) ∈ R does not imply






(ii) R is symmetric because (a, c) ∈ R.




{(1, 3), (3, 1) (1, 7), (7, 1), (3, 7), (7, 3)} ∈ R. Hence, R is not transitive.



(iii) R is transitive because (1, 3) ∈ R and (3, 7) ∈ R 14. R in set A = {1, 2, 3} is








⇒ (1, 7) ∈ R and so on. R = {(1, 1), (2, 2), (3, 3), (1, 2), (2, 3)}



9. See solution of Q.No. 6. (i) Reflexive: Since {(1, 1), (2, 2), (3, 3)} ∈ R




10. R = {(x, y) : x ≥ y, x, y ∈ N} ⇒ R is reflexive.




(i) R is reflexive: Since x ≥ x ⇒ (x, x) ∈ R (ii) Symmetric: Since (1, 2) ∈ R but (2, 1) ∉ R and (2, 3) ∈ R






⇒ R is reflexive. but (3, 2) ∉ R.


(ii) R is not symmetric: x ≥ y does not imply y ≥ x. ⇒ R is not symmetric.






\ R is not symmetric. (iii) Transitive: Since (1, 2) ∈ R and (2, 3) ∈ R does not imply


(1, 3) ∈ R.


(iii) Transitive: Take three natural numbers x, y and z such that


x ≥ y and y ≥ z ⇒x≥z Hence R is not transitive.



Relations and Functions 19

E:\AMIT_WORKS\Exam_Guru\EG_Mathematics-12_(working_02-06-2022)\EG_Mathematics-12_working\Open_Files\Chap_1\Chap_1
\ 16-Aug-2022  Amit   Proof-5 Reader’s Sign _______________________ Date __________


15. R = {(L1, L2) : L1 ^ L2 and L1, L2 ∈ L} 4. A = a set of points in a given plane.




(i) Reflexive: Let L1 ∈ L. Since line L1 cannot be perpendicular R = {(p, q) : distance of p and q from origin is same, p, q ∈ A}




to itself (i) Reflexive: Take p ∈ A.



\ (L1, L1) ∉ R Since (p, p) ∈ R \ R is reflexive.






Hence, R is not reflexive. (ii) Symmetric: Take p, q ∈ A such that distances of p and q



from origin is same.

(ii) Symmetric: Let L1, L2 ∈ L. If line L1 ^ L2 then L2 ^ L1
\ (p, q) ∈ R ⇒ (q, p) ∈ R


Hence (L1, L2) ∈ R and (L2, L1) ∈ R





⇒ R is symmetric.

⇒ R is symmetric.



(iii) Transitive: Take p, q, r ∈ A such that p and q are at equal


(iii) Transitive: Let L1, L2, L3 ∈ L and L1 ^ L2 and L2 ^ L3 then



distances from O and q and r are also at equal distances from


L1 is not perpendicular to L3 O.
\ (L1, L2) ∈ R and (L2, L3) ∈ R does not imply that (L1, L3) Then (p, q) ∈ R and (q, r) ∈ R



∈ R. ⇒ (p, r) ∈ R



Hence, R is not transitive. Hence, R is transitive.


IV. 1. Do it yourself. Since R is reflexive, symmetric and transitive.




2. A = Set of all the books in a library Hence R is equivalence relation,




R = {(x, y) : x and y have the same number of pages and x, y 5. T1 is related to T3 because sides of T1 are 3, 4, 5 and sides of T3




∈ A} are 6, 8, 10 and hence are proportional.
(i) Reflexive: Since number of pages of any book x is equal to 6. Similar solution as given in solution of Q.No. 4 (Short answers-II).





number of pages of itself. Set of elements in set A related to the right angled triangle T

with sides 3, 4 and 5 is a set of all possible triangle in set A.
\ (x, x) ∈ R ⇒ R is reflexive.
7. Solution is similar to solution of Q.No. 5 (Short answers-II).




(ii) Symmetric: Take any two books x, y from the library such


Set of all lines related to the line y = 2x + 4 is y = 2x + C, C is


that number of pages of book x is equal to number of pages





any constant.
of book y then the number of pages of book y is equal to
number of pages of book x. 8. A = {1, 2, 3, 4, 5} and


R = {(a, b) : |a – b| is an even number, a, b ∈ A}
\ (x, y) ∈ R ⇒ (y, x) ∈ R

(i) Reflexive: Take a ∈ A since |a – a| = 0 (an even number)




⇒ R is symmetric.


⇒ (a, a) ∈ R ⇒ R is reflexive


(iii) Transitive: Take three books x, y, z such that x and y are of




(ii) Symmetric: Take a, b ∈ A such that |a – b| is an even number,


equal pages and books y and z are of equal pages. Then x


then |b – a| also will be an even number.
and z will also have equal number of pages.
\ (a, b) ∈ R ⇒ (b, a) ∈ R
\ (x, y) ∈ R and (y, z) ∈ R




⇒ R is symmetric.


⇒ (x, z) ∈ R


(iii) Transitive: Take a, b, c ∈ A such that |a – b| and |b – c| are


Hence, R is transitive.


even numbers. Then |a – c| will also be an even number.

3. (A) A = {x ∈ Z : 0 ≤ x ≤ 12} \ (a, b) ∈ R and (b, c) ∈ R





R = {(a, b) : |a – b| is a multiple of 4 and a, b ∈ A} ⇒ (a, c) ∈ R






(i) Reflexive: Take a ∈ A Hence R is transitive.



Since |a – a| = 0 is divisible by 4 Since R is reflexive, symmetric and transitive.


\ (a, a) ∈ R ⇒ R is reflexive. Hence, R is an equivalence relation.





(ii) Symmetric: Take a, b ∈ A such that |a – b| is divisible by 9. Do it yourself.




4 then |b – a| will also be divisible by 4. 10. R = {(x, y) : x, y ∈ N and 2x + y = 24}


\ (a, b) ∈ R ⇒ (b, a) ∈ R R = {(1, 22), (2, 20), (3, 18), (4, 16), (5, 14),


(6, 12), (7, 10), (8, 8), (9, 6), (10, 4), (11, 2)}




Hence R is symmetric.
⇒ Domain of R = {1, 2, 3, ....., 11} and

(iii) Transitive: Take a, b, c ∈ A such that |a – b| and |b – c| are


Range of R = {2, 4, 6, ......, 20, 22}


divisible by 4 then |a – c| will also be divisible by 4.

Relation R is neither reflexive, nor symmetric, nor transitive.
\ (a, b) ∈ R and (b, c) ∈ R

V. 1. {(a, b) R (c, d) if ad(b + c) = bc(a + d), a, b, c, d ∈ N}


⇒ (a, c) ∈ R \ R is transitive.





(i) Reflexive: Take a, b ∈ N




Set of elements related to 1 = {1, 5, 9}


Since ab(b + a) = ba(a + b)

(B) A = {x ∈ Z : 0 ≤ x ≤ 12}

\ (a, b) R (a, b)⇒ R is reflexive.






R = {(a, b) : a = b and a, b ∈ A} (ii) Symmetric: Take a, b, c, d ∈ N such that (a, b) R (c, d)



Prove in the same way as we have proved in Q.No. 3(A). ⇒ ad(b + c) = bc(a + d) ⇒ cb(d + a) = da(c + b)





Set of elements related to 1 = {1} ⇒ (c, d) R (a, b) ⇒R is symmetric





20 Mathe atics–12
m
E:\AMIT_WORKS\Exam_Guru\EG_Mathematics-12_(working_02-06-2022)\EG_Mathematics-12_working\Open_Files\Chap_1\Chap_1
\ 16-Aug-2022  Amit   Proof-5 Reader’s Sign _______________________ Date __________


(iii) Transitive: Take a, b, c, d, e, f ∈ N such that (a, b) R (c, d) 3. Do it yourself.





and (c, d) R (e, f) 4. Domain of R = {1, 2, 3, ...., 20}



⇒ ad(b + c) = bc(a + d) and Range of R = {1, 2, 3, ...., 37, 39}


cf (d + e) = de(c + f)


5. In a set A = {1, 2, 3}

b+c = a+d d +e c+ f



⇒ and = Relation R = {(1, 1), (1, 2), (1, 3), (2, 1), (2, 2), (2, 3), (3, 1),
bc ad de cf




(3, 2), (3, 3)}
1 1 1 1
⇒ + = + and 1 + 1 = 1 + 1 will be an equivalence relation.
c b d a e d f c



1+1+1+ 1 = 1 +1+ 1 +1 ⇒ 7 ordered pairs will be included.
⇒ (on adding)


c b e d d a f c 6. Hint: A = {1, 2, 3}




1 1 1 1 b+e = a+ f R = {(1, 1), (2, 2), (3, 3), (1, 2), (2, 1)}
⇒ + = + ⇒


b e a f be af S = {(1, 1), (2, 2), (3, 3), (2, 3), (3, 2)}



⇒ a f(b + e) = be (a + f) Here R and S each is an equivalence relation on set A, but their


⇒ (a, b) R (e, f)





⇒ R is transitive. union R ∪ S is not an equivalence relation.



2. R = {(a, b) : (a – b) is divisible by a fixed positive integer, a, b 7. R = {(a, b) R (c, d) if and only if ad = bc; a, b, c, d ∈ N}


∈ Z}







(i) Reflexive: Take a, b ∈ N since ab = ba
(i) Reflexive: Let a ∈ Z.



⇒ (a, b) R (a, b)⇒ R is reflexive.


Since (a – a) = 0 is divisible by n.





(ii) Symmetric: Take a, b, c, d ∈ N such that (a, b) R (c, d)

\ (a, a) ∈ R ⇒ R is reflexive.







(ii) Symmetric: Let a, b ∈ Z such that (a – b) is divisible by n. ⇒ ad = bc ⇒ cb = da







If (a – b) is divisible by n then (b – a) is also divisible by n. ⇒ (c, d) R (a, b)⇒ R is symmetric.





\ (a, b) ∈ R ⇒ (b, a) ∈ R (iii) Transitive: Take a, b, c, d, e, f ∈ N such that (a, b) R (c, d)







⇒ R is symmetric. and (c, d) R (e, f)


(iii) Transitive: Let a, b, c ∈ Z such that (a – b) and (b – c) are ⇒ ad = bc and cf = de ⇒ ad = bc and de = cf






divisible by n. ad bc a b
⇒ = ⇒ =
Then (a – c) also will be divisible by n. de cf e f




[... a – b + b – c = a – c]

⇒ af = be ⇒ (a, b) R (e, f)




Hence (a, b) ∈ R and (b, c) ∈ R ⇒ R is transitive.



⇒ (a, c) ∈ R ⇒ R is transitive 8. Do it yourself.






Topic 3. Functions
Let A and B be two non-empty sets. Then, a relation from set Image and Pre-Image of an Element of a Function
A to set B is called a function if each element of set A has a If the element x ∈ A corresponds to y ∈ B under a function f :
correspondence with exactly one element of set B. A → B, y = f (x), then y is called image of x under f and x is called

A function is denoted by the symbols: y = f (x), y = g (x), pre-image of y under f.
y = h(x), etc. Range of a Function
A function y = f (x) from set A to set B is represented as: In a function f : A → B, y = f (x) the subset of set B containing the

f : A → B and it is read as ‘function f such that A maps into B’. images of the elements of set A is called range of the function f.
Note: In a function f : A → B, y = f (x) if expressed in roster

form, then
(i) No element of set A should be left without correspondence


of only one element of set B.
(ii) No first element of any ordered pair should repeat. For example: f : A → B is a function such that A = {1, 2, 3}


and B = {7, 8, 10, 11}. Let f represents the rule under which
Domain of a Function element 1 of set A corresponds to element 7 of set B; element 2
In a function f : A → B, y = f (x) set A is called domain of the of set A corresponds to element 8 of set B and element 3 of set A
function. corresponds to elements 10 of set B.
This function f : A → B can be expressed in roster form as given,
Co-Domain of a Function f = {(1, 7), (2, 8), (3, 10)} and in arrow diagram as shown alongside.
In a function f : A → B, y = f (x) set B is called co-domain of the It is a function because every element of set A has its exactly one
function. image in set B.

Relations and Functions 21



E:\AMIT_WORKS\Exam_Guru\EG_Mathematics-12_(working_02-06-2022)\EG_Mathematics-12_working\Open_Files\Chap_1\Chap_1
\ 16-Aug-2022  Amit   Proof-5 Reader’s Sign _______________________ Date __________


One-One (or Injective) Function Example 3. A function f : N → N, given by f (x) = 5x + 3 is


A function f : X → Y is called one-one (or injective) if the images (a) one-one but not onto (b) onto but not one-one





of distinct elements of set X under f are distinct i.e., for every (c) bijective (d) None of these.





x1, x2 ∈ X; f (x1) = f (x2) implies x1 = x2. Otherwise f is called Solution. The given function is f : N → N and f (x) = 5x + 3.


many-one function. (i) One-one: Take any two natural numbers x1 and x2 such



that
f (x1) = f (x2) ⇒ 5x1 + 3 = 5x2 + 3






⇒ 5x1 = 5x2 ⇒ x1 = x2




⇒ given function is one-one.



The function f in figure given alongside is one-one. (ii) Onto: The given function is f : N → N and f (x) = 5x + 3.



Onto or (Surjective) Function  y − 3
Taking it as: y = 5x + 3 ⇒ x=
5 




A function f : X → Y is said to be onto (or surjective), if every Now, for every natural number y, there does not exist a natural


element of set Y is the image of some element of set X under f, i.e.,
number x or for any natural number y, y − 3
for every y ∈ Y, there exists an element x ∈ X such that f (x) = y. 5


is not a natural number. For example, for y = 1, there is no


natural number x. Hence, given function is not onto.
Hence, option (a) is the correct answer.


Example 4. A function f : Z → Z given by f (x) = 5x + 3 is
(a) one-one but not onto. (b) bijective
Here, function f1 and f2 are onto functions because every element



(c) onto but not one-one (d) None of these
of Y has a pre-image in X.




Solution. The given function: f : Z → Z given by f (x) = 5x + 3.
Note: A function f : X → Y is onto if and only if Range of f = Y.
(i) One-one: Take two integers x1 and x2,


One-One and Onto or (Bijective) Function such that f (x1) = f (x2) ⇒ 5x1 + 3 = 5x2 + 3



A function f : X → Y is said to be one-one and onto (or bijective) ⇒ 5x1 = 5x2 ⇒ x1 = x2




if f is both one-one and onto. ⇒ given function is one-one.


y −3
(ii) Onto: Take y = 5x + 3 ⇒ x= 5




For every integer y, there does not exist an integer x.

 y−3 
 is not an integer for all y 
5 

Thus, f (x) is not onto.
Function f in figure given alongside is one-one and onto both.

Hence, option (a) is the correct answer.
Example 1. Take a relation from set A = {1, 3, 5, 7} to set B =

Example 5. A function f : R → R is given by: f (x) = 5x + 3. The  
{1, 2, 3, 4, 5} given in roster form as:
function f (x) is

R = {(1, 1), (3, 4), (7, 5)}. It is
(a) one-one but not onto (b) bijective

(a) a function (b) an injective function




(c) onto but not one-one (d) None of these.




(c) not a function (d) a bijective function




Solution. Given function f : R → R is f (x) = 5x + 3.




Solution. It is not a function because element 5 of set A does not
have any image in set B. (i) One-One: Take two real numbers x1 and x2 such that f (x1)



Hence, option (c) is the correct answer. = f (x2)

Example 2. Take a relation from set ⇒ 5x1 + 3 = 5x2 + 3 ⇒ 5x1 = 5x2 ⇒ x1 = x2


  
   

A to set B shown on R.H.S. in arrow ⇒ f (x) is one-one function.



diagram. Which of the following is y −3
(ii) Onto: Take y = 5x + 3 ⇒ x= .
true about it? 5




(a) It is a function. Here for every real number y, there exists a real number x.



(b) It is an injective function.  y−3 
 is a real number for all y 


(c) It is a bijective function. 5 

\ f (x) is an onto function.


(d) It is not a function.





Solution. It is also not a function because element 3 of set A has Since given function f (x) is one-one and onto


two images 7 and 8 in set B. both. \ f (x) is a bijective function.
       


Hence, option (d) is the correct answer. Hence, option (b) is the correct answer.

22 Mathe atics–12
m
E:\AMIT_WORKS\Exam_Guru\EG_Mathematics-12_(working_02-06-2022)\EG_Mathematics-12_working\Open_Files\Chap_1\Chap_1
\ 16-Aug-2022  Amit   Proof-5 Reader’s Sign _______________________ Date __________


Example 6. A function f : N → N defined as f (x) = 5x2 + 3. The Horizontal Line Test for One-One Function
function f (x) is If any horizontal line cuts the graph of a relation in one point only,
(a) bijective (b) one-one but not onto. it is one-one. In the given graph of y = sin x, any line drawn parallel




(c) onto but not one-one (d) None of these to x-axis will cut the graph at many points, therefore the function




Solution. Given function f : N → N is f (x) = 5x2 + 3. y = sin x is not one-one.
(i) One-One: For any two natural numbers x1 and x2, Notes:


let f (x1) = f (x2) 1. The number of functions from a finite set A into finite set



B = [n(B)]n(A).

⇒ 5x12 + 3 = 5x22 + 3 ⇒ 5x12 = 5x22 ⇒ x12 = x22
Example: (i) The number of functions from {1, 2, 3}


  

  

⇒ x1 = x2 (for natural numbers)⇒ f (x) is one-one function




into {6, 7} = 23 = 8.




y −3 (ii) Number of functions from {a, b} into
(ii) Onto: Take y = 5x2 + 3 ⇒ x2 =



5 {2, 3, 4} = (3)2 = 9.




y −3 2. The number of one-one functions that can be defined from
⇒ x=
5



a finite set A into a finite set B = n(B)Pn(A) if n(B) ≥ n(A)



For every natural number y, there does not exist a natural otherwise 0.

y −3 Example: (i) The number of one-one functions from {a,
number x =




5 b, c} into {1, 2} = 0. n(A) > n(B).
\ f (x) is not onto.


(ii) The number of one-one functions from {1,




Hence, option (b) is the correct answer. 2} into {a, b, c} = 3P2 = 6.
(iii) The number of one-one functions from {1,
Vertical Line Test for a Function

2, 3} into {5, 6, 7} = 3P3 = 3! = 6.
A relation is a function if each vertical line meets its graph in only
3. The number of onto functions that can be defined from
one point and relation is not a function if any vertical line cuts


the graph in more than one point. a finite set A containing n elements onto a finite set B
containing 2 elements = 2n – 2.
Example: The number of onto functions from {1, 2, 3}

onto {p, q} = 23 – 2 = 8 – 2 = 6.
4. The number of onto functions that can be defined from


finite set A onto a finite set B is equal to number of ways
of dividing n(A) things into n(B) groups so that no group
is empty if n(A) ≥ n(B) and is 0 otherwise.
Take a function: y = sin x. Its graph is on RHS. You draw any Example: The number of onto functions from {1, 2, 3}

vertical line. It will cut the graph only at one point, hence y = sin onto {p, q} = 3! + 3! = 3 + 3 = 6
x is a function. 1! 2 ! 2 ! 1!

DOMAIN OF FOLLOWING FUNCTIONS

1. (a) The domain of a 2 − x 2 is [–a, a] or –a ≤ x ≤ a.





1
(b) The domain of is (–a, a) or –a < x < a.
a − x2
2


2. (a) The domain of x 2 − a 2 is (–∞, –a] ∪ [a, ∞) or x ≥ a or x ≤ –a.



1
(b) The domain of is (–∞, –a) ∪ (a, ∞) or x > a or x < –a.
x − a2
2


3. (a) The domain of ( x − a ) (b − x ) when a < b is [a, b] or a ≤ x ≤ b.



1
(b) The domain of when a < b is (a, b) or a < x < b.
( x − a ) (b − x )


4. (a) The domain of ( x − a ) ( x − b) when a < b is (–∞, a] ∪ [b, ∞) or x ≥ b or x ≤ a.



1
(b) The domain of when a < b is (–∞, a) ∪ (b, ∞) or x > b or x < a.
( x − a ) ( x − b)


Relations and Functions 23

E:\AMIT_WORKS\Exam_Guru\EG_Mathematics-12_(working_02-06-2022)\EG_Mathematics-12_working\Open_Files\Chap_1\Chap_1
\ 16-Aug-2022  Amit   Proof-5 Reader’s Sign _______________________ Date __________


5. (a) The domain of x − a when a < b is (–∞, a] ∪ (b, ∞) or x > b or x ≤ a.
x−b



(b) The domain of x − a when a > b is (–∞, b) ∪ [a, ∞) or x ≥ a or x < b.
x−b


x−a
(c) The domain of when a < b is [a, b) or a ≤ x < b.
b−x


x−a
(d) The domain of when a > b is (b, a] or b < x ≤ a.
b−x


6. (a) The domain of log (a2 – x2) is (–a, a).



(b) The domain of log (x2 – a2) is (–∞, a) ∪ (a, ∞).


7. (a) The domain of log (x – a) (b – x) when a < b is (a, b).



(b) The domain of log (x – a) (x –b) when a < b is (–∞, a) ∪ (b, ∞).

{ }


8. (a) The domain of tan mx when m ∈ N is R − ( 2n + 1) π , m ∈ Z
2m




(b) The domain of cot nx when m ∈ N is R −

m
: n ∈Z . { }


9. (a) Range of f (x) = x 2 − a 2 is [0, a].




(b) Range of f(x) = a cos x + b sin x + c is c − a 2 + b 2 , c + a 2 + b 2  .
 


10. If f (x) is a function, then



(a) f (x) + f (–x) is an even function. (b) f (x) – f (–x) is an odd function.








x+c
11. The domain of the function f ( x ) = is R – [c] and range = {–1, 1}.
x+c


Thus
Example 7. The domain of x − 1 + 8 − x is
( ) { }

(a) [1, 8] (b) (–8, 8) (c) [1, 8) (d) (1, 8) Range of f = −∞, − 1  ∪  − 1 , ∞ = R − − 1 , − 1
4   20 4 20









Solution. Clearly, x – 1 ≥ 0 and 8 – x ≥ 0 i.e. x ≥ 1 and 8 ≥ x Hence, option (b) is the correct answer.

i.e. x ≥ 1 and x ≤ 8 i.e. 1 ≤ x ≤ 8 ⇒ x ∈ [1, 8] Example 9. Let D be the domain of a real valued function defined






Hence, option (a) is the correct answer.
as: f ( x ) = 25 − x 2. The domain D will be
x+2 (a) [–2, 2] (b) [–3, 3] (c) [–4, 4] (d) [–5, 5]
Example 8. The range of function y = is
x 2 − 8x − 4








(
2

{ }
2
1 Solution. f (x) = 25 − x ⇒ 25 – x ≥ 0
−∞, −  1 1



(a) (b) R − − , − ⇒ 25 ≥ x 2
⇒ –5 ≤ x ≤ 5
4  4 20







(c)  − 1 , ∞ )
(d) None of these.
⇒ Domain = [–5, 5].

 20 Hence, option (d) is the correct answer.





2
Solution. For y to be defined x – 8x – 4 ≠ 0, i.e. x ≠ 4 ± 2 5 Example 10. Let D be the domain of a real valued function defined
1
\ Domain of y = R – {4 + 2 5 , 4 – 2 5 } as: f ( x ) = , find D.
2
4x − 25


x+2 1
Now y = 2 ⇒ x2y – (8y + 1)x – (4y + 2) = 0 Solution. f (x) = ⇒ 4x2 – 25 > 0
x − 8x − 4



2


4x − 25
For x to be real 25 5 5
2
⇒x > ⇒ x> or x < −

(8y + 1)2 + 4y(4y + 2) ≥ 0 ⇒ 80y2 + 24y + 1 ≥ 0 4 2 2
{ }



5 5



( ) ( ) ⇒ Domain = x >
3 1 3
2
1
2 or x < − , x ∈ R
2
⇒ y + y+ ≥0 ⇒ y+ − ≥0 2 2

10 80 20 10



Example 11. Let D be the domain of a real valued function
3 1
⇒ y + 3 ≤ − 1 or y + ≥ [ x2 ≥ a2 ⇒ x ≥ a defined as f ( x ) = x2 − x − 6 .
20 10 20 10


Q
2
or x ≤ –a] Solution. f (x) = x − x − 6 ⇒ x2 – x – 6 ≥ 0




5 1 ⇒ (x –3)(x + 2) ≥ 0 ⇒ x ≥ 3 or x ≤ –2
⇒y≤ − or y ≥ − ⇒ y ≤ − 1 or y ≥ − 1





20 20 4 20 ⇒ Domain ={x ≥ 3 or x ≤ –2, x ∈ R}






24 Mathe atics–12
m
E:\AMIT_WORKS\Exam_Guru\EG_Mathematics-12_(working_02-06-2022)\EG_Mathematics-12_working\Open_Files\Chap_1\Chap_1
\ 16-Aug-2022  Amit   Proof-5 Reader’s Sign _______________________ Date __________


EXERCISE 1.3
I. Multiple Choice Questions (MCQs) 6. A function f : R → R is defined as f (x) = 2x2 + 3. Show






Choose the correct answer from the given options. that f (x) is neither one-one nor onto.


7. A function f : N → N is defined as f (x) = 2x3 + 3. Show

1. A function f : N → N defined as y = x2 is




that f (x) is one-one but not onto.


(a) one-one (b) onto


8. A function f : R → R is defined as f (x) = 2x3 + 3. Show




(c) bijective (d) None of these.




that f (x) is one-one and onto.




2. Domain of the function: f : R → R defined as y = x + 4 is


9. Write Domain and Range of modulus function.


(a) Real numbers (b) Natural numbers



10. Write Domain and Range of signum function.




(c) Negative real numbers (d) None of these.



11. Write Domain and Range of the greatest integer function




3. Domain of a real valued function f ( x ) = 36 − x 2 is:



f : R → R and f (x) = [x].



(a) [–6, 6] (b) (–6, 6) (c) (–6, 6] (d) [–6, 6) III. Short Answer Type Questions-II








4. Number of function from set {1, 2} into set {a, b, c} are 1. Let A be the set of all 50 students of Class X in a school.




(a) (2)3 (b) (3)2 (c) (2)2 (d) (3)3 Let f : A → N be a function defined by f (x) = roll numbers










5. Number of one-one functions that can be defined from a of the student x. Show that f is one-one, but not onto.


set {3, 4, 5} into a set {1, 2} are [NCERT]


(a) 2P3 (b) 0 (c) 2P2 (d) 3P3 2. Show that the function f : N → N given by f (x) = 2x, is












6. The function f : R → [–1, 1] defined by f (x) = cos x is one-one but not onto. [NCERT]




(a) both one-one and onto 3. Prove that the function f : R → R given by f (x) = 2x, is






(b) not one-one, but onto one-one and onto. [NCERT]




(c) one-one, but not onto 4. Show that the function f : N → N given by f (1) =


f (2) = 1 and f (x) = x – 1 for every x > 2 is onto but not


(d) neither one-one, nor onto


one-one. [NCERT]


7. Every function is a

 x + 1 if x is odd


(a) relation. (b) onto function 5. Show that f : N → N given by f ( x ) = 




 x − 1 if x is even



(c) injective function (d) bijective function is both one-one and onto.




8. A function which is injective and surjective is called [NCERT] [HOTS] [A.I. 2011]


(a) onto function only (b) bijective function 6. Show that the function defined by f : R+ → R+ and given by






(c) one-one function (d) None of these. 1
f ( x ) = is one-one and onto, where R+ is the set of all non-
x




9. An onto function from set A to set B means every element zero positive real numbers. Is the result true, if the domain


of set B has a pre-image in R+ is replaced by N with co-domain being same as R+?
(a) Set B × A (b) Set A
7. Prove that the greatest integer function f : R → R given by




(c) Set A × B (d) None of these.


f (x) = [x], is neither one-one nor onto, where [x] denotes





10. A function f : R → R is defined as f(x) = x3 + 1. Then the the greatest integer less than or equal to x.


function has
[NCERT] [HOTS]
(a) no minimum value

8. Show that the signum function f : R → R given by


(b) no maximum value


 1 if x > 0


(c) both maximum and minimum value 
f ( x ) =  0 if x = 0 is neither one-one nor onto.


(d) neither maximum value nor minimum value  − 1 if x < 0 [HOTS]




II. Short Answer Type Questions-I
9. Let A = {3, 7, 9}, B ={2, 4, 10} and let f = {(3, 2), (7, 4),
1. A function f : N → N is given by f (x) = 2x + 3. Show that


(9, 10)} be a function from A to B. Show that f is one-one.



f (x) is one-one but not onto.
10. In a function given below, state whether the function is

2. A function f : Z → Z is given by f (x) = 2x + 3. Show that


one-one, onto or bijective. Justify your answer. f : R → R



f (x) is one-one but not onto. defined by f (x) = 1 + x2. [NCERT] [HOTS]


3. A function f : Q → Q is defined as f (x) = 2x + 3. Show that 11. Let A and B be two sets. Show that f : A × B → B × A such




f (x) is one-one and onto.

that f (a, b) = (b, a) is bijective function. [NCERT]


4. A function f : R → R is defined as f (x) = 2x + 3. Show that 12. Let A = R – {3} and B = R – {1}. Consider the

( )


f (x) is one-one and onto.


function f : A → B defined by f ( x ) = x − 2 . Is

5. A function f : N → N is defined as f (x)= 2x2 + 3. Show that x−3



f (x) is one-one but not onto. f one-one and onto? Justify your answer. [NCERT]



Relations and Functions 25

E:\AMIT_WORKS\Exam_Guru\EG_Mathematics-12_(working_02-06-2022)\EG_Mathematics-12_working\Open_Files\Chap_1\Chap_1
\ 16-Aug-2022  Amit   Proof-5 Reader’s Sign _______________________ Date __________


13. Let A = {– 1, 0, 1, 2}, B = {– 4, – 2, 0, 2} and f, g : 19. Show that the function f : W → W defined by




A → B be functions defined by f (x) = x2 – x, x ∈ A and  x + 1 when x is odd
1
f ( x) =  is bijective function.
g(x) = 2 x − − 1, x ∈ A. Are f and g equal? Justify your  x − 1 when x is even
2
[HOTS] [A.I. 2011 (C), 2012]
answer. 
[HOTS]
IV. Long Answer Type Questions
[Hint: One may note that two functions f : A → B and g :
1. Show that the modulus function f : R → R given by f (x) =

 
A → B such that f (a) = g (a) ∀ a ∈ A, are called equal




functions]. |x| is neither one-one nor onto, where |x| is x if x is positive
14. Find the Domain and Range of each of the following or 0 and |x| is (– x), if x is negative. [HOTS]




functions. 2. Let the function f : R → [–1, 1] defined by f (x) = cos x, ∀




{( }
x ∈ R. Show that f (x) is neither one-one nor onto.[HOTS]
)


(i) f ( x ) =


x, 4 − x 2 : x ∈ R . [HOTS] 3. L e t f : R → R b e t h e f u n c t i o n d e f i n e d a s





1
 2   f ( x) = , ∀ x ∈ R . Find the range of f (x).
(ii) f ( x ) =  x, x − 9  : x ∈ R and x ≠ 3


2 − cos x
 x − 3  [HOTS]


 
{ }


(iii) f ( x ) = ( x, x − x ) : x ∈ R [HOTS] x
4. If f : [0, ∞) → [0, ∞) and f ( x ) = , then prove that



1+ x



f (x) is one-one but not onto.
(iv) f ( x ) =  x, 2 2  : x ∈ R 


 4 − x   5. Let f : R → R be a function defined by f (x) = 2x3 + 5, then





show that f(x) is bijective. [HOTS]
 x + 1 if x is odd


 6. Find the domain of the function:
15. Let f : N → N be defined as f ( x ) =  2 ,

x 1


if x is even (i) f ( x ) = [HOTS]
2 x− x



x ∈ N. State whether function f is bijective. 1
(ii) f ( x ) = [HOTS]

16. Let f : X → Y be a function. Determine a relation R in x+ x





X given by R = {(a, b) : f (a) = f (b)}. Show that R is an
7. Find the domain:


equivalence relation.


(i) f (x) = cos–1(3x – 1) [HOTS]

17. Show that the function f : R → R given by f (x) = ax + b,




−1


where a, b ∈ R, a ≠ 0 is bijective. (ii) f ( x ) = sin x [HOTS]
If f : R → R be the function defined by f (x) = 4x3 + 7. Show x



18.

(iii) f(x) = sin–1 (log2 x) [HOTS]


that f (x) is bijective. [Delhi 2011 (C)]






Answers 1.3
I. 1. (d)
None of these y − 3
(ii) Onto: Take y = 2x + 3 ⇒ x=




2. (a)
Real numbers  2 







3. [–6, 6] 4. (b) (3)2
(a) 5. (b) 0  n(A) > n(B)  y − 3
For every natural number y, is not a natural number.
 2 








6. (b)
not one-one, but onto \ f (x) is not onto.






f (x) =cos x, R → [–1, 1] 2. Similar to solution of Q.No. 1.







One-one: Take x1, x2 ∈ R such that f (x1) = f (x2) 3. Hint: (i) One-one: Take x1, x2 ∈ Q such that






⇒ cos x1 = cos x2 ⇒ x1 = ±x2 f (x1) = f (x2)








Hence, f (x) is not one-one ⇒ 2x1 + 3 = 2x2 + 3 ⇒ x1 = x2





Onto: f (x) =cos x ⇒ f (x) is one-one.






Take y = cos x ⇒ x = cos–1 y y − 3
(ii) Onto: Take y = 2x + 3 ⇒ x=





For every value of y ∈ [–1, 1], cos–1 y exists  2 



\ f (x) is onto. y −3
Now, for every rational number y, is a rational number.

2



7. (a) relation. \ f (x) is onto.





8. (b) bijective function


4. Try yourself as Q.No. 3.



9. (b) Set A



5. Hint: (i) One-one: Take x1, x2 ∈ N such that



10. (b) neither maximum value nor minimum value






II. 1. Hint: (i) One-one: Take x1, x2 ∈ N such that: f (x1) = f (x2)






f (x1) = f (x2) ⇒ 2x12 + 3 = 2x22 + 3 ⇒ x12 = x22






⇒ 2x1 + 3 = 2x2 + 3 ⇒ x1 = x2 ⇒ x1 = x2 [ x1 and x2 are natural numbers]






⇒ f (x) is one-one. ⇒ f (x) is one-one.







26 Mathe atics–12
m
E:\AMIT_WORKS\Exam_Guru\EG_Mathematics-12_(working_02-06-2022)\EG_Mathematics-12_working\Open_Files\Chap_1\Chap_1
\ 16-Aug-2022  Amit   Proof-5 Reader’s Sign _______________________ Date __________


7. f : R → R and f (x) = [x]
y −3



(ii) Onto: Take y = 2x2 + 3 ⇒ x= Hint: One-one: Take x1 and x2 ∈ R such that
2



f (x1) = f (x2) ⇒ [x1] = [x2]
y − 3 is not a natural number.



For every natural number y,


  

2 ⇒ x1 ≠ x2 ⇒ f is not one-one.





\ f (x) is not onto. Onto: y = [x], here for every real number y, there does not exist



a real number x.

6. Hint: (i) One-one: Take x1, x2 ∈ R such that
⇒ f is not onto.


f (x1) = f (x2)






 1 if x > 0
⇒ 2x12 + 3 = 2x22 + 3 ⇒ x12 = x22 
8. Hint: f : R → R and f (x) =  0 if x = 0




⇒ x1 = ± x2 ⇒ f (x) is not one-one function.




 −1 if x < 0





(ii) Onto: Take y = 2x2 + 3 ⇒ x= y −3
2 One-One: Take x1, x2 ∈ R such that f (x1) = f (x2)







y − 3 is not a real number. ⇒ x1 ≠ x2 because f (2) = f (3) but 2 ≠ 3.
For every real number y,


2 ⇒ f (x) is not one-one.


\ f (x) is not onto. Onto: Take y = f (x). Here, we have value of x, only for three





7. Try as we have solved Q.No. 6. values of y i.e., y = – 1, 0, 1. If y = 4. There is no value of



x ⇒ f is not onto.


8. Try as we have solved Q.No. 6.


9. (i) Domain of modulus function = R. 9. True. Try yourself.






(ii) Range of modulus function = non-negative real numbers. 10. Neither one-one nor onto.





 −1; x < 0 Hint: f : R → R and f (x) = 1 + x2.



( )  One-One: Take x1, x2 ∈ R such that f (x1) = f (x2)
Signum function f x =  0; x = 0

 1 x>0 ⇒ x12 = x22 ⇒ x1 = ± x2





10. (i) Domain of signum function = real numbers. ⇒ f is not one-one.




(ii) Range of signum function = {–1, 0, 1}.
Onto: y = 1 + x2 ⇒ x= y −1.


11. (i) Domain of greatest integer function = real numbers.







(ii) Range of greatest integer function = Integers. For every real number y , y −1 is not a real number ⇒ f is not



III. 1. Hint: One-One: Every student has a different roll number onto.



from 1 to 50. No two or more students in the class can have 11. Hint: A and B are two sets: f : A × B → B × A such that f (a, b)




same roll number. Therefore, f is one-one. = (b, a) is bijective.

Onto: Roll numbers of the students are from 1 to 50. There One-one: Take two ordered pairs (a1, b1) and (a2, b2) such that


is no student whose roll number is 51, 52 or 53. So, every

a1, b1 ∈ A and a2, b2 ∈ B. Now assume that
number in the codomain of natural numbers is not a roll
f (a1, b1) = f (a2, b2)
number. Therefore, f is not onto.



2. Try yourself. 3. Try yourself. ⇒ (b1, a1) = (b2, a2) ⇒ b1 = b2 and a1 = a2




⇒ f is one-one.




4. Hint: f : N → N.



One-one: f (1) = f (2) = 1 Onto: For every element (b, a) from co-domain, there exists a

corresponding element (a, b) in domain of f. f is onto.


⇒ f is not one-one.

12. Hint: One-one: Take x1, x2 ∈ R such that f (x1) = f (x2).


Onto: When y = 1, then x = 1 or 2. When x > 2,





then x = y + 1. Hence, for every natural number x − 2 x2 − 2
⇒ 1 = ⇒ x1 = x2
y, (y + 1) is a natural number f is onto. x1 − 3 x2 − 3





 x + 1 if x is odd ⇒ f(x) is one-one.
5. Hint: f : N → N and f ( x ) = 


 x − 1 if x is even x−2 3y − 2


Onto: Take y = ⇒ x= ,y≠1
One-One: Let x1 and x2 be any two odd numbers, and x−3 y −1






f (x1) = f (x2) Here for every real number y ≠ 1, there exists a real number



⇒ x1 + 1 = x2 + 1 ⇒ x1 = x2. ⇒ f is one-one. 3y − 2





Now, take x1 and x2 any two even numbers, and f (x1) = f (x2) y −1




⇒ x1 – 1 = x2 – 1 ⇒ x1 = x2 ⇒ f is onto.


  

13. Yes.
⇒ f is one-one.


Hint: Show that:f (–1) = g(–1) = 2;

Onto: Let y = x + 1, here x is odd ⇒ x = y – 1. For every even



f (0) = g(0) = 0

number y, y – 1 is an odd number. In the same way, prove


when x is even. f (1) = g(1) = 0; f (2) = g(2) = 2



\ f is onto. 14. (i) Domain = [– 2, 2] and Range = [–2, 2]





6. Try yourself. Result is true when R+ is replaced by N. 4 – x2 ≥ 0 ⇒ –2 ≤ x ≤ 2





Relations and Functions 27

E:\AMIT_WORKS\Exam_Guru\EG_Mathematics-12_(working_02-06-2022)\EG_Mathematics-12_working\Open_Files\Chap_1\Chap_1
\ 16-Aug-2022  Amit   Proof-5 Reader’s Sign _______________________ Date __________


(ii) Domain = R – {3}, Range = R – [6]. 2. Try yourself.




(iii) Domain = Real numbers, Range = (– ∞, 0) 1
3. Hint: y =
{ }


1 2 − cos x



(iv) Domain = R – {– 2, 2}, Range = y≥ ; y ∈R 2y − 1
2 ⇒ cos x = = 2− 1


y y



15.
f is onto and not one-one and hence not bijective.
Since –1 ≤ cos x ≤ 1


16.
Hint: Reflexive: Take any element a ∈ X. Since f (a) = f (a)






(a, a) ∈ R
⇒  
⇒ −1 ≤  2 − 1  ≤ 1


R is reflexive.
⇒  y





 
Symmetric: Let a and b be two real numbers. Let (a, b) ∈ R ⇒ −3 ≤  − 1  ≤ −1
 y



⇒ f (a) = f (b) ⇒ f (b) = f (a)




⇒ 3≥ 1 ≥1




⇒ (b, a) ∈ R y





⇒ R is symmetric. 1 ≤ y ≤1



Transitive: Take a, b, c ∈ X such that (a, b) ∈ R, (b, c) ∈ R 3





1
⇒ f (a) = f (b) and f (b) = f (c) ⇒ Range of f(x) =  , 1




 3 




⇒ f (a) = f (c)


4. Try yourself


⇒ (a, c) ∈ R ⇒ R is transitive.



5. Try yourself.


IV. 1. f :R → R and f (x) = |x|.



6. (i) Domain of f (x) =



One-One: Take x1, x2 ∈ R and f (x1) = f (x2)





f


(ii) Domain of f (x) = (0, ∞)


⇒ | x1 | = | x2 |










⇒ x1 ≠ x2 ⇒ f is not one-one.  2
7. (i) D of f (x) = 0, 


 3








Onto: Take y = |x| for every real number y, there does not


exist a real number x. For example, when y = – 1, there is no (ii) D of f (x) = [–1, 1] – {0}




corresponding number x.
1 
⇒ f is not onto. (iii) D of f (x) =  , 2

2 




Case Based Questions
1. There are two small bookshelves, shelf A and shelf B. Both Two books of a shelf are associated to each other by a



the shelves have four books each. Shelf A has different relation. The difference in their number of pages is at
books for science students whereas the shelf B has different most 10.
books for non-science students. On the basis of above information, answer the following:

(i) As per the given definition, the relation on shelf A is:


(a) {(Maths, Physics), (Chemistry, Biology)}


(b) {(Maths, Physics), (Chemistry, Biology), (Physics,



Biology)}
(c) {(Maths, Maths), (Chemistry, Physics), (Biology,


Maths)}
(d) None of above


(ii) With reference to Q1, the relation on shelf A is:


(a) reflexive only (b) symmetric only




(c) transitive only (d) None of these
Number of pages of each book of both the shelves is given




(iii) As per the definition given, the relation on shelf B is:

in the table below:


(a) {(Economics, Accountancy), (Economics, Geography),


Shelf A Shelf B (Accountancy, History)}
Book No. of Book No. of (b) {(Economics, Accountancy), (Economics, History),


Pages Pages (Accountancy, History)}
Maths 132 Economics 145 (c) {(History, Geography), (Accountancy, Geography)}


(d) None of above
Physics 140 Accountancy 149


(iv) With reference to Q3, the relation on the shelf B is:
Chemistry 160 History 154


(a) reflexive only (b) symmetric only




Biology 165 Geography 170 (c) transitive only (d) None of above




28 Mathe atics–12
m
E:\AMIT_WORKS\Exam_Guru\EG_Mathematics-12_(working_02-06-2022)\EG_Mathematics-12_working\Open_Files\Chap_1\Chap_1
\ 16-Aug-2022  Amit   Proof-5 Reader’s Sign _______________________ Date __________


(v) Let {(Maths, Maths), (Physics, Physics), (Chemistry, (v) From the given sets n(A × A), n(B × B) and n(A × B) are




Chemistry), (Biology, Biology)} be a relation defined in respectively:
a different manner on shelf A. Then the relation is: (a) 25, 16 and 20 (b) 20, 10 and 15





(a) reflexive only (b) identity only (c) 22, 14 and 18 (d) 24, 15 and 19








(c) reflexive and identity both (d) None of above Ans. (i) (c) 220 (ii) (d) an equivalence relation









Ans. (i) (a) {(Maths, Physics), (Chemistry, Biology)} (iii) (c) 210 (31) (iv) (d) not a function








(ii) (d) None of these (v) (a) 25, 16, 20






(iii) (b) {(Economics, Accountancy), (Economics, 3. A general election of Lok Sabha is a gigantic exercise.





History), (Accountancy, History)} About 911 million people were eligible to vote and voter
(iv) (c) transitive only. turnout was about 67%, the highest ever.



(v) (c) reflexive and identity both



2. There are two different sections of Class 12: Section A and ONE-NATION


Section B including girls as well as boys. A college girl ONE-ELECTION
Teena forms two sets with these students as her college FESTIVAL OF
project. Let A = {a1, a2, a3, a4, a5} and B = {b1, b2, b3,
DEMOCRACY
b4}, where ai’s and bi’s are the students of Section A and
Section B respectively. GENERAL ELECTION —2019

Let I be the set of all citizens of India who were eligible to


exercise their voting right in general election held in 2019.
A relation ‘R’ is defined on I as follows:

R = {(V1, V2) : V1, V2 ∈ I and both use their voting right
in general election – 2019}
(i) Two neighbours X and Y ∈ I. X exercised his voting right


while Y did not cast her vote in general election – 2019.
Which of the following is true?
(a) (X, Y) ∈ R (b) (Y, X) ∈ R

Section A Section B





Teena decides to explore these sets for various types of (c) (X, X) ∉ R (d) (X, Y) ∉ R





relations and functions using the information given, answer (ii) Mr. ‘X’ and his wife ‘W’ both exercised their voting right


the following: in general election – 2019. Which of the following is true?
(i) Teena wishes to know the number of reflexive relations (a) Both (X, W) and (W, X) ∈ R




defined on set A. How many such relations are possible? (b) (X,W) ∈ R but (W, X) ∉ R


(a) 0 (b) 25 (c) Both (X, W) and (W, X) ∉ R






(c) 220 (d) None of above (d) (W, X) ∈ R but (X, W) ∉ R







(ii) Let R : A → A, R = {(x, y) : x and y are students of same (iii) Three friends F1, F2 and F3 exercised their voting right


sex}. Then relation R is:


in general election – 2019, then which of the following is
(a) reflexive only true?


(b) reflexive and symmetric but not transitive (a) (F1, F2) ∈ R, (F2, F3) ∈ R and (F1, F3) ∈ R




(c) reflexive and transitive but not symmetric (b) (F1,F2 ) ∈ R, (F2, F3) ∈ R and (F1, F3) ∉ R




(d) an equivalence relation (c) (F1, F2 ) ∈ R, (F2, F2) ∈ R but (F3, F3) ∉ R




(iii) Teena and her friend Reena interested to know the number (d) (F1, F2) ∉ R, (F2, F3) ∉ R and (F1, F3) ∉ R


of symmetric relations defined on both the sets A and B,


(iv) The above defined relation R is
separately. She decides to find the symmetric relation on


set A, while Reena decides to find the symmetric relation (a) symmetric and transitive but not reflexive


on set B. What is difference between their result? (b) universal relation


(a) 1024 (b) 210 (15) (c) equivalence relation






10
(c) 2 (31) (d) None of these (d) reflexive but not symmetric and transitive






(iv) Let R : A → B, R = {(a1, b1), (a1, b2), (a2, b1), (a3, b3), (a4, (v) Mr. Shyam exercised his voting right in General Election –




b2), (a5, b2)}, then R is: 2019, then Mr. Shyam is related to which of the following?
(a) neither one-one nor onto (a) All those eligible voters who cast their votes




(b) one-one but, not onto (b) Family members of Mr. Shyam




(c) only onto, but not one-one (c) All citizens of India




(d) not a function (d) Eligible voters of India




Relations and Functions 29

E:\AMIT_WORKS\Exam_Guru\EG_Mathematics-12_(working_02-06-2022)\EG_Mathematics-12_working\Open_Files\Chap_1\Chap_1
\ 16-Aug-2022  Amit   Proof-5 Reader’s Sign _______________________ Date __________


Ans. (i) (d) (X, Y) ∉ R 5. An organization conducted bike race under two different





(ii) (a) Both (X, W) and (W, X) ∈ R categories–boys and girls. Totally there were 250



(iii) (a) (F1, F2) ∈ R, (F2, F3) ∈ R and (F1, F3) ∈ R participants. Among all of them finally three from Category



(iv) (c) equivalence relation 1 and two from Category 2 were selected for the final race.



(v) (a) All those eligible voters who cast their votes. Ravi forms two sets B and G with these participants for
the final race.



4. Sherlin and Danju are playing Ludo at home during


Covid-19. While rolling the dice, Sherlin’s sister Raji Let B = {b1, b2, b3} and G = {g1, g2} where B represents


observed and noted the possible outcomes of the throw the set of boys selected and G the set of girls who were
every time belongs to set {1, 2, 3, 4, 5, 6}. Let A be the selected for the final race.
set of players while B be the set of all possible outcomes.
Then, A = {S, D} and B = {1, 2, 3, 4, 5, 6}

Ravi decides to explore these sets for various types of



relations and functions.
(i) Ravi wishes to form all the relations possible from B to G.


How many such relations are possible?
(a) 26 (b) 25 (c) 0 (d) 23









(i) Let R : B → B be defined by R = {(x, y): y is divisible by (ii) Let R : B → B be defined by R = {(x, y): x and y are students




x} is of same sex}, Then this relation R is
(a) reflexive and transitive but not symmetric (a) equivalence




(b) reflexive and symmetric and not transitive (b) reflexive only




(c) not reflexive but symmetric and transitive (c) reflexive and symmetric but not transitive




(d) equivalence (d) reflexive and transitive but not symmetric




(ii) Raji wants to know the number of functions from A to B. (iii) Ravi wants to know among those relations, how many




How many functions are possible? functions can be formed from B to G?
(a) 62 (b) 26 (c) 6! (d) 212 (a) 22 (b) 212 (c) 32 (d) 23
















(iii) Let R be a relation on B defined by R = {(1, 2), (2, 2), (iv) Let R : B → G be defined by R = {(b1, g1), (b2, g2),




(1, 3), (3, 4), (3, 1), (4, 3), (5, 5)}. Then R is (b3, g1)}, then R is
(a) symmetric (b) reflexive (a) injective






(c) transitive (d) None of these (b) surjective






(iv) Raji wants to know the number of relations possible from (c) neither surjective nor injective


A to B. How many relations are possible?



(d) surjective and injective


(a) 62 (b) 26 (c) 6! (d) 212 (v) Ravi wants to find the number of injective functions from










(v) Let R : B → B be defined by R = {(1, 1), (1, 2), (2, 2), B to G. How many injective functions are possible?


(3, 3), (4, 4), (5, 5), (6, 6)}, then R is (a) 0 (b) 2! (c) 3! (d) 0!








(a) symmetric (b) reflexive and transitive Ans. (i) (a) 2 6
(ii) (a) equivalence









(c) transitive and symmetric (d) equivalence (iii) (d) 23 (iv) (b) surjective










Ans. (i) (a) reflexive and transitive but not symmetric (v) (a) 0



(ii) (a) 62



6. Students of Grade 12, planned to plant saplings along





(iii) (d) None of these straight lines, parallel to each other to one side of the



(iv) (d) 212 playground ensuring that they had enough play area. Let



(v) (b) reflexive and transitive us assume that they planted one of the rows of the saplings



30 Mathe atics–12
m
E:\AMIT_WORKS\Exam_Guru\EG_Mathematics-12_(working_02-06-2022)\EG_Mathematics-12_working\Open_Files\Chap_1\Chap_1
\ 16-Aug-2022  Amit   Proof-5 Reader’s Sign _______________________ Date __________


along the line y = x – 4. Let L be the set of all lines which children. Raji found that the swing traced the path of a
are parallel on the ground and R be a relation on L. parabola as given by y = x2.

(i) Let relation R be defined by R = {(L1, L2): L1 || L2 where (i) Let f : R → R be defined by f (x) = x2 is




L1, L2 ∈ L} then R is (a) neither surjective nor injective



(a) equivalence relation (b) surjective


(b) only reflexive relation



(c) injective


(c) not reflexive relation



(d) bijective


(d) symmetric but not transitive relation



(ii) Let f : N → N be defined by f (x) = x2 is


(ii) Let R = {(L1, L2) : L1 ^ L2 where L1, L2 ∈ L}. Which


(a) surjective but not injective


of the following is true?

(b) surjective
(a) R is symmetric but neither reflexive nor transitive


(c) injective


(b) R is reflexive and transitive but not symmetric


(d) bijective


(c) R is reflexive but neither symmetric nor transitive


(iii) Let f : {1, 2, 3,….} → {1, 4, 9,….} be defined by


(d) R is an equivalence relation


f (x) = x2 is


(iii) The function f : R → R defined by f (x) = x – 4 is (a) bijective



(a) bijective


(b) surjective but not injective


(b) surjective but not injective


(c) injective but not surjective


(c) injective but not surjective


(d) neither surjective nor injective


(d) neither surjective nor injective


(iv) Let : N → R be defined by f (x) = x2 . Range of the function



(iv) Let f : R → R be defined by f (x) = x – 4. Then the range


among the following is



of f (x) is

(a) {1, 4, 9, 16,…} (b) {1, 4, 8, 9, 10,…}
(a) R (b) Z (c) W (d) Q




(c) {1, 4, 9, 15, 16,…} (d) {1, 4, 8, 16,…}








(v) Let R = {(L 1 , L 2 ) : L 1 is parallel to L 2 and L 1 : y =




(v) The function f : Z→Z defined by f (x) = x2 is


x – 4}, then which of the following can be taken as L 2 ?



(a) 2x – 2y + 5 = 0 (b) 2x + y = 5 (a) neither injective nor surjective






(c) 2x + 2y + 7 = 0 (d) x + y = 7 (b) injective






Ans. (i) (a) equivalence relation (c) surjective





(ii) (a) R is symmetric but neither reflexive nor transitive (d) bijective





(iii) (a) bijective (iv) (a) R Ans. (i) (a) neither surjective nor injective









(v) (a) 2x – 2y + 5 = 0 (ii) (c) injective (iii) (a) bijective








7. Raji visited the exhibition along with her family. The (iv) (a) {1, 4, 9, 16,…}





exhibition had a huge swing, which attracted many (v) (a) neither injective nor surjective



Author’s Comments
Questions based on following types are very important for Exams. So, students are advised to revise them thoroughly.
1. Relations: To show that the given relation is reflexive, symmetric and transitive or (equivalence relation).


2. Functions: To show that given function is one-one and onto (bijective).


Relations and Functions 31

E:\AMIT_WORKS\Exam_Guru\EG_Mathematics-12_(working_02-06-2022)\EG_Mathematics-12_working\Open_Files\Chap_1\Chap_1
\ 16-Aug-2022  Amit   Proof-5  Reader’s Sign _______________________ Date __________

COMMON ERRORS
ERRORS CORRECTIONS
(i) In a set A = {1, 2, 3}, a relation R = {(1, 1), (2, 2), (3,
(i) In reflexive relation, every element of set A must be related

2)}. Here R is not reflexive relation because (3, 3) is not to itself.


present in R. Students make a mistake that if only (1, 1)
or (1, 1), (2, 2) are present R is reflexive.
(ii) To check the symmetric relation R.
(ii) Find R–1, if R = R–1 only then R is symmetric.

(iii) To check the transitivity of a relation R.


(iii) Its every ordered pair must be tested.

(iv) If x ∈ N and x12 = x22 ⇒ x1 = ± x2


      (iv) Here x1 = x2, because x1 = –x2 is not possible because x is

a natural number.
(v) If x ∈ R and x12 = x22
  ⇒ x1 = x2
  (v) Here x1 = ± x2, because x is a real number, here negative

number is possible.
(vi) While checking injectivity, surjectivity of a function,
(vi) The first step while checking injectivity, surjectivity, take

students ignore the set in which function is defined, i.e., f care of set under consideration.
: N → N or
f : Z → Z or f : R → R etc
   

(vii) Identification of co-domain, range


(vii) Co-domain is the set of all second elements, i.e., if

f : A → B, then B is the co-domain.


Range is the set of all second entries of the ordered pairs.

REVISION CHART—RELATIONS
mpty
E R elation R eflexive R elation

A relation R in set A is called empty A relation R in set A is called reflexive


relation if no element of set A related if (a, a) ∈ R for all a ∈ A or R is reflexive
to any element of set A, i.e, R = . f in set A if (a, a) ∈ R ∀ a ∈ A.

R elation s

(i) Relation R from set A to set B: Relation R from set A to set B is a subset of Cartesian product (A × B)

or Relation from set A to set B = {(a, b) : a ∈ A and b ∈ B}


If A = {1, 2} and B = {3, 4} then R = {(1, 3), (2, 4)}
(ii) Relation R in set A: Relation R in set A is a subset of Cartesian product (A × A).

If A = {1, 2, 3} then Relation R = {(1, 1), (1, 2), (2, 3)}

Symmetric R elation E quivalence Relation T ran itive


s R elation

A relation R in set A is said to be If a relation R in set A is reflexive, A relation R in set A is said to be


symmetric if (a, b) ∈ R and (b, a) ∈ R, symmetric and transitive, then transitive if (a, b) ∈ R and (b, c) ∈ R
a, b ∈ A or if relation R = its inverse relation R is called an equivalence ⇒ (a, c) ∈ R for a, b, c ∈ A.
relation R–1, then relation R is called relation.
symmetric relation.

32 Mathe atics–12
m
E:\AMIT_WORKS\Exam_Guru\EG_Mathematics-12_(working_02-06-2022)\EG_Mathematics-12_working\Open_Files\Chap_1\Chap_1
\ 16-Aug-2022  Amit   Proof-5  Reader’s Sign _______________________ Date __________

REVISION CHART—FUNCTIONS

O ne-one (or injective function)

A function f : X → Y is called one-one


(or injective) if the images of the
distinct elements of set X under f are
distinct, i.e., for every x1, x2 ∈ X;
Domain of the unction
F o- omain of the
C d Function f

In a function f : A → B, y = f (x); set  

In a function f : A → B, y = f (x); set B


A is called domain of the function f.   is called co-domain of the function f.

F unction s

Let A and B be two non-


empty sets. Then a relation
mage an re-image of an lement
from set A to set B is called a
I d P E

of a F unction R ange of a F unction


function if each element of
If the element x ∈ A corresponds to In a function f : A → B, y = f (x) the  

y ∈ B under the function f : A → B, set A has correspondence subset of B containing the image of
y = f (x) then y is called image of x
     
with exactly one element the elements of set A is called range
under f and x is called pre-image of of set B. of the function f.
y under f.
A function f from set A
to set B is denoted as:
f : A → B and y = f (x).

O nto or (Surjective) function O ne-one an d Onto or (Bijective)


A function f : X → Y is called onto function F unction
if every element of set Y is the image of A function f : X → Y is said to be
some element of set X under f, i.e., for one-one and onto or Bijective, when
every y ∈ Y there exists an element function f is both one-one and onto
x ∈ X such that f (x) = y. 

function.

Relations and Functions 33



E:\AMIT_WORKS\Exam_Guru\EG_Mathematics-12_(working_02-06-2022)\EG_Mathematics-12_working\Open_Files\Chap_2\Chap_2
\ 16-Aug-2022  Amit   Proof-4 Reader’s Sign _______________________ Date __________


Topics
C
overed
2 Inverse Trigonometric Functions

2.1 Inverse of a Function




2.2 Principal Value Branch and Graphs of an inverse trigonometric function.


C hapter map
INVERSE TRIGONOMETRIC FUNCTIONS

Domain and Range Inverse of a Function

sine cosine tangent secant cosecant cotangent

Principal Value Branch

sin–1 x cos–1 x tan–1 x sec–1 x cosec–1 x cot–1 x

We have studied trigonometric functions in detail in Class 11. Let us revise the Domains and Ranges of Trigonometric Ratios.

Trigonometric Ratios, their Domains and Ranges (Recap)


Following are the six trigonometric ratios with their respective domains and ranges:
1. y = sin x, Domain = {x : x ∈ R},


Range = {y : y ∈ [– 1, 1]} or y = sin x : R → [– 1, 1].



2. y = cos x, Domain = {x : x ∈ R},


Range = {y : y ∈ [– 1, 1]} or y = cos x : R → [– 1, 1]

{ }



3. y = tan x, Domain = R − x : x = (2n + 1) π , n ∈ Z . Range = R.
2


{ π
or y = tan x : R − x : x = (2n + 1) , n ∈ Z → R
2 }


4. y = cot x, Domain = R – {x : x = nπ, n ∈ Z}. Range = R


or y = cot x : R – {x : x = n , n ∈ Z} → R

{ }


p
5. y = sec x, Domain = R − x : x = ( 2n + 1) π , n ∈ Z , Range = R – (–1, 1)
2


{ π
or y = sec x : R − x : x = (2n + 1) , n ∈ Z} → R − ( −1,1)
2


6. y = cosec x, Domain = R – {x : x = n , n ∈ Z}, Range = R – (– 1, 1)


p
or y = cosec x : R – {x : x = n , n ∈ Z} → R – (–1, 1)


p
34
E:\AMIT_WORKS\Exam_Guru\EG_Mathematics-12_(working_02-06-2022)\EG_Mathematics-12_working\Open_Files\Chap_2\Chap_2
\ 16-Aug-2022  Amit   Proof-4 Reader’s Sign _______________________ Date __________


Topic 1. Inverse of a Function
If f : X → Y such that f (x) = y, x ∈ X and y ∈ Y, is one-one and 1
(ii) sin −1 x ≠


onto, then we can define a unique function g : Y → X such that sin x



g(y) = x where x ∈ X and y ∈ Y. Here domain of g = range of f, (iii) After this point we shall write inverse of sine function



and range of g = domain of f. Such a function g is called inverse −π π
as y = sin–1 x : [–1, 1] →  ,  .
of function f and is denoted as f –1. Here function g is one-one  2 2 



and onto, g also has its inverse. Inverse of function g is function Graph of y = sin x and y = sin–1 x
f, therefore f and g are inverse of each other.

( ) ( )
−1 −1
Thus: g −1 = f −1 = f or f −1 = g −1 =g

Thus : ( f –1 of )(x) = f –1 [ f (x)] = f –1 (y) = x and













( f of –1 )(y) = f [ f –1(y)] = f (x) = y












Now let us define inverse of trigonometric functions.

Inverse of Sine Function


x = sin q : R → [–1, 1] is the sine function whose domain is R y = sin x :  −π , π  → [ −1,1] y = sin −1 x : [ −1,1] →  −π , π 
(all real numbers) and range is [–1, 1]. For the given domain R,  2 2   2 2 
x = sin q is not one-one and onto function. So, for this domain, Graph of y = sin x Graph of y = sin–1 x
we cannot define inverse of sine function. Inverse of Cosine Function
To make x = sin q, one-one and onto function we shall have x = cos q : R → [–1, 1] is cosine function with domain R and
to re-define the domain of the sine function. The sine function range [–1, 1]. Cosine function with domain R is not a one-one
can become one-one and onto function if we choose its domain and onto function. Hence, we cannot define inverse of x = cos q
as one of the intervals: for this domain R.
−3π −π   −π π   π 3π  To make it one-one and onto function, we shall re-define its
⋅⋅⋅  , , , , , ⋅⋅⋅ .
 2 2   2 2   2 2  domain. If we select one of the intervals out of: ... [– , 0], [0, ],

p
p
[ , 2 ] ... etc. as domain, x = cos q becomes one-one and onto.

−π π
Interval  ,  is taken as domain of the sine function.
p
p
[0, ] is taken as the domain of the cosine function to make it
 2 2 
p
one-one and onto. Hence, x = cos q : [0, ] → [–1, 1] is cosine
−π π
The sine function: x = sin q with domain  ,  and

p
function which is one-one and onto (bijective) function and hence
 2 2  its inverse can be defined.
range [–1, 1] is one-one and onto function and hence its inverse
Inverse of x = cos q : [0, ] → [–1, 1] is q = cos–1 x : [–1, 1]
can be defined.
p
→ [0, ]. This shows that domain of cos–1 x is [–1, 1] and range
­
Inverse of x = sin q is denoted as: q = sin–1 x and read as ‘sine
p

is [0, ].
p
inverse x’, or inverse of (x = sin q) is: This means: –1 ≤ x ≤ 1 and 0 ≤ cos–1 x ≤ .
p
−π π
θ = sin −1 x : [ −1,1] →  ,  Principal Value Branch of cos–1 x
 2 2 
The range of cos–1 x, [0, ] is called principal value branch

–1
p
x is the inverse of sine function. Domain
Hence, q = sin of cos–1 x. This means all the value of cos–1 x will lie in the class

−π π interval [0, ].
and range of inverse of sine function are [–1, 1] and  , 
p
 2 2  Note: (i) cos–1 x is an angle whereas cos x is a number.
respectively.


(ii) cos −1 x ≠ 1 or ( cos x )
−1

Principal Value Branch of sin–1 x cos x




(iii) Now onward, inverse of cos function will be written


−π π as y = cos–1x.
The range  ,  of sin –1 x is called its principal-
 2 2 
Graph of y = cos x and y = cos–1 x
value-branch. This means, now every value of sin–1 x will
−π π
be taken in the interval  ,  .
 2 2 
Principal Value of an Inverse Trigonometric
Function
The value of an inverse trigonometric function which lies in its
principal value branch is called principal value of the inverse
trigonometric functions. Graph of y = cos x : Graph of y = cos–1 x :
Note: (i) q = sin–1 x is an angle whereas sin q is a number [0, ] → [–1, 1] [–1, 1] → [0, ]
p
p


nve se igono et ic Functions 35
I
r
T
r
m
r

E:\AMIT_WORKS\Exam_Guru\EG_Mathematics-12_(working_02-06-2022)\EG_Mathematics-12_working\Open_Files\Chap_2\Chap_2
\ 16-Aug-2022  Amit   Proof-4 Reader’s Sign _______________________ Date __________


In a similar way, we can define inverse of cosec x, inverse of sec x, inverse of tan x and inverse of cot x. Their principal value branches
and graphs are given below:
Inverse of Cosecant Function
−π π
We can define inverse of x = cosec q as q = cosec–1 x : R – (–1, 1) →  ,  − {0} . It is read as cosec inverse x.
 2 2 
−π π
Here q = cosec–1 x has domain R – (–1, 1) and Range is  ,  − {0} .
 2 2 
Principal Value Branch of cosec–1 x
−π π
The range of cosec −1x =  ,  − {0} is the principal value branch of the cosec–1 x.
 2 2 
Note: (i) cosec–1 x is an angle whereas cosec x is a number.


(ii) cosec −1x ≠ ( cosec x ) or
−1 1
.
cosec x


(iii) Now onward, inverse of cosec function will be written as: y = cosec–1 x.


Graph of y = cosec x and y = cosec–1 x

 −π π  Graph of y = cosec −1x = R − ( −1,1) →  −π , π  − {0}


Graph of y = cosec x =  2 , 2  − {0} → ( −1,1)
   2 2 

Inverse of Secant Function


π
We can define inverse of x = sec q as q = sec–1 x : R − ( −1,1) → [ 0, π ] − 2 {}
It is read as ‘secant inverse x’. Hence, q = sec x has domain: R – (–1, 1) and Range: [ 0, π ] − π .
–1
–1
2 {}
Principal Value Branch of sec x
π
The range of sec–1 x, i.e., [ 0, π ] −
2 {}
is the principal value branch of sec–1 x.

Note: (i) sec–1 x is an angle whereas sec x is a number.




(ii) sec −1 x ≠ 1 or (sec x )
−1
sec x


(iii) Now onward, inverse of secant function will be represented as: y = sec–1 x.


Graph of y = sec x and y = sec–1 x

2 {}
Graph of y = sec x : [ 0, π ] − π → R − ( −1,1) Graph of y = sec −1 x = R − ( −1,1) → [ 0, π ] − π
2 {}
36 Mathe atics–12
m
E:\AMIT_WORKS\Exam_Guru\EG_Mathematics-12_(working_02-06-2022)\EG_Mathematics-12_working\Open_Files\Chap_2\Chap_2
\ 16-Aug-2022  Amit   Proof-4 Reader’s Sign _______________________ Date __________


Inverse of Tangent Function
π
x = tan θ : R −  x : x = ( 2n + 1) , n ∈ Z → R is the tangent function with domain R −  x : x = ( 2n + 1) π , n ∈ Z and range R.
 2   2 

( )
Now q = tan–1 x : R → −π , π is the inverse function of x = tan q: with domain R and range −π , π .
2 2
–1
2 2 ( )
Principal Value Branch of tan x

( )
The range of q = tan x, i.e., −π , π is the principal value branch of q = tan–1 x.
–1
2 2
Note: (i) tan–1 x is an angle whereas tan x is a number.


(ii) tan −1 x ≠ ( tan x ) or 1
−1
tan x


(iii) Now onward, inverse of tangent function will be represented as: y = tan–1 x.


Graph of y = tan x and y = tan–1 x

Graph of y = tan x : −π , π → R
2 2 ( ) Graph of y = tan −1 x : R → −π , π
2 2 ( )
Inverse of Cotangent Function
x = cot q : R – {x : x = n , n ∈ Z} → R is a cotangent function with domain: R – {x : x = n , n ∈ Z} and range R.
Hence: q = cot–1 x : R → (0, ) is the inverse function of x = cot q : (0, ) → R. q = cot–1 x : R → (0, ) is read as ‘cot inverse x’.
p
p

Domain and range of q = cot–1 x are R and (0, ) respectively.
p
p
p
p
Principal Value Branch of q = cot–1 x
The range of q = cot–1 x i.e., (0, ) is the principal value branch of q = cot–1 x.
p
Note: (i) cot–1 x is an angle whereas cot x is a number.


1
or ( cot x ) .
−1
(ii) cot −1 x ≠
cot x


(iii) Now onward, inverse function of cotangent will be represented as: y = cot–1 x.


Graph of y = cot x and y = cot–1 x

Graph of y = cot x : R – {x : x = n , n ∈ Z} → R Graph of y = cot–1 x : R → (0, )


p
p
nve se igono et ic Functions 37
I
r
T
r
m
r

E:\AMIT_WORKS\Exam_Guru\EG_Mathematics-12_(working_02-06-2022)\EG_Mathematics-12_working\Open_Files\Chap_2\Chap_2
\ 16-Aug-2022  Amit   Proof-4 Reader’s Sign _______________________ Date __________


The Domain and Ranges (Principal Value Branches) of Inverse Trigonometric Functions
Inverse Trigonometric Functions Domains Ranges (Principal Value Branches)
 −π , π 
y = sin–1 x [–1, 1]
 2 2 
y = cos–1 x [–1, 1] [0, ]

p
 −π , π  − {0}
y = cosec–1 x R – (–1, 1)
 2 2 

y = sec–1 x R – (–1, 1) {}
[0, π ] − π
2

y = tan–1 x R ( )
−π π
,
2 2
y = cot–1 x R (0, )

p
Example 1. The principal value of inverse trigonometric function
q= π
()
\
1 is 3
sin −1




(a) π
2
(b) π (c) π (d) π
⇒ P.V. of sec–1 (2) =
π
3
∈[ 0, π ] −
π
2 {}

3 4 6








Hence, option (c) is the correct answer.

2 ()
Solution. sin −1 1 : P.V.B. of sin −1 x :  −π , π 
 2 2  Example 4. The principal value of inverse trigonometric function
cosec–1 (1) is
Let sin −1
1
2 ()
=q ⇒ sin q =
1
2
(a)
π
3
(b) π
6
(c) π
4
(d) π
2











π  − π π
⇒ sin q = sin –1
Solution. cosec (1) : P.V.B. of cosec x :–1
,  − {0}
6  2 2 


π  −π π  π
⇒ q = ∈ ,  Let cosec–1 (1) = q ⇒ cosec θ = 1 = cosec
6  2 2 2






⇒ P.V. of sin
−1
() 1 π −π π
= ∈ , 
6  2 2 
Since ∈
π  −π π 
, − {0}
2  2 2 
\ θ= π
2


2


π −π π
Hence, option (c) is the correct answer. ⇒ P.V. of cosec–1 (1) = ∈  ,  − {0}
2  2 2 

Example 2. The principal value of inverse trigonometric function
Hence, option (d) is the correct answer.
tan–1 (1) is
π Example 5. The principal value of inverse trigonometric function
(a) π (b) π (c) (d) π
3 4 2 6  − 3








sin −1 
( )
is
Solution. tan (1) : P.V.B. of tan x :
−1 −1 −π π
,  2 
2 2
−π π π
π (a) (b) π (c) (d)
Let tan–1 (1) = q ⇒ tan q = 1 = tan 3 3 6 4








4




⇒ θ= ∈ π
4
− π
(
2 2
,
π
) −1  − 3 
Solution. sin 
2  
−π π
: P.V.B. of sin −1 x :  , 
 2 2 

 
⇒ P.V. of tan–1 (1) = π ∈ −π , π
4 2 2 ( )  − 3
Let sin −1  ⇒ sin θ =
− 3

=q
Hence, option (b) is the correct answer.  2  2


() ( ) ( )
Example 3. The principal value of inverse trigonometric function π −π −π  −π π 
sec–1 (2) is ⇒ sin q = − sin = sin since ∈ ,
3 3 3  2 2 

π
(a) π (b) (c) π (d)
2 3 2 −π




p




{}
\ q=
π 3
Solution. (c) sec ( 2) : P.V.B. of sec x : [ 0, π ] −
−1 −1



2
 − 3  −π  −π π 
Let sec–1 (2) = q ⇒ sin −1  = ∈ ,
 2  3  2 2 
{}




π π π
⇒ sec q = 2 = sec . Since ∈[ 0, π ] − Hence, option (a) is the correct answer.
3 3 2

38 Mathe atics–12
m
E:\AMIT_WORKS\Exam_Guru\EG_Mathematics-12_(working_02-06-2022)\EG_Mathematics-12_working\Open_Files\Chap_2\Chap_2
\ 16-Aug-2022  Amit   Proof-4 Reader’s Sign _______________________ Date __________


Example 6. The principal value of inverse trigonometric function
−1  −2  −π π
Solution. cosec   : P.V.B. of cosec x :  ,  − {0}
( )
−1
−1  3  2 2
cos −1 is
2

(a) π
π 2π π
−1  −2 
Let cosec   = q
 3
⇒ cosec θ =
−2
3
= − cosec ()
π
3





(b) (c) 3 (d) 6
4
( ) ( )
3








−π −π  −π π 
 2 2  { }
( ) −1 ⇒ cosec q = cosec , since ∈ , − 0
Solution. cos −1 : P.V.B. of cos −1 x : [ 0, π ] 3 3


2
−1  −2 

( )
−π −π  −π π 
Let cos −1
( )−1
= q ⇒ cos θ =
−1
= − cos
π \ q=
3
⇒ cosec   =
 3
∈ , − {0}
3  2 2 



2 2 3




( ) ( ) ( )[
Hence, option (b) is the correct answer.
π 2π 2π
cos q = cos π − = cos since ∈ 0, π ] Example 9. The principal value of inverse trigonometric function
3 3 3

sec–1 (–1) is
\ q=

3
−1
⇒ cos − ( ) 1
2
=

3
∈[ 0, π ]
(a)
π
(b)
−π
(c) π
π
(d) 3




2 3









{}
Hence, option (c) is the correct answer.
π
Example 7. The principal value of inverse trigonometric function Solution. sec −1 ( −1) : P.V.B. of sec −1 x : [ 0, π ] −
2
 −1 
tan −1   is Let sec–1 (–1) = q ⇒ sec q = –1 = – sec 0 = sec
 3


{}



p
π Since π ∈[ 0, π ] − π \ q=
π π −π 2


p
(a) (b) (c) (d)
{}
6 4 3 6








⇒ P.V.B. of sec–1(–1) = ∈ [0, ] – π
 
Solution. tan −1  −1  : P.V.B. of tan −1 x : −π , π ( ) 2

p
p
 3 2 2 Hence, option (c) is the correct answer.
Example 10. The principal value of inverse trigonometric
−1  −1   −1 
Let tan   = q
 3
⇒ tan θ =  
 3 function cot −1 − 3 ( ) is




tan q = − tan () ( )
π
6
= tan −
π
6
since
−π
6

−π π
,
2 2 ( )( ) π
(a) 6 (b)

3

(c) 4 (d)

6









( )
\ q=
−π
6
 
⇒ tan −1  −1  = −π ∈ −π , π
 3 6 2 2 ( ) Solution. cot −1 − 3 : P.V.B. of cot −1 x : ( 0, π )


Hence, option (d) is the correct answer.
Let cot −1 (− 3) = q ⇒ cot θ = − 3 = − cot π
6 ( )


Example 8. The principal value of inverse trigonometric function
 −2 

π
6
= cot

cot q = cot π −
6
, since

6
∈( 0, π ) ( ) ( )



cosec −1   is
 3 \ q=

6
⇒P.V.B.of cot −1 − 3 =

6
∈ ( 0, π ) ( )




π −π π π
(a) (b) (c) (d) 4 Hence, option (d) is the correct answer.
3 3 6








EXERCISE 2.1
I. Multiple Choice Questions (MCQs) 3. The principal value of inverse trigonometric function




Choose the correct answer from the given options. sec– 1(2) is
π π

1. The principal value of inverse trigonometric function π
(a) 4 (b) (c) 3 (d) π
2
()










1
sin −1 is 4. The principal value of inverse trigonometric function
2


π π π cosec–1(1) is
(a) π (b) (c) 4 (d)
6 8 3 π π π π








(a) (b) (c) (d)
2. The principal value of inverse trigonometric function 3 6 2 4








( )


tan–1(1) is 5. The value of sin  π − sin −1 −1  is
3 2 


π π π
(a) π (b) (c) 3 (d)
2 4 6 (a) 3 (b) –1 (c) 2 (d) 1
















nve se igono et ic Functions 39
I
r
T
r
m
r

E:\AMIT_WORKS\Exam_Guru\EG_Mathematics-12_(working_02-06-2022)\EG_Mathematics-12_working\Open_Files\Chap_2\Chap_2
\ 16-Aug-2022  Amit   Proof-4 Reader’s Sign _______________________ Date __________


 9π 
6. The principal value of tan–1  tan  is
 8
 
(ii) 2 sin −1  3  − cos −1 −1 + 3 tan −1(1)
 2  2 ( )




(a)
π
8
(b)

8
(c) −
π
8
(d) −

8
 7π 1 
(iii) tan  − 2 sin −1  .
 12 2  ()










[CBSE 2022] III. Long Answer Type Question

7. What is the principal value of sec–1 (–2)?
 3 1
1. Find the value of: 2 sin −1   − cos −1 − + 3 tan −1 (1) ( )


3π 2π 4π π
(a) (b) (c) 3 (d) 2  2  2



2 3
()








 7π 1 
 1  1  2. Find the value of: tan  − 2 sin −1  .
8. The principal value of cos–1   + sin–1  −  12 2 



 2 
2
( ) ( )


−1 −1
π π π π 3. Find the value of: tan −1 (1) + cos −1 + sin −1
(a) (b) (c) (d) 2 2



12 4 3 6








4. Find the value of:
[CBSE 2022]



( )

 −1 π  π  − 3 
II. Short Answer Type Questions-I cos cos −1 +  + sin  − sin −1  .
 2 3 6  2  


1. (i) Write the principal value of tan −1 ( ) ( )
3 − cot −1 − 3



5. Find the value of:
[A.I. 2013]



( ) () ()

π  1 1 
1
(ii) Write the principal value of tan −1 (1) + cos −1 − . cos  + 2 tan −1 (1) + sin 3 sin −1 + 2 cos −1  .
 6   2 2 

2


[Delhi 2013] 6. Find the value of: 3 tan −1 (1) − cos −1 −1 + 2 sin −1 −1 .
2 2 ( ) ( )
( )



 −π 
(iii) Write the value of tan −1 sin . [A.I. 2014] 7. Find the value of:
 2 





(iv) Write the principal value of cos–1[cos (680°)]. 2 tan −1 ( 3 ) − sec ( − 2 ) + cosec
−1 −1  2 
  .


[Delhi 2014] 3

( ) ()

−1 1 8. Find the value of:
(v) Write the value of cos −1 + 2 sin −1 .


( )
2 2


 − 3 −1  −1 
[Foreign 2014] 3 sin −1  + 2 cos −1 + tan −1   .
 2  2  3


2. Find the principal values of each of the following inverse 9. Find the value of:


trigonometric functions:


(i) tan −1(1) + cos −1
−1
2
+ sin −1
−1
2 ( ) ( )

3 sin
−1 −1
2 ( )  − 3
+ 2 sin −1 
 2
−1  −1  
 − tan  3   .

 


Answers 2.1
I. 1. (a)
π
6
sec–1(2) : P.V.B. of sec–1 x = [ 0,π ] −
π
2 {}






sin –1 1
2 ()
−π π
: P.V.B of sin −1 x =  , 
 2 2  Let sec–1 (2) = q ⇒ sec q = 2 = sec
π

3
()




1 π
{}
1
Let sin–1 = q ⇒ sin q = = sin π π
2 2 6 ⇒ q= ∈[ 0, π ] −


3 2



π  −π π  π
⇒ q = ∈ ,  4. (c)
6  2 2 2






π  −π π 
2. (b) cosec–1 (1) : P.V.B. of cosec–1 x =  ,  − {0}
4  2 2



( )

−π , π π
tan (1) : P.V.B. of tan–1 x =
–1 –1
Let cosec (1) = q ⇒ cosec q = 1 = cosec
2 2 2






π π  −π π 
Let tan–1 (1) = q ⇒ tan q = 1 = tan ⇒ q= ∈ , − {0}
4 2  2 2 






⇒ q=
π ∈ −π , π
( ) 5. (d) 1

( ) ()



4 2 2  
sin  π − sin −1 −1  = sin  π + sin −1 1 



π  3 2   3 2 


3. (c)
3



[Q sin–1(–x) = –sin–1 x]

40 Mathe atics–12
m
E:\AMIT_WORKS\Exam_Guru\EG_Mathematics-12_(working_02-06-2022)\EG_Mathematics-12_working\Open_Files\Chap_2\Chap_2
\ 16-Aug-2022  Amit   Proof-4 Reader’s Sign _______________________ Date __________


π π
= sin  +  = sin
 3 6 
π =1
2 ()  −1 1 π
Qsin 2 = 6  () (ii) tan −1 (1) + cos −1 −
1
2
= tan (1) + π − cos
−1 −1 1
2 ( ) ()





–1
π [Q cos (– x) = – cos–1 x]
6. (a)

p
8 π +π− π



=
4 3



7. (b) 2π 3π + 12π − 4π = 11π
3 =



12 12



sec–1(–2) : P.V.B. of sec–1 x = [ 0,π ] −
π
2 {} (iii)
−π
4
(iv) 40° (v)







p
Let sec–1 (–2) = q ⇒ sec q = –2 3π 3π
2. (i) (ii) (iii) 1
4 4










⇒ –sec q = 2
3π 3π



π III. 1. 2. 1 3. 4. 0
⇒ sec( – q) = 2 ⇒ –q= 4 4







3


p



p
1
5. − 2 6. −π
π 2π ∈ 0, π − π
[ ] 2 {} 4





⇒ q= π− =
3 3



π
7. 2 tan −1 ( 3 ) − sec ( − 2 ) + cosec
−1 2 
 
3
−1 



8. (a)
12
( 3 ) − π + sec ( )  



= 2 tan −1 −1
2 + cosec −1  2 
II. 1. (i) tan −1 ( 3 ) − cot ( − 3 ) −1  3


[Q sec–1 (–x) = – sec–1 x]




−1
= tan 3 −  π − cot −1 3 

p


On solving, we get
[Q cot–1 (–x) = – cot–1 x]

π π π π
p
= tan −1 3 − π + cot −1 3 = 2× −π+ + =
3 4 3 4



π − π + π = π − π = −π π
= 8. 6 9. –
3 6 2 2





p
Case Based Questions
1. (iii) ∠BCA = b =

()




B
(a) tan–1 1 (b) tan–1 (2)
2




  (c) tan–1  1 
 3
(d) tan–1 ( 3)




A C
x D d

(iv) ∠ABC =


Two men on either side of a temple of 30 metres high (a) π (b) π
4 6





observe its top at the angles of elevation a and b
π
respectively (as shown in the figure above). The distance (c) π (d)
2 3




between the two men is 40 3 metres and the distance
between the first person A and the temple is 30 3 metres. (v) Domain and Range of cos−1 x =


Based on the above information answer the following: (a) ( −1, 1 ), (0, π) (b) [ −1, 1 ], (0, π)




(i) ∠CAB = a = (c) [ −1, 1 ], [0, π] (d) ( −1, 1 ) ,  − π , π 
 2 2 
()






  1
(a) sin–1  2  (b) sin–1  3
 3 2  1




Ans. (i) (b) sin–1   (ii) (c) cos–1  
   2  2
(d) sin–1  3 





(c) sin–1(2)
 2  π




(iii) (d) tan–1 ( 3 ) (iv) (c)
(ii) ∠CAB = a = 2






(v) (c) [ −1, 1 ], [0, π]
() ()


2
(a) cos–1 1



(b) cos–1 Case Study–II
5 5





2. The Government of India is planning to fix a hoarding
 3
()


–1 –1 4 board at the face of a building on the road of a busy market
(c) cos  2  (d) cos
5 for awareness on COVID-19 protocol. Ram, Robert and




nve se igono et ic Functions 41
I
r
T
r
m
r

E:\AMIT_WORKS\Exam_Guru\EG_Mathematics-12_(working_02-06-2022)\EG_Mathematics-12_working\Open_Files\Chap_2\Chap_2
\ 16-Aug-2022  Amit   Proof-4 Reader’s Sign _______________________ Date __________


Rahim are the three engineers who are working on this
project. “A” is considered to be a person viewing the
(c) tan−1 ()
4
3
(d) tan−1 (4)





hoarding board 20 metres away from the building, standing
at the edge of a pathway nearby. Ram, Robert and Rahim (iii) Measure of ∠EAB =



suggested to the firm to place the hoarding board at three (a) tan−1 (11) (b) tan−1 3

( )





( )
different locations namely C, D and E. “C” is at the height 2 11
of 10 metres from the ground level. For the viewer A, the (c) tan−1 (d) tan−1 2
11





angle of elevation of “D” is double the angle of elevation
of “C”. The angle of elevation of “E” is triple the angle (iv) A' is another viewer standing on the same line of



of elevation of “C” for the same viewer. Look at the observation across the road. If the width of the road is
figure given and based on the above information answer 5 metres, then the difference between ∠CAB and ∠CA′B is
the following:
E (a) tan–1 ()
1
2
(b) tan−1 1
8 ()





D (c) tan−1 ()
2
5
(d) tan−1
11
21 ( )





C
(v) Domain and Range of tan−1x =

( ) ( )


(a) R+, − π , π (b) R–, − π , π
2 2 2 2





10 m

A 5m A 20 m
B


(
(c) R, − π , π
2 2 ) (d) R, 0, ( ) π
2




(i) Measure of ∠CAB =

() Ans. (i) (b) tan–1 1 () (ii) (c) tan–1 4 ()




1
(a) tan−1 (2) (b) tan−1 2 3





2
( )




(c) tan−1 (1) −1
(d) tan (3) (iii) (d) tan–1 11
2
(iv) (b) tan–1 1
8 ()









(ii) Measure of ∠DAB =

() ( )


3 (v) (c) R, − π , π
(a) tan−1 4 (b) tan−1 (3) 2 2







Author’s Comments
Questions based on following types are very important for Exams. So, students are advised to revise them thoroughly.
1. To find principal value of a given trigonometric function.


2. To convert one inverse trigonometric function into another equal trigonometric function.


IMPORTANT FORMULAE
The Domain and Range (Principal Value Branches) of Inverse Trigonometric Functions
Inverse Trigonometric Functions Domains Range (Principal Value Branches)

 −π , π 
y = sin–1 x [–1, 1]  2 2 

y = cos–1 x [–1, 1] [0, ]


p
 −π , π  − {0}
y = cosec–1 x R – (–1, 1)  2 2 

y = sec–1 x R – (–1, 1) {}
[0,π ] − π
2

y = tan–1 x R ( )
−π , π
2 2

y = cot–1 x R (0, )
p
42 Mathe atics–12
m
E:\AMIT_WORKS\Exam_Guru\EG_Mathematics-12_(working_02-06-2022)\EG_Mathematics-12_working\Open_Files\Chap_2\Chap_2
\ 16-Aug-2022  Amit   Proof-4 Reader’s Sign _______________________ Date __________


COMMON ERROR
ERROR CORRECTION
(i) Students generally do not remember the principal value (i) Write down the principal value branch of the respective




branches of different inverse trigonometric functions and trigonometric function.
commit mistakes.

REVISION CHART

I nver e of a s F unction s

If f : X → Y such that f(x) : y, x ∈ X and y ∈ Y, is one-one and onto, then we can define a unique function g : Y → X such that
g(y) = x where x ∈ X and y ∈ Y.
   

Domain an d rincipal
P V alue Branch ( ange)of R

A ll nver e
I s rigonometric
T unction
F s

F unction Domain R ange

 −π π 
sin–1x [–1, 1]  2 , 2
 

cos–1x [–1, 1] [0, ]


p

 −π π 
cosec–1x R – (–1, 1)  2 , 2  − {0}
 

π
sec–1x R – (–1, 1) [0, π ] −  2 
 

 −π π 
tan–1x R  , 
2 2

cot–1x R (0, )
p

nve se igono et ic Functions 43


I
r
T
r
m
r

E:\AMIT_WORKS\Exam_Guru\EG_Mathematics-12_(working_02-06-2022)\EG_Mathematics-12_working\Open_Files\Chap_3\Chap_3
\ 19-Aug-2022  Amit   Proof-4 Reader’s Sign _______________________ Date __________


3 Matrices
Topics overed
C
3.1. Matrix 3.2. Operations on Matrices




l Types of Matrices l Transpose of a Matrix




l Equality of Matrices l Symmetric and Skew Symmetric Matrix




l Multiplication of a Matrix by a Scalar Number


l Addition of Matrices


l Subtraction of Matrices


l Multiplication of a Matrix with Another Matrix


l Inverse of a Square Matrix


C hapter map
MATRIX

Types of Matrices

Row Column Zero Identity Scalar Diagonal Inverse of a


Matrix Matrix Matrix Matrix Matrix Matrix Square Matrix

Order of a 3×3 Order Square


Matrix General Matrix Rectangular Invertible
Matrix Matrix Matrix

Symmetric and Skew


Symmetric Matrix

Operations on Matrices

Addition of Equality of Multiplication


Matrices Matrices of a matrix

Elements of Subtraction Transpose of


a Matrix of Matrices a Matrix

Topic 1. Matrix
Arrangement of numbers in horizontal and vertical lines is called Elements or Members of the Matrix
a matrix. The numbers used to form a matrix are called its elements or
7 −8 5  members. These numbers are enclosed in ( ) or [ ] braces.
 4
 1 3  1 3 0     3 0 2
For example,   ,  −5 2 4 ,  2 0 1 are matrices. For example: 
 4 0      is a matrix. Here 3, 0, 2, –4, 5
0 3 2  −4 5 7 
  and 7 are its elements or members.

44
E:\AMIT_WORKS\Exam_Guru\EG_Mathematics-12_(working_02-06-2022)\EG_Mathematics-12_working\Open_Files\Chap_3\Chap_3
\ 16-Aug-2022  Amit   Proof-4 Reader’s Sign _______________________ Date __________


A matrix is usually denoted by a capital letter, A, B or C, e.g., \ Matrix A can have any one of the possible orders: 1 × 6,
2 3 7 2 × 3, 3 × 2 or 6 ×1.
 3 1 4   Example 1. A matrix has 12 elements. Number of possible orders
A=  ; B = 1 5 8
0 5 7  having it is
 4 6 9 
(a) 5 (b) 4 (c) 6 (d) 7
Rows of a Matrix









Solution. A matrix has 12 elements. Number 12 can be factorised
In a matrix, horizontal lines of numbers are called rows. In a as:
 2 1 3 12 =1 × 12 = 2 × 6 = 3 × 4 = 4 × 3 = 6 × 2 = 12 × 1
matrix A =  4 0 −5 , there are three rows and these are:



  \The number of possible orders of the matrix is 6, i.e.: 1 ×



 6 7 8 12, 2 × 6, 3 × 4, 4 × 3, 6 × 2, 12 × 1.
Hence, option (c) is the correct answer.
 2 1 3 ← First row


 4 0 −5 ← Second row 3 0 2 
 
 6 7 8 ← Third row Example 2. If a matrix A =  1 −5 7  , then the order of the
 
matrix A is 8 4 9
Columns of a Matrix
In a matrix, vertical lines of numbers are called columns. In a (a) 2 × 3 (b) 3 × 3
(c) 3 × 2 (d) 2 × 2
 3 1 −1









 3 0 2 
 
matrix B = 0 5 2 , there are three columns and the three Solution. The given matrix: A =  1 −5 7 
 
 1 4 7  8 4 9
columns are:
Q Since, there are 3 rows and 3 columns in the above given
3 1 −1 matrix, therefore, its order is 3 × 3.
0 5 2 Hence, option (b) is the correct answer.

1 4 7 Example 3. The values of elements: a23, a31, a22, and a13, in the

­
matrix given in Example 2, are
First Second Third
column column column (a) 7, – 5, 8, 2 (b) 7, 2, – 8, 5





(c) 7, 8, –5, 2 (d) 5, 7, –2, 8




Order of a matrix Solution. a23 = 7, a31 = 8, a22 = –5 and a13 = 2.
If a matrix has m-rows and n-columns, then its order is m × n. (It Hence, option (c) is the correct answer.

is read as m by n).
(A) Types of Matrices
In m × n, first number m indicates number of rows, and second
Following are different types of matrices:
number n indicates number of columns in the given matrix
7 1 −1 Row matrix
Take a matrix: A = 
 4 5 0 A matrix having only one row and any number of columns is
 
called a row matrix. For example: [3]1×1, [4 –7]1×2, [5 0 –7]1×3
There are 2 rows and 3 columns, hence its order is 2 × 3 and



are row matrices.
this matrix has 2 × 3 = 6 elements.
3 × 3 Order General Matrix Column Matrix
A matrix having only one column and any number of rows is
 a11 a12 a13 
called a column matrix.
A 3 × 3 order general matrix can be written as: A =  a21 a22 a23 
 a31 a32 a33   1
3  
3× 3
For example: [ 7 ]1×1 ,   ,  0 are column matrices.
This general representation of a 3 × 3 matrix is very important 5
  2 ×1  

because the number 11, 12, 13, etc. written with elements of the  −7  3×1
matrix tell us in which row and column that particular element Zero or Null Matrix
is situated.
A matrix of any order having its all elements zero, is called zero
For example, a23 means, this element is situated in second
matrix or null matrix.
row and third column of the matrix.
If you are given number of elements in a matrix, you can work  0  0 0 0  0 0
Forexample:[0]1×1,[0 0]1×2,   ,   , 
out all possible orders of the given matrix. Suppose a matrix A 0 2 ×1 0 0 0 2 × 3 0 0 2 × 2

has 6 elements, then 6 = 1 × 6 = 2 × 3 = 3 × 2 = 6 × 1. etc. are null or zero matrices.

Mat ices 45
r

E:\AMIT_WORKS\Exam_Guru\EG_Mathematics-12_(working_02-06-2022)\EG_Mathematics-12_working\Open_Files\Chap_3\Chap_3
\ 16-Aug-2022  Amit   Proof-4 Reader’s Sign _______________________ Date __________


Square Matrix Or
A matrix in which number of rows is equal to number of columns A square matrix A = [aij]n×n is a scalar matrix if:
is called a square matrix.
α when i = j
aij = 
 1 3 2  0 when i ≠ j
 3 0
For example, [ −7 ]1×1 ,   and  2 0 5 are
 −5 2 2 × 2  4 0 0
 −5 4 7  3× 3 7 0  0 4 0
For example, [ ]1×1 
5 ,  , are scalar
square matrices.  0 7  2 × 2  

 0 0 4  3× 3
Rectangular Matrix matrices.
A matrix in which number of rows is not equal to number of Note: (i) A scalar matrix is also a diagonal matrix.
columns is called a rectangular matrix.



(ii) A scalar matrix is a times of an identity matrix of same order.



 3 5
1 1 0 Construction of a Matrix when its General Term
For example, [1 2 3]1× 3 ,   and  1 0 are
 2 3 4  2×3 and Order are Given
 4 3 3× 2
To construct a matrix with given general element and order, first
rectangular matrices.
write the general matrix of the required order and then fill in the
Diagonal of a Square Matrix values of the elements with the help of given general element.
If A = [aij]n×n is a square matrix of order n × n then the elements Example 4. A matrix of order 3 × 3 whose general element is aij
a11, a22, a33 ..., an×n of this matrix form the main diagonal or just = 3i + 4j, is given by
diagonal of the square matrix A.
 7 5 11  21 5 11
 
 a11 a12 a13  (a) 14 10 18 (b) 10 14 18
 
Thus: A =  a21 a22 a23 





 is a square matrix of order 17 13 21  3 17 7 
 a31 a32 a33 
3× 3
 7 11 15  7 5 11
3 × 3. Here a11, a22 and a33 elements of the matrix A form the
(c) 10 14 18 (d) 14 18 10
main diagonal of the square matrix.    




13 17 21 13 21 17 
Diagonal Matrix
A square matrix A = [aij]n×n is called diagonal matrix if its all aij Solution. General element of the required matrix of order 3 × 3
= 0 for i ≠ j. is aij = 3i + 4j.
Or To construct it, take a 3 × 3 general matrix:

A square matrix whose all elements except diagonal elements are  a11 a12 a13 
zero is called a diagonal matrix. A =  a21 a22 a23 

 4 0 0  a31 a32 a33 
 3 0  0 3 0 3× 3
For example, [ ]1×1 0 5
7 , and   are Now aij = 3i + 4j
  2× 2



 0 0 7  3×3
(3 × 1 + 4 × 1) (3 × 1 + 4 × 2) (3 × 1 + 4 × 3) 
diagonal matrices.
A = (3 × 2 + 4 × 1) (3 × 2 + 4 × 2) (3 × 2 + 4 × 3)

Identity Matrix or Unit matrix (3 × 3 + 4 × 1) (3 × 3 + 4 × 2) (3 × 3 + 4 × 3) 3×3
A square matrix in which each main diagonal element is 1 and all
other elements are zero, is called an identity matrix or unit matrix.  3 + 4 3 + 8 3 + 12   7 11 15
Or ⇒ A = 6 + 4 6 + 8 6 + 12 ⇒ A = 10 14 18
 



The square matrix A = [aij]n×n is called an identity or unit matrix 13 17 21 3× 3
9 + 4 9 + 8 9 + 12 3× 3
1 when i = j
if: aij =  Hence, option (c) is the correct answer.
0 when i ≠ j
Note: An identity matrix of order n × n is denoted as In×n, e.g., 5i + 3j if i< j

Example 5. A matrix: A = [aij]3×3 when aij =  7 if i= j
1 0
I2× 2 =  . is given by 4i − 3j if i> j

0 1
Scalar Matrix  7 5 8 11 7 14
 
A square matrix whose all non-diagonal elements are zero and all (a) 11 14 9 (b)  9 6 7 
 





diagonal elements are equal is called scalar matrix.  6 7 5  7 5 6

46 Mathe atics–12
m
E:\AMIT_WORKS\Exam_Guru\EG_Mathematics-12_(working_02-06-2022)\EG_Mathematics-12_working\Open_Files\Chap_3\Chap_3
\ 16-Aug-2022  Amit   Proof-4 Reader’s Sign _______________________ Date __________


7 11 14 7 11 14 a matrix whose (ij)th element is k(aij) for all i and j. Thus, kA =


    k[aij]m×n = [kaij]m×n.
(c) 7 5 19 (d)  5 7 19
OR




 9 6 7   9 6 7 
When we multiply a scalar number with a matrix of an order
Solution. General element of the required matrix of order 3 × 3 m × n, then every element of the matrix get multiplied by that
5i + 3j if i < j scalar number.
 3 0 5
is aij =  7 if i = j  
4i − 3j if i > j F o r e x a m p l e : i f m a t r i x A =  1 −1 −2 then

 2 4 8 3× 3
To construct a matrix A, take a 3 × 3 general matrix:

 a11 a12 a13  3 0 5 15 0 25
a  5A = 5  1 −1 −2 =  5 −5 −10
  
 21 a22 a23 
 a31 a32 a33   2 4 8 3× 3 10 20 40 3×33
3× 3
In a similar way if we want to take a number common from
 7 (5 × 1 + 3 × 2) (5 × 1 + 3 × 3)  a matrix, it is taken common from all the elements of the matrix.

⇒ A = ( 4 × 2 − 3 × 1) 7 (5 × 2 + 3 × 3)
 3 −6 0  1 −2 0


( 4 × 3 − 3 × 1) ( 4 × 3 − 3 × 2) 7  Matrix A =   = 3 
9 −15 −18 2 × 3 3 −5 −6 2 × 3
3× 3

7 11 14
⇒ A =  5 7 19 (D) Addition of Matrices


9 6 7  3× 3 If A = [aij]m×n and B = [bij]m×n are two matrices of same order
m × n, then their sum (A + B) is the matrix of the same order m
Hence, option (d) is the correct answer. × n and it is obtained by adding corresponding elements of the
matrix A and matrix B.
(B) Equality of Matrices
A + B = [aij]m×n + [bij]m×n ⇒ A + B = [aij + bij]m×n.
Two matrices A = [aij]m×n and B = [bij]m×n are called equal matrices


if: (i) they are of same order and (ii) each element of matrix A Note: The above definition of addition of matrices shows that only
is equal to corresponding element of the matrix B, i.e., aij = bij. same order matrices can be added and the order of the resultant
matrix also remains the same.
a b c 4 3 −1
For example: If  = , then a = 3 0 5 
d e f  2 × 3  2 7 − 4 2 × 3 Example 7. If two matrices A and B are such that: A =  
1 2 5 2×3
4, b = 3, c = –1, d = 2, e = 7 and f = – 4. 1 1 2 

and B =   , then 3A + 2B is given by
 x− y z 3 5 
3 2 5  2×3
Example 6. If  =  , the values of x, y, z
 2x + 3y w 11 −7 
19 11 2  9 11 19
and w are given by (a)   (b)  
10 25 9  2 25 10





(a) x = –2, y = 1, z = 5, w = 3



(b) x = 3, y = 5, z = –7, w = 7  9 2 19 11 2 19
(c)  (d)  



(c) x = 7, y = –7, z = 3, w = 2 11 25 10  9 10 25







(d) x = 4, y = 1, z = 5, w = –7
3 0 5 


 x− y z 3 5  Solution. We are given two matrices A and B as: A =  
Solution. We are given  = . 1 2 5 2×3
 2x + 3y w 11 −7 
1 1 2 
Since two matrices of equal orders 2 × 2 are given equal and B =  
3 2 5  2×3

so their corresponding elements will also be equal.
 x− y =3  z=5  3 0 5 1 1 2
\ and  . ⇒ 3A + 2B = 3   + 2 
2x + 3y = 11  w = −7 1 2 5 2 × 3 3 2 5  2 × 3





On solving these simultaneous equations for x, y, z and w, we 9 0 15 2 2 4
=  +

get x = 4, y = 1, z = 5 and w = –7.  
3 6 15 2×3  6 4 10  2×3


Hence, option (d) is the correct answer.

11 2 19 
(C) Multiplication of a Matrix by a Scalar Number =  
 9 10 25 2×3


If A = [aij]m×n is a matrix and k is any scalar number, then kA is

Mat ices 47
r

E:\AMIT_WORKS\Exam_Guru\EG_Mathematics-12_(working_02-06-2022)\EG_Mathematics-12_working\Open_Files\Chap_3\Chap_3
\ 16-Aug-2022  Amit   Proof-4 Reader’s Sign _______________________ Date __________


11 2 19  20 5 10   3 −15 12 
⇒ 3A + 2B =   =   − 12 9 −12  2×3
 9 10 25 2 × 3  5 0 25  2×3 



Hence, option (d) is the correct answer.  17 20 −2 
=  
Properties of Addition of Matrices  −7 −9 37  2×3


Following are the properties of addition of matrices.  17 20 −2
1. Addition of matrices is commutative: Let A and B be ⇒ (5A – 3B) =  
 −7 −9 37  2 × 3




two matrices of same order then matrix addition is commutative
means: (A + B) = (B + A). Hence, option (c) is the correct answer.
2. Addition of matrices is associative: Take three matrices Example 9. A matrix X 2×3 if 5A – 7B + 3X = 0, where
A, B and C of same order then matrix addition is associative  2 1 3 0 9 1
means: (A + B) + C = A + (B + C). A=  and B =   is given by
 1 4 7  5 3 5
3. Existence of additive identity of a matrix: Let A and B
be two matrices of same order m × n, then the matrix B will be  30 5 1  30 1 0
(a)   (b)  
called additive identity of matrix A if (A + B) or (B + A) is equal  −10 −58 5
 −10 5 8






to matrix A. This is possible only when matrix B is a zero matrix
of same order as matrix A. Hence, every matrix A has its additive 1  −10 58 −8 1  10 18 8
(c) (d)
identity matrix and it is equal to zero matrix of the same order. 3  30 1 0 2  −2 6 5





4. Existence of additive inverse of a matrix: If sum of two
matrices of same order is zero matrix, then each matrix is called 2 1 3 0 9 1
Solution. It is given that A =  ,B =   and
additive inverse of the other matrix. 5A – 7B + 3X = 0 1 4 7   5 3 5
Let A and B be two matrices of same order and A + B = B + A ⇒ 3X = 7B – 5A



= 0. Then matrix A is additive inverse of matrix B and matrix B is 0 9 1 2 1 3
additive inverse of matrix A. If A + B = 0 then B = –A and A = –B. ⇒ 3X = 7   −5
 5 3 5 1 4 7 



\ For any matrix A, matrix (–A) is its additive inverse.

 0 63 7  10 5 15 
(E) Subtraction of Matrices ⇒ 3X =  − 
35 21 35  5 20 35



If A = [aij]m×n and B = [bij]m×n be two matrices of same order m
 −10 58 −8
× n, then their difference (A – B) is a matrix of same order m × ⇒ 3X =  
 30 1 0




n and it is obtained by subtracting elements of matrix B from the
corresponding elements of matrix A. 1  −10 58 −8
⇒ X =
⇒ (A – B) = [aij]m×n – [bij]m×n ⇒ (A – B) = [aij – bij]m×n 3  30 1 0







The above definition suggests that a matrix B of order m × n can Hence, option (c) is the correct answer.
be subtracted from a same order matrix A and the resultant matrix  x2   x   −2
Example 10. Values of x and y when:  2  − 3   =   are
(A – B) also has same order m×n. y   2y   −9
given by
4 1 2 (a) x = 1, 2; y = 3 (b) x = 2, 3; y = –3
Example 8. If matrix A =   and matrix B =





 1 0 5 2×3 (c) x = –1, 2; y = 3 (d) x = 1, –2; y = 2




 1 −5 4  x2   x   −2
4 , then value of (5A – 3B) will be Solution. Given equation is:  2  − 3   =  
 3 − 4  2×3 y   2y   −9
7 20 21  3 7 21  x 2   3x   −2
(a)   (b)   ⇒  2 −   =  
 3 7 −9  20 3 −6  y   6y   −9








 17 20 −2  20 17 −2  x 2 − 3x   −2   x 2 − 3x = −2
(c)   (d)   ⇒  2  =   ⇒  2
 −7 −9 37   9 7 37   y − 6y   −9   y − 6y = −9







4 1 2  1 −5 4  x 2 − 3x + 2 = 0 ( x − 2) ( x − 1) = 0
Solution. We are given A =   and B = 4 ⇒  2 ⇒ 
3 − 4  2×3  ( y − 3) = 0
2
 1 0 5 2×3  y − 6y + 9 = 0




 4 1 2  1 −5 4  x = 1, 2
⇒ (5A – 3B) = 5  − 3 ⇒ 

 1 0 5 2 × 3 4 3 − 4 2 × 3 y = 3




Hence, option (a) is the correct answer.

48 Mathe atics–12
m
E:\AMIT_WORKS\Exam_Guru\EG_Mathematics-12_(working_02-06-2022)\EG_Mathematics-12_working\Open_Files\Chap_3\Chap_3
\ 16-Aug-2022  Amit   Proof-4 Reader’s Sign _______________________ Date __________


(F) Multiplication of a Matrix with Another Matrix Sum of the Sum of the 
The product AB of two matrices A and B is defined only if number  products of the product of the 
of columns of first matrix A is equal to number of rows of second corresponding corresponding 
 
matrix B. elements of first elements of first 
If matrix A = [aij]m×n has order m × n and another matrix B  row of matrix A row of matrrix A 
= [bjk]n×p has order n × p, then the product AB is defined because and first column and second column 
of matrix B. of the matrix B. 
number of columns of matrix A is n and number of rows of matrix
AB =  
B is also n. Sum of the Sum of the 



The product AB is defined as: AB = [aij]m×n × [bjk]n×p = [cik]  productts of the products of 
corresponding corrresponding 
m×p . elements of second 
To get the element cik of the matrix AB, we take ith row 
elements of second 
 row of matrix row of matrix A 
of matrix A and kth column of the matrix B and multiply them
 A and first column and second coolumn 
elementwise and take the sum of all the product. of matrix B.  2×2
of matrix B
In matrix A = [aij]m×n, ith row is [ai1 ai2 ai3 ... ain] and




in matrix B = [bik]n×p, the kth column is:
⇒ A B =
 b1k 



b   ( 2 × 1) + (1 × −1) + (3 × 0) ( 2 × 2) + (1 × 4) + (3 × 5) 

 2k 
b3k  ( −1 × 1) + ( 4 × −1) + ( 0 × 0) ( −1 × 2) + ( 4 × 4) + (0 × 5) 2× 2
   2 −1+ 0 4 + 4 + 15 
   ⇒ AB =  
bnk 
 −1 − 4 + 0 −2 + 16 + 0 2 × 2



Then (ik)th element of matrix AB = (ik)th element of matrix C  1 23
⇒ AB =  
cik = ai1b1k + bi2b2k + ... ainbnk ⇒ cik = (aijbjk), j = 1, 2, ....n  −5 14  2 × 2



 2 1 3 Hence, option (a) is the correct answer.
Example 11. If matrix A =   and matrix B =
 −1 4 0  2×3 Properties of Matrix Multiplication
 1 2 1. Matrix multiplication is not commutative:
 −1 4  , then AB will be given by


(a) Let A and B be two matrices such that AB is defined,
 


 0 5 3×2 then it may be possible that BA is not defined. For
example, if matrix A has order 3 × 3 and matrix B has
 1 23  23 −5 order 3 × 2, then the product AB is defined but product
(a)   (b)   BA is not defined. Hence, equality AB and BA does
 −5 14  1 14





not exists.
14 5  23 14 (b) Let A and B be two matrices such that both products
(c)   (d)  


 −1 23  11 −1 AB and BA are defined. Now it may be quite possible




that the order of the matrix AB is not equal to order of
 2 1 3 matrix BA. For example, if order of matrix A is 3 × 2
Solution. We are given two matrices: A =  and
 −1 4 0  2×3 and order of matrix B is 2 × 3. Then both the product
 1 2 AB and BA are defined but the order of AB is 3 × 3
B =  −1 4  and order of BA is 2 × 2. Hence AB ≠ BA.
 0 5 3×2 (c) Even if the orders of the matrix (AB) and (BA) are


equal, the two products are generally not equal.
Here matrix A has 3 columns and matrix B has 3 rows, hence
(d) The meaning of the above discussion is not that the

the product AB is defined. The order of the product matrix AB


matrix AB can never be equal to matrix BA. In some
will be 2 × 2. cases we have AB = BA.
 1 2 2. Matrix multiplication is associative: If A, B and C be
 2 1 3  −1 4


AB =   × three matrices of such orders that the products, AB, (AB)
 −1 4 0  2 × 3  


C, (BC) and A(BC) are defined, then matrix multiplication
 0 5 3× 2
is associative, means:
(AB)C = A(BC).

3. Matrix multiplication is distributive over addition


and subtraction: If A, B and C have three matrices of
such orders that (B + C), (B – C), AB, AC, BA and CA

Mat ices 49
r

E:\AMIT_WORKS\Exam_Guru\EG_Mathematics-12_(working_02-06-2022)\EG_Mathematics-12_working\Open_Files\Chap_3\Chap_3
\ 22-Aug-2022  Amit   Proof-4 Reader’s Sign _______________________ Date __________


are defined, then matrix multiplication is distributive over 9 8 17 −5
addition and subtraction means: (c)   (d)  
 3 −4  11 −9





(i) A(B + C) = AB + AC (ii) (B + C)A = BA + CA




(iii) A(B – C) = AB – AC (iv) (B – C)A = BA – CA  4 5  8 3 
Solution. It is given that: A  = ...(1)




4. Existence of multiplicative identity: For every square 
 2 3 10 4




matrix A, there exists an identity matrix I of same order,
such that: AI = IA a b 
Take A =   and put it in (1), we get
1 0 0 c d 



1 0  
I1×1 = [1] , I 2 × 2 =  , =
 3× 3  0 1 0 
I  a b   4 5  8 3
 0 1  0 0 1   c d   2 3 = 10 4
    




Matrix Polynomial  4a + 2b 5a + 3b   8 3
3 2
If f (x) = ax + bx + cx + d is an algebraic polynomial and A is
⇒  4c + 2d 5c + 3d  = 10 4
   





a square matrix, then:
4a + 2b = 8 4c + 2d = 10
f (A) = aA3 + bA2 + cA + dI is called matrix polynomial. ⇒  and 

5 a + 3 b = 3 5c + 3d = 4



Here unit matrix I and square matrix A are of equal orders.
3 1 On solving for a and b, we get a = 9, b = – 14


2
Example 12. If matrix A =   and f (x) = 3x – 4x + 7, then


2 4 On solving for c and d, we get c = 11, d = –17

f (x) is equal to  


 a b   9 −14 

 11 17   28 17 
\ A=  = 

 c d  11 −17 



(a)   (b)  
 23 24 34 45




Hence, option (a) is the correct answer.

14 17  11 23
(c)   (d)   (G) Inverse of a Square Matrix
13 12 12 17 




If A and B are two non-zero square matrices of same order such
that AB = BA = I, then matrix A is called inverse of matrix B
3 1 2
Solution. It is given that A =   and f (x) = 3x – 4x + 7. and matrix B is called inverse of matrix A. Inverse of matrix A is
 2 4

denoted as A–1. Here A and A–1 are inverse of each other.
3 1 3 1 3 1 \AA–1 = A–1 A = I
\ f (A) = 3A2 – 4A + 7I = 3   2 4 – 4 2 4


2 4 Also AI = IA = A and similarly A–1I = IA–1 = A–1

    



1 0  Invertible Matrix
+7   If a matrix A has inverse, it is called invertible matrix.
0 1 
 2 3  2 −3
9 + 2 3 + 4  3 1 1 0 Example 14. If matrix A =   and matrix B =   , then
= 3  − 4  +7   1 2   −1 2
 6 + 8 2 + 16  2 4 0 1


(a) AB ≠ BA


11 7  3 1 1 0 (b) matrices A and B are inverse of each other
= 3  − 4  2 4 + 7  0 1 


14 18      [V. Imp.]


(c) AB = BA = O (d) None of these





 33 21 12 4  7 0   2 3  2 −3
=  − +  Solution. It is given that A =   , B=  
 42 54   8 16   0 7  1 2  −1 2


33 − 12 + 7 21 − 4 + 0   28 17   2 3  2 −3  4 − 3 −6 + 6 
=  =  \ AB =   = 
 42 − 8 + 0 54 − 16 + 7  34 45  1 2   −1 2   2 − 2 −3 + 4 





2  28 17   1 0
f (A) = 3A − 4A + 7I = 
\  =  =I ...(1)
34 45 0 1






Hence, option (b) is the correct answer.  2 −3  2 3  4 − 3 6 − 6 
Also BA =  =

 4 5  8 3   
 −1 2   1 2   − 2 + 2 −3 + 4 



Example 13. A matrix A2×3, when A  =  , is
 2 3 10 4 1 0 
 9 11
=  =I ...(2)
 9 −14 0 1 


(a)   (b)  
11 −17  7 −3 From (1) and (2), we get AB = BA = I.






50 Mathe atics–12
m
E:\AMIT_WORKS\Exam_Guru\EG_Mathematics-12_(working_02-06-2022)\EG_Mathematics-12_working\Open_Files\Chap_3\Chap_3
\ 16-Aug-2022  Amit   Proof-4 Reader’s Sign _______________________ Date __________


\ Matrices A and B are inverse of each other.
1  3 −1
A–1 =



Hence, option (b) is the correct answer. 13  −2 5





 5 1
Example 15. If matrix A =  , and A2 – 8A + 13I = 0, then Hence, option (c) is the correct answer.
2 3


–1
value of A will be 
 5 1 2
Example 16. If matrix A =   , and A – 8A + 13I = 0, then
1 5 −1 1  3 5  2 3
(a) (b)
13 3 7  13  −1 −2 value of A3 will be





1  3 −1 1  −1 5  51 100 151 51
(c) (d) (a)   (b) 
13  −2 5 13  7 3  49 102

102 49









 5 1 102 49  49 50
Solution. It is given that A =  (c)   (d)  
  51 100 150 51





 2 3
and A2 – 8A + 13I = 0 ...(1)  5 1
Solution. It is given that A =  



–1 –1
To find A , multiply by A on both sides of equation (1), we get  2 3


A–1[A2 – 8A + 13I] = A–1 × 0 Take: A2 – 8A + 13I = 0




⇒ A–1A2 – 8A–1A + 13A–1I = 0 ⇒ A2 = 8A – 13I ...(1)





3



To find A , multiply by A on both sides of eqn. (1), we get
⇒ (A–1A)A – 8(A–1A) + 13(A–1) = 0


A3 = 8A2 – 13A




⇒ IA – 8I + 13A–1 = 0




⇒ A3 = 8[8A – 13I] – 13A



[Q A–1A = I and A–1I = A–1]




[Using (1)]
⇒ A – 8I + 13A–1 = 0 ⇒ 3
A = 64A – 104I – 13A






[Q IA = A] ⇒ A3 = 51A – 104I



⇒ 13A–1 = 8I – A  5 1 1 0
A3 = 51 



⇒  − 104  
 1 0   5 1  2 3 0 1



⇒ 13A–1 = 8 − 
0 1   2 3



 255 51  104 0 
 8 0   5 1
⇒ A3=  − 
102 153  0 104



⇒ 13A–1 =  − 
0 8   2 3



151 51
A3 = 
⇒ A3 = 
102 49
 3 −1



⇒ 13A–1 =  
 −2 5 Hence, option (b) is the correct answer.




EXERCISE 3.1
I. Multiple Choice Questions (MCQs) 1  5 7 1  2 −3
(a) − (b) −
Choose the correct answer from the given options. 11  −3 2 11  −5 2





15 x + y   15 8 1  2 5 1  1 5
1. If  = , then the value of x will be (c) − (d)
2 y   x − y 3 11  −3 2 11 7 8






(a) 6 (b) 7  1 −1
4. If matrix A =  and A2 = kA, then the value of k





 −1 1


(c) 5 (d) 8 is equal to




1 2  3 1  7 11 (a) 4 (b) 2




2. If   =  , then the value of x will be (c) 5 (d) 6
3 4  2 5  x 23






5. The value of (x – y + z) from the following equation:


(a) 17 (b) 18  x + y + z  9




 x + z  =  5 is given by
(c) 19 (d) 5    




 y + z  7 
 2 3
3. If A =   and AB = I, then matrix B is equal to (a) 4 (b) 1
 5 2






(c) 2 (d) 3




Mat ices 51
r

E:\AMIT_WORKS\Exam_Guru\EG_Mathematics-12_(working_02-06-2022)\EG_Mathematics-12_working\Open_Files\Chap_3\Chap_3
\ 16-Aug-2022  Amit   Proof-4 Reader’s Sign _______________________ Date __________


0 3  0 4a  α β  2 2
6. If A = 
2 −5  and kA =  −8 5b  , then value of k will be 9. If A = 
γ −α  is such that A = I, then show that 1 – a
     





(a) 4 (b) –4 – bg = 0.




(c) 5 (d) –5 cos θ − sin θ 
10. If f (θ ) =  , prove that




 sin θ cos θ 



 −1 0 −1  1
7. If [ 2 1 3]  −1 1 0  0 = A , then order of the cos (θ + φ ) − sin (θ + φ )
   f (θ ) f ( φ ) =   = f (θ + φ ) .


 0 0 1  −1  sin (θ + φ ) cos (θ + φ )


matrix A will be
 cos θ sin θ 
(a) 1 × 1 (b) 2 × 1 11. If f (θ ) =   , show that f (q). f (–q) = I.



 − sin θ cos θ 







(c) 1 × 2 (d) 2 × 2
 2x + y + z   8 




 2 3
 1 −2 3   12. Solve for x, y and z if:  3y + 2z  = 10
8. If A =     
 and B =  4 5 and BA = [bij], then



−4 2 5  5z  10


   2 1
b21 + b32 equals III. Short Answer Type Questions-II
(a) 18 (b) 17 1 2 2
1. If A =  2 1 2 , verify: A2 – 4A – 5I = 0.




(c) –18 (d) –17  







II. Short Answer Type Questions-I  2 2 1 

 1 0  2 0 1
2. If A =  2 1 3 , find A2 – 5A + 4I and hence find the
2
1. If A =   , find k if A – 8A + kI = 0.
 −1 7  




 1 −1 0
1 2 3  1 matrix X such that A2 – 5A + 4I + X = 0. [Delhi 2015]
  
2. If [1 x 1]  4 5 6  −2 = 0 , find the value of x.


 1 2  x 


 3 2 5  3 3. If [ 2x 3]     = 0 , find x. [Delhi 2015]
 −3 0  3 



3 2  2 4. Construct a matrix:
3. If A =   , find the values of a and b such that A +


1 1 


 2i − 3j 
(i) A =  aij  , when aij = 
aA + bI = 0. 3× 3  2i + 3j 


4. If x and y are matrices of order 2 × 3, solve for x and y.
(ii) A =  aij  , when aij = 3i + 2 j


  1 2 3 3× 3


x + y =  
 3 1 4  7i − 4j if i < j
 
(iii) A =  aij  , when aij =  11 if i = j
 x − y = 3 0 1 3× 3


  1 5 6 5i + 7j if i > j
   
 3i + j if i < j

 cos θ sin θ   sin θ − cos θ  
5. Simplify: cos θ   + sin θ cos θ  
(iv) A =  aij  , when aij = 2i + 3j if i = j
 − sin θ cos θ   sin θ  3× 3




 i + 3j if i > j

cos θ + sin θ 2 sin θ  3i + j if (i + j) is even
6. If A =   , then using PMI prove (v) A =  aij 
, when aij =  i + j
 − 2 sin θ cos θ − sin θ 


3× 3
 2 if (i + j) is odd


that:
5. If x and y each is a matrix of order 2 × 3, solve for x and y
cos n θ + sin n θ 2 sin n θ 


An =    1 − 1 2
 − 2 sin n θ cos n θ − sin n θ   5x + 2y =  
[HOTS]  3 4 7 



7. Construct a matrix of order 3×3 when its general element 3x − 5y = 3 1 5


is aij = 3i + 2j.  5 2 3
  
 3 2 0

   3
8. If A = 1 4 0 , show that A2 – 7A + 10I = 0.
6. If A = 1  and B = [1 2 3], find (i) AB and (ii) BA.



0 0 5  


 2

52 Mathe atics–12
m
E:\AMIT_WORKS\Exam_Guru\EG_Mathematics-12_(working_02-06-2022)\EG_Mathematics-12_working\Open_Files\Chap_3\Chap_3
\ 16-Aug-2022  Amit   Proof-4 Reader’s Sign _______________________ Date __________


0 1 8. Find a matrix [A]2×2 if



7. If matrix A =   , show that:  1 −1 4  7 3 16
 0 0 (i) A ×  =


 
 2 3 1  −7 −13 −4



(i) (aI + bA)2 = a2I + 2abA


(ii) (aI + bA)3 = a3I + 3a2bA  2 −1  −1 −8


 1 0  A =  1 −2
 a − 1 1 1 (ii)     [AI 2017]
2 2 2
8. If A =  and B =   and (A + B) = A + B ,




2 − 1 b − 1  −3 4   9 22
   


then find the values of a and b. 1 2 2
9. Find the condition for which: 9. If A =  2 1 2 , verify : A2 – 4A – 5I = 0 and hence
 





(i) (A + B) (A – B) = A2 – B2  2 2 1 


(ii) (A + B)2 = A2 + B2 find A–1.


 0 1 2  3 8
10. If A =   find the values of x and y when (xI + yA) = A. 10. If A =  2
 − 1 0   and A – mA + nI = 0, find the values of m


 2 1



 0 α and n and hence find A–1.
− tan 
 2
11. If A =   and I is the identity matrix of 11. To promote the making of toilets for women, an
α


 tan 0 



 2  organisation tried to generate awareness through (i) house


order 2 × 2, then show that: calls (ii) letters, and (iii) announcements.
The cost for each mode per attempt is given below:
cos α − sin α 
(I + A) = (I − A)   [V. Imp.] (i) ` 50 (ii) ` 20
 sin α cos α 






(iii) ` 40


IV. Long Answer Type Questions The number of attempts made in three villages X, Y and

Z are given below:
 cos θ sin θ   cos φ sin φ 
1. If A = 
− sin θ cos θ  and B =  − sin φ cos φ  then (i) (ii) (iii)
   


X 400 300 100
 cos (θ + φ) sin (θ + φ) 
show that: AB =   Y 300 250 75
 − sin (θ + φ) cos (θ + φ)  Z 500 400 150
cos θ + sin θ 2 sin θ  Find the total cost incurred by the organisation for the three
2. If f (θ ) =   , then show that:

villages separately, using matrices. [A.I. 2015]
 − 2 sin θ cos θ − sin θ 



(i) [ f (q)]2 = f (2q) (ii) f (q) f (f) = f (q + f)  2 −1  5 2  2 5
12. If A =   , B = 7 4 , C =  3 8 , find matrix D










(iii) f (q) f (–q) = I  3 4     






 cos 2 θ such that CD – AB = 0. [Delhi 2017]
sin θ cos θ 
3. If f (θ ) = 

 , then show that:
sin θ cos θ sin 2 θ   2 1   −3 2 1 0


13. Find the matrix A:   A = 
[ f (q)]2 = f (q).  3 2  5 −3 0 1 





 sin 2 θ sin θ cos θ  14. Three schools A, B and C organized a mela for collecting
4. If f (θ ) = 


 , then show that: funds for helping the rehabilitation of flood victims.
sin θ cos θ cos 2 θ 


π They sold hand made fans, mats and plates from recycled
f (q) f (f) = 0 where (q – f) is an odd multiple of . [V.Imp.]
2 material at a cost of ` 25, ` 100 and ` 50 each. The number




 1 tan θ   1 − tan θ  of articles sold are given below:
5. Show that: 
− tan θ 1   tan θ 1
2
 = sec θ I ( ) School → A B C


   [V. Imp.]

Articles ↓
2 1 1
  Hand Fans 40 25 35
6. If A = 1 2 1  and f (x) = 2x2 – 5x + 7, find f (A).
Mats 50 40 50


1 1 2
Plates 20 30 40
7. Find a matrix [A]2×2 if



Find the funds collected by each school separately by
1 2  2 5 1 2  2 5

(i) A   =  (ii)   A=  selling the given articles. Also find the total funds collected
3 4   − 4 7  3 4   −4 7 




for the purpose. [Delhi 2015]

Mat ices 53
r

E:\AMIT_WORKS\Exam_Guru\EG_Mathematics-12_(working_02-06-2022)\EG_Mathematics-12_working\Open_Files\Chap_3\Chap_3
\ 16-Aug-2022  Amit   Proof-4 Reader’s Sign _______________________ Date __________


15. A trust fund, ` 35,000 is to be invested in two different types has 5 poor and 25 rich children and total monthly collection


of bonds. The first bond pays 8% interest per annum which is ` 26,000. Using matrix method find the monthly fees


will be given to orphanage and second bond pays 10% paid by each child of two types. [Foreign 2016]


interest per annum which will be given to an NGO (Cancer 17. A trust invested some money in two type of bonds. The first



Aid Society). Using matrix multiplication, determine how bond pays 10% interest and second bond pays 12% interest.
to divide ` 35,000 among two types of bonds if the trust

The trust received ` 2,800 as interest. However, if trust had


fund obtains an annual total interest of ` 3,200. [A.I. 2015] interchanged money in bonds, they would have got ` 100 less



16. A coaching institute of English (subject) conduct classes in as interest. Using matrix method, find the amount invested


two batches I and II and fees for rich and poor children are by the trust. Interest received on this amount will be given
different. In batch I, it has 20 poor and 5 rich children and to Helpage India as donation. [A.I. (Punchkula) 2016]


total monthly collection is ` 9,000, whereas in batch II, it

Answers 3.1
I. 1. (c) 5.
 −10 2 21

15 x + y   15 8  
Hint:  = ⇒ BA =  −16 2 37 
y   x − y 3




2  −2 −2 11

⇒ y = 3 and x + y = 8 b21 + b32 = –16 – 2 = –18




⇒ x=5 II. 1. k = 7


2. (a) 17  1 0
Hint: A= 




1 2  3 1 7 11  −1 7 



Hint:  3 4   2 5 =  x
    23 \ A2 – 8A + kI = 0





 3 + 4 1 + 10  7 11  1 0  1 0  1 0 1 0
⇒ 9 + 8 3 + 20 =  x 23
⇒  −1 7   −1 7  − 8  −1 7  + k 0 1 = 0
       


 



 7 11  7 11  1+ 0 0 + 0   8 0  k 0
⇒ 17 23 =  x 23 ⇒ x = 17 ⇒  −1 − 7 0 + 49 −  −8 56 +  0 k  = 0
   



     


 2 −3
3. (b) − 1   1 0   8 0  k 0
11  −5 2 ⇒  −8 49 −  −8 56 +  0 k  = 0



     


4. (b) 2



 1 −1  1 −1  2 −2  −7 0   k 0  0 0
Hint: A2 =   =  ⇒  0 −7  +  0 k  = 0 0
 −1 1  −1 1  −2 2      





 1 −1 ⇒ –7 + k = 0 ⇒ k=7
= 2




 1 2 3  1
 −1 1


⇒ k= 2 2. [1 x 1]  4 5 6  −2 = 0





5. (b) 1  3 2 5  3



x = 2, y = 4, z = 3 ⇒ x–y+z=1  1



6. (b) –4 ⇒ [1 + 4x + 3 2 + 5x + 2 3 + 6x + 5]  −2 = 0





7. (a) 1 × 1  3



Hint: A1×3B3×3C3×1 = A1×3(BC)3×1 = (ABC)1×1.
 1

8. (c) –18
⇒ [ 4 x + 4 5 x + 4 6 x + 8 ]  −2 = 0


 2 3


 1 −2 3    3
Hint: A=   and B =  4 5
 −4 2 5 ⇒ [4x + 4 – 10x – 8 + 18x + 24] = 0



 2 1


⇒ [12x + 20] = 0 ⇒ 12x + 20 = 0




 2 3 −20 −5
   1 −2 3 ⇒ x=
12
=
3
\ BA =  4 5 



 −4 2 5



 2 1 3. a = –4, b = 1




 2 − 12 −4 + 6 6 + 15  3 2  2
Hint: A =   and A + aA + bI = 0
  1 1 


⇒ BA =  4 − 20 −8 + 10 12 + 25



 2 − 4 −4 + 2 6 + 5  \ A2 + aA + bI = 0


54 Mathe atics–12
m
E:\AMIT_WORKS\Exam_Guru\EG_Mathematics-12_(working_02-06-2022)\EG_Mathematics-12_working\Open_Files\Chap_3\Chap_3
\ 16-Aug-2022  Amit   Proof-4 Reader’s Sign _______________________ Date __________


3 2   3 2  3 2  1 0 α β 2
⇒ 1 1  1 1  + a 1 1  + b 0 1 = 0 9. Hint: A = 
γ −α  and A = I
        





9 + 2 6 + 2 3a 2a  b 0 α β α β  1 0
⇒  3 + 1 2 + 1 +  a a  + 0 b  = 0 ⇒  γ −α   γ −α  = 0 1
     




    
11 + 3a + b 8 + 2a + 0 0 0  α 2 + βγ αβ − αβ  1 0
⇒  4+a+0 =
 3 + a + b  0 0 ⇒  2
= 


 γα − γα βγ + α  0 1



⇒ 11 + 3a + b = 0, 2a + 8 = 0
α 2 + βγ 0  1 0


a+4=0 a+b+3= 0 ⇒  = 
 0 βγ + α 2  0 1



⇒ a = –4, b = 1.
⇒ a2 + bg = 1


 2 1 2  −1 1 1



4. x =   and y =   ⇒ 1 – a2 – bg = 0
 2 3 5  1 −2 −1





cos θ − sin θ 
10. Hint: f (θ ) =  
1 2 3  sin θ cos θ 




Hint: x+ y =  
3 1 4 


cos θ − sin θ  cos φ − sin φ 
3 0 1  ⇒ f (θ ) f ( φ ) =   
x− y =   sin θ cos θ   sin φ cos φ 




1 5 6
4 2 4   cos θ cos φ − sin θ sin φ − cos θ sin φ − sin θ cos φ 
2x =  = 

 4 6 10 sin θ cos φ + cos θ sin φ − sin θssin φ + cos θ cos φ 

 2 1 2 cos (θ + φ ) − sin (θ + φ )
⇒ x= =  = f (θ + φ )

 2 3 5  sin (θ + φ ) cos (θ + φ ) 



1 2 3 1 2 3  2 1 2  cos θ sin θ 
y=  −x= −  11. Hint: f (θ ) =  
3 1 4  3 1 4   2 3 5   − sin θ cos θ 



 −1 1 1  cos θ sin θ  cos θ − sin θ 
⇒ y=  f (θ ) f ( −θ ) =   
 1 −2 −1  − sin θ cos θ   sin θ cos θ 



1 0  cos 2 θ + sin 2 θ − sin θ cos θ + sin θ cos θ 
5.   = 
0 1  − sin θ cos θ + sin θ cos θ sin 2 θ + cos 2 θ 



 cos θ sin θ   sin θ − cos θ  1 0
Hint: cos θ   + sin θ cos θ = =I
 − sin θ cos θ   sin θ  0 1


 cos 2 θ sin θ cos θ   sin 2 θ − sin θ cos θ  12. x = 2, y = 2, z = 2.
=  + 


 − sin θ cos θ
2
cos θ  sin θ cos θ sin 2 θ  Hint: Try yourself.


1 2 2
cos 2 θ + sin 2 θ 0  1 0
= 2 2 
=  III. 1. Hint: Put A =  2 1 2 in A2 – 4A – 5I = 0 and verify it.
0 cos θ + sin θ  0 1

  2 2 1 
6. Try yourself.
 −1 −1 −3 1 1 3


5 7 9
2. A2 – 5A + 4I =  −1 −3 −10 and X = 1 3 10 
7. A =  8 10 12


   −5 4 2 5 −4 −2


11 13 15
Hint: A = [aij]3×3 and aij = 3i + 2j Here X = –[A2 – 5A + 4I].


Try yourself.
 a11 a12 a13 

−3
A =  a21 a22 a23  3. x = 0,
2
. Try yourself.



 a31 a32 a33 
 −1 −1 −7 
3 ×1+ 2 ×1 3 ×1+ 2 × 2 3 ×1+ 2 × 3  5 7 9  5 2 11 
1 −5 
= 3 × 2 + 2 × 1 3 × 2 + 2 × 2 3 × 2 + 2 × 3 =  8 10 12 4. (i) A =  −1
7 5 13 



 3 × 3 + 2 × 1 3 × 3 + 2 × 2 3 × 3 + 2 × 3 11 13 15
1 −1 
 3 0
5 

8. Hint: Put the values of A in A2 – 7A + 10I and now get the result.


Mat ices 55
r

E:\AMIT_WORKS\Exam_Guru\EG_Mathematics-12_(working_02-06-2022)\EG_Mathematics-12_working\Open_Files\Chap_3\Chap_3
\ 16-Aug-2022  Amit   Proof-4 Reader’s Sign _______________________ Date __________


 2i − 3j 
Hint: A = [aij]3×3 and aij =  1 11 −3 20 = 1  −12 −8 −29
 2i + 3j  5. x = ,y
31  25 24 41 31  −16 2 6 



 a11 a12 a13  Hint: See the solution of Q.No. 4 (short answer).


   3 6 9
⇒ A =  a21 a22 a23 
6. AB = 1 2 3 , BA = [11]



 a31 a32 a33 



Now put the values of i and j and get the answer.  2 4 6

 5 7 11 0 1
7. Hint: A =  
(ii) A = 11 13 17   0 0





 29 31 35 (aI + bA)2 = a2I2 + b2A2 + 2aIbA



Hint: A = [aij]3×3 and aij = 3 + 2 i j = a2I + b2A2 + 2abA






 a11 a12 a13  [Q I2 = I and IA = A]
  = a2I + b2 × 0 + 2abA
⇒ A =  a21 a22 a23 






 a31 a32 a33   2 0 1 
= a2I + 2abA Q A =   = 0
  0 0  



Now put the values of i and j in aij and get the answer.

8. Hint: a = 1, b = 4
11 −1 −5



(A + B)2 = A2 + B2
(iii) A = 17 11 2 



­
⇒ A + B + AB + BA = A2 + B2 ⇒ AB = –BA
2 2


 22 29 11 



 a −1  1 1  a − b a + 1
AB =   b −1 =  2 − b
7i − 4j if i < j
 2 −1   3 



Hint: A = [aij]3×3 and aij =  11 if i = j
1 1   a −1

5i + 7j if i > j
 –BA = −   
b −1  2 −1


 a11 a12 a13 
   a+2 −2   −a − 2 2 
⇒ A =  a21 a22 a23  = −  = 
ab − 2 −b + 1 
 −ab + 2 b − 1



 


 a31 a32 a33 
Now put the values of i and j in aij and get the answer. ⇒ AB = –BA




\ a+1=2 ⇒ a=1
5 5 6 





3=b–1 ⇒ b=4
(iv) A = 5 10 9 




9. (i) AB = BA, (ii) AB = –BA


6 9 15



(i) Hint: (A + B)(A – B) = A2 – B2
 3i + j if i < j




 ⇒ A2 – AB + BA – B2 = A2 – B2
Hint: A = [aij]3×3 and aij = 2i + 3j if i = j



⇒ –AB + BA = 0 ⇒ AB = BA

 i + 3j if i > j




 (ii) (A + B)2 = A2 + B2



 a11 a12 a13  ⇒ A2 + B2 + AB + BA = A2 + B2
 



⇒ A =  a21 a22 a23  ⇒ AB + BA = 0 ⇒ AB = –BA







 a31 a32 a33 
10. x = ± 1 ; y = ± 1
2 2


Now put the values of i and j in aij and get the answer.

 0 1
 and ( xI + yA ) = A
2
 9 8 81  A=
−1 0
(v) A =  8 81 32   



81 32 729 ⇒ x2I2 + y2A2 + 2xyAI = A


Hint: A = [aij]3×3 and ⇒ x2I + y2A2 + 2xyA = A [Q I2 = I and AI = A]




1 0  0 1  0 1  0 1  0 1
3i + j if
aij =  i + j
(i + j ) is even ⇒ x2   + y2     +2xy  = 
0 1  −1 0  −1 0  −1 0  −1 0


2 if (i + j ) is odd
 x2 0  −1 0  0 2xy   0 1

 a11 a12 a13  ⇒  2
+ y2   +  −2xy 0  =  −1 0
  0 x   0 −1    


Take A =  a21 a22 a23 



 a31 a32 a33   x2 0   −y 2 0   0 2xy 
⇒  + +
2   −2xy
0 x2   0 −y   0 


Now put the values of i and j in aij and get the answer.

56 Mathe atics–12
m
E:\AMIT_WORKS\Exam_Guru\EG_Mathematics-12_(working_02-06-2022)\EG_Mathematics-12_working\Open_Files\Chap_3\Chap_3
\ 16-Aug-2022  Amit   Proof-4 Reader’s Sign _______________________ Date __________


 0 1  0 α
− tan 
=  ⇒ x2 – y2 = 0 and 2xy = 1. 1 0  2
 −1 0 LHS = I + A =  +



0 1   tan α


0 
1  2 
Put y = in x2 – y2 = 0
2x

 1 α
− tan 
⇒ x − 12 = 0
2 4
⇒ 4x – 1 = 0 
= 
2
 = RHS
4x
 tan α






1 
 2 
⇒ x=± 1
2 IV. 1. Try yourself. 2. Try yourself.




3. Try yourself.
1
Since x2 = y2



⇒ y=±
2  sin 2 θ sin θ cos θ 
4. Hint: f (θ ) = 




sin θ cos θ cos 2 θ 



 0 α
− tan  f (q) f (f)
 2




11. A =  


 tan α  sin 2 θ sin θ cos θ   sin 2 φ sin φ cos φ 


0  =
 2  2  
sin θ cos θ cos θ  sin φ cos φ cos 2 φ 


cos α − sin α  On multiplying the above two matrices, we get
RHS = ( I − A )  


 sin α cos α 

 sin θ sin φ sin θ cos φ 
cos (θ − φ )  
1 − tan 2 α cos θ sin φ cos θ cos φ 
  − tan α   −2 tan α 
 1 0  0 2  2 2 

=   − If (q – f) = odd multiple of /2 then cos(q – f) = 0
  2α 
1 + tan 2 α 
 0 1  tan α 1 + tan

p
 0    2 2  sin θ sin φ sin θ cos φ 
 2    = 0
α =0
 2 tan 1 − tan 2 α  cos θ sin φ cos θ cos φ 

 2 2

1 + tan 2 α 1 + tan 2 α   1 tan θ   1 − tan θ 
 2 2 5. Hint: LHS =    tan θ
 − tan θ 1  1 


1 − tan 2 α −2 tan α   1 + tan 2 θ − tan θ + tan θ 
 2 2  = 
 1 tan α  
1 + tan 2α 2α
1 + tan   − tan θ + tan θ 1 + tan 2 θ 
 2 2 2

=   sec2 θ 0  1 0
 − tan α 1   2 tan α 1 − tan 2α
= = sec2 θ  2
 = sec θ I ( )
 2  2 2 2  0 1
 0 sec θ   
1 + tan 2 α 1 + tan 2 α 

 2 2 6. Try yourself.



 1 − tan 2 α + 2 tan 2 α 1  7 −1 
−2 tan α + tan α − tan 3 α  7. (i) A=
 2 2 2 2 2 2 37 −15




 2α 
 1 + tan 1 + tan 2 α 
2 2
=  1 2  2 5
 − tan α + tan 3 α + 2 tan α 2 tan 2α
+ 1 − tan 2α  A  =  
 2 2 2 2 2  3 4   −4 7 


 1 + tan 2 α 1 + tan 2 α 
 2 2  −1 −1
1 2 1 2  2 5 1 2

A   =   
3 4  3 4   −4 7  3 4


 2α − tan α 1 + tan 2 α  
 1 + tan 2 2  2 

 1 + tan 2

1 + tan 2α
2


 2 5 −1  4 −2
AI =    ( )

 −4 7  2  −3 1 


= 


2 (
 tan α 1 + tan 2 α
2 ) 1 + tan 2α
2


 A= ( )
−1  2 5   4 −2
2α 2α 2  −4 7   −3 1 
 1 + tan 2 1 + tan


2 

( )
−1  8 − 15 −4 + 5

A=
 1 α
− tan  2  −16 − 21 8 + 7 


 2
= 
 tan α 1  −1  −7 1 
=
 2  2  −37 15



Mat ices 57
r

E:\AMIT_WORKS\Exam_Guru\EG_Mathematics-12_(working_02-06-2022)\EG_Mathematics-12_working\Open_Files\Chap_3\Chap_3
\ 16-Aug-2022  Amit   Proof-4 Reader’s Sign _______________________ Date __________


1  7 −1   −3 2  2 −1  1 0
=
2 37 −15
⇒ A =  
 5 −3  −3 2 0 1




 −8 −3  −3 2  2 −1
(ii) A=   ⇒ A = 
5 4  5 −3  −3 2






−1 −1
1 2  2 5  2 −1  −3 2
Hint: A =     ⇒ A=  
3 4  −4 7   −3 2  5 −3






3 2  Simplify it.


8. (i) A=   14. A = ` 7,000, B = ` 6,125, C = ` 7,875.
1 −4







Total amount ` 21,000
a c 


Hint: Let A =   Try yourself.
b d 



15. x = ` 15,000, y = ` 20,000



 a c  1 −1 4  7 3 16  Hint: Let investment at 8% = ` x
 b d   2 3 1  =  −7 −13 −4


     and investment at 10% = ` y




Solve for a, b, c, d.  x + y = 35, 000 and

\  8x 10y

1 −2 100 + 100 = 3200



(ii) A=  
3 4 



 x + y = 35, 000
⇒ 
a c 
8x + 10y = 3, 20, 000


Hint: Take A =  
b d 

1 1   x   35, 000 
 −3 2 2  ⇒ 8 10  y  = 3, 20, 000
    


1
9. A–1 =  2 −3 2 
5


−1
 2 2 −3  x  1 1   35, 000 
⇒  y  = 8 10 3, 20, 000
Hint: First verify A2 – 4A – 5I = 0 by putting


     

1 2 2  x  1 10 −1  35, 000 
A =  2 1 2 ⇒  y  = 2  −8 1  3, 20, 000


    


 2 2 1 
 x  1  3, 50, 000 − 3, 20, 000 
Now, A–1[A2 – 4A – 5I] = 0 ⇒  y  = 2  −2, 80, 000 + 3, 20, 000



⇒ A–1A2 – 4A–1A – 5A–1I = 0    


⇒ A – 4I – 5A–1 = 0 ⇒ A −1 = 1 [ A − 4I ] ⇒
 x  1 30, 000 
5  y  = 2  40, 000




   
Put the values of A and I.

 x  15, 000 
 −1 8  ⇒  y  =  20, 000
10. m = 4, n = –13. A −1 = 1 
13  2 −3


   


11. A = ` 30,000, B = ` 23,000, C = ` 39,000 ⇒ x = ` 15,000, y = ` 20,000




16. Fees for poor students = ` 200
 −191 −110


12. D = 
 77 44  Fees for rich students = ` 1000



Hint: Fees for poor students = ` x
Hint: CD – AB = 0


Fees for rich students = ` y

⇒ CD = AB ⇒ D = (C–1) (AB)






1 1 20x + 5y = 9000
13. A =  \ 
 5x + 25y = 26000


1 0


 2 1   −3 2  1 0  4x + y = 1800
Hint:  A =  ⇒ 
 3 2  5 −3 0 1  x + 5y = 5200




−1
 −3 2  2 1  1 0  4 1  x  1800 
⇒ A =    ⇒ 1 5  y  = 5200
 5 −3  3 2 0 1     




58 Mathe atics–12
m
E:\AMIT_WORKS\Exam_Guru\EG_Mathematics-12_(working_02-06-2022)\EG_Mathematics-12_working\Open_Files\Chap_3\Chap_3
\ 16-Aug-2022  Amit   Proof-4 Reader’s Sign _______________________ Date __________


−1
 x   4 1 1800  5x + 6y = 140000
⇒  y  = 1 5 5200 ⇒ 
6x + 5y = 135000





     

 x  1  5 −1 1800  5 6  x  140000


⇒  y  = 19  −1 4  5200 ⇒ 6 5  y  = 135000


         



 x  1  9000 − 5200  −1
 x  5 6 140000
⇒  y  = 19  −1800 + 20800 ⇒  y  = 6 5 135000
   


     



 x  1  3800   200 
⇒  y  = 19 19000 = 1000  x  −1  5 −6 140000
      ⇒  y  = 11  −6 5  135000


    



⇒ x = ` 200 and y = ` 1000.
 x  −1  700000 − 810000 


17. Investment in first bond = ` 10,000 ⇒  y  = 11  −840000 + 675000
   




Investment in second bond = ` 15,000

Hint: Let investment in first bond = ` x  x  −1  −110000 10000
⇒  y  = 11  −165000 = 15000

Investment in second bond = ` y      




10x + 12y = 2800 ⇒ x = ` 10,000, y = ` 15,000
100 100



⇒ 
12x + 10y = 2700


100 100

Topic 2. Operations on Matrices


(A) Transpose of a Matrix  2 3
If A = [aij]m×n be a m × n matrix, then the matrix obtained by  3 2 1  
Solution. It is given that: A =   and B = 1 0
 4 −1 3
changing rows into columns of matrix A is called transpose of  4 5
matrix A. Transpose of a matrix A is denoted by A′ or AT.
 2 3′
In other words, if matrix A = [aij]m×n then its transpose A′  3 2 1  
A + B′ =  4 −1 3 +  1 0 
= [aji]n×m.


  
 4 5
Here: (i) aij of matrix A = aji of matrix A′.
 3 2 1  2 1 4   5 3 5


(ii) If order of matrix A is m × n then order of transpose = + = 
 4 −1 3  3 0 5 7 −1 8


of matrix A or A′ is n × m.

 3 2 −1 Hence, option (c) is the correct answer.

For example: If matrix A =   ,  2 3
 4 −5 6  2×3  3 2 1 1 0
Example 2. If matrix A =   and matrix B =  
 4 −1 3
 3 4  4 5
then A′ =  2 −5 , then value of A′ – (B′)′ will be
 
 −1 6  3×2 2 5 4 2
 2 3

(a) 3 1 (b) 7 −1

  
 3 2 1




   2 −2  3 −2
Example 1. If matrix A =   and matrix B = 1 0
 4 −1 3  4 5  3 5  1 1
, then value of A + B′ will be (c)  4 2 (d)  
   1 −1




 −1 3  −3 −2
 5 5 3  3 5 3
(a)   (b)  
7 8 1 7 4 1   2 3′ ′




3 2 1 ′  
 5 3 5  5 7 8 Solution. A′ – (B′)′ =   −   1 0  
(c)   (d)    4 −1 3   
7 −1 8  −1 2 3   4 5 




 

Mat ices 59
r

E:\AMIT_WORKS\Exam_Guru\EG_Mathematics-12_(working_02-06-2022)\EG_Mathematics-12_working\Open_Files\Chap_3\Chap_3
\ 16-Aug-2022  Amit   Proof-4 Reader’s Sign _______________________ Date __________


 3 4 Theorem: If A is a square matrix then prove that: P = A + A′ is
′  3 4  2 3  1 1
 2 −1 −  2 1 4   2 −1 −  1 0 =  1 −1 symmetric matrix.
=    3 0 5 =       Proof: Take any square matrix A.
 1 3  


 1 3  4 5  −3 −2
P = A + A′



Hence, option (d) is the correct answer. ⇒ P′ = (A + A′)′





⇒ P′ = A′ + (A′)′
Properties of Transpose Matrix





[Q (A + B)′ = A′ + B′]
1. For any matrix A, (i) (A′)′ = A and (ii) (kA)′ = kA′.
⇒ P′ = A′ + A [Q (A′)′ = A]


2. For any two matrices A and B of same order:





⇒ P′ = A + A′ [Q Matrix addition is


(i) (A + B)′ = A′ + B′





commutative]


(ii) (A – B)′ = A′ – B′ ⇒ P′ = P


(iii) (mA + nB)′ = mA′ + nB′





⇒ P = (A + A′) is a symmetric matrix.





(iv) (mA – nB)′ = mA′ – nB′ Theorem: If matrix A is a square matrix, then prove that: Q =


Here m and n are scalar number. (A – A′) is a skew symmetric matrix.

3. Let A, B and C be three matrices such that their respective Proof: Take a square matrix A


orders are: m × n, n × p and p × q. Then Q′ = A – A′



(i) (AB)′ = B′A′ Q′ = (A – A′)′ = A′ – (A′)′





(ii) (ABC)′ = C′B′A′ [Q (A – B)′ = A′ – B′]


⇒ Q′ = A′ – A [Q (A′)′ = A]
(B) Symmetric and Skew Symmetric Matrix




⇒ Q′ = –A + A′
A square matrix A is called symmetric if its transpose is equal to




[ Matrix addition is commutative]
the matrix A itself.
⇒ Q′ = –(A – A′)
OR




⇒ Q′ = – Q
A square matrix A is called symmetric if A′ = A



⇒ Q is a skew symmetric matrix.

 7 4 8 Theorem: Each diagonal element of a skew symmetric matrix
For example, matrix A =  4 3 9 is symmetric matrix is zero.
 
8 9 5 Proof: Let A = [aij]m×m be a skew symmetric square matrix.
\ A′ = –A ⇒ [aij]′ = –[aij]
7 4 8



⇒ [aji] = [–aij]
because A′ = 4 3 9 = A.


  ⇒ aji = –aij ∀ i and j ...(i)
8 9 5


To take only diagonal elements, put j = i in (1), we get

 x A B \ aii = –aii ⇒ 2aii = 0



In general matrix A =  A y C is a symmetric matrix. ⇒ aii = 0
 

 B C z  Since aii are the elements of main diagonal, hence all the

elements of the main diagonal of a skew symmetric matrix are zero.
A square matrix A is called skew symmetric if its transpose is Theorem: A matrix which is both symmetric and skew symmetric
equal to negative of the matrix A itself. Each diagonal element is a zero matrix.
of a skew symmetric matrix is zero. Proof: Let A = [aij]m×m be square matrix which is both symmetric
Or and skew symmetric.
A square matrix A is called skew symmetric matrix if A′ = –A. Since A is a skew symmetric matrix, A′ = –A
 0 5 −7  ⇒ [aij] = –[aij] ⇒ [aji] = –[aij]




  ⇒ aji = –aij ...(1)
For example, matrix A =  −5 0 −8 is a skew symmetric




matrix because  7 8 0 Since A is a symmetric matrix \ A′ = A
⇒ [aij]′ = [aij] ⇒ [aji] = [aij]
 0 −5 7   0 5 −7 




⇒ aji = aij ...(2)
A′ =  5 0 8 = −  −5 0 −8 = −A




From (1) and (2), we get: aij = –aij

 −7 −8 0  7 8 0 ⇒ 2aij = 0 ⇒ aij = 0 ∀ i and j





Hence all the elements of matrix A are zero i.e., A is a zero matrix.
 0 A −B
Theorem: If A is a square matrix, then prove that (i) P = AA′ is
In general, matrix P =  −A 0 C is a skew symmetric
 a symmetric matrix and (ii) Q = A′A is also symmetric matrix.
 B −C 0
matrix. Proof: Take any square matrix A.

60 Mathe atics–12
m
E:\AMIT_WORKS\Exam_Guru\EG_Mathematics-12_(working_02-06-2022)\EG_Mathematics-12_working\Open_Files\Chap_3\Chap_3
\ 16-Aug-2022  Amit   Proof-4 Reader’s Sign _______________________ Date __________


(i) Take P = AA′  0 3 −1


⇒ P′ = (AA′)′ ⇒ P′ = [(A′)′]A′  
= −  −3 0 −8 = −Q




[Q (AB)′ = (B′A′)]


 1 8 0 
⇒ P′ = AA′ [Q (A′)′ = A]




⇒ P′ = P ⇒ Q is a skew symmetric matrix.







\ P is a symmetric matrix. 2 4 2 2 1 3
(ii) Take Q = A′A  1 0 −3  4 0 5
(iii) R =AA′ ⇒ R=  






⇒ Q′ = (A′A)′  3 5 4   2 −3 4 




⇒ Q′ = A′(A′)′ [ (AB)′ = B′A′]
 24 −4 34




⇒ Q′ = A′A = Q [Q (A′)′ = A]
 




⇒ Q′ = Q ⇒ R =  −4 10 −9 = R′








\ Q′ is also a symmetric matrix.  34 −9 50

 2 4 2 \ R is a symmetric matrix.



Example 3. If matrix A =  1 0 −3 , (iv) S = A′A
 




 3 5 4 2 1 3  2 4 2 
 4 0 5  1 0 −3
(i) work out P = (A + A′) and show that P is a symmetric ⇒ S=   






matrix.  2 −3 4   3 5 4 
(ii) work out Q = (A – A′) and show that Q is a skew symmetric
14 23 13


matrix.  
(iii) work out R = (AA′) and show that R is a symmetric matrix.
=  23 41 28 = S′


 13 28 29 


(iv) work out S = (A′A) and show that S is a symmetric matrix.


\ S is a symmetric matrix.
 2 4 2


Theorem: Any square matrix can be expressed as the sum of a symmetric
Solution. It is given that: matrix A =  1 0 −3 and a skew symmetric matrices.
 
 3 5 4 Proof: Let matrix A be a square matrix.
\ A= A
 2 4 2  2 1 3



1 1
(i) P = A + A′
  
⇒ P =  1 0 −3 +  4 0 5
 ⇒ A = [ 2A ] = [ A + A ]
2 2







 3 5 4  2 −3 4 1
⇒ A = ( A + A′ ) + ( A − A′ )
2




 4 5 5
⇒ P =  5 0 2
 ⇒  1
2 
1
A =  ( A + A′ ) +  A − A′ 
2
 
( )



  



 5 2 8 Symmetric matrix Skew symmetric matrix

 4 5 5  3 5 2
 1 7 4
⇒ P′ = 5 0 2 = P
 Example 4. If matrix A =   , express matrix A as a sum of
 


 5 2 8  4 −5 8

⇒ P is a symmetric matrix. symmetric and skew symmetric matrices




(ii) Q = A – A′  3 5 2
 




Solution. It is given that: A =  1 7 4
2 4 2 2 1 3
 1 0 −3 −  4 0 5  4 −5 8
⇒ Q=    


 3 5 4   2 −3 4    3 5 2  3 1 4 
1 1
⇒ ( A + A′ ) = 2   1 7 4 +  5 7 −5
 0 3 −1 2


  4 −5 8  2 4 8
    
⇒ Q =  −3 0 −8


 1 8 0  
 3 3 3
 6 6 6
 0 −3 1 1   −1
  = 6 14 −1 = 3 7 
⇒ Q′ =  3 0 8 2 2
6 −1 16 


3 −1 8


 −1 −8 0 
 2 

Mat ices 61
r

E:\AMIT_WORKS\Exam_Guru\EG_Mathematics-12_(working_02-06-2022)\EG_Mathematics-12_working\Open_Files\Chap_3\Chap_3
\ 16-Aug-2022  Amit   Proof-4 Reader’s Sign _______________________ Date __________


  3 5 2  3 1 4     
1 1   1 7 4  −  5 7 −5  3 3 3  0 2 −1
Also ( A − A′ ) = 2    
2   −1  9




  4 −5 8  2 4 8  ⇒ A = 3 7 +  −2 0 
   
 2  2




  3 −1 8  1 −9 0
0 2 −1  
 0 4 −2    2
  
2
1   0 9
 Symmetric matrix Skew symmetric matrix
=  −4 0 9  =  −2
2 2
 2 −9 0  


 3 5 2
 1 −9 0   
 2  So, it is clear that matrix A =  1 7 4 can be expressed


1 1  4 −5 8
Now A = ( A + A′ ) + ( A − A′ )
2 2 as the sum of symmetric and skew symmetric matrices.



EXERCISE 3.2
I. Multiple Choice Questions (MCQs) 2 3
3 5


(c) a = ,b= (d) a = − ,b=
Choose the correct answer from the given options. 7 2 3 2






 3  3 5
5. A =   is written as A = P + Q. If P is a symmetric matrix
1. If A =  5 and B = [1 0 4] , then (AB)′ equals 7 9


 


 2 and Q is skew symmetric matrix, then value of matrix P will be

 −1 2 1  2 5 8
     1 5 7 8
(a)  7 2 3 (b)  3 2 1 (a)   (b) 
3 4  9 4








 5 11 5  5 4 9
 3 7 3 6
(c)  (d) 
 3 5 2  3 2 5 6 7

6 9




   
(c)  0 0 0 (d) 7 8 11
 cos θ sin θ 




12 20 8 7 0 5 6. If f (θ ) =   , then [ f (q)]′ is equal to
 − sin θ cos θ 





a + b 2  6 5 ′
2. If  = , then a equals (a) f (q) (b) f (–q)

b   2 2





 5


 θ
(c) f (2q) (d) f  
 2

(a) 3 (b) 4








(c) 6 (d) 5  2 1 3
7. If matrix A =  3 0 4 , then value of AA′ will be




 cos α sin α   


3. If A =  π 1 5 1 
 , a satisfying 0 < α < 2 and A +
 − sin α cos α 


A′ = 2 I 2 then value of a is  5 7 11 10 25 7 
   
(a) 17 3 8 (b) 10 7 8
π π




(a) (b) 14 18 10 11 12 3
3 4




π π 14 18 10 15 5 8
(c) (d)
 
6 2 (c) 18 25 7 




(d)  7 8 11
 




 0 2b −2 10 7 27  13 12 13
4. Matrix A = 3  1 3 is given to be symmetric



3a 3 −a  3 4  2
8. If A =   , then (A )′ equals
1 5


matrix. The values of a and b will be
3 1 2 1 (a) (A′)2 (b) A′
(a) a = , b = (b) a = , b =




5 5 5 5 (c) A3 (d) AI








62 Mathe atics–12
m
E:\AMIT_WORKS\Exam_Guru\EG_Mathematics-12_(working_02-06-2022)\EG_Mathematics-12_working\Open_Files\Chap_3\Chap_3
\ 16-Aug-2022  Amit   Proof-4 Reader’s Sign _______________________ Date __________


 3 1 1 1 1
 2 1 3
, B =  −1 4 and C =  2 −1  
9. If A =      5 7  , (AB 3. If matrix A = 1 1 1 then using P.M.I. prove that
 4 7 −1  





 0 3 1 1 1
+ C)′ equals 3n −1 3n −1 3n −1
(a) (B′A′ + C′) (b) (A′B′ + C′)  
A n = 3n −1 3n −1 3n −1 where n ∈ N. [V. Imp.]




(c) (C′ + BA′) (d) (C′ + AB′)


3n −1 3n −1 3n −1
 




10. If a matrix A is both symmetric and skey symmetric, then


A necessarily a 3 − 4 
(a) Diagonal matrix 4. If A =   , then using P.M.I. prove that:
1 − 1 





(b) Zero square matrix
1 + 2n − 4n 


(c) Square matrix An =  : n ∈N [V. Imp.]
 n 1 −2n 



(d) Identity matrix


II. Short Answer Type Questions-I 0 1 n
5. If A =   , then using P.M.I. prove that: (aI + bA) =
 0 0



 2 3  1 5
1. If matrix A =  1 0 and matrix B =  3 2 , then find anI + nan – 1 bA, n ∈ N.
   


 3 4 0 −1 a b
AB′.
6. If A =   , a ≠ 1, using P.M.I. prove that:
 0 1


 cos θ sin θ 
2. If f (θ ) =   , show that [ f (q)]′ = f (–q).

 n
(
b an − 1 

)
 − sin θ cos θ  A = a



n
a − 1 , n ∈N .


0 1 
5 2   
3. If matrix A =   , express matrix A as a sum of a
1 4 


 cos θ − i sin θ 
symmetric and a skew symmetric matrix. 7. If f (θ ) =  , then show that: (i) [ f (q)]′
cos θ 



i sin θ


 3 1 2
= f (–q) (ii) f (q) [ f (q)]′ = (cos 2q) I.




4. If A =   is a 2 × 2 matrix. Verify: |5A| = (5) |A| =
 4 5 


cos θ + sin θ − 2 sin θ 
25 |A|. 8. If f (θ ) =   , then show that:
2 sin θ cos θ − sin θ 



5. Solve for x:
( x + 3) ( x − 1) x 1
= (i) [ f (q)]–1 = f (–q) (ii) f (q) f (–q) = I.
( x + 2) (3x − 1) −17 x








1 2 2
1 −1 2
9. If A = 2 1 −2 satisfies AA′ = 9I, find the values of

6. Find the value of the determinant: 3 2 3 .  


 x 2 y 


2 1 1
x and y.
0 sin α − cos α
 3 4
7. Evaluate: ∆ = − sin α 0 sin β  2 3 1  
10. If matrix A =   and matrix B = 1 5 then


cos α − sin β 0  1 4 5 


 2 3
 3 1  2 3 verify: (AB)′ = B′A′.
8. If A =   and B =   , verify | AB | = | A | | B |.






 4 5 1 3  3


 
9. If A is a skew symmetric matrix of order 3, then prove that 11. If A =  1 and B = [1 –5 7] verify: (i) (AB)′ =




det A = 0. [Delhi 2017]  −7 

III. Short Answer Type Questions-II B′A′ (ii) (BA)′ = A′B′.

1. Using P.M.I., prove that (A′)n = (An)′, where n ∈ N.  3 1 2


 cos θ sin θ   
2. If matrix A =  12. If matrix A =  2 4 1
 , then using P.M.I. prove



 − sin θ cos θ   1 5 3


 cos nθ sin nθ  (i) work out P = (A + A′) and show that: P is a symmetric
that: A n =   where n ∈ N.


 − sin nθ cos nθ  matrix.

Mat ices 63
r

E:\AMIT_WORKS\Exam_Guru\EG_Mathematics-12_(working_02-06-2022)\EG_Mathematics-12_working\Open_Files\Chap_3\Chap_3
\ 16-Aug-2022  Amit   Proof-4 Reader’s Sign _______________________ Date __________


(ii) work out Q = (A – A′) and show that: Q is a skew The cost (in thousands rupees); average expenditure (in



symmetric matrix. rupees) and average returns (in rupees) of each ox, camel
(iii) work out R = (AA ′) and show that: R is a symmetric and tractor is given below:


matrix.  Cost Average Average 
(in thousand `) exp. (in `) income (in `)
(iv) work out S = (A′A) and show that: S is a symmetric  
Ox


matrix.  3.0 20 20 
Camel  
 4.0 20 40 
 5 3 1 Tractor
 50 100 200 
13. If matrix A = 7 4 5 express matrix A as a sum of
 



 2 9 3 Find by using matrix multiplication, the total cost, average


symmetric and skew symmetric matrices. expenditure and average income on oxen, camels and tractors.
IV. Long Answer Type Questions 7. In a legislative assembly election, a political group hired a



 0 1 3 7  public relation firm to promote its candidate in three ways,
1. If matrix A =   and B =   , then verify that
 −1 0 5 2  telephones, house-calls and letters. The cost per contact


P = BAB′ and Q = B′AB are skew symmetric matrix. (in paise) is given in matrix A.


[HOTS] Cost per contact in paise

 40  Telephones
 0 −2 −3  0 5 −2 A=  
2. If matrix A =  2 0 −5 and matrix B =  −5 0 3
   100  House calls
 


 3 5 0  2 −3 0  50  Letters

are two skew symmetric matrices, then show that P = (AB
The number of contacts of each type made in two cities X

and Y is given below by matrix B.
– BA) is also a skew symmetric matrix. [HOTS]

Telephones House Calls Letters 

B =  1, 000 500 5, 000  X
a b c 
3. If matrix A =  b c a  where a, b and c are real positive  3, 000 1, 000 10, 000  Y
 



 c a b 
Find the total amount spent by the group in the two cities
numbers. abc = 1 and A′A = I, then find the value of a3 + b3 + c3.

X and Y.
[HOTS]
8. The shop of a particular school has 10 dozen Chemistry



 cos α sin α  –1 books, 8 dozen Physics books and 10 dozen Economics
4. If A =   and A′ = A , find the value of a.
 − sin α cos α 


[HOTS] books. The selling prices are ` 80, ` 60 and ` 40 each

()
respectively. Find the total amount the book shop will
 −1 x 
 sin ( πx ) tan −1  receive from selling all the books using matrix algebra.
1 π
5. If A =   and
π  −1 x
()
cot −1 ( πx ) 9. A manufacturer produces three products x, y and z which


sin


 π 
he sells in two markets. Annual sales are indicated below.

1
B= 
− cos −1 ( πx ) tan −1
x 
π ()
 , then find (A – B).
Market
x
Products
y z
π −1 x
() − tan ( πx )

−1
sin I 10,000 2,000 18,000
 π 
II 6,000 20,000 8,000
6. Farm A and Farm B have oxen, camels and tractors as (a) If unit sale prices of x, y and z are ` 2.50, ` 1.50 and ` 1.00




given below: respectively, find the total revenue from each market with
Oxen Camels Tractors  the help of matrix algebra.
A  20 5 4  (b) If unit cost of the above three commodities are ` 2.0, `



B  10 6 6  1.00 and 50 paise, respectively. Find the gross profit.

64 Mathe atics–12
m
E:\AMIT_WORKS\Exam_Guru\EG_Mathematics-12_(working_02-06-2022)\EG_Mathematics-12_working\Open_Files\Chap_3\Chap_3
\ 16-Aug-2022  Amit   Proof-4 Reader’s Sign _______________________ Date __________


Answers 3.2
 3 5 2  2 3
1 3 0 
I. 1. (c)  0 0 0 Hint: Work out AB′ = 1 0 
  5 2 −1

 3 4 


12 20 8 

 3  3 0 12   cos θ sin θ 
Hint: AB =  5 [1 0 4] =  5 0 20 2. Hint: f (θ ) =  
     − sin θ cos θ 




 2  2 0 8 
cos θ − sin θ 
Now find (AB)′. ⇒  f (θ ) ′ =   and
 sin θ cos θ 



2. (b) 4 3. (b) π
4  cos ( −θ ) sin ( −θ ) 





f ( −θ ) =  
4. (d) a =
−2 , b = 3  − sin ( −θ ) cos ( −θ )
3 2




cos θ − sin θ 
 0 2b −2  = 
  sin θ cos θ 
Hint:  3 1 3  is symmetric matrix.



3a 3 −a  [Q cos(–q) = cos q and sin(–q) = –sin q]

\ 2b = 3 and 3a = –2 ⇒ b = 3 and a = −2 =  f (θ ) ′


2 3




10 3 1  0 1
 3 6 3. A = 1  +
5. (d)   2  3 8 2  −1 0


6 9



1 A+A 1 1
Hint: P = ( ′) Hint: A = ( A + A′ ) + ( A − A′ )
2 2 2


6. (b) f (–q)
 3 1 15 5 




4. Hint: A =   ⇒ 5A =  
 cos θ sin θ   4 5  20 25


Hint: f (θ ) =  
 − sin θ cos θ 

|5A| = 15 × 25 – 20 × 5


cos θ − sin θ  = 375 – 100 = 275 = 25 × 11 = (5)2|A|
⇒  f (θ ) ′ =   and


 sin θ cos θ  Q |A| = 15 – 4 = 11 ⇒ |5A| = 25|A|






 cos ( −θ ) sin ( −θ )  cos θ − sin θ  5. x = 2, –9 6. D = –6
f ( −θ ) =   =





 − sin ( −θ ) cos ( −θ )  sin θ cos θ  7. D = 0 8. Try yourself.





[Q cos(–q) = cos q and sin(–q) = –sin q] III. 1. Try yourself.
2. Try yourself. 3. Try yourself.
=  f (θ ) ′




4. Try yourself. 5. Try yourself.





14 18 10  6. Try yourself.
7. (c) 18 25 7 


   cos θ −i sin θ 



10 7 27  7. (i) Hint: f (q) =  

i sin θ cos θ 





8. (a) (A′)2
 cos ( −θ ) −i sin ( −θ )



13 8 
(A )′ = (A′ ) 2 2
=
⇒ f (–q) = 
i sin ( −θ ) cos ( −θ ) 







32 29
9. (a) (B′A′ + C′)  cos θ i sin θ  ′
=   =  f (θ )



 −i sin θ cos θ 


 7 10 
( AB + C)′ = B′A′ + C′ =   (ii) Hint: f (q)[ f (q)]′
14 36






10. (b) zero square matrix  cos θ −i sin θ   cos θ i sin θ 
=



 
17 12 −3 i sin θ cos θ   −i sin θ cos θ 
II. 1. AB′ =  1 3 0

 cos 2 θ − sin 2 θ i sin θ cos θ − i sin θ cos θ 
 23 17 −4 = 
i sin θ cos θ − i sin θ cos θ cos 2 θ − sin 2 θ 

Mat ices 65
r

E:\AMIT_WORKS\Exam_Guru\EG_Mathematics-12_(working_02-06-2022)\EG_Mathematics-12_working\Open_Files\Chap_3\Chap_3
\ 16-Aug-2022  Amit   Proof-4 Reader’s Sign _______________________ Date __________


cos 2θ
=
0  1 0 13. A = 1 ( A + A′ ) + 1 ( A − A′ )
 = cos 2θ 0 1 2 2



 0 cos 2 θ   
5 5 3  0 −2 − 1 

= (cos 2q)I.  2  2

8. Try yourself. See the solution of Q.No. 7. =  5 4 7  +  2 0 −2 


9. x = –2, y = –1 3  1 
 2 7 3   2 2 0 





Hint: AA′  
Symmetric Matrix Skew Symmetric Matrix

 x + 2y + 4 
 1 0 
9  0 −29
  IV. 1. P = Q =  is skew symmetric matrix.
= 9 0 1
2x − 2y + 2   29 0
 9 
 
 x + 2y + 4 2x − 2y + 2 x2 + y 2 + 4   0 19 19
 9 9 9  2. P = ( AB − BA ) =  −19 0 −19
 




1 0 0  −19 19 0
= 9 0 1 0 is skew symmetric matrix.


0 0 1
3. a3 + b3 + c3 = 4



Now solve for x and y. 4. a can have any value.



11 17  1
10. (AB)′ = B′ A′ =   5. A − B = I
 26 29 2




 3 1 −7  6. Total cost in Average Average
11. (i) ( AB) = B′ A′ = −15 −5 35
′  Thousand expenditure in income in
  rupees rupees rupees



 21 7 −49
280 900 1400
(ii) (BA)′ = A′B′ = –15


 6 3 3 354 920 1640
 
12. (i) P = (A + A′) =  3 8 6 = P′
7. X = ` 3400, Y = ` 7200



 3 6 6


8. ` 20,160
⇒ P is symmetric matrix


9. (a) Total revenue from market I = ` 46,000


 0 −1 1



(ii) Q = A – A′ =  1 0 −4 = −Q′ Total revenue from market II = ` 53,000



 −1 4 0 (b) ` 15,000, ` 17,000


⇒ Q is skew symmetric matrix. Hint:



14 12 14 (a) Work out the product:
(iii) R = AA′ = 12 21 25 = R ′


 2.50


14 25 35 10, 000 2, 000 18, 000  
 6, 000 20, 000 8, 000  1.50 
⇒ R is symmetric matrix.  
1.00 


14 16 11

(iv) S = A′A = 16 42 21 = S′ 10, 000 2, 000 18, 000 
(2.50 − 2.00) 
  


11 21 14  (b)   (1.50 − 1.00) 
 6, 000 20, 000 8, 000  


 (1.00 − 0.50) 
⇒ S is symmetric matrix.


66 Mathe atics–12
m
E:\AMIT_WORKS\Exam_Guru\EG_Mathematics-12_(working_02-06-2022)\EG_Mathematics-12_working\Open_Files\Chap_3\Chap_3
\ 16-Aug-2022  Amit   Proof-4 Reader’s Sign _______________________ Date __________


Case Based Questions
1. Three schools A, B and C organized a mela for collecting 2. Two farmers Velu and Krishna cultivates only three




funds for helping the rehabilitation of flood victims. varieties of rice namely Basmati, Permal and Naura. The
They sold hand made fans, mats and plates from recycled sale (in `) of these varieties of rice by both the farmers
material at a cost of ` 25, ` 100 and ` 50 each. in the month of September and October are given by the
following matrices A and B.



The number of articles sold are given below: September sales in `


School → A B C  Basmati Permal Naura 
Articles ↓ A = 10, 000 20, 000 30, 000 Velu


 


Hand made Fans 40 25 35  50, 000 30, 000 10, 000  Krishna

Mats 50 40 50 October sales in `



Plates 20 30 40  Basmati Permal Naura 
B =  5, 000 10, 000 6, 000  Velu
Based on the above information, answer the following  


 20, 000 10, 000 10, 000 Krishna


questions:
(i) The total money collected by the school A is Based on the above information, answer the following

questions:


(a) ` 700 (b) ` 7000
(i) The combined sales in September and October for farmer




(c) ` 6125 (d) ` 7875


Krishna for all varieties is




(ii) The total amount of money collected by schools B and
(a) ` 90000 (b) ` 130000


C is




(c) ` 8000 (d) ` 13500
(a) ` 14000 (b) ` 15725




(ii) The combined sales in September and October for farmer




(c) ` 21000 (d) ` 13125


Velu of all varieties is




(iii) The total amount of money collected by all the three
(a) ` 86000 (b) ` 90000


schools A, B and C is




(c) ` 80000 (d) ` 81000
(a) ` 15775 (b) ` 14000




(iii) The variety of rice which has the lowest selling value in




(c) ` 21000 (d) ` 17125


the month of September for the farmer Velu is




(iv) Number of handmade fans made by the students of (a) Permal (b) Naura


schools A, B and C are 20, 30 and 40 respectively and




(c) Basmati
number of plates made by the students of schools A,



(d) All of these have the same price
B and C are 40, 25 and 35 respectively. Moreover the


number of mats made by students of respective schools (iv) The variety of rice which has the highest selling value


remain unchanged. Then the total money collected by in the month of September for the farmer Krishna is?
all schools is (a) Naura (b) Permal




(a) ` 21250 (b) ` 6750 (c) Basmati



(d) All of these have the same price




(c) ` 21000 (d) ` 7000


(v) If both farmers receive 2% profit on gross sales, compute




(v) Total number of articles sold by all the three schools is


the profit for each farmer for each variety sold in October.


(a) 230 (b) 130
 Basmati Permal Naura 





(c) 430 (d) 330
 120  Velu




Ans. (i) (b) ` 7000 (ii) (a) ` 14000 (iii) (c) ` 21000 (a)  100 200








(iv) (a) ` 21250 (v) (c) 330  400 200 200  Krishna





Mat ices 67
r

E:\AMIT_WORKS\Exam_Guru\EG_Mathematics-12_(working_02-06-2022)\EG_Mathematics-12_working\Open_Files\Chap_3\Chap_3
\ 16-Aug-2022  Amit   Proof-4 Reader’s Sign _______________________ Date __________


 Basmati Permal Naura  (i) Total revenue of market A is



 (a) ` 64,000 (b) ` 60,400
(b)  100 200 120  Velu





(c) ` 46,000 (d) ` 40,600


 400 300 200  Krishna





(ii) Total revenue of market B is



 Basmati Permal Naura  (a) ` 35,000 (b) ` 53,000






 100  Velu (c) ` 50,300 (d) ` 30,500
(c)  100 200





(iii) Cost incurred in market A is


 400 300 120  Krishna



(a) ` 13,000 (b) ` 30,100
 Basmati Permal Naura 






(c) ` 10,300 (d) ` 31,000
 120  Velu





(d)  100 200 (iv) Profit in market A and B respectively are





 410 200 200  Krishna (a) (` 15,000, ` 17,000)




(b) (` 17,000, ` 15,000)
Ans. (i) (b) `1,30,000 (ii) (d) ` 81,000



(c) (` 51,000, ` 71,000)




(iii) (c) Basmati (iv) (c) Basmati




(d) (` 10,000, ` 20,000)






 Basmati Permal Naura  (v) Gross profit in both market is
 120  Velu



(v) (a)  100 200 (a) ` 23,000 (b) ` 20,300








 400 200 200  Krishna (c) ` 32,000 (d) ` 30,200





Ans. (i) (c) `46,000 (ii) (b) ` 53,000 (iii) (d) ` 31,000
3. A manufacturer produces three stationery products Pencil,







(iv) (a) (` 15,000, ` 17,000) (v) (c) ` 32,000


Eraser and Sharpener which he sells in two markets.





4. Amit, Biraj and Chirag were given the task of creating a


square matrix of order 2.
Below are the matrices created by them. A, B , C are the

matrices created by Amit, Biraj and Chirag respectively.
 1 2  4 0 2 0
A= 
−1 3 B =  1 5 C =  1 −2
     

If a = 4 and b = −2, based on the above information answer

the following:
(i) Sum of the matrices A, B and C, A + (B + C) is


 1 6  6 1
(a)   (b)  
2 7 7 2




7 2  2 1
(c)   (d)  
 1 6 7 6




(ii) (AT)T is equal to


 1 2 2 1
(a)   (b)  
 −1 3  3 −1




 1 −1  2 3
(c)   (d)  
 2 3  −1 1




Annual sales are indicated below.

(iii) (bA)T is equal to
Products (in numbers)


Market
Pencil Eraser Sharpener  −2 −4  −2 2
(a)   (b)  
A 10,000 2000 18,000  2 −6  −4 −6




B 6000 20,000 8,000  −2 2  −6 −2
(c)  (d) 
The unit Sale price of Pencil, Eraser and Sharpener are `  −6 −4  2 4





2.50, ` 1.50 and ` 1.00 respectively, and unit cost of the
above three commodities are ` 2.00, ` 1.00 and ` 0.50 (iv) AC − BC is equal to


respectively.  −4 −6  −4 −4
(a)  (b)  
Based on the above information answer the following:  −4 4  4 −6





68 Mathe atics–12
m
E:\AMIT_WORKS\Exam_Guru\EG_Mathematics-12_(working_02-06-2022)\EG_Mathematics-12_working\Open_Files\Chap_3\Chap_3
\ 16-Aug-2022  Amit   Proof-4 Reader’s Sign _______________________ Date __________


 −4 −4  −6 4 (iv) How much amount is given to each child by Seema?



(c)  (d)  (a) ` 32 (b) ` 30
 −6 4  −4 −4









(c) ` 62 (d) ` 26





(v) (a + b) B is equal to (v) How much amount Seema spends in distributing the





 0 8  2 10 money to all the students of the Orphanage?
(a)   (b)   (a) ` 609 (b) ` 960
10 2  8 0









(c) ` 906 (d) ` 690





 8 0  2 0 Ans. (i) (a) 5x – 4y = 40





(c)   (d)   5x – 8y = –80
 2 10  8 10





5 −4  x   40
(ii) (c)   =  (iii) (d) 32
 7 2 1 2
(ii) (a)  5 −8  y   −80






Ans. (i) (c)   
 1 6 −
 1 3







(iv) (b) ` 30 (v) (b) ` 960






 −2 2
(iii) (b)   5. In a city there are two factories A and B. Each factory
 −4 −6







produces sports clothes for boys and girls. There are three
 −4 −4 8 0 types of clothes produced in both the factories, type I, II
(iv) (c)  (v) (c) 
 −6 4 
 2 10 and III. For boys the number of units of types I, II and III






respectively are 80, 70 and 65 in factory A and 85, 65 and
4. On her birthday, Seema decided to donate some money to 72 are in factory B. For girls the number of units of types


children of an orphanage home. If there were 8 children
I, II and III respectively are 80, 75, 90 in factory A and 50,
less, everyone would have got ` 10 more. However, if
there were 16 children more, everyone would have got ` 55, 80 are in factory B.
10 less. Let the number of children be x and the amount
distributed by Seema for one child be y (in `).

Based on the above information, give the answer of the


Based on the information given above, answer the following questions:

following questions: (i) If P represents the matrix of number of units of each type


(i) The equations in terms x and y are produced by factory A for both boys and girls, then P is


(a) 5x – 4y = 40 (b) 5x – 4y = 40 given by:




5x – 8y = –80 5x – 8y = 80 Boys Girls
I II III


(c) 5x – 4y = 40 (d) 5x + 4y = 40 I 85 50
(a) II  65 55 (b) Boys  50 55 80 




5x + 8y = –80 5x – 8y = – 80 Girls 85 65 72





 
III 72 80


(ii) Which of the following matrix equations represent the


information given above? Boys Girls
I II III
I 80 80
5 4   x   40 5 −4  x   40 (c) Boys 80 75 90 (d)
(a)    =  (b)   =  II 70 75




5 8  y   −80 5 −8  y   80 Girls 80 70 65  




III  65 90

5 −4  x   40 5 4  x   40 (ii) If Q represents the matrix of number of units of each
(c)    =  (d)   =


  type produced by factory B for both boys and girls, then
5 −8  y   −80 5 −8  y   −80




(iii) The number of children who were given some money Q is given by:


by Seema, is Boys Girls
I II III
(a) 30 (b) 40 I 85 50
(a) II  65 55 (b) Boys  50 55 80 




Girls 85 65 72





(c) 23 (d) 32  
III 72 80




Mat ices 69
r

E:\AMIT_WORKS\Exam_Guru\EG_Mathematics-12_(working_02-06-2022)\EG_Mathematics-12_working\Open_Files\Chap_3\Chap_3
\ 16-Aug-2022  Amit   Proof-4 Reader’s Sign _______________________ Date __________


Boys Girls (i) (ii) (iii)
I II III X 400 300 100
I 80 80
(c) Boys 80 75 90 (d)
II 70 75 Y 300 250 75




Girls 80 70 65   Z 500 400 150
III  65 90
Also, the chance of making of toilets corresponding to one
(iii) The total production of sports clothes of each type for


attempt of given modes is:


boys is given by the matrix:
(i) 2 % (ii) 4 %
I II III I II III





(a) (b) (iii) 20 %
[ 130 137]
165 [ 165 137]
130






Based on the above information, give the answer of the
I II III I II III


(c) (d) following questions:
[ 135 137]
165 [ 135 165]
137




(i) The cost incurred by the organisation on village X is:



(iv) The total production of sports clothes of each type for (a) ` 10,000 (b) ` 15,000







girls is given by the matrix: (c) ` 30,000 (d) ` 20,000





I II III I II III (ii) The cost incurred by the oraganisation on village Y is:
(a) (b)
[130 130 170] [170 130 130]







(a) ` 25,000 (b) ` 18,000






I II III (c) ` 23,000 (d) ` 28,000
(c) (d) None of these
[130 170 130]





(iii) The cost incurred by the organisation on village Z is:






(v) Let R be a 3 × 2 matrix that represents the total (a) ` 19,000 (b) ` 39,000







production of sports clothes of each type for boys and (c) ` 45,000 (d) ` 50,000




girls, then transpose of R is:
(iv) The total number of toilets that can be expected after the


promotion in village X is:
(a) 165 135 137  (b) 130 130 170
130 130 170 165 135 138 (a) 20 (b) 30









(c) 40 (d) 50
165 132 130 168




(c) 135 130 (d) 130 135 (v) The total number of toilets that can be expected after the


   




137 170 170 137  promotion in village Z, is:
Boys Girls Boys Girls (a) 26 (b) 36





I 80 80 I 85 50 (c) 46 (d) 56




Ans. (i) (d) II 70 75 (ii) (a) II  65 55 Ans. (i) (c) `30,000 (ii) (c) ` 23,000 (iii) (b) ` 39,000






   






III  65 90 III 72 80 
(iv) (c) 40 (v) (d) 56





I II III 6. Three car dealers, say A, B and C, deal in three types of
I II III


(iii) (c) [165 135 137 ] (iv) (a) cars, namely Hatchback car, Sedan car, SUV car. The
[130 130 170]





sales figure of 2019 and 2020 show that dealer A sold
(v) (a) 165 135 137  120 Hatchback, 50 Sedan, 10 SUV cars in 2019 and 300
130 130 170



Hatchback, 150 Sedan, 20 SUV cars in 2020; dealer B
6. To promote the making of toilets for women, an sold 100 Hatchback, 30 Sedan, 5 SUV cars in 2019 and


organisation tried to generate awareness through (i) houses 200 Hatchback, 50 Sedan, 6 SUV cars in 2020, dealer C
 
(ii) e-mails and (iii) announcements. sold 90 Hatchback, 40 Sedan, 2 SUV cars in 2019 and 100
Hatchback, 60 Sedan, 5 SUV cars in 2020.

The cost for each model per attempt is given below:

(i) ` 50 (ii) ` 20
Based on the above information, give the answer of the




(iii) ` 40

following questions:


The number of attempts made in the village X, Y and Z are

given below: (i) The matrix summarising sales data of 2019 is:


70 Mathe atics–12
m
E:\AMIT_WORKS\Exam_Guru\EG_Mathematics-12_(working_02-06-2022)\EG_Mathematics-12_working\Open_Files\Chap_3\Chap_3
\ 19-Aug-2022  Amit   Proof-4 Reader’s Sign _______________________ Date __________


Hatchback Sedan SUV Hatchback Sedan SUV
A 300 150 20  A 420 200 30 
(a) B  200 50 6  (c) B  300 80 11 
   




C 100 60 5  C 190 100 7 

Hatchback Sedan SUV (d) None of these



A 120 50 10 
(b) B  100 30 5  (iv) The increase in sales from 2019 to 2020 is given by the
 




C 90 40 2  matrix:
Hatchback Sedan SUV
Hatchback Sedan SUV A 180 100 10 
A 100 30 5  (a) B  
(c) B  10 20 1
120 50 10   



  C 100 20 3 


C 90 40 2 
Hatchback Sedan SUV
Hatchback Sedan SUV A 10 20 3 
A 200 50 6  (b) B  
(d) B  100 20 1
100 30 5   



  C 180 100 10 


C 300 150 20 
Hatchback Sedan SUV
(ii) The matrix summarising sales data of 2020 is: A 180 100 10 
(c) B  


100 20 1
 


Hatchback Sedan SUV C 10 20 3 
A 300 150 20 
(a) B  200 50 6 
  Hatchback Sedan SUV


C 100 60 5  A 100 20 3 
(d) B  180 100 10 
 


Hatchback Sedan SUV C 10 20 3
A 120 50 10  
(b) B  100 30 5 
 


C 90 40 2  (v) If each dealer receive profit of ` 50,000 on sale of a

Hatchback, ` 100,000 on sale of a Sedan and ` 200,000
Hatchback Sedan SUV

 
A 100 30 5  on sale of a SUV, then amount of profit received in the
(c) B  120 50 10 
  year 2020 by each dealer is given by the matrix.


C 90 40 2 
A 30000000 A 12000000 
Hatchback Sedan SUV (a) B 15000000  (b) B 16200000 
   




A 200 50 6  C 12000000  C 34000000
(d) B  100 30 5 
 


C 300 150 20  A 34000000 A 15000000 
(3) B 16200000  (d) B 30000000
   




(iii) The total number of cars sold in two given years, by each C 12000000  C 12000000 


dealer, is given by matrix:
Hatchback Sedan SUV
Hatchback Sedan SUV A 120 50 10 
A 190 100 7 Ans. (i) (b) B  100 30 5 
  



(a) B  300 80 11  C 90 40 2 
 


C 420 200 30 
Hatchback Sedan SUV
Hatchback Sedan SUV A 300 150 20 
A 300 80 11 (ii) (a) B  200 50 6 
  



(b) B  190 100 7  C 100 60 5 
 


C 420 200 30 

Matrices 71

E:\AMIT_WORKS\Exam_Guru\EG_Mathematics-12_(working_02-06-2022)\EG_Mathematics-12_working\Open_Files\Chap_3\Chap_3
\ 22-Aug-2022  Amit   Proof-4 Reader’s Sign _______________________ Date __________


Hatchback Sedan SUV Based on the above information, give the answer of the


A 420 200 30  following questions:
(iii) (c) B  300 80 11 
  (i) If ` 15000 is invested in bond X, then:



C 190 100 7 



Investment X Y
Hatchback Sedan SUV (a) X 15000  ; B = [0.1 0.08] interest rate
A= 



A 180 100 10  Y  20000
(iv) (c) B  100 20 1 
  Interest rate



C 10 20 3 X Y
 (b) A = Investment [15000 20000] ; X  0.1 
B= 
Y 0.08



A 34000000
(v) (c) B 16200000  Interest rate
  X Y



C 12000000  (c) A = Investment [20000 15000]; X 0.08
B= 
Y  0.1 




6. A trust fund has ` 35,000 that must be invested in two
(d) None of these




different types of bonds, say X and Y. The first bond pays (ii) If ` 15,000 is invested in bond X, then total amount of



10% interest p.a. which will be given to an old age home interest received on both bonds is:
and second one pays 8% interest p.a. which will be given (a) ` 2,000 (b) ` 2,100






to WWA (Woman Welfare Association). (c) ` 3,100 (d) ` 4,000






(iii) If the trust obtains an annual total interest of ` 3200,


then the investment in two bonds is:
(a) ` 15,000 in X, ` 20,000 in Y



(b) ` 17,000 in X, ` 18,000 in Y


(c) ` 20,000 in X, ` 15,000 in Y
IBMA



(d) ` 18,000 in X, ` 17,000 in Y
TRUST FUND


(iv) The total amount of interest received on both bonds is


given by:
  
  
(a) AB (b) A′B




(c) B′A (d) None of these




(v) If the amount of interest given to old age home is ` 500,


then the amount of investment in bond Y is:
(a) ` 20,000 (b) ` 30,000




(c) ` 15,000 (d) ` 25,000




Interest rate
X Y X 0.1 
Ans. (i) (b) A = Investment [15000 20000] ; B = 
Y 0.08



(ii) (c) ` 3,100



(iii) (c) ` 20,000 in X, ` 15,000 in Y



(iv) (a) AB
Let A be 1 × 2 matrix and B be a 2 × 1 matrix, representing



(v) (b) ` 30,000

the investment and interest rate on each bond respectively.



72 Mathe atics–12
m
E:\AMIT_WORKS\Exam_Guru\EG_Mathematics-12_(working_02-06-2022)\EG_Mathematics-12_working\Open_Files\Chap_3\Chap_3
\ 16-Aug-2022  Amit   Proof-4 Reader’s Sign _______________________ Date __________


Author’s Comments
Questions based on following types are very important for Exams. So, students are advised to revise them thoroughly.
1. To express a square matrix into sum of symmetric and skew-symmetric matrices.


2. Questions based on property |adj A| = |A|n – 1.


3. To find inverse of a square matrix. (Most Important)


COMMON ERRORS
Errors Corrections
(i) Wrong notation for matrices. (i) Matrices: ( ) or [ ].






(ii) Solution of equations: (ii) If AX = B, then X = A–1B.




If AX = B, then X = BA–1 is wrong.

(iii) Not multiplying the first row elements with the second (iii) By memorising tips like Run and Jump remember while




matrix corresponding column elements and add multiplying two matrices. More practice on various order
matrices for multiplication.

IMPORTANT FORMULAE
1. Multiplication of a Matrix by a Scalar Number


If A = [aij]m×n is a matrix and k is any scalar number, then kA = k[aij]m×n = [kaij]m×n.

2. Addition of Matrices


If matrix A = [aij]m×n and matrix B = [bij]m×n, then

A + B = [aij]m×n + [bij]m×n = [aij + bij]m×n.


3. Subtraction of Matrices


If matrix A = [aij]m×n and matrix B = [bij]m×n then

(A – B) = [aij]m×n – [bij]m×n = [aij – bij]m×n


4. Multiplication of a matrix with another matrix


[aij]m×n × [bjk]n×p = [cik]m×p

5. If A is the inverse matrix of matrix A then AA–1 = A–1A = I.
–1


6. If A′ is the transpose of matrix A, then


(i) (A′)′ = A (ii) (A + B)′ = A′ + B′




(iii) (A – B)′ = A′ – B′ (iv) (AB)′ = B′A′




(v) (ABC)′ = C′B′A′.


7. If A is a square matrix, then


(i) (A + A′) is a symmetric matrix. (ii) (A – A′) is a skew-symmetric matrix.




(iii) (AA′) and (A′A) are symmetric matrix.


8. Any square matrix A can be expressed as a sum of symmetric and skew-symmetric matrix:


A= 1
( A + A′ ) +
2 
1
(A − A′
2  )


 
Symmetric matrix Skew-symmetric matrix

o s 73
E
rr
r

E:\AMIT_WORKS\Exam_Guru\EG_Mathematics-12_(working_02-06-2022)\EG_Mathematics-12_working\Open_Files\Chap_3\Chap_3
\ 16-Aug-2022  Amit   Proof-4 Reader’s Sign _______________________ Date __________


REVISION CHART

M atrix
An arrangement of numbers in horizontal and vertical lines is called a matrix.
4 1 3 
A =  0 −4 5  is a matrix
 


 columns of the matrix
rows of the matrix

4, 1, 3, 0, –4 and 5 are elements of the matrix. Matrix A has order 2 × 3 (Two rows ×
three columns)

ype of
T s M atrice s

(i) Row matrix, (ii) Column matrix, (iii) Zero or null matrix, (iv) Square matrix, (v) Rectangular matrix,
(vi) Diagonal matrix, (vii) Identity or Unit matrix, (viii) Scalar matrix

E quality of Matrice s M ultiplication of a M atrix Add ition an d Su traction of matrice


b s

Matrix A = [aij]m×n and matrix an a Scalar


d M atrix If A = [aij]m×n and B = [bij]m×n are two
B = [bij]m×n are called equal Let matrix A = [aij]m×n and matrices of equal order then
matrices if they are of same k is a scalar number then (A + B) = [aij]m×n + [bij]m×n = [aij + bij]m×n

order and aij = bij. kA = k [aij]m×n = [kaij]m×n. (A – B) = [aij]m×n – [bij]m×n = [aij – bij]m×n

M ultiplication of a atrix y another


M b Matrix Symmetric M atrix
The product AB of two matrices A and B is defined only if A square matrix A is called symmetric if its transpose
number of columns of the first matrix A is equal to number is equal to the matrix A itself or a square matrix A is
of rows of the second matrix. symmetric if A = A′.
If A = [aij]m×n and [bjk]n×p then
 x A B

A =  A y C = A ′
AB = [aij]m×n × [bjk]n×p = [cik]m×p

If A, B and C are three matrices such that A = [aij]m×n


 B C z 
B = [bjk]n×p and C = [ckl]p×q then
(i) AB ≠ BA
(ii) (AB)C = A(BC)
\ A is a symmetric matrix.

Skew-Symmetric M atrix

A square matrix A is called skew


symmetric matrix if its transpose
is equal to negative of the matrix
A itself, i.e., A′ = –A.
 0 A −B 
A′ =  − A 0 C  = − A
 B − C 0 
\ A is a skew-symmetric matrix.

74 Mathe atics–12
m
E:\AMIT_WORKS\Exam_Guru\EG_Mathematics-12_(working_02-06-2022)\EG_Mathematics-12_working\Open_Files\Chap_4\Chap_4
\ 16-Aug-2022  Amit   Proof-5 Reader’s Sign _______________________ Date __________


Topics
C
overed
4 Determinants

4.1 Determinant 4.2 Area of a Triangle






4.3 Adjoint of a Square Matrix 4.4 Inverse of a Non-singular Square Matrix




4.5 Applications of Matrices


C hapter map
DETERMINANTS

Determinants of a Minors and Solution of


Square Matrix Co-factors Determinant Equation

Singular Evaluation of Area of a


Matrix Determinants Triangle

INVERSE OF A MATRIX BY ELEMENTARY OPERATIONS OF A MATRIX

Adjoint of a Square Inverse of a Application of


Matrix Non-Singular Square Martrix Martrices

Topic 1. Determinant
(A) Determinant of a Square Matrix Determinant of a (2 × 2) Matrix
To every square matrix A = [aij]n×n we can associate a number  a11 a12 
calculated with a rule. This number is called determinant of the If matrix A =   , then determinant of matrix
 a21 a22 
matrix. Determinant of a matrix is denoted by (det. A) or | A | or D.


 a11 a12 a13  a11 a12
  A = A = det A = = a11a22 − a12a21 .
Suppose A is a matrix which is denoted as A=  a21 a22 a23  , a21 a22
 
 a31 a32 a33  Note: Value of a determinant can be any positive, negative or

zero real number.
a11 a12 a13
7 2
then det A = | A | = D is denoted as a21 a22 a23 Example 1. If matrix A =   , then determinant A will be
1 3


a31 a32 a33 given by
(a) 19 (b) 17 (c) 15 (d) 16
Here we shall learn how to determine the special number








called determinant or value of determinant of a square matrix. 7 2
Solution. It is given that: matrix A =  
Note: Only square matrices have determinants, rectangular 1 3
7 2

matrices do not have determinants. \ | A | = det A = = 7 × 3 − 2 × 1 = 21 − 2 = 19


1 3


Determinant of a (1 × 1) Matrix \ det A = 19.



If A = [a]1×1 is a 1 × 1 order matrix, then det A = | a | = a. Hence, option (a) is the correct answer.



75
E:\AMIT_WORKS\Exam_Guru\EG_Mathematics-12_(working_02-06-2022)\EG_Mathematics-12_working\Open_Files\Chap_4\Chap_4
\ 16-Aug-2022  Amit   Proof-5 Reader’s Sign _______________________ Date __________


Theorem: If A is a 2 × 2 matrix, then | mA | = m2| A |. \ M11 = Minor of the element a11 of the det. A







 a11 a12  a11 a12 a13
Proof: Let A=   be a 2 × 2 matrix
 a21 a22  = a21 a22 a23




 a11 a12  a31 a32 a33
\ mA = m  
 a21 a22  a22 a23



⇒ M11 =
a32 a33




 ma11 ma12 
⇒ mA =  
 ma21 ma22  ⇒ M11 = (a22 × a33 – a32 × a23)






ma11 ma12 Now, M12 can be worked out by deleting first row and second
⇒ | mA | column of the determinant A.
ma21 ma22





a11 a12 a13
⇒ | mA | = ma11ma22 – ma12 ma21
M12 = a21 a22 a23


\



⇒ | mA | = m2(a11a22 – a12a21)






a31 a32 a33



⇒ | mA | = m2| A |.




a21 a23



 3 1 ⇒ M12
Example 2. If a 2 × 2 matrix A =   , then | 3A | equals a31 a33






 4 5
⇒ M12 = (a21 a33 – a31 a23)
(a) 4 | A | (c) 6 | A | (b) 3 | A |
(d) 9 | A |




Similarly M13 can be worked out by deleting first row and third








 3 1 column of the determinant A.
Solution. It is given that: matrix A =  
 4 5
M13 = minor of element a13 of the det. A


3 1 a11 a12 a13
⇒ |A| = = 15 − 4 = 11 ...(1) a21 a22
4 5




⇒ M13 = a21 a22 a23 ⇒ M13 =
a31 a32





 3 1  9 3  a31 a32 a33
Now 3A = 3  = 
 4 5 12 15



⇒ M13 = (a21a32 – a31a22)



9 3 Now you have understood the way we work out minors of different
⇒ | 3A | = = 135 − 36
12 15 elements of det A.





⇒ | 3A | = 99 = 9 × 11  2 1 3


Example 3. If matrix A =  −1 4 7  , then matrix of minors



⇒ | 3A | = 9 × | A | = 9| A | [Using (1)]  










 1 8 5
Hence, option (d) is the correct answer. of determinant will be

(B) Minors 12 36 11  −12 11 2
To work out the value of the determinant of a 3 × 3 matrix or (a)  7 8 9 (b)  36 18 5
   




determinant of a 3 × 3 order matrix: We need to define minors  2 3 5  6 7 2
and cofactors of different elements of the matrix or determinant
of the matrix.  −36 −12 −12  −19 7 15
 
Minor of an element of a square matrix or determinant of (c)  −19 7 15 (d)  5 17 9





a square matrix: Minor of an element aij of a square matrix or  −5 17 9  12 12 31
determinant of a square matrix is the determinant obtained by
deleting its ith row and jth column in which the element aij lies.  2 1 3
Minor of an element aij is denoted by Mij. Solution. It is given that A =  −1 4 7 
 
Note: Minor of an element of a determinant of order n × n  1 8 5

(n ≥ 2), is a determinant of order (n – 1) × (n – 1).
2 1 3
Now take a matrix A of order 3 × 3:
⇒ | A | = −1 4 7
 a11 a12 a13  a11 a12 a13





  1 8 5
A =  a21 a22 a23  ⇒ A = a21 a22 a23
4 7

 a31 a32 a33  a31 a32 a33 \ M11 = = 20 − 56 = −36
3× 3
8 5



Then minor of the element a 11 = M 11 , where M 11 is the
determinant obtained by deleting first row and first column of the −1 7
M12 = = −5 − 7 = −12
determinant A. 1 5


76 Mathe atics–12
m
E:\AMIT_WORKS\Exam_Guru\EG_Mathematics-12_(working_02-06-2022)\EG_Mathematics-12_working\Open_Files\Chap_4\Chap_4
\ 16-Aug-2022  Amit   Proof-5 Reader’s Sign _______________________ Date __________


−1 4 below before the minors of the different elements to get their
M13 = = −8 − 4 = −12 respective cofactors.
1 8


+ − +
1 3
M21 = = 5 − 24 = −19 − + −
8 5


+ − +
2 3
M22 = = 10 − 3 = 7  3 1 −2
1 5


Example 4. If matrix A =  4 5 9 , then matrix of co-factors
2 1  
M23 = = 16 − 1 = 15  8 7 6
1 8


of A will be given by
1 3
M31 = = 7 − 12 = −5 19 −35 11  11 13 −14
4 7 (a) 34 −4 7  (b)  34 −24 25


  





2 3  8 6 −9  −7 12 18
M32 = M 32 = = 14 + 3 = 17
−1 7


 −33 48 −12  −33 48 −12
2 1  
M33 = = 8 +1 = 9 (c)  −20 34 −13 (d)  −10 34 −3
 





−1 4  19 −35 −11


 19 −35 11
\ Matrix of minors of determinant of matrix
 3 1 −2

 −36 −12 −12 Solution. It is given that A =  4 5 9
 

A =  −19 7 15  8 7 6


 −5 17 9
3 1 −2
Hence, option (c) is the correct answer.
⇒ |A| = 4 5 9






(C) Cofactors 8 7 6
Cofactor of an element aij of a determinant is denoted by Aij and Calculation of co-factors:
is defined as: 5 9
A11 = ( −1) = + (30 − 63) = −33
1+1
Aij = (–1)i + j(Mij) M11 = + M11 = +
7 6




Here Mij is the minor of the element aij.
4 9
A12 = ( −1) = − ( 24 − 72) = 48
1+ 2
Let us find out the minors and cofactors of the different elements M12 = −M12 = −
8 6


of a square matrix A or determinant of square matrix A, i.e., |A|.
 a11 a12 a13  a11 a12 a13 4 5
A13 = ( −1) = + ( 28 − 40) = −12
1+ 3
M13 = +M13 = +
  8 7


A =  a21 a22 a23  ⇒ A = a21 a22 a23



 a31 a32 a33  a31 a32 a33 1 −2
A21 = ( −1) = − ( 6 + 14) = −20
2 +1
M 21 = −M 21 = −
7 6


a22 a23
M11 = = ( a22a33 − a32a23 ) 3 −2
a32 a33 A22 = ( −1)
2+ 2
= + (18 + 16) = 34


M 22 = + M 22 = +
8 6


[By eliminating first row and first column of matrix A or det. A]
and A11 = (–1)1+1 M11 = (–1)2 M11 = M11 3 1
A23 = ( −1) = − ( 21 − 8) = −13
2+ 3
M 23 = −M 23 = −



8 7


⇒ A11 = (a22a33 – a32a23)



a21 a23 1 −2
A31 = ( −1) = + (9 + 10) = 19
3+1
M12 = = ( a21a33 − a31a23 ) M 31 = +M 31 = +
a31 a33 5 9




and A12 = (–1)1+2M12 = (–1)3M12 = –M12 3 −2
A32 = ( −1) = − ( 27 + 8) = −35
3+ 2
M 32 = − M 32 = −



⇒ A12 = – (a21a33 – a31a23) 4 9





a21 a22 3 1
= ( a21a32 − a31a22 ) A33 = ( −1) = + (15 − 4) = 11
3+ 3
M13 = M 33 = +M 33 = +
a31 a32 4 5




and A13 = (–1)1+3M13 = (–1)4M13 = M13  −33 48 −12



⇒ A13 = (a21a32 – a31a22) 
\ Matrix of cofactors of Matrix A =  −20 34 −13




In place of writing (–1)i+j in front of the minors of a determinant  19 −35 11
of a matrix of order 3 × 3, we can put the +, – signs as shown Hence, option (c) is the correct answer.

ete inants 77
D
rm

E:\AMIT_WORKS\Exam_Guru\EG_Mathematics-12_(working_02-06-2022)\EG_Mathematics-12_working\Open_Files\Chap_4\Chap_4
\ 16-Aug-2022  Amit   Proof-5 Reader’s Sign _______________________ Date __________


Value of the Determinant of a Square Matrix A of D1 = a11A21 + a12A22 + a13A23 = 0;
order 3×3 D2 = a11A31 + a12A32 + a13A33 = 0
 a11 a12 a13  3 4 2
Take a general matrix A of order 3×3: A =  a21 a22 a23 
Example 5. In the determinant 2 1 −3 , the sum of the
 a31 a32 a33 
1 6 5
Value of its determinant D, can be worked out in 6 ways, along
R1, along R2, along R3, along C1, along C2 and along C3 products of the elements of first row with cofactors of second
row is
(i) Value of the determinant along
(a) 0 (b) 5


R1 = a11A11 + a12A12 + a13A13





(c) 6 (d) 8


(ii) Value of the determinant along







R2 = a21A21 + a22A22 + a23A23 3 4 2


(iii) Value of the determinant along Solution. It is given that: D = 2 1 −3 . It is required to find


R3 = a31A31 + a32A32 + a33A33 1 6 5


(iv) Value of the determinant along the sum of the products of the elements of the first row with


C1 = a11A11 + a21A21 + a31A31 cofactors of second row.


(v) Value of the determinant along D = a11A21 + a12A22 + a13A23


C2 = a12A12 + a22A22 + a32A32



4 2 3 2 3 4


(vi) Value of the determinant along = 3 ( −1) +4 + 2 ( −1)
6 5 1 5 1 6



C3 = a13A13 + a23A23 + a33A33



You can calculate the value of determinant of a square matrix of
= –3(20 – 12) + 4(15 – 2) – 2(18 – 4)


order 3 × 3 using any one of the above discussed formulae, the = –3(8) + 4(13) – 2(14)
value of the determinant will be same.


= –24 + 52 – 28 = 52 – 52 = 0
Theorem (without proof): If elements of a row or column of


⇒ D= 0
determinant of a square matrix are multiplied with cofactors of



any other row or column of the matrix, then their sum is zero. Hence, option (a) is the correct answer.

 a11 a12 a13  Theorem (without proof): The determinant of the product of two
matrices of same order is equal to the product of their respective
For example, A =  a21 a22 a23  then:
determinants, that is | AB | = | A | | B |, where A and B are two square






 a31 a32 a33  matrices of same order.

EXERCISE 4.1
I. Multiple Choice Questions (MCQs)
 5 7 9
Choose the correct answer from the given options.  
5. If A =  9 8 7  , then |A| =
 1 2  1 3


1. If A =   and B =   , the value of |AB| will 19 14 9
3 −1  −1 1


be (a) 5 (b) 0 (c) 3 (d) 9








(a) –28 (b) –8 1 k 3




(c) –18 (d) 5 6. If = 3 k −2 = 0, then k =




( x + 3) ( x − 1)



x 1 2 3 −1
2. For = , value of x will be
( x + 2) (3x − 1) −17 x


(a) – 1 (b) 0




(a) 9, 2 (b) 2, –9 (c) 9, –2 (d) –9, –2 (c) 1 (d) None of these.












0 sin α − cos α 0 p−q p−r
3. If ∆ = − sin α 0 sin β , then value of will be 7. q − p 0 q − r is equal to





cos α − sin β 0 r− p r−q 0
(a) 5 (b) 0 (a) 0 (b) (p – q) (q – r)(r – p)








(c) 7 (d) 1 (c) pqr (d) 3pqr








 3 1  2 3 1 2
4. If A =   and B = 1 3 , then | AB | equals 8. If A =   , then |2A| is equal to
 4 2


4 5


   


|A| (a) 2 |A| (b) 4 |A|
(a) | A | | B | (b) (c) | A |2 | B | (d) | B | | A |


















|B|








(c) 3 |A| (d) None of these.





78 Mathe atics–12
m
E:\AMIT_WORKS\Exam_Guru\EG_Mathematics-12_(working_02-06-2022)\EG_Mathematics-12_working\Open_Files\Chap_4\Chap_4
\ 16-Aug-2022  Amit   Proof-5 Reader’s Sign _______________________ Date __________


9. What positive value of x makes following pair of 2 3 4


determinant equal?
15. The value of the determinant −5 6 −10 is
2x 3 16 3



= 1 7 2
5 x 5 2
(a) – 440 (b) 0 (c) 328 (d) 488









(a) 1 (b) 2 (c) 3 (d) 4 II. Short Answer Type Questions-I








1 ω ω 6 8 α 2  3
1. If A =   and |A | = 125, then find the value of a.
The value of the determinant ω 6 ω 3 ω 7 where ω3 = 1  2 α 



10.


ω8 ω 7 1 6 −1
is 2. If = 15, then find the value of x.
3 x



(a) 1 (b) 2 (c) 3 (d) 9 3 x 3 2








1 0 1  3. What is the values of x for which = ?
x 1 4 1



11. If A = 0 1 2 , then |3A| is equal to
  sin 10° − cos 10°


0 0 4 4. What is the value of ?
sin 80° cos 80°



(a) 3 |A| (b) 9 |A|


 5 10 3




(c) 27 |A| (d) None of these.
5. The matrix  −2 −4 6 is a singular matrix, then find the





6 −3 2  



 −1 −2 b 
12. The value of D = 2 − 1 2 is value of b.


−10 5 2 III. Long Answer Type Questions
(a) 0 (b) 1 (c) 8 (d) 12  1 17 
 3 5








1 2 k 1. If A =   and B =   , what is the value of |AB| ?
 2 0 0 −10


 
13. If k 4 5 = 0, then the possible values of k are
 1 2


5 6 7 2. If A =   , then what is the value of k if | 2A | = k| A | ?
8 8  4 2
(c) −3, −1 (d) 3,




 
(a) –3, 3 (b) −3, −
3 3 3 1 2 3








14. If Cij denotes the cofactor of element Pij of the matrix
3. Find the value of the determinant 0 sec x tan x .


 1 −1 2


P =  0 2 −3 . 0 tan x sec x
 3 2 4
0 2 0
(a) 5 (b) 24 (c) – 24 (d) – 5
4. What is the value of the determinant 2 3 4 ?








[CBSE 2020]


4 5 6

Answers 4.1
I. 1. (a) –28 2. (b) x = 2, –9 0 p−q p−r


3. (b) D = 0 4. (a) |A| |B| 5. (b) 0 Let D= q− p 0 q−r






r− p r−q 0



5 7 9
We have |A| = 9 8 7 Expanding along R1, we get
19 14 9




Expanding along first row, we get D = – (p – q) q − p q − r
r−p 0



|A| = 5(72 – 98) –7(81 – 133) + 9(126 – 152) q− p 0
+ (p – r)


= 5(–26) – 7(–52) + 9(–26) r− p r−q


= –130 + 364 – 234 = 364 – 364 = 0 = (p – q)(q – r)(r – p) + (p – r)(q – p)(r – q)



6. (d) None of these = (p – q)(q – r)(r – p) – (p – q)(q – r) (r – p) = 0.




Expanding ∆ along R1, we get 8. (b) 4 |A|





(– k + 6) – k(– 3 + 4) + 3(9 – 2k) = 0 1 2  2 4
We have 2A = 2  4 2 = 8 4

⇒ –k + 6 – k + 27 – 6k = 0    





⇒ 33 – 8k = 0 2 4
⇒ |2A| = 8 4 = 2(4) – 8(4) = 8 – 32 = – 24


33



⇒ k=
8 1 2



7. (a) 0 Also |A| = 4 2 = 1(2) – 4(2) = 2 – 8 = – 6






ete inants 79
D
rm

E:\AMIT_WORKS\Exam_Guru\EG_Mathematics-12_(working_02-06-2022)\EG_Mathematics-12_working\Open_Files\Chap_4\Chap_4
\ 19-Aug-2022  Amit   Proof-5 Reader’s Sign _______________________ Date __________


⇒ 4|A| = 4 (– 6) = – 24 1 0 1   3 0 3



Hence, |2A| = 4|A|. 3A = 3 0 1 2 = 0 3 6
0 0 4 0 0 12





In general, |kA| = kn|A|, where n is the order of determinant.  

9. (d) 4 3 0 3



2x 3 16 3 ∴ |3A| = 0 3 6
=



We have 5 x 5 2 0 0 12

On expanding, we get Expanding along C1, we get



2x2 – 15 = 32 – 15 3 6 0 3 0 3
|3A| = 3 0 12 − 0 0 12 + 0 3 6


⇒ 2x2 – 15 = 17






⇒ 2x2 = 32 ⇒ x2 = 32 ÷ 2 = 16 = 3(36 – 0) – 0 + 0 = 108.







⇒ x = ± 16 ⇒ x = ±4 1 0 1





The positive value of x = 4 Again, |A| = 0 1 2




0 0 4

Hence, for x = 4, the given pair of determinant is equal.
Expanding along C1, we get

10. (c) 3





1 2
1 ω 6 ω8 |A| = 1 −0+0
0 4


   
∆ = ω 6 ω 3 ω 7
ω8 ω 7 1 = 4–0=4



27 |A| = 27(4)
( ω ) ( ω ) .ω 3 2 3 2


\


2
1 = 108



∆ = (ω ) 3 2
ω ( ω ) .ω 3 3 2
Hence, |3A| = 27|A|.

( ω ) .ω ( ω ) .ω 1
3 2 2 3 2 12. (a) 0



6 −3 2
1 1 ω2 ∆ = 2 −1 2
= 1 1 ω [Q ω3 = 1] − 10 5 2



ω2 ω 1 Expanding in terms of elements along the first row, we get
Expanding along first row, we get −1 2 2 2 2 −1
5 2 ( ) − 10 2
∆ = 6 − −3 +2
∆ = 1(1 – ω2) – 1(1 – ω3) + ω2(ω – ω2) − 10 5
= 1 – ω2 – 1 + ω3 + ω3 – ω4 = 6 (– 2 – 10) + 3 (4 + 20) + 2 (10 – 10)




= 1 – ω2 – 1 + 1 + 1 – ω3.ω = – 72 + 72 + 2 (0) = 0.


= 2 – (ω2 + ω)


8
13. (d) 3, 14. (a) 5 15. (b) 0


[Q 1 + ω + ω2 = 0 Q ω + ω2 = –1] 3






= 2 – (– 1) = 2 + 1 = 3 II. 1. ±3 2. 2 3. ±2 2 4. 1 5. Any value










11. (c) 27 |A| III. 1. 100 2. 4 3. 1 4. 8









Topic 2. Area of a Triangle
If vertices of a triangle ABC are: A(x1, y1), B(x2, y2) and C(x3, y3), Example 1. Using determinants, the area of the triangle whose
then the area of the triangle is given by the following formula: D vertices are: A(1, 4), B(5, –3) and C(4, 7) is given by
1
=  x1 ( y2 − y3 ) + x2 ( y3 − y1) + x3 ( y1 − y2 ) (a) 16 (b) 15 (c) 16.5 (d) 14
2








Solution. The vertices of triangle ABC are: A(1, 4), B(5, –3)
Now this expression can be written in the form of determinant
and C(4, 7)
x1 y1 1
1 1 4 1
as: D = x y2 1 1
2 2 ⇒ D= 5 −3 1
x3 y3 1 2



4 7 1
Notes: (i) The value of this determinant can be positive or
1


negative, and area of a triangle is always a positive ⇒ D = 2 [1(–3 – 7) – 4(5 – 4) + 1(35 + 12)]



quantity, so always take absolute value of the above
determinant 1
= 2 [–10 – 4 + 47]


(ii) If the area of triangle is given, and you are to work out
1 1
D = 2 [47 – 14] = 2 [33] = 33 = 16.5


value of some other variable, use both positive and ⇒
2



negative values of the determinant for calculation and
Area of the DABC = 16.5
(iii) If three points are collinear then the area of the triangle

\


will be zero. Hence, option (c) is the correct answer.

80 Mathe atics–12
m
E:\AMIT_WORKS\Exam_Guru\EG_Mathematics-12_(working_02-06-2022)\EG_Mathematics-12_working\Open_Files\Chap_4\Chap_4
\ 16-Aug-2022  Amit   Proof-5 Reader’s Sign _______________________ Date __________


Example 2. Use determinants, and tell which three points, are 1
collinear ⇒ 2 [5(–1 – 4) – 1(k – 11) + 1(4k + 11)] = 0



(a) A(1, 4), B(3, 7) and C(4, 8) 1
⇒ 2 [–25 – k + 11 + 4k + 11] = 0


(b) A(2, 4), B(0, 1) and C(4, 7)






(c) A(5, 3), B(4, 4) and C(5, 4)
1


(d) A(7, 3), B(6, 1) and C(5, 3) ⇒ 2 [3k – 3] = 0 ⇒ 3k = 3 ⇒ k = 1






Solution. The three points are: A(2, 4), B(0, 1) and C(4, 7)
2 4 1 Hence, option (c) is the correct answer.
1 Example 4. The value of x (using determinants) if the area of the
⇒ D= 2 0 1 1



4 7 1 triangle ABC is 35 square units with vertices A(x, 4), B(2, –6) and
1 C(5, 4) will be given by
= 2 [2(1 –7) – 4(0 – 4) + 1(0 – 4)]


(a) 2, –12 (b) –2, 12 (c) –2, –12 (d) 2, 12









1
= 2 [–12 + 16 – 4] Solution. The three points are: A(x, 4), B(2, –6) and C(5, 4)


1 x 4 1
= 2 ×0=0 1
\ D= 2 −6 1 = ±35


2




\ The three given points are collinear. 5 4 1
Hence, option (b) is the correct answer. ⇒ x[–6 –4] – 4[2 – 5] + 1[8 + 30] = ±70

Example 3. The value of k (using determinants) when the three
⇒ –10x + 50 = ±70 ⇒ –x + 5 = ±7
points A(5, 1), B(k, –1) and C(11, 4) are collinear, will be given by



(a) 4 (b) 2 ⇒ –x + 5 = 7 or –x + 5 = –7







(c) 1 (d) 3 ⇒ x = –2 or x = 12








Solution. The three given points are: A(5, 1), B(k, –1) and C(11, 4) ⇒ x = –2, 12

5 1 1
1 Hence, option (b) is the correct answer.
⇒ D= k −1 1
2


11 4 1
EXERCISE 4.2
I. Multiple Choice Questions (MCQs) 2. Find the equation of the line joining the points A(1, 2) and


Choose the correct answer from the given options. B(3, 6) using determinant method.
1. The area of the triangle (using determinants) with vertices: 3. An equilateral triangle ABC has each side equal to a. If




A (– 2, 5), B (– 6, – 7), and C (– 5, – 4) is given by the coordinates of its vertices are A(x1, y1), B(x2, y2) and  
(a) 6 (b) 5 (c) 0 (d) 8 x1 y1 1
3








2. Three points which are collinear given by C(x3, y3), then show that x2 y2 1 = (a)2
2


(a) A (5, 7), B (0, 1) and C (3, 5) x3 y3 1


(b) A (2, 4), B (0, 1) and C (4, 7) 4. The points (2, –3), (k, –1) and (0, 4) are collinear then find




(c) A (0, 1), B (5, 6) and C (1, 5) the value of k.


(d) none of these 5. Find the area of the triangle with vertices (1, 1), (3, 2) and




3. For what value of x, the three points A(x, 2 – 2x), (2, 3) in sq. units.


B(– x + 1, 2x) and C(– 4 – x, 6 – 2x) are collinear? III. Long Answer Type Questions
1 3 3 1 1. Using determinants, find the value of x if the area of the
(a) 1, − (b) 2, − 2 (c) –2, 2 (d) –1,


2 2 DABC is 35 square units with vertices A (x, 4), B (2, –6)








II. Short Answer Type Questions and C (5, 4).
1. Find the area of the triangle formed by three points 2. Find the equation of a line joining the points A(1, 3)


A(p + 1, 1), B(2p + 1, 3) and C(2p + 2, 2p) and show that


and B(0, 0) using determinants and find the value of k if
the points will be collinear if
D(k, 0) is a point such that the area of DABD is 3 sq. units.
p = 2 or −1 [A.I. 2013]
2

ete inants 81
D
rm

E:\AMIT_WORKS\Exam_Guru\EG_Mathematics-12_(working_02-06-2022)\EG_Mathematics-12_working\Open_Files\Chap_4\Chap_4
\ 16-Aug-2022  Amit   Proof-5 Reader’s Sign _______________________ Date __________


Answers 4.2
I. 1. (c) 0 Equating (1) and (2), we get


2. (b) A(2, 4), B(0, 1) and C(4, 7) x1 y1 1


1 3 2
2 4 1 x2 y2 1 = a
2 4


Q ∆ = 0 1 1 = 0 ⇒ Three points are collinear. x3 y3 1


4 7 1 x1 y1 1
⇒ x2 y2 1 = 3 a2
x 2 − 2x 1
2




1 x3 y3 1
3. (d) x = –1, Hint: Evaluate: ∆ = −x + 1 2x 1 =0
2



−4 − x 6 − 2x 1 4. The given points (2, –3), (k, –1) and (0, 4) are collinear if



1
II. 1. p = 2 or − 2 −3 1
2
Area of the ∆ = 1 (2p 2 − 3p − 2) . k −1 1 = 0
2 0 4 1




Points are collinear when p = 2 or − 1 Applying R2 → R2 – R1 and R3 → R3 – R1, we get
2



2 −3 1
Hint: Evaluate the determinant for area of the D: k −2 2 0 = 0

−2 7 0



p +1 1 1
1 2 1 3 1
p+ Expanding along C3, we get
2

2p + 2 2p 1 k −2 2
= 0
−2 7



2. 2x = y


1 2 1 ⇒ 7k – 14 + 4 = 0 ⇒ 7k – 10 = 0



  
Hint: Evaluate: 3 6 1 = 0 10
\ k=

x y 1 7



3
5.
3 a2 2


3.
2 x 4 1


x1 y1 1 1 2
III. 1. x = – 2, 12. Hint: Solve for x: −6 1 = ±35
Hint: Since D = 1 x2 y2 1 ...(1) 2
2 5 4 1




x3 y3 1 2. k = ±2


3 a2 1 3 1
and D = ...(2)
4 Hint: 1 0 0 1 = ±3



2


[Area of an equilateral D with side ‘a’] k 0 1

Topic 3. Adjoint of a Square Matrix


Before discussing adjoint of a square matrix, first know about  2 3 −1
the singular matrix.  x + 4 −1 2 
Solution. A=  


Singular Matrix 3x + 1 2 −1
If value of the determinant of a square matrix is zero, then the
matrix is called singular matrix,otherwise it is called non-singular 2 3 −1
matrix. ⇒ | A | = x + 4 −1 2





 2 3 −1 3x + 1 2 −1
Example 1. If matrix A = x + 4 −1 2  is a singular matrix,

R1 → R1 – R3 and R2 → R2 + 2R3
 

3x + 1 2 −1
−3x + 1 1 0
then the value of x equals
| A | = 7x + 6 3 0


3


(a) 5 (b) 3x + 1 2 −1
16 13




3 3 Evaluate the det. along C3
(c) (d) −

7 16 | A | = –1[–9x + 3 – 7x – 6] = –[–16x – 3] = 16x + 3








82 Mathe atics–12
m
E:\AMIT_WORKS\Exam_Guru\EG_Mathematics-12_(working_02-06-2022)\EG_Mathematics-12_working\Open_Files\Chap_4\Chap_4
\ 16-Aug-2022  Amit   Proof-5 Reader’s Sign _______________________ Date __________


Since the given matrix is singular. 3 14 9  −13 −14 19

−3
16x + 3 = 0 ⇒ x= . (c) 8 7 14 (d)  2 −4 2
16   










  
3 −5 4  7 10 −9
Hence, option (d) is the correct answer.

Adjoint of a Square Matrix  1 4 3


 
The adjoint of a square matrix A is defined as the transpose of the Solution. Here A =  2 −1 4
matrix of the cofactors of the matrix A. It is denoted as (adj A)  3 2 5
 a11 a12 a13  Cofactors of matrix A are:
Take a matrix A =  a21 a22 a23 
−1 4

 a31 a32 a33  A11 = + = ( −5 − 8) = −13
2 5



Then matrix C = (a matrix of cofactors of matrix A)
2 4

 A11 A12 A13  A12 = − = − (10 − 12) = 2
3 5



⇒ C =  A 21 A 22 A 23 



 A31 A32 A33  2 −1
A13 = + = ( 4 + 3) = 7
3 2



Then (adj A) = Transpose of matrix C = C′
 A11 A 21 A31  4 3
A21 = − = − ( 20 − 6) = −14
(adj of matrix A) = C′ =  A12 A 22 A32  2 5




 A13 A 23 A33  1 3
A22 = + = (5 − 9) = −4
3 5


 4 1
Example 2. If matrix A =  , then its adjoint matrix is
 3 5 1 4
given by A23 = − = − ( 2 − 12) = 10
3 2


 −1 5 5 −4 4 3
(a)  (b)   A31 = + = (16 + 3) = 19
 −1 4
 4 −1  3 2






1 3
 5 −1  4 3 A32 = − = − ( 4 − 6) = 2
(c)  (d)   2 4


  −1 4
 −3 4




1 4
A33 = + = + ( −1 − 8) = −9
 4 1 2 −1


Solution. A =  
 3 5
 −13 2 7 
Cofactors of matrix A  
\ C =  −14 −4 10

A11 = (–1)1+1(5) = 5 A12 = (–1)1+2(3) = –3



 19 2 −9


A21 = (–1)2+1(1) = –1 A22 = (–1)2+2(4) = 4


 −13 −14 19
 5 −3  2 −4 2
\ Cofactors of matrix A is C =   ⇒ (adj A) = C′ = 
 −1 4




 7 10 −9
 5 −1
⇒ Adj A = C′ =   Hence, option (d) is the correct answer.
 −3 4


Theorem: If A be any given square matrix of order n × n, then:
Hence, option (c) is the correct answer.
A(adj A) = (adj A) A = |A| I. Here I is a unit matrix of order n × n.
 1 4 3
 2 −1 4 Theorem: If A is a square matrix then: (adj A)′ = (adj A′)
Example 3. If matrix A =   , then its adjoint will be
 3 2 5 Theorem: If A and B be two non singular square matrices of same
order then adj (AB) = (adj B). (adj A).
 2 −4 2 13 14 11
Theorem: If A is non-singular square matrix of order 3 × 3, then
(a) 7 10 8 (b)  7 4 2
    |adj A| = |A|2 and in general if A is non-singular square matrix of




 3 −7 5  8 7 9
order n × n, then |adj A| = |A|n – 1

ete inants 83
D
rm

E:\AMIT_WORKS\Exam_Guru\EG_Mathematics-12_(working_02-06-2022)\EG_Mathematics-12_working\Open_Files\Chap_4\Chap_4
\ 16-Aug-2022  Amit   Proof-5 Reader’s Sign _______________________ Date __________


EXERCISE 4.3
I. Multiple Choice Questions (MCQs)
 2 3 1
Choose the correct answer from the given options. A =  1 2 −1
5. If   is a non-singular square matrix of



2 3 1  3 4 2
 
1. If matrix A = 1 4 2 , then (adj A) equals
order 3 × 3, then show that |adj A| = |A|2


 3 7 5
cos x sin x 1 0
6 8 −2  5 6 4 6. If A = and A(adj A) = k   , then find
  − sin x cos x 0 1



(a)  1 3 7  (b)  3 2 7 
 




 5 4 5  −1 2 4 the value of k.

 6 −8 2  3 8 −7  III. Long Answer Type Questions


 
(c)  1 7 −3 (d)  3 2 4  1 −1 1





 −5 −5 5  −5 5 5  
1. If A =  0 2 −3 and B = (adj A), and C = 5A, then what



II. Short Answer Type Questions-I  2 1 0

 2 5 1 | adj B |
is the value of ?
1. If matrix A =  −1 2 4 , then verify |C|
 


 4 3 5 2. If k is a scalar and I is a unit matrix of order 3, then find


A (adj A) = (adj A) A = | A | I. the value of adj (kI).



 2 5 2 10 0 
2. If matrix A =  3 1 5 , then verify 3. For any 2 × 2 matrix A, if A(adj A) =   , then
   0 10




 4 −1 −3 find |A|.

(adj A′) = (adj A)′ 3 1


4. If A =  2 −3 , then find |adj A|.

 


 1 2 3  3 1 2
 
3. If matrix A =  1 0 2 and B =  2 5 0 , then  2 0 0
  5. If A =  0 2 0 , then find the value of |adj A|.


 −1 1 −1  4 3 5  0 0 2


verify adj (AB) = (adj B) . (adj A)
6. A is a square matrix of order 3 and |A| = 7. Find the value




 3 7 2 of |adj A|.
 
4. If matrix A =  4 5 3 , then find (adj A) A without
7. If A is a square matrix of order 3 such that |Adj A| = 64,


7 −1 4


then what is the value of |A|?
finding (adj A).
Answers 4.3
3. (adj A) . (adj B)
 6 −8 2




  4. |A| = 26, and (adj A) A = |A| I = 26 I.
I. 1. (c) (adj A) =  1 7 −3


 −5 −5 5  8 −2 −5
5. | A | = 1 and (adj A) =  −5 1 3
 −2 −22 18 


 6 −9  −2 1 1
II. 1. |A| =90; (adj A) =  21
 −11 14 9 and | (adj A) | = 1 ⇒ | (adj A) | = |A|2 = 1.











cos x sin x
1 0 0 6. |A| =
 0 1 0 − sin x cos x



and A(adj A) =(adj A) A = 90  
= cos2 x + sin2 x = 1


0 0 1


A (adj A) =(adj A) A = 90 I = | A |I Since (adj A)A = |A| I2







 2 29 −7  ⇒ (adj A)A = 1I = I



  But A(adj A) = kI
2. (adj A′) =(adj A)′ =  13 −14 22



\ kI = I ⇒k=1



 23 −4 −13





84 Mathe atics–12
m
E:\AMIT_WORKS\Exam_Guru\EG_Mathematics-12_(working_02-06-2022)\EG_Mathematics-12_working\Open_Files\Chap_4\Chap_4
\ 16-Aug-2022  Amit   Proof-5 Reader’s Sign _______________________ Date __________


1 −1 1 1 0 0
III. 1. |A| = 0 2 −3 2. Let I =  0 1 0
0 0 1






2 1 0
= 1(0 + 3) – 0 + 2(3 – 2) = 3 + 2 = 5
1 0 0  k 0 0 


Now, A11 = 3, A12 = – 6, A13 = – 4;
\ kI = k 0 1 0 =  0 k 0 



A21 = 1, A22 = – 2, A23 = – 3; 0 0 1  0 0 k 






A31 = 1, A32 = 3, A33 = 2


 3 1 1 k 2 0 0  1 0 0
adj (kI) = 0 k 0  = k 0 1 0
 2 2
B= adj A =  − 6 − 2 3 
 − 4 − 3 2 2 0 0 1





0 0 k 
B11 = 5, B12 = 0, B13 = 10,
= k2I


B21 = –5, B22 = 10, B23 = 5,





B31 = 5, B32 = –15, B33 = 0 10 0 
3. A(adj A) = 
 0 10


 5 −5 5





adj B =  0 10 −15
10 5 0 1 0


⇒ A(adj A) = 10  = 10 I
0 1




 1 −1 1
= 5  0 2 −3 = 5A We know that A(adj A) = (adj A) A = |A| I
 2 1 0



\ We get |A| = 10
adj B = C ⇒ |adj B| = |C|



\
4. –11 5. 64





| adj B |



\ =1 6. 49 7. ±8
|C|







Topic 4. Inverse of a Non-Singular Square Matrix
We will discuss the concept of inverse of a non-singular square 3 4 
matrix. If A and B are two non-singular square matrices of same Solution. A=   ⇒ | A | = 21 – 4 = 17



1 7 


order and they are such that AB = BA = I, then matrix A is inverse
of matrix B and matrix B is inverse of matrix A. Inverse of a ⇒ Matrix A is non-singular and hence its inverse exists. Let


matrix A is denoted as A–1. us find its cofactors.
Now we know that: (adj A) A = A(adj A) = | A | I A11 = + (7) = 7,




 1    A12 = – (1) = –1,
(adj A ) A = A  A1 (adj A )


⇒  A21 = – (4) = – 4,
 A   





1 A22 = +(3) = 3.
⇒ (adj A ) is inverse of matrix A.

A  7 −1


⇒ Cofactor’s matrix C =  
1
(adj A )  −4 3



Hence A–1 =
A



 7 −4
Also: (i) AI = IA = A ⇒ adj A = C′ =  
 −1 3





(ii)A–1I = IA–1 = A–1
1 1  7 −4


(iii)AA–1 = I and A–1A = I. ⇒
A
( adj A ) = 
17  −1 3
A–1 =





Notes: (i)If a square matrix A has inverse, it is called invertible
Hence, option (b) is the correct answer.


matrix.
(ii) A square matrix A is invertible if and only if matrix A  2 3 1
Example 2. If a square matrix A =  1 4 5 , then find A–1


is a non-singular matrix.  
Theorem: Inverse of non-singular square matrix is unique.  −1 2 3
3 4  –1 and verify that AA–1 = A–1A = I
Example 1. If non-singular matrix A =   , then A equals
1 7   2 3 1
1  −7 −3 1  7 −4 Solution. A =  1 4 5
(a) (b)  


13  −1 4 17  −1 3  −1 2 3




1  −4 −3 1  2 3 ⇒ | A | = 2(12 – 10) – 3(3 + 5) + 1(2 + 4)
(c) (d)


13  2 7  17  5 7 



= 2×2–3×8+1×6






ete inants 85
D
rm

E:\AMIT_WORKS\Exam_Guru\EG_Mathematics-12_(working_02-06-2022)\EG_Mathematics-12_working\Open_Files\Chap_4\Chap_4
\ 16-Aug-2022  Amit   Proof-5 Reader’s Sign _______________________ Date __________


⇒ | A | = 4 – 24 + 6 = –14 On pre-multiplying A–1 on both sides of the above equation, we






⇒ Matrix A is non-singular and hence A exists. –1 have
A–1(AB)(AB)–1 = A–1I


 2 3 1



Let us find cofactors of the matrix A =  1 4 5 ⇒ (A–1A)B(AB)–1 = A–1
 




[Q Multiplication of matrices is associative and A–1I = A–1]

 −1 2 3


Cofactors of matrix A ⇒ IB(AB)–1 = A–1 [Q A–1A = I]




⇒ B(AB)–1 = A–1 [Q IB = B]

A11 = + (12 – 10) = 2 A12 = – (3 + 5) = –8





On pre-multiplying B–1 on both sides, we get




A13 = + (2 + 4) = 6 A21 = – (9 – 2) = – 7


B–1B(AB)–1 = B–1A–1





A22 = + (6 + 1) = 7 A23 = – (4 + 3) = – 7



⇒ I(AB)–1 = B–1A–1 [Q B–1B = I]





A31 = + (15 – 4) = 11 A32 = – (10 – 1) = – 9




⇒ (AB)–1 = B–1A–1





A33 = + (8 – 3) = 5




Hence, (AB)–1 = B–1A–1.





 2 −8 6 3 4  2 1


C =  −7 7 −7  Example 3. If A =   and B =   are two square
1 5 3 7



 11 −9 5 matrices of same order, then verify that (AB)–1 = B–1A–1
[V. Imp.]
 2 −7 11


   3 4   2 1 
⇒ (adj A) = C′ =  −8 7 −9 Solution. A=   , B = 3 7
1 5



   



 6 −7 5
3 4  2 1  18 31
 2 −7 11 ⇒ AB =   = 
1 5  3 7  17 36



1 1 
⇒ (A–1) =
A
(adj A ) = −14  −8 7 −9 ⇒ |AB| = 648 – 527 = 121



 6 −7 5



1  36 −31
⇒ (AB)–1 = ...(1)
 2 −7 11 121  −17 18




−1  
⇒ A–1 = 14  −8 7 −9 | B | = 14 – 3 = 11







 6 −7 5
1  7 −1
⇒ B–1 = , | A | = 11
11  −3 2






 2 3 1  2 −7 11
AA–1 = −1  1 4 5  −8 7 −9 1  5 −4
14   
A–1 =



 −1 2 3  6 −7 5 11  −1 3



 −14 0 0 1 0 0 1  7 −1  5 −4
−1  0 0 = 0 1 0 = I B–1A–1 =
= −14 11 × 11  −3 2  −1 3
14 




 0 0 −14 0 0 1 
Theorem: If A and B are two invertible matrices of same order, 1  36 −31
= ...(2)
then (AB)–1 = B–1A–1. 121  −17 18



Proof: From the definition of inverse of a matrix, we have: (AB) From (1) and (2), we get

(AB)–1 = I (AB)–1 = B–1A–1.


EXERCISE 4.4
I. Multiple Choice Questions (MCQs) 1
(a) det A (b)
Choose the correct answer from the given options. det A




1. Let A be a non-singular matrix of order (3 × 3). Then |adj (c) 1 (d) 0






A| is equal to
 2 3
(a) |A| (b) |A|2 3. If A =  1 −2 and A–1 = aA, then the value of a is
 






(c) |A|3 (d) 3|A|

(a) 7 (b) – 7



2. If A is an invertible matrix of order (2 × 2), then det (A–1)




1
(c) 1 (d) − .


is equal to 7 7




86 Mathe atics–12
m
E:\AMIT_WORKS\Exam_Guru\EG_Mathematics-12_(working_02-06-2022)\EG_Mathematics-12_working\Open_Files\Chap_4\Chap_4
\ 16-Aug-2022  Amit   Proof-5 Reader’s Sign _______________________ Date __________


 2 3 –1 3 4  7 8 
4. If A =   , then 19A is equal to 2. Find matrix [A]2 × 2, if A  = 2
.
 5 −2 1 2 1




(a) A (b) 2A
 3 2   2 5  7 2 




(c) A
1
(d) 3A
3. Find matrix [A]2×2 if  A =  
1 4  2 3  − 4 3



2




−1 III. Long Answer Type Questions
 x y3 1 8  x y
5. If A =   =   , then 2 0  =
2 0   2 0  3 1  4 5


  1. If A =  –1 –1 –1
 , B=   , then verify (AB) = B A .
 4 0  3 4



 0 −2 1 0
(a)   (b)    5 0 4 1 3 3
 −2 1 0 1




   
2. If A =  2 3 2 and B−1 = 1 4 3 , find (AB)–1.
 0 1



 0 −8  2 1 2 1  1 3 4
(c)   (d) 
 −2 1 1 1




 −  1 3 3
2 4
3. If A = 1 4 3 , then verify A–1A = I
 2 2 0 −1  



6. If A =   and B =
–1 –1 –1
 1 0 , then (B A ) = 1 3 4
 −3 2  


1 −1 1 
 2 −2  3 −2
(a)   (b)   4. If A =  2 −1 0 , then A 2 = A–1.
 2 3  2 2  






 1 0 0
 2 2 1 1  3 2
(c)  −2 3 (d)  −2 2  1 tan x 
10 10 5. If A =  , then show that A′A –1 =
    1 




 − tan x


7. If A is an invertible matrix of order 2, then det (A –1) is
cos 2x − sin 2x 


equal to
 sin 2x cos 2x 
(a) det A (b) 1 (c) 1 (d) 0  
det (A)  1 − 1 1








0 1 2  2 2 2
II. Short Answer Type Questions   
6. If A = 1 2 3 and A–1 =  − 4 3 c  , then show that



 2 −1  5 2  2 5  3 a 1   5 3 1
1. Let A =   , B=   and C =   . Find the  2
 3 4 7 4  3 8 2 2 


a = 1, c = –1.
matrix D such that CD – AB = 0.
Answers 4.4
I. 1. (b) |A|2 4. (a) A




Hint: If A is a matrix of order n × n then  2 3
A=  , then

|adj A| = |A|n – 1  5 −2




1
2. (b) 2 3
det A |A| = 5 −2 = – 4 – 15 = – 19 ≠ 0





3. (c) 1
7 A A   −2 −3



adj A =  A11 A 21  = 
a b   d −b   12 22   −5 2


Adjoint of the matrix  c d  is 
   −c a 

adj. A 1  −2 −3 1  2 3
 −2 −3 \ A–1 = = =
\ adj A =  |A| − 19  −5 2 19  5 −2



 −1 2



 2 3
2 3 ⇒ 19A–1 =  5 −2 = A
Also |A| = =–4–3=–7≠0  



1 −2



adj A 1  −2 −3  0 1
⇒ A–1 = =  2
|A| −7  −1 2 5. (d) 



1 1



1  2 3  − 
= 2 4
7  1 −2


 x y3  1 8
1 Given that   = 
⇒ a=  2 0   2 0

7



ete inants 87
D
rm

E:\AMIT_WORKS\Exam_Guru\EG_Mathematics-12_(working_02-06-2022)\EG_Mathematics-12_working\Open_Files\Chap_4\Chap_4
\ 16-Aug-2022  Amit   Proof-5 Reader’s Sign _______________________ Date __________


By equality of matrices, we get
 −1 8 −12 1 3 3

A −1 = −  0 1 −2 , B− 1 = 1 4 3
1
x = 1, y3 = 8 ⇒ y = (8)3 = 2 2.





 x y 1 2  1 −10 15 1 3 4
Now,  2 0  = 
   2 0



 2 − 19 27 
1 2  0 −2
\ adj  =  (AB) − 1 = B− 1A − 1 = −  2 − 18 25
 2 0 
  −2 1




 3 − 29 42
1 2 1 2
Also determinant  = 2 0 =0–4=–4
 2 0  7 −3 −3

−1 −1 3. A −1
=  −1 1 0 and A − 1A = I
 x y 1 2 1  0 −2
= 



\ =  −1 0 1
 2 0   2 0 −4  −2 1



 0 1 0 0 1
 
=  1 12  4. A −1
= A = 0 −1 2
2

 2 − 4 





 1 −1 1

6. (a)  2 −2  1 − tan x   1 − tan x 


 2 3 5. A′A −1 =  × 12 




1  sec x  tan x 1 
 tan x



(B–1 A–1)–1 = (A–1)–1 (B–1)–1 = AB


 2 2 0 −1 0 1 2
=  − 3 2  1 0 6. |A| = 1 2 3
  





3 a 1
 0 + 2 − 2 + 0  2 − 2
=  0 + 2 3 + 0 =  = 0 – 1(1 – 9) + 2(a – 6) = 2a – 4
   2 3




1 The cofactors of different elements of |A|
7. (b)

det (A)



A11 = 2 – 3a, A12 = 8, A13 = a – 6,
Since A is invertible matrix, therefore, AA–1 = I


A21 = 2a – 1, A22 = –6, A23 = 3

Now |AA–1| = | I | = 1 ⇒ |A| |A–1| = 1







1 A31 = –1, A32 = 2, A33 = – 1
⇒ | A–1 | =


|A|



\ A–1 = 1
Adj. A
–11 A



i.e., det (A ) =
det (A)



 2 − 3a 2a − 1 −1
II. 1. Hint: CD = AB 1  8
= 6 2 ...(1)
( 2a − 4)  a − 6 3 −1


⇒ C – 1CD = C– 1(AB) ⇒ D = C – 1 (AB)






−1
 7 8  3 4 
2. Hint: A–1 =     1 − 1 1
1 2 1 2




 2 2 2
Given A–1 =  −4 3 c ...(2)
7 8  1  2 −4  5 3 1




=  ×    2 − 2 2 
1 2 2  −1 3


 2 5 3 2 
−1
 7 2 −1 = 1
From (1) and (2), we get [On comparing a13]
3. Hint: A   =    − 4 3 2a − 4 2


 2 3  1 4




 
2 =c
and [On comparing a23]
1  4 −2  7 2 1  36 2 2a − 4


= × =
10  −1 3  −4 3 10  −19 7  Now, –2 = 2a – 4





−1 ⇒ 2a = 2
1  36 2  2 5 −1  3 −5



and A= =
10  −19 7   2 3 40  −2 3 ⇒ a = 1 and 2
=c



2 (1) − 4



15 19   20 −19
III. 1. AB =   , (AB) −1 = −1  2
16 20 4  −16 15 ⇒
−2
= c





 4 − 5 − 1 − 1  0 − 1 ⇒ c = –1
B–1 =   , A = 4 − 4 3 



− 3 4  Hence, a = 1 and c = –1.


 

88 Mathe atics–12
m
E:\AMIT_WORKS\Exam_Guru\EG_Mathematics-12_(working_02-06-2022)\EG_Mathematics-12_working\Open_Files\Chap_4\Chap_4
\ 16-Aug-2022  Amit   Proof-5 Reader’s Sign _______________________ Date __________


Topic 5. Applications of Matrices
Here we shall discuss to solve the system of linear equations in On putting the values of A–1 and B in X = A–1B, we get


two and three variables and shall also discuss how to check the −1  −1 −2 12
consistency of the system of linear equations. ⇒ X=
7  −2 3  1 




Consistent System of Linear Equations: A system of
linear equations is said to be consistent if its solution (one or −1  −12 − 2 −1  −14
⇒ X = =
more) exists. 7  −24 + 3  7  −21


Inconsistent System of Linear Equations: A system of linear  2  x   2
equations is said to be inconsistent if its solution does not exist. ⇒ X =   ⇒   =   ⇒ x = 2, y = 3
 3  y   3




Solution of system of Linear Equations Using Inverse of Example 2. Using matrix method, find the value of 2x + y – z for
a Matrix: Take a system of linear equations in three variable:
 5x − y + z = 4
 a1x + b1y + c1z = d1 
the following linear equations 3x + 2y − 5z = 2 .

a2 x + b2 y + c2 z = d 2  x + 3y − 2z = 5

a x + b y + c z = d
 3 3 3 3  5x − y + z = 4

This can be expressed in matrix form as: Solution. The given system of equation is: 3x + 2y − 5z = 2
 a1 b1 c1   x   d1   x + 3y − 2z = 5

a b c   y  = d  1  x   4
 2 2 2    2 5 −1
 a3 b3 c3   z   d3      
AX = B ⇒ 3 2 −5  y  =  2


  
 a1 b1 c1   x  d1   1 3 −2  z   5
 a b c  , X =  y  and B =  d  5 −1 1
Let A =  2 2 2     2 ⇒ X = A B , A = 3 2 −5
–1 
 a3 b3 c3   z   d3   



 1 3 −2
Then the above matrix equation can be written as: AX = B . ⇒ | A | = 5 (–4 + 15) + 1(–6 + 5) + 1(9 – 2)





Case I: If A is a non-singular matrix then its inverse exists. = 5 × 11 – 1 + 7 = 61 ≠ 0


On pre-multiplying A–1 on both sides of AX = B, we get: ⇒ Matrix A is non-singular and hence its inverse exists.

Cofactors of matrix A are:
A–1(AX) = (A–1B) ⇒ (A–1A)X = A–1B

A11 = + (– 4 + 15) = 11 A12 = – (– 6 + 5) = 1


[Q Matrix multiplication is associative]








–1
A13 = + (9 – 2) = 7 A21 = – (2 – 3) = 1
[Q A–1A = I]


⇒ IX = A B




A22 = + (– 10 – 1) = –11 A23 = – (15 + 1) = – 16












⇒ X = A −1B [Q IX = X] A31 = + (5 – 2) = 3 A32 = – (– 25 – 3) = 28







A33 = + (10 + 3) = 13



This equation will give you a unique solution of the given



system of linear equation if A–1 exists. This method of solving 11 1 7 11 1 3
 1 −11 −16  1 −11 28
system of linear equations is called Matrix Method. ⇒ C=   ⇒ (adj A) = C′ =  



Case II. If A is a singular matrix, then | A | = 0. In this case  3 28 13  7 −16 13


A–1 does not exists. Now calculate [(adj A) B]. 11 1 3
If [(adj A) B] ≠ 0, then the system of linear equations has 1 1 
no solution. This type of system of linear equations is called
⇒A = –1
A
(adj A ) = 61  1 −11 28



inconsistent system of linear equations.  7 −16 13
If [(adj A) B] = 0, then the system of equations is consistent On putting matrices A–1 and B in X = A–1B, we get

with infinite number of solutions. 11 1 3  4
1 
Example 1. Using matrix method, find the values of x and y for ⇒ X= 1 −11 28  2
61 



3x + 2y = 12  7 −16 13  5
the following linear equations  .
 2x − y = 1  44 + 2 + 15   61 
3x + 2y = 12 1   1  
Solution. The given equations are:  ⇒ X= 4 − 22 + 140  = 122
61  61



 2x − y = 1  28 − 32 + 65  61 
 3 2  x  12 −1 1   x 1 
⇒  2 −1  y  =  1  ⇒ AX = B ⇒ X = A B      
     X = 2 ⇒  y  =  2 ⇒ x = 1, y = 2, z = 1.




 



 3 2 −1  −1 −2 1   z  1 
Here A=   ⇒ A −1 =
 2 −1 7  −2 3





 2x + y – z = 2 (1) + 2 – 1 = 2 + 2 – 1 = 3

ete inants 89
D
rm

E:\AMIT_WORKS\Exam_Guru\EG_Mathematics-12_(working_02-06-2022)\EG_Mathematics-12_working\Open_Files\Chap_4\Chap_4
\ 16-Aug-2022  Amit   Proof-5 Reader’s Sign _______________________ Date __________


EXERCISE 4.5
I. Multiple Choice Questions (MCQs) 2. Using matrix method, solve the following system of



Choose the correct answer from the given options.  2x − 3y + 5z = 11

1. Solution of system of equations 2x + 8y + 5z = 5, 
equations: 3x + 2y − 4z = −5


x + y + z = –2 and x + 2y – z = 2 is  x + y − 2z = − 3
(a) x = –3, y = 2, z = –1 (b) x = –3, y = –2, z = 1 
[Delhi 2011] [A.I. 2017]




(c) x = 3, y = –2, z = –1 (d) None of these




II. Short Answer Type Question-I  2 3 10
x+ y + z = 4
 2x + 5y = 1 
1. Solve by matrix method:   4 6 5
3x + 2y = 7 3. Solve by matrix method:  − + = 1


 x y z



[NCERT Exemplar]  6 9 20
 x + y − z = 2

1 2 1
 
2. If P = 1 −1 −2 , find P −1 , and solve the following [NCERT Exemplar] [Delhi 2011, 2017]




1 1 3  x + y + z = 3
 1 1 1 
system of equations:  2x − y + z = 2  
4. If A = 1 0 2 , find A–1 and use A–1 to solve the
 x − 2y + 3z = 2




3 1 1   x+ y+ z = 6
3. Using matrix method examine the consistency of the 
following system of equations  x + 2z = 7


following system of equations:
3x + y + z = 12
2x + 3y = 10 
(i)  [Delhi 2011, 2012 (C), AI 2011, 2012 (C)]
 7x − 4y = 6


3x + 2y = 16  − 4 4 4  1 −1 1
(ii)  
5. If P = −7 1 3 and A =  1 −2 −2 , find
 
6x − 4y = 14 




 5 −3 −1  2 1 3
 3x + 5y = 7
(iii) 
9x + 15y = 21 PA and use this to solve the following system of



4. Using matrix method, examine the consistency of the  x− y+z = 4



following system of equations: equations  x − 2y − 2z = 9 .
 x + y − 2z = 1  2x − y + z = 4  2x + y + 3z = 1

 
(i)  2x + 3y − 5z = 3 (ii)  x + 2y + 3z = 2 [Delhi 2011, 2012 (C), AI 2011, 2012]




3x − 4y + z = −4 3x − 2y + z = 5

   1 −1 2  −2 0 1
5. Two schools A and B decided to award prizes to their 0 2 −3  9 2 −3
6. Use product    to solve the


students for three values, team, spirit, truthfulness


 3 −2 4  6 1 −2
and tolerance at the rate of ` x, ` y and ` z per student
system of equations:



respectively. School A, decided to award a total of ` 1,100

for the three values to 3, 1 and 2 students respectively while x + 3z = 9, –x + 2y – 2z = 4, 2x – 3y + 4z = –3.

school B decided to award ` 1,400 for the three values to 1,
[Delhi 2017]

2 and 3 students respectively. If one prize for all the three

values together amount to ` 600, then  2 3 1
 

(i) represent the above situation by a matrix equation after 7. If A =  1 2 2 find A–1 and hence solve the system


forming linear equations.


 −3 1 −1
(ii) is it possible to solve the system of equations so of equations:


obtained using matrices?
2x + y – 3z = 13, 3x + 2y + z = 4, x + 2y – z = 8.
(iii) which value you prefer to be awarded most and why?



[Delhi 2015 (C)] [Delhi 2017]


III. Long Answer Type Questions  −4 4 4  1 −1 1
1. Using matrix method, solve the following system of 8. Determine the product:  −7 1 3  1 −2 −2 and
  




 x + 2y − 3z = −4  5 −3 −1  2 1 3

equations:  2x + 3y + 2z = 2 [A.I. 2012] use it to solve the system of equations:

3x − 3y − 4z = 11
 x – y + z = 4, x – 2y – 2z = 9, 2x + y + 3z = 1. [A.I. 2017]


90 Mathe atics–12
m
E:\AMIT_WORKS\Exam_Guru\EG_Mathematics-12_(working_02-06-2022)\EG_Mathematics-12_working\Open_Files\Chap_4\Chap_4
\ 16-Aug-2022  Amit   Proof-5 Reader’s Sign _______________________ Date __________


9. A school wants to award its students for the value of (ii) solve these equations using matrices.




honesty, regularity and hard-work with a total cash award (iii) which values are reflected in this question?



of ` 6000. The three times award money for hard work [A.I. 2013 (C)]
added to that given for the honesty amount to ` 11000. The


13. Two schools P and Q want to award their selected students
award money given for honesty and hard-work together is



on the values of discipline, politeness and punctuality. The
double the one given for regularity. Represent the above school P wants to award ` x each, ` y each and ` z each
situation algebraically and find the award money for each

 
for the three respectively values to its 3, 2 and 1 students
value, using matrix method. with a total award money of ` 1000. School Q wants

 
10. The management committee of a residential colony to spend ` 1,500 to award its 4, 1 and 3 students on the



decided to award some of its members (say x) for honesty, respective values (by giving the same award money for
some (say y) for helping others and some others (say z) the three values as before) If the total amount of awards
for supervising the workers to keep the colony neat and for one prize on each value is ` 600, using matrices, find
clean. The sum of all the awardees is 12. Three times the the award money for each value. [Delhi 2014]


sum of awardees for cooperation and supervision added 14. Two schools A and B want to award their selected students



to two times the number of awardees for honesty is 33. on the values of sincerity, truthfulness and helpfulness. The
If the sum of the number of awardee for honesty and school A wants to award ` x each, ` y each and ` z each,
supervision is twice the number of awarded for helping for the three respectively values to 3, 2 and 1 students
others, using matrix method, find the number of awardee respectively with a total award money of ` 1,600. School
of each category. [A.I. 2013] B wants to spend ` 2,300 to award its 4, 1 and 3 students

11. Two institutions decided to award their employees for on the respective values (by giving the same award money
to the three values as before).


the three values of resourcefulness, competence and
determination in the form of prizes at the rate of ` x, ` y
If the total amount of award for one prize on each value is
and ` z respectively per person. The first institution decided ` 900, using matrices find the award money for each value.
to award respectively 4, 3 and 2 employees with a total [A.I. 2014]

prize money of ` 37000 and the second institution decided 15. A total amount of ` 7,000 is deposited in three different


to award respectively 5, 3 and 4 employees with a total saving bank accounts with annual interest 5%, 8% and
prize money of ` 47000. If all the three prizes per person 1
8 % respectively. The total annual interest from these
together amount to ` 12000. Then using matrix method, 2
three accounts is ` 550. Equal amounts have been deposited
find the values of x, y and z. [Delhi 2013 (C)]
in the 5% and 8% saving account. Find the amount

12. Two factories decided to award their employees deposited in each of the three accounts with the help of


for three values of (a) adaptable to new techniques matrices. [Delhi 2014 (C)]
(b) careful and alert in difficult situations and (c) keeping

16. Two schools P and Q want to award their selected students
calm in tense situations at the rate of ` x, ` y and ` z per


for values of sincerity, truthfulness and hard work at the rate
person respectively. The first factory decided to honour
of ` x, ` y, and ` z for each respective value per student.
respectively 2, 4 and 3 employees with a total prize
School P awards its 2, 3 and 4 students on the above
money of ` 29,000. The second factory decided to honour respective values with a total prize money of ` 4,600.
respectively 5, 2 and 3 employees with the prize money

 
School Q wants to award is 3, 2 and 3 students on the
of ` 30,500. If the three prizes per person together cost respective values with a total award money of ` 4,100.
` 9,500. Then If the total amount of award money for one prize on each
(i) represent the above situation by matrix equations and value is ` 1500, using matrices find out the award money


form linear equations for each value. [A.I. 2014 (C)]

Answers 4.5
I. 1. (a) x = – 3, y = 2, z = – 1 3. (i) Given system of equations is consistent with unique solution.



II. 1. x = 3, y = –1; Hint: | A | = –29 ≠ 0.



 2 −5
Hint: A −1
= −1  (ii) Given system of equations is consistent with unique solution.
11  −3 2



Hint: | A | = –24 ≠ 0



x = 1  −1 −5 −3 (iii) Given system of equations is consistent with infinite
2. y = 1 Hint: P −1 = −1  −5 2 3


solutions.
9  


z = 1  2 1 −3 Hint: | A | = 0 and (Adj A)B = 0.



 −1 −5 2 4. (i) Given system of equations is consistent with infinite



A −1
= P ( )
−1 ′
= −1  −5 2
9
1 solutions.

 −3 3 −3 Hint: | A | = 0 and (Adj A)B = 0.



ete inants 91
D
rm

E:\AMIT_WORKS\Exam_Guru\EG_Mathematics-12_(working_02-06-2022)\EG_Mathematics-12_working\Open_Files\Chap_4\Chap_4
\ 16-Aug-2022  Amit   Proof-5 Reader’s Sign _______________________ Date __________


(ii) Given system of equations is inconsistent with no solution. 8. x = 3, y = –2, z = –1




Hint: | A | = 0 and (Adj A) B ≠ 0.  −4 4 4  1 −1 1




5. Ans. x = ` 100, y = ` 200, z = ` 300  1 3  1 −2 −2
and BA =  −7





Hint: Equations are




 5 −3 −1  2 1 3

3x + y + 2z = 1100
= 8I

x + 2y + 3z = 1400



1

x + y + z = 600 ⇒ A–1 = B , Now solve.
8




 −1 1 −1 9. x = ` 500, y = ` 2000 and z = ` 3500.
1
A = −  2
−1
1 −7 



3 x + y + z = 6000
 1 −2 5 
Hint: Equations are: x + 0y + 3z = 11000



 −6 17 13 x − 2y + z = 0
III. 1. x = 3, y = – 2, z = 1; Hint: A −1 = 1  14 5 −8
67    6 −3 3

 −15 9 −1
and A −1 = 1  2 0 −2
6 


 0 1 −2  −2 3 −1
2. x = 1, y = 2, z = 3 Hint: A −1
=  −2 9 −23 x = 3



 −1 5 −13 
10. y = 4



x = 2  75 150 75 z = 5   x + y + z = 12
 1 
3. y = 3 Hint: A = −1
110 −100 30 
Hint: Equations are: 2x + 3y + 3z = 33
1200 



z = 5

 72 0 −24  x − 2y + z = 0

x = 3  −2 0 2  9 −3 0
1
4. y = 1 Hint: A −1
=  5 −2 −1
 1
and A =  1 0 −1
−1
4 3 




z = 2  1 2 −1  −7 3 1
x=3  x = ` 4000
 
5. y = − 2 Hint: PA = 8 I 11. y = ` 5000


z = ` 3000 



z = − 1 
⇒ PAA–1 = 8IA–1 ⇒ PI = 8A–1 x + y + z = 12000

Hint: Equations are: 4x + 3y + 2z = 37000




1
⇒ 8A–1 = P ⇒ A–1 =

P. 5x + 3y + 4z = 47000
8




 −4 4 4  6 −1 −1
1
\ A −1 =  −7 1 3 and A −1 = −1  −6 −1 2
8 3  



 5 −3 −1  −3 2 −1
 1 −1 2  2 0 1 x = ` 2500
6. x = 0, y = 5, z = 3 and 0 2 −3  −3 2 −3 
   12. y = ` 3000




 3 −2 4  4 1 −2 z = ` 4000
x + y + z = 9, 500
1 0 0 
PB = 0 1 0 Hint: Equations are: 2x + 4y + 3z = 29, 000
 

5x + 2y + 3z = 30, 500


0 0 1
⇒ PB = I ⇒ B = P–1  −6 1 1
 −1
1





−1 and A =  −9 2
 1 0 3

 16 −3 −2
⇒ −1 2 −2
 = (P–1)′
  x = ` 100



 2 −3 4 
13. y = ` 200


7. x = 1, y = 2, z = –3 z = ` 300 


 −4 4 4 3x + 2y + z = 1000
1  5
A = − −–1
1 −3 Hint: Equations are: 4x + y + 3z = 1500
16



 7 −11 1 x + y + z = 600

92 Mathe atics–12
m
E:\AMIT_WORKS\Exam_Guru\EG_Mathematics-12_(working_02-06-2022)\EG_Mathematics-12_working\Open_Files\Chap_4\Chap_4
\ 16-Aug-2022  Amit   Proof-5 Reader’s Sign _______________________ Date __________


 −2 −1 5 x = ` 1125
and A −1 =
−1  −1 2 −5 15. y = ` 1125 
5  




 3 −1 −5 z = ` 4750 
Hint: Equations are:
x = ` 200


 x + y + z = 7, 000
14. y = ` 300   17 −1 1
 1  17 −1 −7 


z = ` 400 
−1
10x + 16y + 17z =1,10, 000 and A =
8


x − y= 
0  −26 2 6
3x + 2y + z = 1, 600 
 x = ` 500
Hint: Equations are: 4x + y + 3z = 2, 300 
16. y = ` 400 


x + y + z = 900 



z = ` 600 
 −2 −1 5 Hint: Equations are:
and A −1 = −1  −1 2 −5


5   x + y + z = 1, 500  1 −1 1

 3 −1 −5  and −1 1 
2x + 3y + 4z = 4, 600 A =  6 0 −2
2


This is same as Q. No. 10. 3x + 2y + 3z = 4,100  −5 1 1

Case Based Questions
1. Two schools A and B want to award their selected students (ii) With reference to Q1, inverse of matrix is given by




on the values of sincerity, truthfulness and helpfulness.  2 −1 5 
The school A wants to award ` x each, ` y each and ` z 1
(a) − 1 2 −5
each for the three respective values to 3, 2 and 1 students 5 


respectively with a total award money at ` 1600. School  3 −1 −5
B wants to spend ` 2300 to award its 4, 1 and 3 students  −2 −1 5 
on the respective values (by giving the same award money 1
(b) − −1 2 −5
to the three values as before). 5


 3 −1 −5

 −2 −1 5 
1 
(c)  −1 2 −5
5


 3 −1 −5

 −2 −1 5
1
If the total amount of award for one piece on each value (d) −1 2 5
5



is ` 900, then answer the following questions.  3 −1 5
(i) Represent the above situation by a matrix equation.
(iii) The award money for sincerity amount to


 3 2 1  x  1600 


     (a) ` 300 (b) ` 200 (c) ` 400 (d) ` 500
(a)  4 1 3  y  =  2300








(iv) The total award money for sincerity and truthfulness


1 1 1  z   900 


amount to
(a) ` 300 (b) ` 400 (c) ` 500 (d) ` 300
 3 2 1  x  1600 








(v) What is the award money for helpfulness?
(b) 4 2 3  y  =  2300



     (a) ` 200 (b) ` 500 (c) ` 300 (d) ` 400


1 1 1  z   900 








 3 2 1  x  1600 
3 2 1  x  1600   4 1 3  y  =  2300
Ans. (i) (a)     
    



(c) 9 1 3  y  =  2300 1 1 1  z   900 


1 1 1  z   900 
 −2 −1 5 
1
3 2 3  x  1600  (ii) (b) − −1 2 −5 (iii) (b) ` 200
5





(d) 9 2 1  y  =  2300  3 −1 −5
    


1 1 1  z   900  (iv) (c) ` 500 (v) (d) ` 400





ete inants 93
D
rm

E:\AMIT_WORKS\Exam_Guru\EG_Mathematics-12_(working_02-06-2022)\EG_Mathematics-12_working\Open_Files\Chap_4\Chap_4
\ 16-Aug-2022  Amit   Proof-5 Reader’s Sign _______________________ Date __________


2. Gaurav purchases 3 pens, 2 bags and 1 instrument box and 3. Manjit wants to donate a rectangular plot of land for




pays ` 41. From the same shop, Dheeraj purchases 2 pens, a school in his village. When he was asked to give
1 bag and 2 instrument boxes and pays ` 29, while Ankur dimensions of the plot, he told that if its length is decreased
purchases 2 pens, 2 bags and 2 instrument boxes and pays by 50 m and breadth is increased by 50 m, then its area
will remain same, but if length is decreased by 10 m and
` 44.
breadth is decreased by 20 m, then its area will decrease
 
by 5300 m2. Based on the information given above, answer
the following questions:

(i) The equations in terms of x and y are



(a) x – y = 50, 2x – y = 550



(b) x – y = 50, 2x + y = 550



Read the above information, answer the following (c) x + y = 50, 2x + y = 550



questions. (d) x + y = 50, 2x + y = 550l


(i) Translate the given problem into a system of equations
(ii) Which of the following matrix equation is represented




(a) 3x + 2y + z = 41 (b) 3x + 2y + 2z = 41 by the given information?




2x + y + 2z = 29 2x + y + 2z = 20  1 −1  x   50  1 1  x   50
(a)     =  (b)    =  


2x + 2y + 2z = 44 2x + 2y + 2z = 44 2 1  y  550  2 1  y  550






(c) 3x + 2y + z = 41 (d) 3x + 2y + z = 41  1 1  x   50  1 1  x   −50
(c)  = (d)    = 




2x + y + 2z = 29 2x + y + z = 29     
 2 −1  y  550  2 1  y   −550






x + 2y + 2z = 44 x + 2y + 2z = 44 (iii) The value of x (length of rectangular field) is


–1


(ii) With reference to Q1, A is equal to (a) 150 m (b) 400 m (c) 200 m (d) 320 m










− 2 − 2 3 − 2 −2 3 (iv) The value of y (breadth of rectangular field) is
1  (b) 1 


(a) − 0 4 − 4 0 4 − 4 (a) 150 m. (b) 200 m. (c) 430 m. (d) 350 m.
4  4












 2 − 2 − 1  2 (v) How much is the area of rectangular field?
− 2 − 1


(a) 60000 sq.m. (b) 30000 sq.m.
− 2 2 3




2 − 2 3 (c) 30000 sq.m. (d) 3000 sq.m.
1 1
 4 − 4




(c) −  0 4 − 4 (d) 0 Ans. (i) (b) x – y = 50, 2x + y = 550
4 4







 2 − 2 − 1 
 2 2 − 1  1 −1  x   50
(ii) (a)  = (iii) (c) 200 m
2 1  y  550





(iii) The cost of one pen is equal to


(a) ` 2 (b) ` 5 (c) ` 10 (d) ` 15 (iv) (a) 150 m (v) (b) 30000 sq.m.





4. Yash scored 40 marks in a test, getting 3 marks for each








(iv) What is the combined cost of one pen and one bag?


right answer and losing 1 mark for each wrong answer.


(a) ` 12 (b) ` 15 (c) ` 17 (d) ` 25
Had 4 marks been awarded for each correct answer and 2








(v) What is the combined cost of one pen and one marks been deducted for each incorrect answer, then Yash


instrument box? would have scored 50 marks. Let number of right answer
(a) ` 7 (b) ` 12 (c) ` 17 (d) ` 25 is x and number of wrong answer is y.








Ans. (i) (a) 3x + 2y + z = 41



2x + y + 2z = 29

2x + 2y + 2z = 44

− 2 − 2 3
1 
(ii) (a) − 0 4 − 4  (iii) (a) ` 2
4





 2 − 2 − 1
(iv) (c) ` 17 (v) (a) ` 7





94 Mathe atics–12
m
E:\AMIT_WORKS\Exam_Guru\EG_Mathematics-12_(working_02-06-2022)\EG_Mathematics-12_working\Open_Files\Chap_4\Chap_4
\ 16-Aug-2022  Amit   Proof-5 Reader’s Sign _______________________ Date __________


Based on the information given above, answer the 5. Three shopkeepers Sanjeev, Rohit and Deepak are using



following questions: polythene bags, handmade bags (prepared by prisoners)
(i) The equations in terms of x and y are and newspaper’s envelope as carry bags. It is found that
the shopkeepers Sanjeev, Rohit and Deepak are using


(a) 3x – y = 40, 2x – y = 25
(20, 30, 40), (30, 40, 20) and (40, 20, 30) polythene bags,


(b) x – 3y = 40, x – y = 25
handmade bags and newspaper’s envelopes respectively.


(c) 3x + y = 40, 2x + y = 25 The shopkeepers Sanjeev, Rohit and Deepak spent ` 250,


(d) x + 3y = 40, 2x + 2y = 25


` 270 and ` 200 on these carry bags respectively.


(ii) Which of the following matrix equations represent the Using the concept of matrices and determinants, answer



above information? the following questions:
1 3  x   40  3 1  x   40 (i) What is the cost of one polythene bag?
(a)  =  (b)    =  



    2 1  y   25 (a) ` 1 (b) ` 2 (c) ` 3 (d) ` 5
1 2  y   25












(ii) What is the cost of one handmade bag?
1 −3  x   40  3 −1  x   40



  =   (a) ` 1 (b) ` 2 (c) ` 3 (d) ` 5
(c)    =   (d) 
1 −2  y   25  2 −1  y   25









(iii) What is the cost of one newspaper envelope?






(iii) Using matrix method, the number of right answer given (a) ` 1 (b) ` 2 (c) ` 3 (d) ` 5











by Yash is (iv) Keeping in mind the social condition, which shopkeeper



(a) 5 (b) 15 (c) 20 (d) 40 is best?
(a) Sanjeev (b) Rohit








(iv) The number of wrong answer given by Yash is





(c) Deepak (d) None of these.


(a) 15 (b) 20 (c) 5 (d) 25




(v) Keeping in mind the environment conditions, which








(v) How many questions were there in the test?


shopkeeper is best?


(a) 20 (b) 25 (c) 35 (d) 40
(a) Sanjeev (b) Rohit








Ans. (i) (a) 3x – y = 40, 2x – y = 25




(c) Deepak (d) None of these.



 3 −1  x   40




Ans. (i) (a) ` 1 (ii) (d) ` 5
(ii) (d)     =   (iii) (b) 15





 2 −1  y   25 (iii) (b) ` 2 (iv) (b) Rohit










(iv) (c) 5 (v) (a) 20 (v) (a) Sanjeev








Author’s Comments
Questions based on following types are very important for Exams. So, students are advised to revise them thoroughly.
1. To find inverse of a non-singular square matrix, i.e. 2 × 2 or 3 × 3 order matrix.


2. To solve a system of linear equation of two variables or three variables. (Most Important)


3. To form the three linear equations of the given word problem and solving them.


4. To find the value of a determinant without expanding the determinant.


IMPORTANT FORMULAE
1. Area of a triangle: If A(x1, y1), B(x2, y2) and C(x3, y3) are the vertices of a DABC. Then the area of the triangle ABC


x1 y1 1
1
D= x y2 1
2 2


x3 y3 1
Note: Take positive value of D if the value of D comes negative.

2. If the three points A(x1, y1), B(x2, y2) and C(x3, y3) are collinear, then


x1 y1 1
x2 y2 1 = 0
x3 y3 1

 a11 a12 a13 
3. If matrix A =  a21 a22 a23  , then
 


 a31 a32 a33 

ete inants 95
D
rm

E:\AMIT_WORKS\Exam_Guru\EG_Mathematics-12_(working_02-06-2022)\EG_Mathematics-12_working\Open_Files\Chap_4\Chap_4
\ 16-Aug-2022  Amit   Proof-5 Reader’s Sign _______________________ Date __________


 A11 A 21 A31 
(i) Adjoint of a square matrix A = A12 A 22 A32  , where Aij is the cofactor of ijth element of matrix A.

 




 A13 A 23 A33 
(ii) A(adj A) = (adj A)A = |A|I (iii) (adj A)′ = (adj A′)




(iv) adj (AB) = (adj B) (adj A) [Matrices A and B are square matrices of same order]



(v) |adj A| = |A|n – 1 If A is a square matrix of order n.


4. Inverse of a square matrix A −1 = 1 [ adj A ]
A


(i) AI = IA = A (ii) AA–1 = I and A–1A = I (iii) A–1I = IA–1 = A–1






5. If A and B are two square matrices of the same order then (AB)–1 = B–1A–1.


a1x + b1y + c1z = d1

6. a2 x + b2 y + c2 z = d 2 is a system of linear equations.


a x + b y + c z = d
 3 3 3 3

 a1 b1 c1   x  d1 
Take A =  a2 b2 c2  ≠ 0 and X =  y  , B =  d 2  then solution of linear equations X = A −1B
     

 a3 b3 c3   z   d3 
 a11 a12 a13 
7. Value of the matrix A =  a21 a22 a23 


 a31 a32 a33 
= a11A11 + a12A12 + a13A13


= a21A21 + a22A22 + a23A23 Expanding along the Rows


= a31A31 + a32A32 + a33A33


= a11A11 + a21A21 + a31A31


= a12A12 + a22A22 + a32A32 Expanding along the Columns


= a13A13 + a23A23 + a33A33


where A11, A12, ..., are cofactors.

COMMON ERRORS
ERRORS CORRECTIONS
(i) Wrong notation for determinants (i) Determinant: | |




a1 + α1 a2 + α 2 a3 + α 3
(ii) b1 + β1 b2 + β 2 b3 + β3 (ii) The determinant will be broken up in any number of




c1 + γ 1 c2 + γ 2 c3 + γ 3 determinant. At a time, only one row/column can be
divided, rest of the rows/columns remains the same.
a1 a2 a3 α1 α 2 α 3
= b1 b2 b3 + β1 β 2 β3
c1 c2 c3 γ1 γ 2 γ 3

(iii) Directly try to expand (iii) By using rules of determinants try to make maximum




number of zeroes in a row or column and expand.
(iv) Students commit error in calculating the value of determnant (iv) Students are advised to calculate value of determinant of




of a matrix because they try it orally. a matrix by using two sticks.
(v) (a) For finding co-factor not taking proper sign. (v) Find co-factors by using (–1)i+j and remember to take




(b) Not taking transpose of a co-factor matrix. transpose.

96 Mathe atics–12
m
E:\AMIT_WORKS\Exam_Guru\EG_Mathematics-12_(working_02-06-2022)\EG_Mathematics-12_working\Open_Files\Chap_4\Chap_4
\ 16-Aug-2022  Amit   Proof-5 Reader’s Sign _______________________ Date __________


REVISION CHART

Determinant
The determinant of a square matrix is a real number associated with the matrix. Every square matrix has a determinant. i.e. To every square
matrix [aij] of order n, we can associate a number (real or complex) called determinant of the square matrix A, where ajj = (i, j)th element of A.
It is denoted by |A| or det. A, if the matrix is denoted by some alphabet (A, B, ...), otherwise it is denoted by D (Delta.)

M inor of an E lement of a M atrix C o-factor

a11 a12 a13  Co-factor of aij element of a matrix is denoted by Aij and defined as Aij = (–1)i+j Mij.
If matrix A = a21 a22 a23  then a11 a12 a13 
a31 a32 a33  Value of the determinant of the matrix A = a21 a22 a23  is denoted by D.
minor aij is denoted by Mij. a31 a32 a33 
Mij is the determinant obtained by D = a11A11 + a12A12 + a13A13 (along first row)
eliminating ith row and jth column of In a similar way the value of the determinant can be calculated along 2nd row, 3rd
the matrix A. row, 1st column, 2nd column and 3rd column.

A rea of a triangle
x1 y1 1
1
If A(x1, y1), B(x2, y2) and C(x3, y3) are vertices of a DABC. Then area of the DABC = x2 y2 1
2
x3 y3 1
(i) The value of this determinant is sometimes negative, take only positive value of this determinant.

(ii) If value of this determinant is zero, then the three points A, B and C are collinear.

Ad joint of a Square M atrix


The adjoint of a square matrix is defined as the transpose of the matrix of the cofactors of the matrix. It is denoted by (adj. A).

I nver e of a non- ingular


s s P ropertie of
s Ad joint
square matrix A : If A and B are square matrix of order n × n and |A| ≠ 0 and |B| ≠ 0, then
   

1 (i) A(adj A) = (adj A)A = | A |I. (ii) adj(AB) = (adj B) (adj A).
A = −1
(adj A)
   

A (iii) | adj A | = |A|n – 1 if A is a matrix of order n × n.



   
   

(iv) (adj A)′ = (adj A′)


   

(v) adj(adj A) = |A|n – 2 A



   

Solution of on i tent Sy tem of inear quation


C s s s L E s

Take a system of linear equations in three variables:


a1x + b1y + c1z = d1

pplication of atrice a2 x + b2 y + c2 z = d 2
a x + b y + c z = d
A s M s

To solve the system of linear equations in two  3 3 3 3

and three variables and the consistency of the This can be written in matrix form as:
system of linear equations.
 a1 b1 c1   x   d1 
 a b c   y  = d 
 2 2 2    2
a3 b3 c3   z  d 3 

ete inants 97
D
rm

E:\AMIT_WORKS\Exam_Guru\EG_Mathematics-12_(working_02-06-2022)\EG_Mathematics-12_working\Open_Files\Chap_5\Chap_5
\ 16-Aug-2022  Amit   Proof-4 Reader’s Sign _______________________ Date __________


5 Continuity And Differentiability
Topics overed
C
5.1 Continuity and Discontinuity of a Function at a Point 5.2 Differentiability of a Function




5.3 Differentiation 5.4 Derivative of a Composite Function




5.5 Product Rule and Quotient Rule of Differentiation 5.6 Implicit Functions




5.7 Differentiation by using Trigonometrical Substitution 5.8 Logarithmic Differentiation




5.9 Differentiation of Parametric Functions 5.10 Higher Order Derivatives




C hapter map
CONTINUITY

Differentiability of a Function Direct Differentiation

Derivative of a Product Rule, Quotient Higher Order


Composite Function Rule of Differentiation Derivatives

Differentiation Logarithmic Differentiation by Implicit


of a Parametric Differentiation using Trigonometrical Functions
function Substitution

Topic 1. Continuity and Discontinuity of a Function at a Point


A function y = f (x), x ∈ D is called continuous at a point x = m ∈ D, if there is no interruption in the graph of f (x) at x = m. The graph
is unbroken at m and there is no gap or hole in the graph of the function at that point. If there is interruption or break in the graph
or there is a gap or hole in the graph of the function at the point x = m ∈ D, then the function y = f (x), x ∈ D is called discontinuous
at x = m ∈ D.

98
E:\AMIT_WORKS\Exam_Guru\EG_Mathematics-12_(working_02-06-2022)\EG_Mathematics-12_working\Open_Files\Chap_5\Chap_5
\ 16-Aug-2022  Amit   Proof-4 Reader’s Sign _______________________ Date __________


How to prove that a given function is continuous or not at a
(iii) RHL = lim+ f ( x ) = lim+ (3x + 2)
given point?




x→2 x→2
1. If a function y = f (x); x ∈ D, has a single definition for all 3 ( 2 + h ) + 2 = lim (8 + 3h )
= hlim


the points in its domain D, then to prove its continuity at →0 h→0



a point x = m ∈ D, work out: = 8+3×0=8+0=8



(i) f (m), (ii) single limit: lim f ( x ) and if f (m) Since f (2) = lim f ( x ) = lim f ( x ) = 8


x → 2− x → 2+






x→m
= lim f ( x ) then the function is continuous at ∴ f (x) is continuous at x = 2.
x→m


x = m ∈ D. Hence, option (b) is the correct answer.
 
 
 x 2 if x ≤ 0
Example 2. The function: f ( x ) =  is
4x + 3 if x > 0
discontinuous at
(a) x = –1 (b) x = 1 (c) x = 0 (d) x = –2









 x 2 if x≤0
Solution. f (x) = 


4x + 3 if x>0



Discontinuity at x = 0:


(i) f (0) = (0)2 = 0





LHL = lim f ( x ) = lim x 2 = lim ( 0 − h )
2
(ii)
x → 0− x → 0− h→0



2. If the given function y = f (x), x ∈ D has two or three

h→0
(
= lim 02 − 2 × 0 × h + h 2 )




definitions for different values of x in the domain D, like:
= lim ( 0 + 0 + h ) = 0 = 0
2 2
 g(x) if x ≥ m h→0


y = f ( x) =  ;
φ(x) if x < m RHL = lim+ f ( x ) = lim ( 4x + 3)

(iii)
x → 0+



x→0
h(x) if x > m
 = lim  4 ( 0 + h ) + 3 = lim [ 4h + 3]
y = f ( x ) =  g(x) if x = m h→0 h→0




φ(x) if x < m
 = 4×0+3=3
Then to prove the continuity or discontinuity of the given


f (0) = lim f ( x ) ≠ lim f ( x )

function at x = m, work out: Since

x → 0− x → 0+



(i) f (m), (ii) LHL = lim f ( x ) ,
h → m− f (x) is discontinuous function at x = 0.




(iii) RHL = lim + f ( x )

\
h→m Hence, option (c) is the correct answer.


If f ( m ) = lim f ( x ) = lim f ( x ) , t h e n f ( x ) i s  3x + x
h → m− +  if x≤0

h→m
Example 3. Prove that the function: f (x) =  x

continuous at x = m.  3 if x=0
is discontinuous function at x = 0.
3x + 2 if x > 2
  3x + x
Example 1. Function f ( x ) = 8 if x = 2 at x = 2 will be a  if x ≤ 0
5x − 2 if x < 2 Solution. f (x) =  x




 3 if x = 0
(a) discontinuous function (b) continuous function
(i) f (0) = 3





(c) signum function (d) None of these



 3x + x 




3x + 2 if x>2 (ii) LHL = lim f ( x ) = lim 
 − x→0 
− x 



x→0
Solution. f (x) = 8 if x=2

3 (0 − h) + 0 − h


5x − 2 if x<2
 = lim
h→0 (0 − h)


Continuity at x = 2:
 3 ( −h ) + − h 


(i) f (2) = 8 = lim  
−h





h→0  
(ii) LHL = lim− f ( x ) = lim− (5x − 2)
(
−3h + h
)
−2h
( )



x→2 x→2
= lim = lim
−h h → 0 −h
= lim 5 ( 2 − h ) − 2 = lim [8 − 5h ]
h→0


= lim ( 2) = 2.
h→0 h→0


= 8–5×0=8 h→ 0




ontinuity nd i e entiability 99
C
A
D
ff
r

E:\AMIT_WORKS\Exam_Guru\EG_Mathematics-12_(working_02-06-2022)\EG_Mathematics-12_working\Open_Files\Chap_5\Chap_5
\ 16-Aug-2022  Amit   Proof-4 Reader’s Sign _______________________ Date __________


(iii) RHL = lim f ( x ) = lim 
x→0 
 3x + x 
x 
= lim
h→0
( 3h + h
h )
= lim
4h
h→0 h
( )
= lim ( 4) = 4
h→0



+ +



x→0

Since f (0) ≠ lim− f ( x ) ≠ lim+ f ( x )


 3 (0 + h) + 0 + h 





x→0 x→0
= lim  
h→0  (0 + h) 


The given function f (x) is discontinuous at x = 0.



\

EXERCISE 5.1
I. Multiple Choice Questions (MCQs) II. Short Answer Type Questions-I
Choose the correct answer from the given options.  cos 3x − cos x
 if x≠0
1. For what value of the constant k, the function 1. Prove that the function f ( x ) =  x2




 kx , if x < 0  −4 if x=0
 is continuous at x = 0.
f ( x) =  x is continuous at x = 0?
3, if x ≥ 0 2. Find the value of k, so that the function



(a) –2 (b) 2 (c) 3 (d) –3  x 2 − 2x − 3 
if x ≠ − 1









2. For what value of k, the function f ( x ) =  x + 1 


 k if x = − 1
 ( x + 3)2 − 36 
 if x ≠ 3
f ( x) =  x−3 becomes continuous at x = –1.
if x = 3



k 3. Find the value of k so that the function

is continuous at x = 3?

 x3 + x 2 − 16x + 20 

(a) 7 (b) 12 (c) 5 (d) 9  if x≠2
f ( x ) =  (x − 2) 2 








 sin 3x if x ≠ 0 
 k if x=2
3. The function f ( x ) =  x is a continuous 
becomes continuous at x = 2.


function at 
 2x + 3 if x = 0

(a) x = 0 (b) x = 1 (c) x = –1 (d) x = 2 
 ( )
3
−x if x<
1








2 2
 x−5 
 if x≠5  1
4. The function f ( x ) =  x − 5 is discontinuous 4. If the function: f ( x ) =  k if x = is a
2




at  1 if x=5 
(a) x = 5 (b) x = –5 (c) x = –1 (d) x = –2
1

2
+ x if ( ) x>
1
2








5. The function f (x) = | x | is continuous at 1
continuous function at x = , then find the value of k.





(a) x = 0 (b) x = 1 (c) x = –1 (d) x = 2 2










5. Find the point of discontinuity of f (x), where f (x) is defined
 x + 2 if x ≤1
6. The function f ( x ) = 


is discontinuous at | x | + 3 if x≤−3
 x − 2 if x >1



as : f ( x ) =  − 2x if −3 < x < 3 .
(a) x = 0 (b) x = 1 (c) x = –1 (d) x = –2  6x + 3 if
 x≥3








 x −1 2
 if x ≠1  1 − cos 2x 
7. The function f ( x ) =  x − 1 is continuous at   if x ≠ 0
6. If the function f ( x ) =  2x 2 


 2 if x =1


 k if x = 0

(a) x = 0 (b) x = 1 (c) x = –1 (d) x = 2
is continuous at x = 0, find the value of k.








8. The function f (x) = 2x – | x | is continuous at






7. Show that the function f (x) defined as:
(a) x = 1 (b) x = 0 (c) x = 3 (d) x = 2










 e3 x − e −5 x 
 , if x ≠ 0  sin x 
9. The function f(x) =  x  x + cos x if x>0


k , if x = 0 
 f ( x) =  2 if x=0
is continuous at x = 0 for the value of k, as [CBSE 2022]


 


(a) 3 (b) 5 (c) 2 (d) 8 x2
   if x<0








2
10. The function f (x) = | x – 3 |, x ∈ R, is continuous at   1 − 1 − x 





(a) x = 2 (b) x = –2 (c) x = 3 (d) x = 0 is continuous at x = 0.









100 Mathe atics–12
m
E:\AMIT_WORKS\Exam_Guru\EG_Mathematics-12_(working_02-06-2022)\EG_Mathematics-12_working\Open_Files\Chap_5\Chap_5
\ 16-Aug-2022  Amit   Proof-4 Reader’s Sign _______________________ Date __________


8. Find the value of a for which the function defined as: 

{ }


 π(x + 1)   1 − cos 4x 
if x < 0
a sin 2
if x≤0   x2

f ( x) =  
5. If the function f ( x ) =  a if x = 0 is
 tan x − sin x if x>0



 x3 
 x

is continuous at x = 0.   if x > 0

 16 + x − 4 

III. Long Answer Type Questions
is continuous at x = 0, find the value of a.
 k cos x  π 6. Find the value of k for which the function:
 π − 2x  if x ≠



2  1 + kx − 1 − kx 
1. If f ( x ) =  is continuous function
π   if −1 ≤ x < 0


 5 if x =  x 
 2 f ( x) = 
  2x + 1
at x = π , find the value of k. [CBSE 2022]   x − 1  if 0 ≤ x ≤ 1
2 


 x sin
2. Prove that the function f ( x ) = 
1
x () if x≠0
is continuous at x = 0.
7. Find the values of a and b so that the function




 0 if x=0  24 if x≤3
is continuous at x = 0. 
f ( x ) = ( 2ax + 3b ) if 3 < x < 5 is continuous at

 5x + 4 − 4 x + 5  36 if x≥5
 if x ≠1 
3. If the function f ( x ) =  x −1 x = 3 and x = 5 respectively.


 
 
 k if x =1
 x − 4 + a if x<4
is continuous at x = 1, find the value of k.  x−4

 (ax + b) if x > 2 8. If f ( x ) =  a + b if x = 4 is continuous at


  x−4
4. If f ( x ) =  (7x − 4) if x = 2 is a continuous function + b if x>4



(3ax − 2b) if x < 2
  x−4
at x = 2, find the values of a and b. x = 4, then find a and b.
 
 
Answers 5.1
I. 1. (d) –3 3. (a) x = 0


 kx Hint:
 if x<0

Hint: f (x) =  −x (i) f (0) = 2 × 0 + 3 = 3





( )
 3



if x≥0
(ii) L.H.L. = lim f ( x ) = lim sin 3x
x → 0− x → 0− x



 −k if x<0
=   sin 3( 0 − h ) 
 3 if x≥0 − sin 3h


= lim 
h→ 0  (0 − h ) 
 = hlim
→0 −h


⇒ f (0) = 3, LHL = –k, RHL = 3

 sin 3h  × 3 = 1 × 3 = 3




⇒ k = –3 = lim 
h → 0  3h  





2. (b) 12 Similarly, RHL = 3



 ( x + 3) − 36 2 Since
 if x≠3

Hint: f (x) =  x−3 f(0) = lim f ( x ) = lim f ( x ) = 3
­
 x → 0− x → 0+






 k if x=3
⇒ f (x) is continuous at x = 0.



 ( x + 3 + 6) ( x + 3 − 6) 4. (a) x = 5
 if x≠3
=  x−3


 − ( x − 5)


 k if x=3 if x−5< 0
 x−5

 x + 9 if x≠3  x−5
Hint: f (x) =  if x−5≥ 0
= 
 x−5




 k if x=3


 1 if x=5
⇒ f (3) = k, LHL = RHL = 12 ⇒ k = 12 




  

ontinuity nd i e entiability 101
C
A
D
ff
r

E:\AMIT_WORKS\Exam_Guru\EG_Mathematics-12_(working_02-06-2022)\EG_Mathematics-12_working\Open_Files\Chap_5\Chap_5
\ 16-Aug-2022  Amit   Proof-4 Reader’s Sign _______________________ Date __________


x<5 2. Hint: f (– 1) = k, LHL = – 4, RHL = – 4
 −1 if



 x3 + x 2 − 16x + 20
=  1 if x≥5 3. Hint: f (x) =
( x − 2)2




 1 if





 x=5

⇒ f (x) ≠ lim− f ( x ) ≠ lim+ f ( x ) ⇒ f (x) =


( x + 5)( x − 2)2 ⇒ f (x) = x + 5



( x − 2)2




x→5 x→5





⇒ f (x) is discontinuous at x = 5.
 x + 5, if x ≠ 2



5. (a) x = 0 f (x) = 


 k, if x = 2


\




Hint: f (0) = lim f ( x ) = lim f ( x ) = 0 Now, prove that f (2) = LHL = RHL = 7

x → 0− x → 0+




()


6. (b) x = 1 1
4. Hint: f = k (Given).
2






Hint: f (1) =3, LHL = 3, RHL = –1. Prove that LHL = 1, RHL = 1 ⇒ k=1



7. (b) x = 1




5. Hint: f (3) = 21, LHL = –6, RHL = 21








 x + 1 if x ≠1 ⇒ f(x) is discontinuous at x = 3.
Hint: f (x) = 


 2 if x =1

6. k = 1





Now f (1) = lim ( x + 1) = lim ( x + 1) = 2 
( )

x →1− x →1+  sin x + cos x , if



x>0
⇒ f (x) is continuous at x = 1.  x




7. Hint: f (x) =  2, if x=0


8. (b) x = 0






 


2
x
Hint: f (x) = 2x – | x | , if x<0
  1 − 1 − x 2 





f (0) = 2 × 0 – | 0 | = 0


(i) f (0) = 2







L.H.L. = lim− f ( x ) = lim  2 ( 0 − h ) − 0 − h  x2 
h→ 0 (ii) LHL = lim 


x→0

x → 0−  1 − 1 − x 2 



= lim ( −2h ) − h  = lim ( −3h ) = 0
h→ 0 h→ 0 On rationalisation, we get



R.H.L. = lim f ( x ) = lim 2x − x  x 2 1 + 1 − x 2  
x → 0+ x → 0+   
= lim  


= lim 2 ( 0 + h ) − 0 + h −  x 2 


x→0  



h→ 0

= lim 2h − h = lim h = 0
= lim− 1 + 1 − x
2
( )


h→ 0 h→ 0


x→0
⇒ f (0) = lim f ( x ) = lim f ( x ) = 0  2
= lim 1 + 1 − ( 0 − h )  = 2

x → 0− x → 0+



h→ 0  


⇒ f (x) is continuous at x = 0.
( )

sin x


9. (d) 8 (iii) RHL = lim + cos x
x → 0+ x





10. (c) x = 3
 sin ( 0 + h ) 


Hint: = lim  + cos ( 0 + h )
h→ 0  0+h 



(i) f(3) = | 3 – 3 | = 0
sin h


 



= lim  + cos h  = 1 + 1 = 2
(ii) LHL = lim 3 − h − 3 = lim − h = 0 h→ 0  h 


h→ 0 h→ 0



(iii) RHL = lim 3 + h − 3 = lim h = 0 ⇒ f (0) = LHL of f (x) = RHL of f (x) = 2.









h→ 0 h→ 0
  π ( x + 1) 
⇒ f(3) = lim f ( x ) = lim f ( x ) = 0 a sin  2  if x≤0
x → 3− x → 3+  



8. Hint: f (x) = 

 tan x − sin x




 −2 sin 2x sin x , if x≠0 if x>0
  x3
II. 1. Hint: f (x) =  x2

 π ( 0 + 1) 
()


 −4, if x=0 π =a
(i) f (0) = a sin   = a sin
 2  2
( )( )




 sin 2x sin x , if
 −4 x≠0
⇒ f (x) =  2x x   π ( x + 1)  

(ii) LHL = lim−  a sin 
2  



 −4, if x=0 x→0  



Now prove that   π ( 0 − h + 1)  
= lim  a sin   

f (0) = –4, LHL = –4 and RHL = –4. h→ 0
  2 





102 Mathe atics–12
m
E:\AMIT_WORKS\Exam_Guru\EG_Mathematics-12_(working_02-06-2022)\EG_Mathematics-12_working\Open_Files\Chap_5\Chap_5
\ 16-Aug-2022  Amit   Proof-4 Reader’s Sign _______________________ Date __________


= a sin
π =a
() Since f (x) is continuous at x =
π
2


2



RHL = lim  tan x −3 sin x 
x → 0+  x  2 ()
f π =LHL = RHL


\




k=k
 tan ( 0 + h ) − sin ( 0 + h )  ⇒ 5= ⇒ k = 10
2 2






= lim  
(0 + h)3
()
h→ 0  


  1 , if
 x sin x≠0
2. Hint: f (x) =  x
 tan h − sin h 






= lim    0, if x=0
h→ 0  h3


(i) f (0) = 0


  1 − cos h  
()




 sin h  cos h   (ii)

LHL = lim  x sin
1 
= hlim   x → 0−  x 




→0  h3 
( )


   1 
= lim ( 0 − h ) sin
h→ 0  0 − h 



()
 tan h   sin h  1 2 
= lim 
h→ 0 

h   h 2  (1 + cos h )  = lim  h sin 1  = 0 × ( any value)
h 


h→ 0 



1 =1 = 0
= 1 × (1) ×
2


()


1+1 2  1  0


(iii) Similarly RHL = lim  x sin =
Since f (x) is continuous at x = 0 x→0 
+ x 



f(0) = LHL = RHL Since f (0) = LHL = RHL = 0






\


1 1 ⇒ f (x) is continuous at x = 0.


⇒ a=a= ⇒ a=   5x + 4 − 4x + 5 
2 2






x −1  if x ≠ 1
3. Hint: f (x) = 
 k cos x
( ) x≠ π  




if  k if x = 1
 π − 2x 2 
III. 1. Hint: f (x) = 
First simplify

π


 5 if x=

 2 5x + 4 − 4x + 5 (5x + 4) − ( 4x + 5)
First simplify cos x =
2 π−x ( ) x −1 ( x − 1)  5x + 4 + 4x + 5 



2
=
( x − 1) =
1
π−x π
Put = y, x → , then y → 0 ( x − 1)  5x + 4 + 4x + 5  5x + 4 + 4x + 5

2 2



π−y (i) f (1) = k

⇒ x=



2
LHL = lim  1 



(ii) 
( ) k →1  5x + 4 + 4x + 5 




cos x cos π − y
2 sin y
( )
= =  
π 2 y 2y 1

\


2 −x = lim  
2 h → 0  5 (1 − h ) + 4 + 4 (1 − h ) + 5 


 
Given: f ( ) = 5
π 1 =1
2 =
3+3 6





  1
(iii) RHL = . Evaluate it yourself.
 k cos x  6



( )
LHL = lim−  
π−x Since f (x) is continuous at x = 1.


x→ π  2



2  2  1
f (1) = LHL = RHL =
6

k sin ( 0 − h )

\


k sin y 1 1 1
= lim− = lim ⇒ k= = ⇒ k=
2y h→ 0 2 (0 − h ) 6 6 6


y→0





4. Hint: f(2) = 10, LHL = 6a – 2b
−k  sin h  = k × 1 = k




= lim   RHL = 2a + b
h → 0 −2  h  2 2




⇒ 6a – 2b = 10 and 2a + b = 10
k



In a similar way evaluate RHL = . Solve these equations.
2


ontinuity nd i e entiability 103
C
A
D
ff
r

E:\AMIT_WORKS\Exam_Guru\EG_Mathematics-12_(working_02-06-2022)\EG_Mathematics-12_working\Open_Files\Chap_5\Chap_5
\ 16-Aug-2022  Amit   Proof-4 Reader’s Sign _______________________ Date __________


 Since f (x) is continuous at x = 0



  1 − cos 4x  if x < 0 f (0) = LHL = RHL
 
 x2 



\


 ⇒ a=8=8 ⇒ a=8
5. Hint: f (x) =  a if x = 0











  1 + kx − 1 − kx
 x 
 , if −1 ≤ x < 0
  if x > 0 6. Hint: f (x) = 
x

( )
16 + x − 4 


 2x + 1





 , if 0 ≤ x ≤1
  x −1

( )
2
 8 sin 2x if x < 0
 2x  2k
 if −1 ≤ x < 0
a if x = 0  1 + kx + 1 − kx
=  = 


 2x + 1



 if 0 ≤ x <1
 x 
 x −1
  if x > 0
 16 + x − 4 
(i) f (0) = k (ii) LHL = k


(i) f (0) = a








(iii) RHL = lim 2x + 1 = −1



 sin 2x 2 
( ) x → 0+ x − 1




(ii) LHL = lim 8 
x → 0−  2x



 ⇒ k = –1




  sin 2 ( 0 − h )  2  7. Hint:
= lim 8   
h→ 0   2 (0 − h )  



(i) Continuity at x = 3 gives 6a + 3b = 24.


 


sin 2h  8 1 = 8
2 (ii) Continuity at x = 5 gives 10a + 3b = 36.
= lim 8  = ×( )
2


h→ 0  2h  Solve them.



(iii) RHL = 8 (Evaluate it) 8. a = 1, b = –1





Topic 2. Differentiability of a Function
Differentiability of a Function at a Point (Graphical (iii) Take a function:
Definition)
 x 2 if x≤2
A function f (x) is differentiable at a point x = c in the domain f ( x) = 

D, if there is no break, gap or hole in the graph of f (x) at x = c. 2x if x>2

Example. and draw its graph. In the
(i) Take a function: y = x + 4, x 2 graph of given function there
∈ R. Draw its graph. In the is a break at x = 2, hence it is
graph of y = x2 + 4, x ∈ R, not differentiable at x = 2.


there is no break, gap or hole
 x 2, if x≤2
at any point. Hence y = x2 + y=
4 is differentiable for all real 2x, if x>2
values of x.
y = x2 + 4, x ∈ R (iv) Take a function
 x 2 if x<2
(ii) Take a function: y = | x |, x ∈ f ( x) = 


R and draw its graph. In the 2x if x>2
graph of y = | x |, x ∈ R there and draw its graph. In the




is a break at x = 0. Hence y = graph there is a hole at x = 2,
| x |, x ∈ R is not differentiable hence the given function is not


at x = 0. differentiable at x = 2.
 x 2, if x<2
y = |x|, x ∈ R y=
2x, if x>2

104 Mathe atics–12


m
E:\AMIT_WORKS\Exam_Guru\EG_Mathematics-12_(working_02-06-2022)\EG_Mathematics-12_working\Open_Files\Chap_5\Chap_5
\ 16-Aug-2022  Amit   Proof-4 Reader’s Sign _______________________ Date __________


(i) It is differentiable in (a, b).
(v) Take a function



(ii) It is right differentiable at x = a.
 x 2 if x≤2



f ( x) =  (iii) It is left differentiable at x = b.



3x if x>2 A function f (x) is differentiable at a point x = x0 ∈ D if its left
and draw its graph. It is clear hand derivative (LHD)
from the graph that there is  f ( x0 − h ) − f ( x0 ) 
a gap in the graph at x = 2. =  lim  is equal to its right hand
 h→ 0 −h
f ( x0 + h ) − f ( x0 ) 


Hence the given function is not 
differentiable at x = 2.
derivative (RHD) =  hlim h 
 →0 


 x 2, if x≤2
y=
Note: (i) If a function f (x) is differentiable at x = x0, then the
3x, if x>2



function f (x) is continuous at x = x0.
(ii) If a function f (x) is continuous at x = x0, then the
Differentiability of a Function at a Point



function f (x) may or may not be differentiable at
Left Hand Derivative Right Hand Derivative x = x0.
(LHD) (RHD) (iii) To prove that a function f (x) is differentiable



at a point x = x 0, prove that its LHD = RHD at
f(x) f(x)
x = x0.
y y Example 1. Show that the function f (x) = 3x2 + 5 is differentiable
at x = 2.
f ( 2 + h ) − f ( 2)
Solution. [R.H.D of f (x) at x = 2] = lim
h



h→0
LHD RHD
 3 ( 2 + h )2 + 5  −  3 ( 2 ) 2 + 5 
= lim    
h→0 h


O x=a
x
O x=a
x
= lim 
(
3 4 + 4h + h 2 + 5 − 17
 )
h→0 h


Differentiability over an Interval 2
lim 17 + 12h + 3h − 17 = lim (12 + 3h )
= h→0
Open interval (a, b) h h→0


y = 12 + 3 × 0 = 12


f ( 2 − h ) − f ( 2)
[LHD of f (x) at x = 2] = lim
h→0 −h

 3 ( 2 − h )2 + 5  −  3 ( 2 ) 2 + 5 
   
= lim
h→0 −h


(
3 4 − 4h + h 2 + 5 − 17
= lim   )
x h→0 −h


O a c b
A function f(x) is said to be differentiable over (a, b) if it is 17 − 12h + 3h 2 − 17
= lim
differentiable at each and every point c Œ (a, b). h→0 −h


Closed interval [a, b] = lim (12 − 3h ) = 12 − 0 = 12
h→0


y
Since [RHD of f (x) at x = 2] = [LHD of f (x) at x = 2]

RHD f (x) is differentiable at x = 2 and f ′(2) = 12.

\

LHD  x 2 if x>5
Example 2. Prove that the function f ( x ) =  is
continuous but not differentiable at x = 5. 5x if x≤5
Solution.
(i) Continuity of f (x) at x = 5:



f (5) = (5x)x = 5 = 5 × 5 = 25
x


O a
A function f(x) is said to be differentiable over [a, b] if:
b
RHL = lim+ f ( x ) = lim+ x = lim (5 + h )
2
( ) h→ 0
2


x →5 x →5

ontinuity nd i e entiability 105


C
A
D
ff
r

E:\AMIT_WORKS\Exam_Guru\EG_Mathematics-12_(working_02-06-2022)\EG_Mathematics-12_working\Open_Files\Chap_5\Chap_5
\ 16-Aug-2022  Amit   Proof-4 Reader’s Sign _______________________ Date __________


= hlim (
25 + 10h + h 2 ) f (1) = (5a + 3b)




lim f ( x ) = lim+ (5ax + 3b ) = lim 5a (1 + h ) + 3b 
→0


= 25 + 10 × 0 + (0)2 = 25 x →1+ h→ 0



x →1


= (5a + 3b)
LHL = lim− f ( x ) = lim− (5x )



lim f ( x ) = lim x 2 + 5 = lim (1 − h ) + 5
2
( )


x →5 x →5
− −  
5 (5 − h ) = lim ( 25 − 5h )
h → 0



x →1 x →1
= hlim
→0 h→0 2 2
= lim 1 + h − 2h + 5 = lim 6 − 2h + h  = 6


h→0 h→0



= 25 – 5 × 10 = 25
Hence 5a + 3b = 6 ...(1)


Since f (5) = lim+ f ( x ) = lim− f ( x ) = 25




x →5 x →5 [RHD of f (x) at x = 1]





f (x) is continuous at x = 5. f (1 + h ) − f (1)
= lim


\
h→0 h



(ii) Differentiability of f (x) at x = 5

5a (1 + h ) + 3b  − [5a + 3b ]


f ( 5 + h ) − f ( 5) = lim 
[R.H.D. of f (x) at x = 5] = lim h
h

h→0



h→0

5a + 5ah + 3b − 5a − 3b 5ah
= lim
(5 + h)2 − (5 × 5) = lim 25 + 10h + h2 − 25 = lim
h
= lim
h→0 h
= 5a
h→0



h→0 h h→0 h


f (1 − h ) − f (1)
[LHD of f (x) at x = 1] = lim
(10 + h) = (10 + 0) = 10 −h


= hlim h→0


→0


[L.H.D. of f (x) at x = 5] (1 − h )2 + 5 − (5a + 3b )

= lim  

f ( 5 − h ) − f ( 5) 5 ( 5 − h ) − ( 5 × 5) −h
= lim = lim h→0


h→0 −h h→0 −h


1 − 2h + h 2 + 5 − 6
25 − 5h − 25 = lim [Q 5a + 3b = 6]
= lim = lim (5) = 5 h→0 −h



h→0 −h h→0


−2h + h 2
Since [R.H.D. of f (x) at x = 5] ≠ [L.H.D. of f (x) at x = 5] = lim 2 − h = 2
= lim
−h


h→0 h→0



Thus f (x) is not differentiable at x = 5.
Since f (x) is differentiable at x = 1



5ax + 3b if x ≥ 1

Example 3. If the function f ( x ) =  2 is [RHD of f (x) at x = 1] = [LHD of f (x) at x = 1]


 x + 5 if x < 1

\
2
differentiable at x = 1, then find the values of a and b. ⇒ 5a = 2 ⇒ a=
5



    
5ax + 3b if x ≥ 1 2
Solution. f (x) =  2 Now put a = in 5a + 3b = 6
 x + 5 if x < 1

5



Since f (x) is differentiable at x = 1, therefore, it is continuous ⇒ 2 + 3b = 6 ⇒ b= 4


also at x = 1. 3



    
2 4
⇒ f (1) = lim f ( x ) = lim f ( x ) , When f (x) is differentiable at x = 1, a = , b =
5 3


x →1+ x →1−

\



EXERCISE 5.2
I. Multiple Choice Questions (MCQs) (a) f is discontinuous at x = 1


Choose the correct answer from the given options. (b) f is differentiable at x = 1


 x + 2, − 1 < x ≤ 3 (c) f is continuous but not differentiable at x = 1



1. If f (x) =  5 , x=3 then at x = 3, f ′(x) = (d) None of these.


 8 − x,


 x>3
II. Short Answer Type Questions-I
(a) 1 (b) – 1




(c) 0 (d) Does not exist. 5ax + 3b if x ≥ 1
1. I f t h e f u n c t i o n f ( x ) =  2 is




 x + 1, when x < 2  x + 5 if x < 1



2. Let f (x) =  , then f ′ (2) =
2 x − 1 when x ≥ 2 differentiable at x = 1, find the values of a and b.


(a) 0 (b) 1 2. For what value of λ, the function defined by:




(c) 2 (d) Does not exist.


λ x 2 + 2 ( )




 x , 0 ≤ x ≤1 if x≤0
3. If f (x) =  , then f ( x) =  is continuous at x = 0?
 2 x − 1, 1< x  4x + 6 if x>0


106 Mathe atics–12
m
E:\AMIT_WORKS\Exam_Guru\EG_Mathematics-12_(working_02-06-2022)\EG_Mathematics-12_working\Open_Files\Chap_5\Chap_5
\ 16-Aug-2022  Amit   Proof-4 Reader’s Sign _______________________ Date __________


3. F i n d t h e v a l u e s o f a a n d b i f t h e f u n c t i o n : x = 1, find the values of a and b.


2
 x if x ≤ 1
f ( x) =  is differentiable at x = 1. 3ax 2 + 2bx − 14 if x > 1
ax + b if x > 1 6. If f ( x ) =  3
is differentiable at
 5ax − 4bx if x ≤ 1



4. Find the values of a and b if the function:
x = 1, find the values of a and b.


 x 2 + 3x + a if x ≤1
f ( x) =  is differentiable at x = 1.
 bx + 2 if x >1



 3ax 2 + 5bx if x ≥ 1
7. If a function defined as f ( x ) = 
5ax − 3b + 18 if x < 1



4ax + b + 10 if x ≥1
5. If f ( x ) =  is differentiable at
2
 4ax − bx if x <1 is differentiable at x = 1, find the values of a and b.



Answers 5.2
f ( x) − f (3) f (3 − h) − f (3) (1 − h) − 1 −h
I. 1. (d) L f ′(3) = lim = lim ,h>0 = lim = lim = lim (1) = 1
x → 3− x−3 h→0 −h h→0 −h h→0 − h h →0






= lim (3 − h + 2) − 5 [ f (3) = 5] f (1 + h) − f (1)
−h R f ′(1) = lim




h→0
h→0 h



5−h−5 −h
= lim= lim = lim (1) = 1 2(1 + h) − 1 − 1
−h h→0 − h = lim


h→0 h→0
h



h→0
f ( x) − f (3) f (3 + h) − f (3)
R f ′(3) = lim = lim
x → 3+ x−3 h→0 h 2 + 2h − 1 − 1


= lim
8 − (3 + h) − 5 8−3−h−5

h


h→0
= lim = lim
h h


h→0 h → 0 2h
= lim = lim (2) = 2
−h h →0 h h →0


= lim = lim (− 1) = – 1
h→0 h h→0


As L f ′(1) ≠ R f ′(1), so f (x) is not differentiable at x = 1.

As R f ′(3) ≠ L f ′(3), so f (x) is not differentiable at x = 3. Hence Hence, (c) is the correct answer.

(d) is the correct answer.

II. 1. Hint: It is given that f (x) is differentiable at x = 1
f (2 + h) − f (2)




2. (d) RHD = R f ′ (2) = lim ∴ f (x) is continuous also at x = 1
h




h→0

(i) Continuity of the f (x) at x = 1 gives: 5a + 3b = 6.

{2(2 + h) − 1} − (2 ⋅ 2 − 1)


= lim (ii) Differentiability of the f (x) at x = 1 gives 5a = 2.  
h→0 h




2. = 3
4 + 2h − 1 − 3 2h


l
= lim = lim = lim 2 = 2 3. a = 2, b = –1
h→0 h h →0 h h →0





Hint: (i) Continuity of f (x) at x = 1 gives: a + b = 1

f (2 − h) − f (2)


LHD = L f ′ (2) = lim (ii) Differentiability of f (x) at x = 1 gives:

h→0 −h





[RHD of f (x) at x = 1] = a
( − h)

2 − h +1− 3

= lim = lim = lim (1) = 1 [LHD of f (x) at x = 1] = 2
−h h→0 − h

h →0


h→0

⇒ a=2
As LHD ≠ RHD, so f ′ (2) does not exist.


4. a = 3, b = 5

Hence, (d) is the correct answer.



3. (c) f (1 – 0) = lim f ( x)  lim f (1  h) Hint: (i) Continuity of f (x) at x = 1 gives a – b = –2


x  1 h0




(ii) Differentiability of f (x) at x = 1 gives b = 5


= lim (1 − h) = 1 – 0 = 1 5. a = –2, b = –4
h→0




Hint: (i) Continuity of f (x) at x = 1 gives: a + 2b = –10
f (1 + 0) = lim f ( x)  lim f (1  h)


x  1 h0 (ii) Differentiability of f (x) at x = 1 gives:





= lim 2(1 + h) – 1 = lim 1 + 2h 2a – b = 0.
h→0 h→0




6. a = 2, b = 3
= 1 + 2(0) = 1


Hint: (i) Continuity of f (x) at x = 1, gives a – 3b = –7


Now, f (1 – 0) = f (1 + 0) = 1


(ii) Differentiability of f (x) at x = 1 gives:



f (1) = 1 ∴ lim f ( x)  1  f (1)

and

x 1 3a – 2b = 0



  



so f (x) is continuous at x = 1 7. a = –5, b = 1



f (1 − h) − f (1) Hint: (i) Continuity of f (x) at x = 1, gives a – 4b = –9
Now, L f ′(1) = lim


h→0 −h (ii) Differentiability of f (x) at x = 1 gives: a + 5b = 0







ontinuity nd i e entiability 107
C
A
D
ff
r

E:\AMIT_WORKS\Exam_Guru\EG_Mathematics-12_(working_02-06-2022)\EG_Mathematics-12_working\Open_Files\Chap_5\Chap_5
\ 16-Aug-2022  Amit   Proof-4 Reader’s Sign _______________________ Date __________


Topic 3. Differentiation
Derivative of a function f (x) is defined as: (iii) y = tan–1 x ⇒ x = tan y






f ( x + h) − f ( x) On differentiating both sides w.r.t. y, we get:
f ′ ( x ) = lim .


h h→0 dx dy 1 1
    
⇒ = sec2 y ⇒ = =
Here the derivative of the function exists if this limit exists dy dx sec 2 y 1 + tan 2 y





d
finitely. The derivative of f (x) is denoted by f ′(x) or  f ( x ) or if dy 1
dx 

=


dy ⇒
y = f (x) by or y′. The process of finding derivative of a function dx 1 + x2





dx
is called differentiation.

dx (
d tan −1 x = 1 , x ∈ R
1 + x2
)



Standard Formulae for Differentiation
dy (iv) y = cot–1 x ⇒ x = cot y
n
= nx n − 1 dy





1. y = x ⇒ 2. y = αx n ⇒ = (α n x n − 1) On differentiating both sides w.r.t. y, we get:
dx dx





dy dy dx
3. y = e x ⇒ = ex 4. y = a x ⇒ = a x log a ⇒ = –cosec2 y
dx dx dy








dy 1 dy = −1 −1 −1
5. y = log e x ⇒ = ⇒ 2 = =
dx x dx cosec y 1 + cot y 1 + x 2
2






6. y = log a x ⇒
dy
=
1
dx x log e a

d cot −1 x = −1
dx (
1 + x2
,x∈R )




dy (v) y = sec–1 x ⇒ x = sec y
dy
7. y = k ⇒




=0 8. y = sin x ⇒ = cos x On differentiating both sides w.r.t. y, we get:
dx dx





dy dx
9. y = cos x ⇒ = − sin x ⇒ = sec y tan y
dx dy





dy dy 1
=
1
10. y = tan x ⇒ = sec 2 x ⇒
dx
=
dx sec y sec y − 1 x x 2 − 1
2





dy
11. y = cot x ⇒
dx
= − cosec 2 x
⇒ (
d sec −1 x = 1
)
, x ∈ R – (–1, 1)


dx x x2 − 1


dy
12. y = sec x ⇒ = sec x tan x
dx (vi) y = cosec–1 x ⇒ x = cosec y






dy On differentiating both sides w.r.t. y, we get:
13. y = cosec x ⇒ = − cosec x cot x

dx dx


⇒ = – cosec y cot y
We have not derived the standard formulae of differentiation for dy




inverse trigonometric functions. Let us do that: dy −1 1
(i) y = sin–1 x ⇒ x = sin y ⇒ = =
dx cosecy cosec y − 1 x x 2 − 1
2







On differentiating both sides w.r.t. y, we get:

dx
dy
= cos y ⇒
dy
=
1
dx cos y

d
dx
(
cosec −1x =
−1
x x2 − 1
)
, x ∈ R – (–1, 1)





dy 1 1 Example 1. If f (x) = ax2 + bx + 5 and f ′(4) = 15 and f ′(2) = 11,
=



⇒ =
dx 1 − sin 2 y 1 − x2 then values of a and b will be



(a) a = 2, b = 7 (b) a = 1, b = 2
d
(
sin −1 x =
1
)




⇒ dx = –1 < x < 1 (c) a = 1, b = 7 (d) a = 3, b = –3
1 − x2






Solution. f (x) = ax2 + bx + 5 ⇒ f ′(x) = a(2x) + b(1) + 0
y = cos–1 x


(ii) ⇒ x = cos y



⇒ f ′(x) = 2ax + b ⇒ f ′(4) = 8a + b = 15 ...(1)




On differentiating both sides w.r.t. y, we get:







f ′(2) = 4a + b = 11 ...(2)

dx dy −1




⇒ = – sin y ⇒ = On solving (1) and (2), we get: a = 1, b = 7.
dy dx sin y






Hence, option (c) is the correct answer.
dy −1
=
−1
⇒ = 1 dy
dx 1 − cos 2 y 1 − x2 Example 2. If y = x + , then : 2x + y equals



x dx

d
dx(cos −1 x = )
−1
1 − x2
, –1 ≤ x ≤ 1
(a) 2 x (b)
x
(c) x x (d) 3 x


2








108 Mathe atics–12
m
E:\AMIT_WORKS\Exam_Guru\EG_Mathematics-12_(working_02-06-2022)\EG_Mathematics-12_working\Open_Files\Chap_5\Chap_5
\ 16-Aug-2022  Amit   Proof-4 Reader’s Sign _______________________ Date __________


1
1 −1 dy 1
Solution. y= x+ = x2 + x 2 ...(1) ⇒ 2x = x− ...(2)
dx x





x



Now (1) + (2) gives:
−1 −3
dy 1 2 1 2 


⇒ dx =  x  −  x  dy 1 1
2  2  ⇒ 2x +y= x− + x+



dx x x




dy 1 1
− dy
⇒ dx = 2 x 2x x ⇒ 2x +y = 2 x



dx




dy 1  1  Hence, option (a) is the correct answer.
⇒ dx = 2x 
x− 
x



EXERCISE 5.3
I. Multiple Choice Questions (MCQs) 3. If y = a sin x + b cos x, then



Choose the correct answer from the given options. 2
dy
2 (a) y 2 +   = a 2 + b2
 1  dy  dx 



1. If f ( x ) =  x −  , then dx is given by
 x


2
 dy  2 2
1 1 1 1 (b) y +   = a + b
(a) 1 − 2 (b) 1 + 2 (c) 1 − x 3 (d) 1 + 3 dx



x x x








2
dy
2. If f (x) = ax2 + bx + 11, and f ′(4) = 61 and f ′(2) = 33, the (c) y −   = a 2 + b 2
 dx 




values of a and b will be
(a) a = 7, b = –3 (b) a = 5, b = –3 dy
2
(d) y 2 −   = a 2 − b2




(c) a = 7, b = 5 (d) a = 3, b = 2  dx 






Answers 5.3
1 dy
1. (a) 1 − 2 2. (c) a = 7, b = 5 Hint: y = a sin x + b cos x. Find and put these values of y
x dx




2
dy 
3. (a) y 2 +  = a 2 + b2 dy dy
2
 dx  and in y 2 +   , you will get a2 + b2.



dx  dx 

Topic 4. Derivative of a Composite Function


A function of function is called composite function. Composite dy − f ′ ( x) −f ′ ( x )
1
functions are differentiated with the help of a rule called Chain 4. y = ⇒ = =
f ( x) dx 2f ( x ) f ( x ) 3


Rule. 2  f ( x ) 2
Chain Rule is the main rule for differentiation. The one who dy
Example 1. If y = (3x2 + 4)5, then is equal to
knows chain rule, knows differentiation. dx
2 3
(a) 20x(x + 4) (b) 30x(3x2 + 5)4

Chain Rule




(c) 30x(3x3 + 5)4 (d) 30x(3x2 + 4)4

If y is a function of u and u is a function of x such that y = f (u) and




2 5
Solution. y = (3x + 4)

u = g(x) then y = f [ g(x)]. Now the chain rule to differentiate it is:


dy
( ) ( )


4 d
dy dy du ⇒ = 5 3x 2 + 4 3x 2 + 4 = 5(3x2 + 4)4 × 6x
× dx dx



=
dx du dx = 30x(3x2 + 4)4


Differentiation of Algebraic Functions by Chain


Hence, option (d) is the correct answer.
Rule 1 dy
Example 2. If y = 3 , then dx is given by
n
1. y =  f ( x ) ⇒
dy
dx
= n  f ( x )
n −1
 f ′ ( x ) 3
2x + 5 ( )


dy − n f ′ ( x ) (a) −18x 2
(b) −18x 2
1
2. y = n ⇒ dx =
(3x + 5) ( 2x + 5 )
3 4




n +1 3 3
 f ( x )  f ( x )


f ′ ( x) (c) 18x 2 (d) 18x 2
dy
f ( x) ⇒
3. y = =
(4x + 3) (3x + 5)
2 3 4




2
dx 2 f ( x )


ontinuity nd i e entiability 109
C
A
D
ff
r

E:\AMIT_WORKS\Exam_Guru\EG_Mathematics-12_(working_02-06-2022)\EG_Mathematics-12_working\Open_Files\Chap_5\Chap_5
\ 16-Aug-2022  Amit   Proof-4 Reader’s Sign _______________________ Date __________


f ′ ( x)
1
d 3
dy −3 dx 2x + 5 ( −18x 2 ) 7. y = sin–1 f (x) ⇒
dy
dx
=


Solution. y = ⇒ = = 1 −  f ( x )
2





(2x + 5) ( ) ( )
3 3 dx 4 4
2x3 + 5 2x3 + 5


dy −f ′ ( x )
Hence, option (b) is the correct answer. 8. y = cos–1 f (x) ⇒ =
dx


2
1 −  f ( x )





Differentiation of Exponential and Logarithmic
Functions by Chain Rule dy f ′ ( x)
9. y = tan–1 f (x) ⇒ =
dy  f ( x )  dx 1 +  f ( x ) 2


f ( x)
 f ′ ( x )





1. y = e ⇒ = e  
dx   


dy − f ′ ( x)
dy  f ( x ) 
2. y = a ( ) ⇒ y = cot–1 f (x) =
f x 10.
 [ log a ]  f ′ ( x )
= a ⇒
dx 1 +  f ( x ) 2


dx 





 


dy f ′ ( x ) f ′ ( x)
3. y = log  f ( x ) ⇒ = y = sec–1 f (x)
dy
dx f ( x) 11. ⇒ =


dx


2
f ( x )  f ( x ) − 1





Logarithmic Formulae
  dy − f ′ ( x)
1. log(xy) = log x + log y 2. log  x  = log x − log y 12. y = cosec–1 f (x) ⇒
dx
=


2
 y f ( x )  f ( x ) − 1









 log e x  dy
3. log(x)m = m log x 4. log y x =  Example 3. If y = sin(4x2 + 5), then dx equals
 log e y 




loge f ( x ) (a) 6x cos (4x2 + 5) (b) 8x cos (4x2 + 5)
6. a a ( ) = f ( x )
log f x
5. e = f ( x)




(c) 3x cos (4x2 + 5) (d) 9x cos (4x2 + 5)




Differentiation of Trigonometric and Inverse



2
Solution. y = sin(4x + 5)
Trigonometric Functions by Chain Rule



1. y= sinm f (x) ⇒
dy
dx
= cos 4x + 5 
2
(d
dx
) (
4x 2 + 5 )







dy

dx
=  m sin (m − 1) f ( x ) cos f ( x )  f ′ ( x )
  = cos ( 4x + 5) (8x ) = 8x cos(4x 2 2
+ 5)
 





m
2. y= cos f (x) Hence, option (b) is the correct answer.




dy  m cos(m − 1) f ( x )  − sin f ( x )  f ′ ( x )
⇒ =   Example 4. If y= 1 − sin x ,
dx



1 + sin x


m
3. y= tan f (x)
( )

dy π x
then 2   + sec 2 − equals



dy  m tan (m − 1) f ( x ) sec 2 f ( x ) f ′ ( x )
⇒ =     dx  4 2
dx



4. y= cotm f (x) (a) 5 (b) 4 (c) 2 (d) 0









( )



dy
⇒ =  m cot (m − 1) f ( x )  − cosec 2 f ( x )  f ′ ( x ) 1 − cos π − x
dx    1 − sin x 2 π x
( )



( )
Solution. y= = = tan −
5. y= secm f (x) 1 + sin x π 4 2


1 + cos − x




dy  2
⇒ = m sec(m − 1) f ( x ) sec f ( x ) tan f ( x ) ×  f ′ ( x )
dx  dy π x −1
( )  dy  + sec 2 π − x = 0
( )


6. y = cosecm f (x) ⇒ = sec 2 −   ⇒ 2
dx 4 2  2   dx  4 2






dy 
⇒ = m cosec(m − 1) f ( x )  − cosec f ( x ) cot f ( x ) ×  f ′ ( x ) Hence, option (d) is the correct answer.
dx 

EXERCISE 5.4
Multiple Choice Questions (MCQs) 70x −70x
Choose the correct answer from the given options. (a) (b)
(15x + 8) (7x + 8)
6 2 6




2
5
dy
1. If y = 3x 2 + 5 ( ) 3 then dx is equal to −50x


( 5x )
6
2 2 (c) 2
+9 (d) None of these
( ) ( 2
)




(a) 5x 3x + 5 2 3 (b) 10x 3x + 5 3




dy
2 2 3. If y = 4x3 + x 2 + 8 then dx is equal to
(
(c) 5x 3x − 5 2
) 3 (d) 10x 3x − 5 ( 2
) 3


5x ( 2x + 1)




x (2x + 5)
1 dy (a) (b)
2. If y = , then equals 3 2
2x + 2x + 2 3x 3 + 2x 2 + 3
(7x + 8)




5
2 dx


110 Mathe atics–12
m
E:\AMIT_WORKS\Exam_Guru\EG_Mathematics-12_(working_02-06-2022)\EG_Mathematics-12_working\Open_Files\Chap_5\Chap_5
\ 16-Aug-2022  Amit   Proof-4 Reader’s Sign _______________________ Date __________


3x (2x + 1) 9. Differential of log [log (log x5)] w.r.t. x is



(c) (d) None of these 5 5
4x3 + 3x 2 + 8 (a) (b)




5 5
x log( x ) log (log x ) x log (log x 5 )





1 dy
4. If y = , then = 5x 4
5x4
3 2
x + 3x + 4x + 5 dx


(c) (d)
log x 5 log (log x 5 )
( ) log ( x 5 ) log (log x 5 )





− 2x 2 + 4x + 3 −3x 2 − 6x + 4
(a) (b) 2(x 2 − 3x 2 + 4x + 5) [CBSE 2022]
(
2 x 3 + 2x 2 + 3x + 5 )


dy




log x
10. If y = e5 log x + 52 log5 x + 7 9 7 , then equals
( 2
) dx



− 3x + 6x + 4 6 7
(c) 3 (d) None of these (a) 5x + 2x + 9x (b) 7x + 3x + 2x8
2





2(x3 + 3x 2 + 4x + 5) 2 (c) 5x3+ 3x + 7x8 (d) 5x4+ 2x + 9x8








dy II. Short Answer Type Questions-I
5. If y = x + x 2 + a 2 , then equals
dx


dy
y 2y 1. If y = esin x + cos x, then find .
dx



(a) x + a2
2 (b) 2x 2 + a 2




3x 2 + tan x dy
y 2. If y = 7 , then find .
dx



(c) x2 − a2 (d) None of these




dy
n
−1 2
3. If y = sin x , then prove ( ) 2x
.
dy
=
6. If y =  x + x + a  , then 1 − x4 dx
2 2



equals
  dx


−1 dy 1
ny yn 4. If y = tan x, then show = .
dx 2 x (1 + x)


(a) x 2 + a 2 (b)
x2 + a2
( )




−1 dy
ny 5. If y = cos x x , then find .
dx


(c) x 2 − a 2 (d) None of these
3




2 dy
7. If y = 6. If y = sin 2 5x + 7, then show that .
x +1 + x − 1 , then dx




dy y dy  15x   12 
(a) x2 − 1   = (b) x 2 − 1   = 2y =  
2 2
 sin 5x + 7 cos 5x + 7  .
 dx  2  dx 




 2 5x 2 + 7   

dy
(c) x2 + 1   = y (d) None of these dy
 dx  7. If y = sec x , then find .




dx


1− x
8. If y = , then 8. If y = cos(log x + ex), then show that
1+ x




dy x 1 x
= − sin log x + e  ×  + e  .
(a) 1 − x 2 (
dy
+ y = 0. ) ( )
(b) 1 + x 2 dy + y = 0. dx x 

dx dx




1  dy  sec x − 1 dy
(c) 2   + y = 0. (d) None of these 9. If y = , then what is the value of ?
x dx sec x + 1 dx






Answers 5.4
2 ny
I. 1. (b) 10x 3x 2 + 5 ( ) 3 2. (b)
−70x
(7x 2 + 8)6
6. (a)
x + a2
2





n

3x ( 2x + 1) (
− 3x 2 + 6x + 4 ) Hint: y =  x + x 2 + a 2 
 



3. (c) 4. (c) 3
4x3 + 3x 2 + 8
( )




2 x3 + 3x 2 + 4x + 5 2
dy
n −1 
2x 
= n  x + x + a 
2 2
y ⇒
  1 + 
dx  2 x2 + a2 



5. (a)
x2 + a2


n −1 
dy x + x2 + a2 
= n  x + x + a  
2 2
Hint: y = x + x2 + a2 ⇒ 
dx    x 2 + a 2 







dy 2x x x2 + a2
⇒ = 1+ = + =
ny
dx 2 x2 + a2 x2 + a2



x + a2
2


dy y dy y

dx
= 7. (a) x2 − 1   =
2
x +a 2  dx  2





ontinuity nd i e entiability 111
C
A
D
ff
r

E:\AMIT_WORKS\Exam_Guru\EG_Mathematics-12_(working_02-06-2022)\EG_Mathematics-12_working\Open_Files\Chap_5\Chap_5
\ 16-Aug-2022  Amit   Proof-4 Reader’s Sign _______________________ Date __________


Hint: y= x +1 + x −1 dy
⇒ (1 – x2) +y=0
dx






dy 1 1
⇒ = +
dx 2 x +1 2 x −1 5



9. (a)
x log( x ) log (log x 5 )
5



1  x +1 + x −1
=  
2 x2 − 1 10. (d) 5x4 + 2x + 9x8




dy y
x2 − 1   =
5 2 9
⇒ Hint: y= e
log x
+ 5log5 x + 7log7 x
 dx  2






⇒ y = x5 + x2 + x9
dy
( )




2
8. (a) 1 − x +y=0
dx II. 1. [esin x + cos x] (cos x – sin x)



( )
1
Hint: y = 1− x 2  3x2 + tan x 2 
1+ x 2. 7 (log 7) × (6x + sec x) 






2 3x 2 + tan x 
dy 1 + x  −1 
⇒ =   −3 x
dx 1 − x  (1 + x )2  5.



 
2 1 − x3



dy − 1− x
⇒ = sec x tan x
dx (1 − x )(1 + x ) 1 + x 7.



4 x


dy 1 − x = −y 1 sec2 x
⇒ (1 – x2) = − 9.
dx 1+ x 2 2





Topic 5. Product Rule and Quotient Rule of Differentiation
Product Rule 2 2
(a) 2x (x + 2) cos(x + 22) − sin (x + 2)
dy
= f ′ ( x) g ( x) + f ( x) g′ ( x) ( x + 2)


1. If y = f (x) g(x), then
dx




2. If y = f (x) g(x) f(x), 3x (x + 3) cos(x + 2) − sin (x + 2)
(b)


(x + 2) 2




then dy = f ′(x)g (x)f(x) + f (x) g′(x) f(x) + f(x) g(x) f′(x)
dx








2 2
Example 1. If y = (3x2 + 2) sin x2, then dy is equal to (c) 3x (x + 3) cos (x + 22) + sin (x + 2)
( x + 2)


dx
2 2 2
(a) 6x cos (3x + 5) + 2 sin x 5x (x 2 + 2) cos2 x + sin 2 x



(b) 3x sin x2 + 2x2 (3x2+ 5) (d)
(x + 2)2




(c) 5x cos x2 (3x2 + 5) + 2 sin x2
( )


(d) 6x sin x2 + 2x (3x2+ 2) cos x2 sin x 2 + 2
Solution. y=
( x + 2)


Solution. y = (3x2 + 2) sin x2





dx  dx  (
dy =  d 3x 2 + 2  sin x 2 + 3x 2 + 2 d sin x 2
dx
) ( ) ( ) dy  ( )  ( ) dx ( )
d sin x 2 + 2  − sin x 2 + 2  d x + 2
( x + 2) dx



⇒ =
= 6x sin x2 + (3x2 + 2)(cos x) 2x dx ( x + 2 )2




= 6x sin x2 + 2x(3x2 + 2)cos x2.
( x + 2)  2x {cos ( x 2 + 2)} − sin ( x 2 + 2)


Hence, option (d) is the correct answer. dy
⇒ =
Quotient Rule dx ( x + 2 )2


f ( x) dy g ( x ) f ′ ( x ) − f ( x ) g ′ ( x )
If y =
g ( x)
then
dx
=
 g ( x )
2
=
( )
2x ( x + 2) cos x 2 + 2 − sin x 2 + 2 ( )
( x + 2) 2

( ) , then dy will be given by




sin x 2 + 2
Example 2. If y =
( x + 2) dx Hence, option (a) is the correct answer.

112 Mathe atics–12


m
E:\AMIT_WORKS\Exam_Guru\EG_Mathematics-12_(working_02-06-2022)\EG_Mathematics-12_working\Open_Files\Chap_5\Chap_5
\ 16-Aug-2022  Amit   Proof-4 Reader’s Sign _______________________ Date __________


EXERCISE 5.5
I. Multiple Choice Questions (MCQs) II. Short Answer Type Questions-I
Choose the correct answer from the given options.
2
1. If y = a log  x + x 2 + a 2  + x dy
dy x 2 + a 2 , then find
1. If y = (3x 2+ 7) 2 (5x 2 − 4)3, then equals 2   2 dx



dx


(a) 6x (3x2 + 7) (5x2 – 4)2 (25x2 + 27) 2
2. If y = − a log  x + x 2 − a 2  + x x 2 − a 2 , then


2
(b) 6x (3x – 7) (5x + 4) (25x – 27) 2 2
2   2





(c) 3x (6x2 + 7) (3x2 – 4)2 (8x2 + 15)
find dy


(d) None of these. dx




dy
2. If y = (cos x3) (sin 2 x5), then equals
dx  a + b cos x dy


3. If y = sin − 1  , then show that
(a) x2 sin x2 [ –3sin x3sin x5)  b + a cos x  dx





(b) x2 sin x5 [ –3 sin x3sin x5 + 10 x2 cos x3cos x5]


(c) x2 sin x5 [ 3 sin x3sin x5 – 10 x2 cos x3cos x5]  − b2 − a 2 
=  b + a cos x 


(d) None of these.  

  


3. If y =
a 2 sin − 1 x + x a 2 − x 2 , then dy
2 a 2 dx ()
is equal to
4. If f ( x) =
x +1
x 2 + 1, g ( x ) =  2 



 x + 1


(a) a2 + x2 (b) a2 – x2




(c) a2 − x2 (d) a2 + x2 and h(x) = (2x – 3), then find f ′[h′{g′(x)}].







Answers 5.5
2 2 2 2
I. 1. (a) 6x(3x + 7)(5x – 4) (25x + 27)
­
II. 1. x2 + a2 2. x2 − a2

­



2 5 3 5 2 3 5
2. (b) x sin x [–3 sin x sin x + 10x cos x cos x ] 2
­
4.


5


3. (c) 2
a −x 2


Topic 6. Implicit Functions
A relation between x and y in which y cannot be expressed easily
⇒ dy = 1 − 2x − y
in terms of x, is called implicit function x2 + y2 + x2y + 7 = 0, dx 3y 2 + x − 1



x3 + sin(xy) = 2 etc are some examples of implicit functions.
Example 2. If y = x + x + x + ............... , then prove that
Differentiation of Implicit Functions
dy = 1 .
To differentiate implicit function, remember: dx 2y − 1
d n
dx
( )
y = ny n −1
dy
dx
and
d n
dy
x = nx n −1
dx
dy
( ) Solution. y= x + x + x + ...............




dy
Example 1. If x2 + y3 + xy = x + y, then find w.r.t. x
dx ⇒ y2 = x + x + x + x + ........... = x + y



Solution. x2 + y3 + xy = x + y
dy dy
On differentiating w.r.t. x, we get ⇒ 2y = 1+
dx dx




dy dy dy
2x + 3y2 +x +y×1= 1+ dy
dx dx dx ⇒ ( 2y − 1) = 1


dx


dy dy 1
⇒ (3y2 + x – 1) = 1 – 2x – y ⇒ =
dx dx 2y − 1






ontinuity nd i e entiability 113
C
A
D
ff
r

E:\AMIT_WORKS\Exam_Guru\EG_Mathematics-12_(working_02-06-2022)\EG_Mathematics-12_working\Open_Files\Chap_5\Chap_5
\ 16-Aug-2022  Amit   Proof-4 Reader’s Sign _______________________ Date __________


EXERCISE 5.6
I. Multiple Choice Questions (MCQs)
2. If sin(xy) – cos(x – y) = y2, then find dy
Choose the correct answer from the given options. dx



1. If x3 + 8xy + y3 = 64, then
dy
= 3. If xy + y2 = y + tan x, then find dy
dx



dx


8x + 3 y 2 2 4. If 1 − x 2 + 1 − y 2 = a ( x − y ) , then prove that
(a) (b) 8 y + 3 x



3x 2 + 8 y 3 y + 8x2




dy 1 − y2
=
3x 2 + 8 y dx 1 − x2


(c) – (d) None of these dx cos a
8x + 3 y 2 5. If sin y = x cos (a + y), then prove that =




dy cos 2 (a + y)



dx
2. If y = x sin y, then = [CBSE 2022]
dy



cos2(a + y)
1 − sin y 1 − x sin y 6. If cos y = x cos (a + y), then prove that dy =
(a) (b) dx sin a



x cos y x cos y




1 − x cos y 1 − x cos y 7. If y = sin x + sin x + sin x + ... , then prove that
(c) (d)



sin y x sin y




dy  cos x 
=
II. Short Answer Type Questions-I
dx  2y − 1

 y
1. Find the value of dy at x = 1, 8. If log x 2 + y 2 = tan − 1 , then prove that
dx  x




π 2
y= if sin y + cos (xy) = k. dy  x + y 
4 =

dx  x − y 

dy dx
2. If 5x2 + 3y2 + xy = 7, then find × .
dx dy


9. If y 1 − x 2 − x 1 − y 2 = 1 , then prove that


III. Long Answer Type Questions
dy 1 − y2
1. If x3 + x2y + xy2 + y3 = 81, then find dy . dx
=
dx 1 − x2




Answers 5.6
3 3
I. 1. (c) We have, x + 8xy + y = 64 dy sin y
⇒ =

Differentiating both sides w.r.t. x, we get dx 1 − x cos y




 dy  dy dx 1 − x cos y
3x2 + 8 1y + x  + 3 y 2 =0 or =
 dx  dx


dy sin y



dy dy Hence, (c) is the correct answer.
⇒ 3x 2 + 8 y + 8 x + 3y2 =0

dx dx II. 1. Hint: sin2 y + cos(xy) = k




dy dy dy
⇒ 3x 2 + 8 y + (8 x + 3 y 2 ) =0 ⇒ 2 sin y cos y − sin ( xy )  x + y  = 0
dx dx  dx






dy  3x 2 + 8 y  π
= –  dy
2 Find and put x = 1 and y =
dx  8x + 3 y  dx 4

\




Hence, (c) is the correct answer.
2. 1

2. (c) y = x sin y


III. 1. Hint: x3 + x2y + xy2 + y3 = 81


Differentiating both sides w.r.t. x, we get


dy dy dy dy
dy ⇒ 3x2 + 2xy + x2 + y2 + 2xy + 3y2 =0
= x cos y + sin y dx dx dx


dx dx


dy
Now find .
dy dx

⇒ (1 − x cos y ) = sin y
dx 2. Hint: sin(xy) – cos(x – y) = y2





114 Mathe atics–12
m
E:\AMIT_WORKS\Exam_Guru\EG_Mathematics-12_(working_02-06-2022)\EG_Mathematics-12_working\Open_Files\Chap_5\Chap_5
\ 16-Aug-2022  Amit   Proof-4 Reader’s Sign _______________________ Date __________


 dy dy dx = cos a
⇒ cos ( xy )  x + y  + sin ( x − y ) 1 −  ⇒
 dx   dx  dy cos 2 (a + y)




dy cos y
= 2y 6. Hint: x =
dx cos (a + y)




dy Now differentiate w.r.t. y.
Now find .
dx



3. Hint: xy + y2 = y + tan x 7. Hint: y = sin x + sin x + sin x + ......





dy dy dy ⇒ y2 = (sin x) + y.
⇒ x + y + 2y = + sec2 x



dx dx dx dy dy dy cos x

⇒ 2y = cos x + ⇒ =
dx dx dx 2y − 1



   
dy
Now find .
dx  y

8. Hint: log x 2 + y 2 = tan −1
 x



4. Hint: 1 − x2 + 1 − y 2 = a ( x − y )


Put x = sin q and y = sin f. ⇒
2
+ ( )
1 log x 2 y 2 = tan −1  y 
 x




1 − sin 2 θ + 1 − sin 2 φ = a (sin θ − sin φ ) dy
x −y
1  2x + 2y dy  = 1 × dx

( )

2 x 2 + y 2  dx  y2 x2



⇒ (cos q + cos f) = a(sin q – sin f) 1+ 2
x

 θ+φ θ − φ  θ+φ θ − φ dy
⇒ 2 cos cos = a 2 cos sin x+ y
dx =  x   1 dy − y 
2
 2 2   2 2  ⇒
 

2 2 
 x + y 2   x dx x 2 
2


x +y
⇒ θ−φ
cot =a
2 dy


Now simplify it and find .
⇒ q – f = 2 cot–1 a dx



⇒ sin–1 x – sin–1 y = 2 cot–1 a
9. Hint: y 1 − x 2 − x 1 − y 2 = 1



1 1dy
⇒ − =0 Put x = sin q and y = sin f
2
1 − y dx 2

1− x

sin f cos q – sin q cos f = 1
5. Hint: sin y = x cos (a + y)

sin(f – q) = sin π



sin y 2



⇒ x=
cos (a + y)



π
⇒ f–q=
Differentiate w.r.t. y 2

  

dx = cos ( a + y ) cos y − sin y {− sin ( a + y ) . 1} ⇒ sin −1 y − sin −1 x = π
⇒ 2
dy cos 2 ( a + y )




1 dy
dx = cos (a + y) cos y + sin (a + y) sin y ⇒ − 1 =0
⇒ 1 − y 2 dx 1 − x2


dy cos 2 ( a + y )


dx = cos ( a + y − y ) ⇒ dy
=
1 − y2

dy cos 2 (a + y) dx 1 − x2




Topic 7. Differentiation by using Trigonometrical Substitution
Before starting differentiation of inverse trigonometric functions, tan A − tan B
revise and commit to your memory the following formulae. 6. tan ( A − B) =
1 + tan A tan B


1. sin (A + B) = sin A cos B + cos A sin B.
2 tan θ


2. sin (A – B) = sin A cos B – cos A sin B. 7. sin 2θ = 2 sin θ cos θ =
1 + tan 2 θ




3. cos (A + B) = cos A cos B – sin A sin B.
8. cos 2q = cos2q – sin2q = 1 – 2 sin2q = 2 cos2q – 1


4. cos (A – B) = cos A cos B + sin A sin B.



1 − tan 2 θ


5. tan (A + B) = tan A + tan B =
1 + tan 2 θ


1 − tan A tan B


ontinuity nd i e entiability 115
C
A
D
ff
r

E:\AMIT_WORKS\Exam_Guru\EG_Mathematics-12_(working_02-06-2022)\EG_Mathematics-12_working\Open_Files\Chap_5\Chap_5
\ 16-Aug-2022  Amit   Proof-4 Reader’s Sign _______________________ Date __________


2 tan θ y = sin −1 sin 2 x  = 2 x
9. tan 2θ = 10. 1 + cos θ = 2 cos 2 θ ⇒
1 − tan 2 θ




2




dy 1
⇒ =
11. 1 − cos θ = 2 sin 2 θ 12. sin 3q = 3 sin q – 4 sin3 q dx x




2




13. cos 3q = 4 cos3 q – 3 cos q Example 2. If y = tan −1 ( 1 + sin x
cos x )
, then find
dy
dx
.


( )
3 tan θ − tan 3 θ
14. tan 3θ = 1 + sin x
1 − 3 tan 2 θ Solution. y = tan −1


cos x



−1  x + y 
15. tan 
 1 − xy 
= tan −1 x + tan −1 y 1 − cos π + x
−1  2 ( ) 


( ) 
⇒ y = tan 
π




 x− y   sin + x
16. tan
−1 −1 −1
 1 + xy  = tan x − tan y  2

2 sin ( π + x )


 2 
   4 2 
17. 2 tan −1 x = tan −1  2x 2 
−1

 2 sin ( + ) cos ( + ) 
= tan 
1 − x  π x π π 





 4 2 4 2 

 ( 4 2) 
18. 2 tan −1 x = sin −1  2x 2   sin π + x 
 = tan  tan ( 4 + 2 )
1+ x 


⇒ y = tan −1  π x  −1

 cos ( + ) 
 π x
 1 − x2   



−1 −1
19. 2 tan x = cos 
 1 + x 2   4 2 


20. sin x = cos ( )
π
2
−x ⇒ y=
π x
+
4 2

dy
dx
= 0+
1
2

dy
dx
=
1
2



  
  


21. sin x = − cos π + x
2 ( ) 22. cos x = sin π + x
2 ( )  3x − x3  −1
Example 3. If y = tan −1  2 ,
 1 − 3x  3
<x<
1
3
, then find
dy
dx
.




23. cos x = sin π − x
2 ( ) 24.
1 − tan x
1 + tan x ( )
= tan
π
4
−x Solution.
 3x − x3 
y = tan −1  2




 1 − 3x 


25.
1 + tan x
1 − tan x
= tan
π
4
+x ( ) Put x = tan q ⇒ q = tan–1 x



  


 3 tan θ − tan 3 θ 
⇒ y = tan −1  2 
 2 tan x   1 − 3 tan θ 
, then find dy .



Example 1. If y = sin −1 
 1 + tan 2 x  dx
⇒ y = tan–1[tan 3q] = 3q = 3 tan–1 x



 2 tan x  dy 3
Solution. y = sin −1  ⇒ =
 1 + tan 2 x  dx 1 + x2





EXERCISE 5.7
I. Multiple Choice Questions (MCQs)  sin x + cos x  dy
3. If y = tan −1   , then is
Choose the correct answer from the given options.  cos x − sin x  dx


1. If f (x) = x tan–1 x, then f ′ (1) =
1 p


π (a) (b) (c) 0 (d) 1
(a) 1 + (b) 1 + π 2 4








4 2 4




II. Short Answer Type Questions-I
1 π
(c) − (d) 2  1 − cos x  dy
2 4 1. If y = tan − 1 




, then find
 sin x  dx


 x + 1 −1  x − 1  dy
2. If y = sec −1  then =
 x − 1  + sin  x + 1  , dx −1 1 − cos x dy


    2. If y = tan , then find
1 + cos x dx


1
(a) 0 (b)
x +1  cos x + sin x  dy




3. If y = tan − 1  , then find
(c) 1 (d) None of these  cos x − sin x  dx






116 Mathe atics–12
m
E:\AMIT_WORKS\Exam_Guru\EG_Mathematics-12_(working_02-06-2022)\EG_Mathematics-12_working\Open_Files\Chap_5\Chap_5
\ 16-Aug-2022  Amit   Proof-4 Reader’s Sign _______________________ Date __________


−1 dy  5 cos x − 12 sin x  dy
4. If y = cot (cosec x + cot x), then find 8. If y = cos −1   , then find dx
dx  13




III. Long Answer Type Questions
 1 + cos x − 1 − cos x 
−1 1 − x dy 9. If y = tan −1   , then find dy
1. If y = cos  1 + x  , then find dx  1 + cos x + 1 − cos x  dx



 


 a−x dy
2. If y = tan − 1   , then find dx  1 + sin x 2 − 1 − sin x 2  dy
 a + x  y = tan −1
  , then find


10. If dx
 1 + sin x 2 + 1 − sin x 2 



−1  2
x +1 
dy
3. If y = tan  x  , then find dx
 1 − 4   1 + x 2 −1


dy
11. If y = tan −1   , then find
2x x dx
− 1 1 − 3   



dy
4. If y = cos  2x  , then find dx
 1 + 3 


 1+ x + 1− x  dy
12. If y = tan −1   , then find
 2 × 5x  dy  1+ x − 1− x  dx



5. If y = sin − 1  2x  , then find dx
1 + 5


 
dy
13. If y = sin  x 1 − x − x 1 − x  , then find
−1 2
−1  a − x dy   dx



6. If y = tan   , then find dx
 1 + ax 


 1 − cos x + 1 + cos x  dy
 ax + b  dy 14. If y = sin −1   , then find dx
7. If y = tan − 1  , then find 2
 a − bx   


dx


Answers 5.7
–1
I. 1. (b) f (x) = x tan x  π 
tan + tan x 



Differentiating w.r.t. x, we get  1 + tan x   4
⇒ y = tan −1  −1
 = tan  

1  1 − tan x   1 − tan π tan x 



f ′ (x) = x × + tan −1 x
1 + x2  4 


x −1 π  π
⇒ f ′ (x) = + tan −1 x ⇒ y = tan tan  + x  = + x
1 + x2 4  4






Put x = 1, then dy
= 1

1 1 π 1 π dx
+ tan −1 (1) =

\


f ′ (x) = + = +
1 + (1) 2 1+1 4 2 4 Hence, (d) is the correct answer.



Hence, (b) is the correct answer. −1 1 − cos x  x x
II. 1. Hint: y = tan  = tan −1  tan  =

 x + 1 −1  x − 1   sin x   2  2



2. (a) If y = sec−1 
 x − 1  + sin  x + 1 
( )




    2. Hint: y = tan −1 1 − cos x = tan −1 tan x = x
1 + cos x 2 2




x −1
−1  −1  x − 1  π
( )
= cos 
 x + 1  + sin  x + 1  = 2 3. Hint: y = tan −1
cos x + sin x


   
cos x − sin x




( ) ( )
 −1 −1 π
−1 1 + tan x −1  π 
Q sin x + cos x = 2  = tan 1 − tan x = tan  tan 4 + x 
  


dy
∴ = d  π  = 0 = π+x
dx dx  2  4





Hence, (a) is the correct answer. 4. Hint: y = cot–1(cosec x + cot x)





 sin x + cos x  −1 1 + cos x 
3. (d) y = tan −1   = cot 
 cos x − sin x   sin x 






 sin x cos x 
 +
cos x 
⇒ ( )
x
y = cot −1 cot =
2
x
2



−1 cos x
= tan 
cos x sin x  1− x
( )


 −  III. 1. Hint: y = cos −1
 cos x cos x  1+ x



ontinuity nd i e entiability 117
C
A
D
ff
r

E:\AMIT_WORKS\Exam_Guru\EG_Mathematics-12_(working_02-06-2022)\EG_Mathematics-12_working\Open_Files\Chap_5\Chap_5
\ 16-Aug-2022  Amit   Proof-4 Reader’s Sign _______________________ Date __________


Put x = tan2 q ⇒ q = tan −1 x ⇒
−1
y = tan x + tan
−1 b
a ()








2 
−1 1 −
⇒ y = cos  tan 2 θ  = cos −1 cos 2θ Now find
dy
.
1 + tan θ  dx




⇒ y = 2θ = 2 tan −1 x 8. Hint: Put 5 = cos α and 12 = sin α
13 13






Now differentiate. −1  1 + cos x − 1 − cos x 
9. Hint: y = tan  

 1 + cos x + 1 − cos x 





a−x
2. Hint: y = tan −1
a+x  2 cos x − 2 sin x 




−1  2 2
Put x = a cos ⇒ θ = cos x
−1 ⇒ y = tan 
x + 2 sin x 
a




2 cos



θ

 2 2 
⇒ y = tan −1 1 − cos θ = θ = 1 cos −1 x
1 + cos θ 2 2 a () 1 − tan x 



⇒ y = tan −1  2
Now differentiate. x




1 + tan 2 

x +1 
−1  2

( )
3. Hint: y = tan  x  π x 
1 − 4  = tan −1  tan − 




 4 2 



  dy −1
x
−1  2 × 2 
π−x
⇒ y= tan ⇒ y= ⇒ =
4 2 dx 2
 x 2
( )








1 − 2   1 + sin x 2 − 1 − sin x 2 
−1
Put 2x = tan q 10. Hint: y = tan  

 1 + sin x 2 + 1 − sin x 2 






 2 tan θ 
⇒ y = tan −1  Write
1 − tan 2 θ 




x 2 + sin x 2
–1 1 + sin x 2 = cos and
⇒ y = tan [tan 2q] = 2q 2 2





⇒ y = 2tan–1(2x) 2 2
1 − sin x 2 = cos x − sin x



Now differentiate. 2 2



2x   2 
 x
4. Hint: y = cos −1 1 − 32x  −1 
sin  x 2 dy
1 + 3  y = tan  2 = ⇒ =x




2 2
x dx




Put 3x = tan  cos 
 2



θ
 2  11. Hint: Put x = tan .
⇒ y = cos −1 1 − tan 2 θ  = cos −1 cos 2θ


θ
1 + tan θ  −1  1 + x + 1 − x 



12. Hint: y = tan  
⇒ y = 2q = 2 tan (3 ) –1 x
 1+ x − 1− x 







Now differentiate. Rationalise Dr.


 2 × 5x   2
5. Hint: y = sin −1  2x  y = tan −1 1 + x + 1 − x + 2 1 − x 
1 + 5  (1 + x ) − (1 − x ) 







Put 5x = tan q ⇒ q = tan–1(5x)
 2





 2 tan θ  −1 1 + 1 − x
⇒ y = sin −1  = sin–1(sin 2q) ⇒ y = tan  
1 + tan 2 θ   x 






⇒ y = 2q = 2 tan–1(5x) Put x = cos q




Now differentiate. dy
Simplify and find .

  dx

6. Hint: y = tan −1  a − x 
 1 + ax  y = sin −1  x 1 − x − x 1 − x 2 




13. Hint:
 




⇒ y = tan −1 a − tan −1 x
 
( x)



2
Now differentiate. y = sin − 1  x 1 − − x 1 − x2 
( )

 


ax + b
7. Hint: y = tan −1
a − bx




Now put x = sin θ and x = sin φ
Divide Nr. and Dr. by a

2 

  y = sin −1 sin θ 1 − sin 2 φ − sin φ 1 − sin θ 
x+ b   
−1 


y = tan  a
b 
()
–1
y = sin [sin q cos f – cos q sin f]


1 − x 


 a  y = sin–1[sin(q – f)] = q – f


118 Mathe atics–12
m
E:\AMIT_WORKS\Exam_Guru\EG_Mathematics-12_(working_02-06-2022)\EG_Mathematics-12_working\Open_Files\Chap_5\Chap_5
\ 16-Aug-2022  Amit   Proof-4 Reader’s Sign _______________________ Date __________


y = sin −1 x − sin −1 x

−1 
y = sin  sin 
 2  ( ) ( )
x  1  + cos x  1  
2  2  


 2




dy

( )
Now find
dx  x π  x π
= sin −1 sin +  = +

 2 4  2 4



 
14. Hint: y = sin −1  1 − cos x + 1 + cos x  Find
dy
.
 2  dx





Topic 8. Logarithmic Differentiation
Function of the type y = [ f (x)]g (x) are differentiated by taking log Solution. y = (x)tan x + (tan x)x





on both the sides Let xtan x = u and (tan x)x = v. Now the above equation



becomes:
log y = log [ f (x)]g (x)
y= u+v







⇒ log y = g(x) log [f (x)] dy du dv
⇒ = +



dx dx dx
 g ( x) f ′ ( x) 




1 dy
⇒ = g ′ ( x ) log  f ( x ) +   Note: If y = u + v, then log y ≠ log u + log v
y dx  f ( x) 




y = xtan x + (tan x)x



dy  g ( x) f ′ ( x)  du dv
⇒ = y  g ′ ( x ) log  f ( x ) + Now let us find and
dx  f ( x ) 
dx dx



Take: u = xtan x ⇒ log u = tan x log x
dy g ( x)  g ( x) f ′ ( x) 



   
⇒ =  f ( x )  g ′ ( x ) log  f ( x ) +
dx f ( x )  1 du
= sec 2 x log x +
tan x



 log
u dx x


Example 1. If y = (sin x + cos x ) (3x2 +5) , then find dy . du tan x  tan x 
dx ⇒ = x sec 2 x log x +
dx  x 



Solution. y = (sin x + cos x ) (3x2 +5) Take v = (tan x)x ⇒ log v = x log tan x





On taking log on both sides, we get 1 dv x sec 2 x
⇒ = log tan x +

log y = (3x2 + 5) log(sin x + cos x) v dx tan x





1 dy
⇒ = (6x) log (sin x + cos x) + (3x2 + 5) × 2
dv = ( tan x ) x log tan x + x sec x 
( )
y dx ⇒  tan x 



cos x − sin x dx 
2



sin x + cos x

dy 2
du dv dy du dv
⇒ = (sin x + cos x)3x + 5 On putting the values of and in = + ,
dx dx dx dx dx dx





(
cos x − sin x 
6x log (sin x + cos x ) + 3x + 5 sin x + cos x 
2
)( ) dy
= x tan x sec 2 x log x +
tan x 

We get +
dx  x 



dy
Example 2. If y = xsin x × (tan x)x, then find .
dx  2 
Solution. y = xsin x× (tan x)x ( tan x )x log tan x + x tan
sec x
x 
 



⇒ log y = log(xsin x) + log(tan x)x



⇒ log y = sin x log x + x log tan x x.... dy
Example 4. If y = x x , then find .
dx




1 dy
y dx (
= cos x log x +
sin x 
x
+  log tan x + x

sec 2 x 
tan x  ) x x....
⇒ y = xx
x....
= x y ⇒ log y = y log x



Solution. y= x


dy
⇒ = xsin x (tan x)x 1 dy y dy dy  1  y
dx − log x =



⇒ = + log x ⇒
y dx x dx dx  y  x



cos x log x + sin x + log tan x + 2x cosec 2x 
 x 

dy  1− y log x  y dy y2
Example 3. Differentiate y = (x)tan x + (tan x)x, then find
dy
.

dx  y  = x ⇒
dx
=
x (1− y log x )



dx

ontinuity nd i e entiability 119


C
A
D
ff
r

E:\AMIT_WORKS\Exam_Guru\EG_Mathematics-12_(working_02-06-2022)\EG_Mathematics-12_working\Open_Files\Chap_5\Chap_5
\ 16-Aug-2022  Amit   Proof-4 Reader’s Sign _______________________ Date __________


EXERCISE 5.8
I. Multiple Choice Questions (MCQs) dy
3. If yx = xy, then find .
Choose the correct answer from the given options. dx



dy
1. If xx = yy, then dy = 4. If y = (log x)(log x), then find .
dx dx





x
(a) − y (b) − 5. If y = sin (xx), then find
dy
.
x y dx






x 1 + log x III. Long Answer Type Questions
(c) 1 + log   (d)
 y 1 + log y




1. If (x)y = ex – y, then prove that dy = log x .
dy dx (1 + log x )2



y x
2. If x × y = 1, then is equal to
dx


x x dy
y ( x log y − y ) y ( x log y − y ) 2. If y = x x or y = x (x ), then find .
(a) (b) dx



x ( y log x − y ) x ( y log x + y )




dy y
y ( x log y + y ) y ( x log y + y ) 3. If xmyn = (x + y)m + n, then prove that = .
(c) (d) − dx x



x ( y log x − x) x ( y log x + x)




dy
4. If (cos x)y = (sin y)x, then find .
dy dx
3. If y = (sin x)tan x, then


is equal to
dx


dy
5. If y = (tan x)tan x ÷ (sin x)sin x, then find .
(a) (sin x)tan x (1 + sec2 x log sin x) dx




(b) tan x (sin x)tan x – 1 cos x dy
6. If y = (x)cot x + (sin x)x, then find


(c) (sin x)tan x · sec2 x · log sin x .
dx




(d) tan x · (sin x)tan x – 1
dy
7. If y = (sin x ) + sin −1 x, then find
x


.
II. Short Answer Type Questions-II dx


(x − 3) (x − 4) (x + 1) dy x ..... dy
1. If y = , then find . 8. If y = x x , then find .
(x 2 + 2) (x3 + 5) dx dx




dy (log x )(log x ) ...... dy
2. If y = (x + 3)2(x + 4)3(x + 5)4, then find . 9. If y = ( log x ) , then find .
dx dx




Answers 5.8
x y
I. 1. (d) x =y
x  dy  y 




Taking logarithms at both sides, we get  + log x  +  + log y  = 0
 y  dx  x 




x log x = y log y
⇒ 1 ( x + y log x ) dy + 1 ( y + x log y ) = 0


Differentiating both sides w.r.t. x, we get y dx x




 1   1  dy 1 dy 1
 x × + log x ⋅ 1 =  y ⋅ y + log y ⋅ 1 dx ⇒ ( x + y log x) = − ( y + x log y )
 x   x


 y dx



dy dy y ( x log y + y )
⇒ (1 + log x) = (1 + log y) ⇒ = −
dx dx x ( y log x + x)






dy 1 + log x Hence, (d) is the correct answer.
⇒ =

dx 1 + log y 3. (d) y = (sin x)tan x







Hence, (d) is the correct answer. Taking logarithms of both sides

log y = tan x · log sin x

2. (d) xy · yx = 1


Differentiating both sides w.r.t. x, we get




Taking logarithms of both sides w.r.t. x


y log x + x log y = 0 [ log 1 = 0] 1 dy 2
⋅ = tan x ⋅ cot x + log sin x ⋅ sec x



Differentiating both sides w.r.t. x, we get y dx



1 dy 1 dy dy
y ⋅ + (log x) + x⋅ + log y ⋅ 1 = 0 ⇒ = y (1 + log sin x sec2 x)
x dx y dx dx





120 Mathe atics–12
m
E:\AMIT_WORKS\Exam_Guru\EG_Mathematics-12_(working_02-06-2022)\EG_Mathematics-12_working\Open_Files\Chap_5\Chap_5
\ 16-Aug-2022  Amit   Proof-4 Reader’s Sign _______________________ Date __________


dy dy xx  x − 1
+ dv (log x) 
tan x
or = (sin x) (1 + sec2 x log sin x) = x x
dx dx  dx 






Hence, (a) is the correct answer.
Now put the value of dv .

dy dx


(x − 3) (x − 4) (x + 1)
II. 1. = 1 3.
dy
=
y
dx 2 (x 2 + 2) (x3 + 5) dx x



Hint: xmyn = (x + y)m + n
 1 3 2 
− 2 x − 3x 
1 1 2




× + + ⇒ m log x + n log y = (m + n) log (x + y)
 x − 3 x − 4 x + 1 x + 2 x + 5



Hint: Take log on both sides and then differentiate. ⇒ m +  n  dy =  m + n  1 + dy 
x  y  dx  x + y   dx 



dy 2 + 3 + 4 
2. = (x + 3)2 (x + 4)3(x + 5)4 × 
dx  x + 3 x + 4 x + 5  m +  n  dy = m + n +  m + n   dy 



Hint: Take log on both sides and then differentiate. x  y  dx x + y  x + y   dx 




dy y ( y − x log y ) Work out dy . It will come .
y
3. =
dx x ( x − y log x ) dx x




Hint: yx = xy dy log (sin y) + y tan x
4. =
log (cos x) − x cot y



⇒ x log y = y log x dx






x dy y dy log Hint: (cos x)y = (sin y)x
⇒ log y + = + x




y dx x dx ⇒ y log cos x = x log sin y





dy dy
Now find . ⇒ −y tan x + log cos x = x cot y + log sin y
dx dx



dy
dy 1 + log ( log x )  Find .
4. = (log x)(log x)  dx

dx x 
  (tan x) tan x


dy
5. = [sec2 x log(tan x) + sec2 x
Hint: y = ( log x )log x dx (sin x)sin x


– (cos x) log(sin x) – cos x]



⇒ log y = (log x) [log(log x)]
Hint: y = [(tan x)tan x] ÷ [(sin x)sin x]



1 dy =  1 log ( log x ) + log x 




y dx x{ } x log x  ( tan x )tan x
  ⇒ y=



(sin x )sin x



dy log x 1 + log ( log x ) 
⇒ = ( log x )  
⇒ log y = tan x log(tan x) – sin x log(sin x)

( )
dx x



 



1 dy tan x sec2 x
dy ⇒ = sec2 x log ( tan x ) +
5. = xx cos (xx) (1 + log x) y dx tan x



dx
( )
− cos x log (sin x ) − sin x cos x


Hint: y = sin (xx) sin x



Put v = xx ⇒ y = sin v dy (tan x) tan x
= [sec2 x log(tan x) + sec2 x



  

dv x dx (sin x)sin x


⇒ = x (1 + log x)
dx – (cos x) log(sin x) – cos x]



We have y = sin v, on differentiating we get dy cot x
6. = (x)cot x  − (cosec 2 x) log x 



dy
= (cos v) dv ...(1) dx  x 


dx dx + (sin x)x [log(sin x) + x cot x]



Put the value of v = x and dv = xx(1 + log x) in eq. (1)
x
Hint: y = (x)cot x + (sin x)x
dx





⇒ y=u+v
dy log x



III. 1. = dy du + dv
⇒ = ...(1)
dx (1 + log x )2



dx dx dx




cot x x
Hint: (x)y = ex – y Now u=x and v = (sin x)



⇒ log u = (cot x)(log x) and log v = x log(sin x)



x
⇒ y log x = x – y ⇒ y=



1 + log x Now find du and dv and put their values in equation (1).





dy log x dx dx

⇒ = dy 1
dx (1 + log x )2 7. = (sin x)x [log (sin x) + x(cot x)] +



dx 2 x 1− x


dy x
Hint: y = (sin x)x + sin −1 x ⇒y=u+v
2. = x x  x x −1 + x x log x (1 + log x )




dx dy


du + dv
x ⇒ = ...(1)
Hint: y = x x = x v where v = xx dx dx dx




u = (sin x)x and v = sin −1 x



Now
dv = xx(1 + log x)



dx ⇒ log u = x log(sin x) and v = sin −1 x





ontinuity nd i e entiability 121
C
A
D
ff
r

E:\AMIT_WORKS\Exam_Guru\EG_Mathematics-12_(working_02-06-2022)\EG_Mathematics-12_working\Open_Files\Chap_5\Chap_5
\ 16-Aug-2022  Amit   Proof-4 Reader’s Sign _______________________ Date __________


Differentiate and find du and dv and put their values in 9.
dy
=
y2
dx dx (x log x) [1 − y log (log x) ]

equation (1). dx



dy y2 ......................
8. = (log x )(log x )
dx x(1 − y log x) Hint: y = ( log x )





 x x.... 
x x....
x


 ⇒ y = (log x)y
Hint: y = x ⇒y=x = xy ⇒ y = xy




⇒ log y = y log(log x)

⇒ log y = y log x







1 dy dy y dy  1  y dy
⇒ = log x + ⇒ − log x = Now differentiate and find .
y dx dx x dx  y  x dx






Topic 9. Differentiation of Parametric Functions
If y = f ( ) and x = g( ) be two functions of same independent
 2 tan θ  = tan −1 tan 2θ = 2θ
y = tan −1  ( )
θ
θ
variable , then such functions are called parametric functions ⇒
1 − tan 2 θ 




θ
and is called a parameter.
θ
y = f (q) dy 2
⇒ y = 2 tan–1 p ⇒ =

dp 1 + p 2


dx


dy





⇒ = f ′(q) and x = g(q) ⇒ = g ′ (θ )
dθ dθ




  
dy  2 
 dy   2
 dθ f ′ (θ )
dy dp  1 + p  dy
dy Now = = =1 ⇒ =1

( )
⇒ = = dx dx  2  dx
g ′ (θ )



  
dx dx



dp  1 + p 2 

 2p  −1  2p  dy
Example 1. If x = sin −1  2  and y = tan 
, then Example 2. If y = 3 cos q – 2 cos3 q and x = 3 sin q – 2 sin3 q, then find
1+ p   1 − p 2  .
dx
dy Solution. y = 3 cos q – 2 cos3 q and x = 3 sin q – 2 sin3 q
prove that = 1.


dx Take y = 3 cos q – 2 cos3 q
 2p  −1  2p 



x = sin −1  dy
Solution. 2  and y = tan  ⇒ = –3 sin q + 6 cos2 q sin q
1+ p   1 − p 2  dθ





 2p  dy
Take x = sin −1  and put p = tan q ⇒ = 3 sin q[2 cos2 q – 1] = 3 sin q cos 2q
 1 + p 2  dθ






⇒ q = tan–1 p Take x= 3 sin q – 2 sin3 q






dx
⇒ x = sin −1  2 tan θ2  = sin −1 [sin 2θ ] = 2θ ⇒ = 3 cos q – 6 sin2 q cos q
1 + tan θ  dθ






= 3 cos q[1 – 2 sin2 q] = 3 cos q cos 2q


dx 2  dy 
⇒ x = 2 tan–1 p ⇒ =
dp 1 + p 2 dy
=  d θ  = 3 sin θ cos 2θ = tan θ






dx  dx  3 cos θ cos 2θ



 2p 
Take y = tan −1  and put p = tan q  dθ 
 1 − p 2 



dy
⇒ = tan q
⇒ q = tan–1 p dx






EXERCISE 5.9
I. Multiple Choice Questions (MCQs) 1 t dy
2. If cos x = and sin y = , then =
Choose the correct answer from the given options. 1 + t2
2 dx


1+ t
dy 1− t 1
1. If x = a (t – sin t) and y = a (1 – cos t), then =
dx (a) –1 (b) 1 (c) (d)


1+ t 2
1 + t2








t t
(a) tan (b) − tan   II. Short Answer Type Questions-II
2 2




 y = 3 sin θ − 2 sin 3 θ and dy
t t 1. If  , then find
(c) cot   (d) − cot   3 dx
2  x = 3 cos θ − 2 cos θ


2




122 Mathe atics–12
m
E:\AMIT_WORKS\Exam_Guru\EG_Mathematics-12_(working_02-06-2022)\EG_Mathematics-12_working\Open_Files\Chap_5\Chap_5
\ 16-Aug-2022  Amit   Proof-4 Reader’s Sign _______________________ Date __________



 x = a cos θ + log tan
2. If 
θ
2 { ( )} and, then find
dy
dx
at θ =
π
4

x = a
5. If 
sin −1 t
and
, then show that
dy −y
dx
=
x
.




 y = a sin θ  y = a cos t
−1
 
 x = a (cos θ + θ sin θ) and dy
3. If  , then find .  x = a (sin θ − cos θ) eθ and dy π
 y = a (sin θ − θ cos θ) dx 6. If  , then find at θ =


θ dx 4
 y = a (sin θ + cos θ) e



 x = a (θ − sin θ) and dy π
4. If  then find at θ = .
y = a (1 + cos θ) dx 3


Answers 5.9
I. 1. (c) x = a (t – sin t) 1
dy




dx 1 + t2
∴ = a (1 − cos t ) dy
= dt = =1
dt



dx dx 1


\


dy dt 1 + t 2
and y = a (1 − cos t ) ⇒ = a sin t
dt



Hence, (b) is the correct answer.


dy
dy a sin t 2 sin t / 2 cos t / 2 dy
∴ = dt = = II. 1. = cot θ
dx


dx dx a (1 − cos t ) 2 sin 2 t / 2



dt Hint: dy = 3 cos θ cos 2θ and


= cos t / 2 = cot t dx = 3 sin θ cos 2θ
sin t / 2 2




Hence, (c) is the correct answer.
2. dy

1  1  =1
∴ x = cos  −1  dx at θ = π


2. (b) cos x = , 4
1 + t2  1 + t2 






  2 

Put t = tan q, then Hint: dy = a cos θ and dx = a  cos θ 
dθ dθ  sin θ 


 1 
x = cos −1  
3.
dy
= tan q
 1 + tan 2 θ 


  dx


dx a θ cos θ, dy a sin
 1    Hint: = = θ θ
= cos −1   = cos −1  1  dθ dθ

 sec2 θ   sec θ 


  dy
4. = − 3
⇒ x = cos–1 (cos q) = q = tan–1 t dx at θ = π


3



dx 1
⇒ = dy dx
dt 1 + t2 Hint: = − 2a sin θ cos θ ; = 2a sin 2 θ



dθ 2 2 dθ 2

t dy y
Again, sin y = 5. = −
dx x



2


1+ t
−1 t −1 t
 t  Hint: xy = a sin × a cos

⇒ y = sin −1    sin −1 t + cos −1 t 
 1 + t2 



 
   2 
=a

 tan α 
Put t = tan a, then y = sin −1 
⇒ xy = a
( π4 )
 1 + tan 2 α 

 


Now differentiate.

 tan α 
−1 dy
⇒ y = sin   6. =1
 sec α  dx at θ = π





4
= sin–1 (sin a) = a = tan–1 t
dx = 2a (sin θ) eθ


Hint:
dy 1 dθ

⇒ =
dt 1 + t2 dy



and = 2a (cos θ) eθ


ontinuity nd i e entiability 123
C
A
D
ff
r

E:\AMIT_WORKS\Exam_Guru\EG_Mathematics-12_(working_02-06-2022)\EG_Mathematics-12_working\Open_Files\Chap_5\Chap_5
\ 16-Aug-2022  Amit   Proof-4 Reader’s Sign _______________________ Date __________


Topic 10. Higher Order Derivatives
Let y = f (x) be a function. dy
On putting the value of (y – b) in ( x − a ) + ( y − b ) = 0, we
dx


dy get
Then first order derivative = = y′ = y1 = f ′ ( x )
dx   dy  2 
1 +  dx   dy
d 2y (x – a) =  
Second order derivative = = y′′ = y2 = f ′′ ( x ) 2
dx 2  d y  dx



 dx 2 
d 3y
Third order derivative = = y′′′ = y3 = f ′′′ ( x ) Now on putting the values of (x – a) and (y – b) in (x – a)2 +
dx3


(y – b)2 = c2, we get
Example 1. If (x – a) 2 + (y – b) 2 = c 2 , then prove that 2 2
  dy  2    dy  2 
3 1 +
  dx   dy 2 1 + 
  dy  2 2
    +   dx   = c2
1 +  dx    d y   dx   d y 
2 2



  = c.  dx 2   dx 2 
d 2y
2
dx 2   dy  2 
1 +  dx     dy  2 
Solution. (x – a)2 + (y – b)2 = c2 2
⇒   1 +    = c


2 dx
 d y 



On differentiating w.r.t. x, we get  
 dx 2 

⇒ ( x − a ) + ( y − b) dy
dx
= 0
  dy  2 
3



1 +  dx  
Again differentiating w.r.t. x, we get  
⇒ = c2

2 2
 d y



2 2
d y  dy 
1 + ( y − b) + = 0  dx 2 
dx 2  dx 


3
  dy  2    dy  2  2
1 +  dx   1 +  dx  
⇒ (y – b) = –  2
 ⇒   = c
 d y 



d 2y



 dx 2 
dx 2

EXERCISE 5.10
I. Multiple Choice Questions (MCQs) 5
(c) (d) None of these.
Choose the correct answer from the given options. 16 t 6




d2y 1 d 2u du
1. If x = at2, y = 2 at, then = 4. u = v3 log , show that v 2 − 2 + 3v 2 =
dx 2 v dv dv




1
(a) − (b) − 1 (a) 1 (b) 2
t2




t3




(c) 0 (d) None of these.




1 1
(c) (d) − II. Long Answer Type Questions
2at 3 2at 3




d 2y
2 d y 2 1. If y = x sin x, then find .
2. If x = 3
, y = t – 1, then = dx 2


t 2
dx 2


d 2y
15 2. If y = sin–1 x, then find .
(a) 15t2 (b) 16t2 (c) (d) 15 t 7 dx 2


16t 3 16








d 2y
d y 2 3. If y = a sin x + b cos x, then prove that + y = 0.
3. Let y = t10 + 1 and x = t8 + 1, then dx 2


=
dx 2


 d 2y 
5 8
4. If y = x + tan x, then prove that cos2 x  2  − 2y + 2x = 0.
(a) t (b) 20t  dx 


 
2




124 Mathe atics–12
m
E:\AMIT_WORKS\Exam_Guru\EG_Mathematics-12_(working_02-06-2022)\EG_Mathematics-12_working\Open_Files\Chap_5\Chap_5
\ 16-Aug-2022  Amit   Proof-4 Reader’s Sign _______________________ Date __________


5. If y = tan x + sec x, then prove that 2
(1 − x )  ddxy  − x  dy
2  +m y = 0
2


2
2d y dx 
2


(1 − sin x)  2  = cos x.
 dx 

21. If y = cos(cos x), then prove that



6. If y = cosec x + cot x, then prove that  d 2y   dy 
 dx 2  − ( cot x )  dx  + y sin x = 0
2


 d 2y 


(1 − cos x)2  2  = sin x.
 dx  22. If y = (sin x + cos x)ex, then prove that

 


2
7. If y = a sin mx – b cos mx, then prove that d y2 + m 2 y = 0 d 2y dy
− 2   + 2y = 0
dx dx 2  dx 



 d 2y   dy  −1
8. If y = tan–1 x, then prove that (1 + x 2)  2  + 2x
 dx 
=0 23. If y = sin x , then prove that
 dx  1 − x2


 
 


 d 2y   dy   d 2y  dy
9. If y = sin–1 x, then prove that (1 − x 2)  2  − x
 dx 
=0 (1 − x 2)  2  − 3x   − y = 0
 dx   dx   dx 


 
 

10. If y = cosec–1 x, then prove that  d 2y  dy
24. If y = (sin x)ex, then show that  2  − 2   + 2y = 0


2
 d y dy  dx   dx 
x (x 2 − 1)  2  + (2x 2 − 1)   = 0



 dx   dx 
25. If xy = aex + be–x + x2, then prove that



11. If y = (tan–1 x)2 , then prove that  d 2y  dy
x  2  + 2   − xy + x 2 − 2 = 0


 d 2y   dx   dx 

dy
(x 2 + 1) 2  2  + 2x (x 2 + 1)   = 2
 dx   dx  d 2y  dy 

26. If y = (ax + b)e3x, then prove that 2 − 6  dx  + 9y = 0
12. If y = (cot–1 x)2, then show that dx




 d 2y  dy  y = θ sin θ + cos θ and d 2y
( x 2 + 1) 2  2  + 2x ( x 2 + 1)   = 2 27. If  , then find .
 dx   dx   x = θ cos θ − sin θ dx 2



2
d 2 y  1   dy  y
13. If y = (x)x, then prove that 2 −   dx  − x = 0  y = 3 sin θ − 2 sin 3 θ and d 2y π
dx  y 28. If  , then find 2 at θ = 2


3 dx
 x = 3 cos θ − 2 cos θ


14. If y = Peax + Qebx, then prove that


 −1  2t 
d 2y  dy  =0  y = sin  and
2 − (a + b)  dx  + aby   1 + t 2  d 2y
dx

29. If  , then find .
4
 x = cos −1  1 − t 
2 dx 2


15. If y =  x + x 2 + 1 , then prove that
    1 + t 2 



2
 d y dy
(1 + x 2)  2  + x   − 16y = 0  x = a cos θ + b sin θ
 dx   dx  30. If  , then prove that

 y = a sin θ − b cos θ


16. If y = sin(log x), then prove that


 d 2y  dy
y2  2  − x   + y = 0
2
2d y dy
x  2  + x  + y = 0  dx   dx 
 dx 

 dx 

17. If y = a cos(log x) + b sin(log x), then prove that  x = cos θ


2 31. If  , then prove that
2d y dy
x  2  + x  + y = 0
3
 y = sin θ


 dx   dx 

2
 d2y dy
n n
y  2  +   = 3 sin2 q (5 cos2 q – 1)
18. If y = P  x + x 2 − 1 + Q  x − x 2 − 1 , then prove that  dx   dx 
   



2 d 2y
( x − 1)  ddxy  + x  dy
2
2
 −n y = 0
dx 
2
32. If xmyn = (x + y)m + n, then prove that
dx 2
=0



−1x
19. If y = e m cos , then prove that d 2y
33. If ey(x + 1) = 1, then prove that = 0
dx 2




2
(1 − x )  ddxy  − x  dy
2
2
 −m y = 0
dx 
2
34. If y = ex(tan–1 x), then prove that



 d2y
 sin −1 y 
20. If x = sin 
 m 
, then prove that ( ) (
1 + x2  2  − 2 1 − x + x2
 dx 
dy
dx
2
)
+ (1 − x ) y = 0



ontinuity nd i e entiability 125
C
A
D
ff
r

E:\AMIT_WORKS\Exam_Guru\EG_Mathematics-12_(working_02-06-2022)\EG_Mathematics-12_working\Open_Files\Chap_5\Chap_5
\ 16-Aug-2022  Amit   Proof-4 Reader’s Sign _______________________ Date __________


Answers 5.10
dx d 2u
I. 1. (d) x = at2, ∴ = 2at ⇒ = − 5v − 6v log v
dt dv 2








dy d 2u du
and y = 2at, ∴ = 2a
dt v −2 = − 5v 2 − 6v 2 log v + 2v 2 + 6v 2 log v





2
dv dv


\


dy
dy 2a 1 d 2u du
Now = dt = = ⇒ v 2
−2 + 3v 2 = 0
dv dv




dx dx 2at t



dt Hence, (c) is the correct answer.


2
d y d2y d2y x
∴ = d  dy  × dt = d 1 1
 × II. 1. = 2 cos x − x sin x 2. = 3
dx 2 dt  dx  dx dt  t  2at dx 2 dx 2






(1 − x 2) 2
1 1 1 3. Hint: y = a sin x + b cos x
× =− = −





t 2 2at 2at 3 dy


⇒ = a cos x – b sin x
Hence, (d) is the correct answer. dx





2 dx − 4 d 2y
2. (d) x= 2, ∴ = 3 ⇒ = –a sin x – b cos x = –y
t dt t dx 2










4. Hint: y = x + tan x
dy





Also y = t3 – 1, ∴ = 3t 2
dt dy d 2y
= 1 + sec2 x = 2 sec2 x tan x





⇒ ⇒
dx dx 2





dy
dy 3t 2 3 2
Now,
dx
= dt =
dx  − 4 
= − t5
4
⇒ (cos x) ddxy
2
2
= 2 tan x = 2y – 2x





dt  t 3  2

d y d  dy  dt 2
3 1 15 7
⇒ (cos x) ddxy
2
2
– 2y + 2x = 0


=  × = − (5t 4 ) × = t
dx 2 dt  dx  dx 4 −4 16 5. Hint: y = tan x + sec x


 3 




 t  dy
⇒ = sec2 x + sec x tan x
dx



Hence, (d) is the correct answer.

3. (c) We have y = t10 + 1 and x = t8 + 1 dy
⇒ = sec x(sec x + tan x)
dx






t8 = x – 1 ⇒ (t2)4 = (x – 1)



Now,
dy
( )





t2 = (x – 1)1/4 1 + sin x = 1 + sin x
⇒ = sec x
dx cos x cos 2 x

\





y = t10 + 1 = (t2)5 + 1 = (x – 1)5/4 + 1
dy 1

\


Now differentiating both sides w.r.t. x we get ⇒ =
dx 1 − sin x




dy 5
= ( x − 1)1/ 4 d 2y cos x
dx 4 ⇒ 2 =


dx (1 − sin x )2



Again differentiating both sides w.r.t. x, we get
6. Hint: y= cosec x + cot x

d2y 5 5




= ( x − 1) −3/ 4 = dy
dx 2
16 16( x − 1)3/ 4 ⇒ = –cosec x cot x – cosec2 x


dx



d2y 5 5 dy
= = [ x – 1 = t8] ⇒ = –cosec x(cot x + cosec x)
dx 2 8 3/ 4
16t 6 dx



16(t )

\




Hence, (c) is the correct answer. ⇒
dy
dx
= (
−cosec x 1 + cos x
sin x )




1
4. (c) u = v3 log = v3 (log 1 – log v) dy 1 + cos x = −1
v = −





dx sin 2 x 1 − cos x



⇒ u = – v3 log v
d 2y



du sin x
1 ⇒ =
⇒ = −  v3 ⋅ + log v ⋅ 3v 2  = − v 2 − 3v 2 log v dx 2 (1 − cos x )2



dv  v 



7. Hint: y = a sin mx – b cos mx
d 2u 1
= − 2v − 3 v 2 ⋅ + ( log v ) 2v 




and dy
dv 2  v  ⇒ = am cos mx + bm sin mx



dx



2
d u d 2y
⇒ = − 2v − 3v − 6v log v ⇒ = –am2 sin mx + bm2 cos mx
dv 2 dx 2






126 Mathe atics–12
m
E:\AMIT_WORKS\Exam_Guru\EG_Mathematics-12_(working_02-06-2022)\EG_Mathematics-12_working\Open_Files\Chap_5\Chap_5
\ 16-Aug-2022  Amit   Proof-4 Reader’s Sign _______________________ Date __________


d 2y dy
⇒ = –m2(a sin mx – b cos mx) 14. Hint: y = Peax + Qebx ⇒ = Paeax + Qbebx
dx 2 dx










d 2y
d 2y d 2y ⇒ = Pa2eax + Qb2ebx ...(1)
⇒ = –m2y ⇒ + m 2y = 0 dx 2





dx 2 dx 2





 dy = Paeax + Qbebx
8. Hint: y = tan–1 x 
⇒  dx




dy 1 dy



⇒ = ⇒ (1 + x2) =1  y = Peax + Qebx
dx 1 + x2 dx





2
Solve these two equations for Peax and Qebx. It gives:
(1 + x ) ddxy + 2x  dy  =0


2
⇒  dy − by   dy − ay 
dx 
2
 dx   dx 


ax bx
Pe =  and Qe = − 
9. Hint: y = sin–1 x a−b  a−b 



   




dy 1 dy
⇒ = ⇒ 1 − x2 =1 Put these values of Peax and Qebx in equation (1)
dx 1 − x2 dx






d 2y
d 2y dy = Pa2eax + Qb2ebx
⇒ 1− x 2
+ −2x   = 0 dx 2



dx 2 2 1 − x 2  dx 


d 2y
2 ⇒ = a2(Peax) + b2(Qebx)
(1 − x ) ddxy − x  dy  =0 dx 2




­
2

dx 
2
 dy − by   dy ay 


10. Hint: y = cosec–1 x d 2y 2  dx  2  dx − 
⇒ = a  a−b  − b  a −b 
dx 2




dy −1 dy




2    
⇒ = ⇒ x x − 1 = −1
dx x x2 − 1 dx





d 2y 1  2 dy dy
 d 2y    dy ⇒ = a − a 2by − b 2 + ab 2 y 
⇒ x x 2 − 1  2  +  x 2 − 1 + x × 2x  =0 dx 2 a − b  dx dx 



 dx   2 x − 1  dx
2


d 2y  dy 
⇒ 2 − ( a + b )  dx  + aby = 0
2
⇒ (
x x2 − 1 ) ddxy + (2x − 1) dy =0 2 dx

2 dx


4
11. Hint: y = (tan–1 x)2 15. Hint: y =  x + x 2 + 1
 








dy 2 tan −1 x dy
⇒ = ⇒ ( x 2 + 1) = 2 tan −1 x 3
dx x2 + 1 dx dy
= 4  x + x 2 + 1 1 + 2x 





dx    2 x 2 + 1 



2
⇒ ( x + 1) ddxy + 2x  dy
2
2
= 2
dx  1 + x 2
dy 4
1


= 4  x + x + 1 ×
2

dx   x2 + 1



2
( x + 1) d y dy
( )
2 2 2
⇒ 2 + 2x x + 1 dx = 2
dx dy


⇒ x2 + 1 = 4y
y = (cot–1 x)2 dx


12. Hint:




−2 cot −1 x d 2 y  2x  dy

dy
dx
=
1 + x2
⇒ x2 + 1
dy
dx
= −2 cot −1 x ( ) ⇒ x2 + 1 +
dx 2  2 x 2 + 1  dx
=4
dy
dx





2
⇒ ( x + 1) ddxy + 2x  dy
2 = 2
dx  x + 1
2 2 ⇒
2
2
( x + 1) ddxy + x dy =4 x2 + 1
dy


2 dx dx


2
( x + 1)  ddxy  + 2x ( x + 1)  dy  =2
2 2
( x + 1) ddxy + x dy
2 2
⇒ ⇒ 2
= 4 × 4y
2 dx 


2 dx


x
13. Hint: y=x 2
( x + 1) ddxy + x dy
2




1 dy ⇒ 2 − 16y = 0
⇒ log y = x log x ⇒ = 1 + log x dx


y dx





dy 16. Hint: y = sin(log x)
⇒ = y(1 + log x)




dx dy cos ( log x )



 dy  = cos log x
⇒ = ⇒ x ( )
() dx x  dx 
2
d y dy
(1 + log x ) + y 1x





⇒ 2 = dx
dx
sin ( log x )



 d 2 y  dy
2 ⇒ x 2  + =−
d y dy  dy  1 y  dx  dx x


⇒ = +
dx 2 dx  dx  y x



2  d 2y dy
d 2 y 1  dy  y ⇒ x2  2  + x + y = 0
⇒ − − =0  dx  dx


dx 2 y  dx  x


ontinuity nd i e entiability 127
C
A
D
ff
r

E:\AMIT_WORKS\Exam_Guru\EG_Mathematics-12_(working_02-06-2022)\EG_Mathematics-12_working\Open_Files\Chap_5\Chap_5
\ 16-Aug-2022  Amit   Proof-4 Reader’s Sign _______________________ Date __________


17. Hint: y = a cos(log x) + b sin(log x)
 d 2y
1 − x 2  2  + −2x   =
dy −m 2 sin m sin −1 x ( )




dy −a sin ( log x ) b cos ( log x ) ⇒
⇒ = +  dx  2 1 − x 2  dx  1 − x2



dx x x



2
⇒ x  dy 
 dx  = –a sin(log x) + b cos(log x)
⇒ (1 − x )  ddxy  − x  dy
2
2
 +m y=0
dx 
2






 d 2 y  dy −a cos ( log x ) b sin ( log x ) dy
⇒ x 2  + = − 21. Hint: y = cos(cos x) ⇒ = [sin(cos x)]sin x
 dx  dx x x dx







d 2y
d 2y
2 dy ⇒ = [cos(cos x)(– sin x)]sin x + [sin(cos x)]cos x
⇒ x  2  + x  + y = 0 dx 2





 dx   dx 


18. Hint: y = P  x + x − 1 + Q  x − x − 1 2
n
2
n ⇒
d 2y
dx 2
2
( ) cos x  dy 
= − sin x cos ( cos x ) + sin x  dx 




   
( )




d 2y dy
= − sin x ( y ) + cot x
2
n −1  ⇒
dy
= Pn  x + x 2 − 1 1 + 2x  dx 2 dx




⇒ 
dx  
 2 x2 − 1 



d 2y dy
⇒ − ( cot x )   + y sin 2 x = 0
n n dx 2  dx 



dy Pn  x + x 2 − 1 Qn  x − x 2 − 1 22. Hint: y = (sin x + cos x)ex
   
⇒ = −





dx 2
x2 − 1 dy



x −1 ⇒ = (sin x + cos x)ex + (cos x – sin x)ex
n dx




n
dy
⇒ x 2 − 1   = Pn  x + x 2 − 1  −Qn  x − x 2 − 1
  dy
 dx    ⇒ = y + (cos x – sin x)ex



dx



 d y 2
2x  dy  d 2y dy
⇒ x2 − 1  2  + ⇒ = + (–sin x –cos x)ex + (cos x – sin x)ex
 dx  2 x 2 − 1  dx 


dx 2 dx



n n
d 2y dy  dy − y
Pn 2  x + x 2 − 1  Qn 2  x − x 2 − 1 ⇒ = –y+
=   +   dx 2 dx  dx 



x2 − 1 x2 − 1 d 2y dy
⇒ − 2   + 2y = 0
2 dx 2  dx 
( x − 1)  ddxy  + x  dy


2  −n y=0 2

dx  2
sin −1 x


23. Hint: y= ⇒ 1 − x 2 ( y ) = sin −1 x
1 − x2






m cos −1 x
19. Hint: y= e
dy




−2x 1
dy −m em cos −1 x ⇒ 1 − x2   + ( y) =
⇒ =  dx  2
1 − x2


dx 2 1− x
1 − x2




dy
1 − x2 ( )
−1
= −mem cos x
⇒ (1 − x ) dy
2
dx
− xy = 1


dx



2
On differentiating, we get ⇒ (1 − x ) ddxy − (2x)  dy
2
2
 − x  dy  = 0
dx   dx 



 d y 2 2
dy
1 − x 2  2  + −2x   = m em cos x
−1 2

 dx  2 1 − x 2  dx  1− x 2 ⇒ (1 − x ) ddxy − 3x  dy
2
2
 =0
dx 


24. Hint: y = (sin x)ex





 d 2y 
⇒ (  dx 
) dy
1 − x 2  2  − x   − m2 y = 0
 dx  ⇒
dy
dx
= (cos x)ex + (sin x)ex





dy
 sin −1 y  ⇒ = (cos x)ex + y
20. Hint: x = sin  dx



 m 




d 2y dy
⇒ = (cos x)ex – (sin x)ex +
sin y−1
dx 2 dx



⇒ = sin–1 x ⇒ sin–1 y = m sin–1 x
m
­
d 2y  dy  dy





–1 ⇒ =  − y − y +
⇒ y = sin[m sin x] dx 2  dx  dx







dy
=
m cos m sin −1 x ( ) ⇒
d 2y
dx 2
dy
− 2   + 2y = 0
 dx 


dx 1 − x2



25. Hint: xy = aex + be–x + x2





dy
1 − x 2   = m cos m sin −1 x
 dx  ( ) ⇒
dy
x   + y = ae x − be −x + 2x
 dx 




128 Mathe atics–12
m
E:\AMIT_WORKS\Exam_Guru\EG_Mathematics-12_(working_02-06-2022)\EG_Mathematics-12_working\Open_Files\Chap_5\Chap_5
\ 16-Aug-2022  Amit   Proof-4 Reader’s Sign _______________________ Date __________


d 2y

 d 2 y  dy dy
x 2  +
 dx  dx dx
+ = ae x + be −x + 2 ⇒
dx 2
(
= − cosec2 θ

dx )






1
 d 2y  dy = −cosec2 θ ×
⇒ x  2  + 2   = xy − x 2 + 2 3 sin θ cos 2θ



 dx   dx 


cosec3 θ 
= − =1

 d 2y  dy
x  2  + 2   − xy + x 2 − 2 = 0 3 cos 2θ  3



θ= π
 dx   dx  2


26. Hint: y = (ax + b)e3x  −1  2t 
 y = sin  1 + t 2 




dy 29. Hint:  
⇒ = 3(ax + b)e3x + ae3x 2
 x = cos −1  1 − t 




dx



 
 1 + t 2 
dy
⇒ = 3y + ae3x Put t = tan q
dx




d 2y  −1  2 tan θ 
 y = sin  1 + tan 2 θ  = sin (sin 2θ ) = 2θ
 dy  + 3ae3x −1
⇒ = 3
dx 2  dx  ⇒ 



 2
 x = cos −1  1 − tan θ  = cos −1 ( cos 2θ ) = 2θ



2
d y  dy  + 3  dy − 3y  
⇒ = 3
 dx   dx    1 + tan θ 
2
dx 2



d 2y  dy  + 3  dy  − 9y ⇒  y = 2θ = 2 tan −1 t
⇒ = 3  −1
dx 2  dx   dx   x = 2θ = 2 tan t






d 2y  dy   dy = 2
⇒ 2 − 6  dx  + 9y = 0  dt 1 + t 2
dx


⇒ 
 dx = 2 2


 y = θ sin θ + cos θ
27. Hint:   dt 1 + t
 x = θ cos θ − sin θ



dy d 2y
 dy = sin θ + θ cos θ − sin θ = θ cos θ ⇒ =1 ⇒ =0
dx dx 2





 dθ
⇒   dy 
 dx = cos θ − θ sin θ − cos θ = −θ sin θ


dy  d θ  a cos θ + b sin θ  −x 
 dθ 30. Hint: = = =
dx  dx  −a sin θ + b cos θ  y 


dy dy / d θ θ cos θ  dθ 
⇒ = = = − cot θ
dx dx / d θ −θ sin θ



  dy  
d 2y  y − x  dx  

dx 2
(
= cosec2 θ

dx ) d 2y
⇒ 2 =−
dx y2




 

( )
d 2y 1  
⇒ = cosec2 θ

dx 2 −θ sin θ  d 2y dy



⇒ y2  2  − x   + y = 0
2
d y cosec θ 3  dx   dx 
⇒ = −
dx 2 θ




 x = cos θ
 y = 3 sin θ − 2 sin 3 θ 31. Hint:  3
 y = sin θ



28. Hint:  3
 x = 3 cos θ − 2 cos θ



 dx = − sin θ
 dy = 3 cos θ − 6 sin 2 θ cos θ  dθ
 dθ ⇒ 
 dy = 3 sin 2 θ cos θ


⇒ 
 dx = −3 sin θ + 6 cos 2 θ sin θ  dθ


 dθ dy
⇒ = –3 sin q cos q
dx
(
 dy = 3 cos θ 1 − 2 sin 2 θ
)



 dθ d 2 y = −3  − sin 2 θ + cos 2 θ  d θ
⇒  ⇒   dx
(
 dx = 3 sin θ 2 cos 2 θ − 1) dx 2





( )
 dθ d y2
−1
⇒ = 3 sin 2 θ − cos 2 θ 
 dy = 3 cos θ cos 2θ dx 2 sin θ



 dθ
⇒  d 2y  cos 2 θ − sin 2 θ 
 dx = 3 sin θ cos 2θ
⇒ 2 = 3 


dx  sin θ 



 dθ
 d 2 y   dy  2
dy dy / d θ 3 cos θ cos 2θ ⇒ y 2  +
⇒ = = = cot θ  dx   dx 


dx dx / d θ 3 sin θ cos 2θ



ontinuity nd i e entiability 129
C
A
D
ff
r

E:\AMIT_WORKS\Exam_Guru\EG_Mathematics-12_(working_02-06-2022)\EG_Mathematics-12_working\Open_Files\Chap_5\Chap_5
\ 16-Aug-2022  Amit   Proof-4 Reader’s Sign _______________________ Date __________


 2 2  dy −1 × x + 1 = −1
= 3 sin 3 θ  cos θ − sin θ  + [ −3 sin θ cos θ ]
2
⇒ =
sin θ dx ( x + 1)2 1 x + 1




 

= 3 sin2 q[cos2 q – sin2 q] + 9 sin2 q cos2 q d 2y 1  dy  −1
⇒ = 2 and  dx  = x + 1
( )

= 3 sin2 q cos2 q – 3 sin4 q + 9 sin2 q cos2 q dx 2 ( )
x + 1





= 12 sin2 q cos2 q – 3 sin4 q 2
d 2y  dy 

= 3 sin2 q[4 cos2 q – sin2 q] ⇒ =
dx 2  dx 





= 3 sin2 q[5 cos2 q – 1] 34. Hint: y = ex(tan–1 x)





32. Hint: xmyn = (x + y)m + n
dy e x + e x tan −1 x




⇒ m log x + n log y = (m + n) log(x + y) ⇒ =
dx 1 + x2






m + n dy = m + n 1 + dy 
⇒ dy ex + y
x y dx x + y  dx  ⇒ = ...(1)


dx 1 + x2






dy  my − nx  my − nx
=
dx  y ( x + y )  n ( x + y )
⇒ ( 1 + x2
dy
dx
)
= ex + (1 + x2)y

⇒ dy y
= ⇒ (1 + x ) ddxy + 2x dy
2
dx 2



dx x


dy
x  − y
= ex +
dy
dx
( )
1 + x 2 + y ( 2x )



2
d y  dx  dy y
⇒ 2 = = 1  −  2
dx x 2 x  dx x  (1 + x ) ddxy + (2x − 1 − x ) dy
2 2
= ex + y(2x)


⇒ 2dx


d 2y 1  y y  2
⇒ = −
dx 2 x  x x 
=0
⇒ (1 + x ) ddxy − ( x
2 2
− 2x + 1 ) dy


2 dx


1
33. Hint: ey(x + 1) = 1 ⇒ ey =
x +1
...(1)
(
2  dy 
= 1 + x  dx − y + 2xy ) [Using (1)]










dy −1 2
⇒ ey
dx
=
( + 1)2
x (1 + x ) ddxy − (2x
2
2
2
− 2x + 2 ) dy
dx
+ (1 − x )
2
y=0



dy

1 −1 2

( x + 1) dx =
( x + 1)2
[Using (1)]
(1 + x ) ddxy − 2( x
2 2
− x +1 ) dy + (1 − x )
2
y=0




2 dx

Case Based Questions
1. A potter made a mud vessel, where the shape of the pot is (i) When x > 4, what will be the height in terms of x?




based on
(a) x – 2 (b) x – 3
f(x) =| x − 3 | + | x − 2 | , where f (x) represents the height




(c) 2x – 5 (d) 5 – 2x


of the pot.




(ii) For x = 3, we have f (3) equal to


(a) 1 (b) 2




(c) 3 (d) 4




dy
(iii) What is at x = 3 ?
dx


(a) 2 (b) – 2




(c) Function is not differentiable


(d) 1


(iv) When the x value lies between (2, 3), then the function


is
(a) 2x – 5 (b) 5 – 2x




(c) 1 (d) 5





130 Mathe atics–12
m
E:\AMIT_WORKS\Exam_Guru\EG_Mathematics-12_(working_02-06-2022)\EG_Mathematics-12_working\Open_Files\Chap_5\Chap_5
\ 16-Aug-2022  Amit   Proof-4 Reader’s Sign _______________________ Date __________


(v) If the potter is trying to make a pot using the function (i) The derivative of log (log x) is




f(x) = [x], will he get a pot or not? Why? 1 1
(a) (b)
(a) Yes, because it is a continuous function log x x







(b) Yes, because it is not continuous function
1 x


(c) No, because it is a continuous function (c) (d)
x log x log x








(d) No, because it is not continuous function
(ii) The derivative of sin (log x) is




Ans. (i) (c) 2x – 5 (ii) (a) 1 cos (log x)
(a) cos (log x) (b)





(iii) (c) Function is not differentiable x








(iv) (c) 1 (log x) sin x
(c) sin (d)



x log x





(v) (d) No, because it is not continuous function



2. Mrs Pallavi Jain, a teacher of Blue Marble School, is (iii) The derivative of cot (4x2 + 3) at x = 2 is





teaching chain rule to his students in the classroom. Let 2 2
f be a real valued function which is a composite of two (a) −8 2 cos ec (11) (b) −16 cosec (4 2 )





functions u and v; i.e., f = vou. Suppose t = u(x) and if both (c) 1 (d) 2





dt dv df dv dt (iv) The derivative of tan [sin(cosx)] is
and exist, we have = .



dx dt dx dt dx (a) cos x sin (cos x). sec2 (sin (cos x)]



(b) –sin x .cos (cos x). sec2 {sin (cos x)}


(c) cos x sin (sin x). tan (sin (cos x)]



(d) None of these


π2
(v) The derivative of sec [tan ( x )] at x = is
16


π 4
(a) sec 2 (1) (b) tan 2 (1)
4 π




4 4
(c) π sec [tan (1)] (d) sec (1). tan (1)
π




1
Ans. (i) (c) (ii) (b) cos (log x)
x log x x





Chain rule may be extended as follows. Suppose f is a real 2
(iii) (a) −8 2 cosec (11)

valued function which is a composite of three functions u,



2
v and w, i.e., f = (w.u.).v (iv) (b) −sin x.cos (cos x). sec {sin (cos x)}



df d ( wou ) dt dw ds dt 4
If t = v (x) and s = u (t), then = . = . . (v) (d) sec (1). tan (1)
dx dt dx ds dt dx π




Author’s Comments
Questions based on following types are very important for Exams. So, students are advised to revise them thoroughly.
1. To prove continuity and differentiability of a given function at a point. (Most Important)


2. Differentiation of implicit function, differentiation using logarithms, differentiable and second order derivatives. (Most Important)



3. Find the values of a and b when the given function is continuous.


4. Find the value of k, when the function is continuous.


5. Derivatives of parametric functions.


ontinuity nd i e entiability 131
C
A
D
ff
r

E:\AMIT_WORKS\Exam_Guru\EG_Mathematics-12_(working_02-06-2022)\EG_Mathematics-12_working\Open_Files\Chap_5\Chap_5
\ 16-Aug-2022  Amit   Proof-4 Reader’s Sign _______________________ Date __________


IMPORTANT FORMULAE
1. Continuity of a function f (x) at a point x = a




A function y = f (x) is continuous at x = a, if f ( a ) = lim f ( x ) = lim f ( x ) = n .


x→ a − x→ a+

Here lim f ( x ) is called left hand limit and lim f ( x ) is called right hand limit of f (x).


x→ a − x→ a+

2. Differentiability of a function f (x) at x = x0



A function f (x) is differentiable at x = x0, if its left hand derivative


 f ( x0 − h ) − f ( x0 ) 
(LHD) = lim   is equal to its right hand derivative
h→0  −h 


 f ( x0 + h ) − f ( x0 ) 
(RHD) = lim  
h→0  h 


3. Standard formulae for direct differentiation:


(i)
d n
dx ( )
x = nx n −1 (ii)
d
dx ( )
αx n = α  nx n −1 (iii)
d x
dx ( )
e = ex






(iv)
d x
dx
( )
a = a x log a (v)
d
dx
(loge x ) = 1x d 1 1
(vi) dx ( log a x ) = x × log a






d d d
(vii)
dx
(k ) = 0 (viii)
dx
(sin x ) = cos x (ix)
dx
(cos x ) = − sin x






d d d
(x)
dx
( tan x ) = sec2 x (xi)
dx
(cot x ) = − cosec2 x (xii)
dx
(sec x ) = sec x tan x






(xiii)
d
dx
(cosec x ) = − cosec x cot x (xiv)
d
dx (
sin −1 x =
1
)
1 − x2
d −1
(xv) dx cos x = (
−1
1 − x2
)






dx (
(xvi) d tan −1 x = 1 2
1+ x
) (xvii)
d
dx (
cot −1 x =
−1
)
1 + x2
(xviii)
d
dx (
sec −1 x =
1
)
x x2 − 1






(xix)
d
dx
(
cosec −1 x =
−1
x x2 − 1
)


4. Differentiation of algebraic functions by chain rule:


n dy n −1 1 dy −nf ′ ( x )
(i) If y =  f ( x ) ⇒ = n  f ( x ) f ′ ( x ) (ii) If y = n ⇒ dx =
dx  f ( x )  f ( x )
n +1




dy f ′ ( x) 1 dy f ′ ( x)
(iii) If y = f ( x) ⇒ = (iv) If y = ⇒ =−
dx 2 f ( x ) f ( x) dx 2  f ( x )
3/ 2




132 Mathe atics–12
m
E:\AMIT_WORKS\Exam_Guru\EG_Mathematics-12_(working_02-06-2022)\EG_Mathematics-12_working\Open_Files\Chap_5\Chap_5
\ 16-Aug-2022  Amit   Proof-4 Reader’s Sign _______________________ Date __________


5. Differentiation of trigonometric and inverse trigonometric functions by chain rule


dy 
(i) y = sinm f (x) ⇒ = m sin m −1 f ( x ) cos f ( x ) f ′ ( x )
dx 




dy 
(ii) y = cosm f (x) ⇒ = −m cos m −1 f ( x ) sin f ( x ) f ′ ( x )

dx 


dy 
(iii) y = tanm f (x) ⇒ = m tan m −1 f ( x ) sec 2 f ( x ) f ′ ( x )
dx 



dy 
(iv) y = cotm f (x) ⇒ = −m cot m −1 f ( x ) cosec 2 f ( x ) f ′ ( x )
dx 



dy 
(v) y = secm f (x) ⇒ = m sec m −1 f ( x ) sec f ( x ) tan f ( x ) f ′ ( x )
dx 



dy 
(vi) y = cosecm f (x) ⇒ = −m cosec m −1 f ( x ) cosec f ( x ) cot f ( x ) f ′ ( x )
dx 



dy f ′ ( x) dy −f ′ ( x )
(vii) y = sin–1 f (x) ⇒ = (viii) y = cos −1 f ( x ) ⇒ =
dx dx

2
1 −  f ( x )
2
1 −  f ( x )




dy f ′ ( x) dy −f ′ ( x )
(ix) y = tan −1 f ( x ) ⇒ (x) y = cot f ( x ) ⇒
−1
= =
dx 1 +  f ( x ) 2 dx 1 +  f ( x ) 2




   

dy f ′ ( x) dy −f ′ ( x )
(xi) y = sec −1 f ( x ) ⇒ = (xii) y = cosec −1 f ( x ) ⇒ =
dx 2 dx 2
f ( x )  f ( x ) − 1 f ( x )  f ( x ) − 1




6. Product rule:


dy
(i) y = f ( x ) g ( x ) ⇒ = f ′ ( x) g ( x) + f ( x) g′ ( x)
dx


dy
(ii) y = f ( x ) g ( x ) φ ( x ) ⇒ = f ′ ( x ) g ( x ) φ ( x ) + f ( x ) g ′ ( x ) φ ( x ) + f ( x ) g ( x ) φ′ ( x )
dx


f ( x) dy g ( x ) f ′ ( x ) − f ( x ) g ′ ( x )
7. Quotient rule: y = ⇒ =
g ( x) dx 2
 g ( x )


8. Revise following trigonometric and inverse trigonometric formulae


(i) sin(A + B) = sin A cos B + cos A sin B (ii) sin(A – B) = sin A cos B – cos A sin B




(iii) cos(A + B) = cos A cos B – sin A sin B (iv) cos(A – B) = cos A cos B + sin A sin B




tan A + tan B tan A − tan B
(v) tan ( A + B) = (vi) tan ( A − B) =
1 − tan A tan B 1 + tan A tan B




2 tan θ
(vii) sin 2θ = 2 sin θ cos θ =
1 + tan 2 θ


ontinuity nd i e entiability 133
C
A
D
ff
r

E:\AMIT_WORKS\Exam_Guru\EG_Mathematics-12_(working_02-06-2022)\EG_Mathematics-12_working\Open_Files\Chap_5\Chap_5
\ 16-Aug-2022  Amit   Proof-4 Reader’s Sign _______________________ Date __________


2 2 2 2 1 − tan 2 θ
(viii) cos 2θ = cos θ − sin θ = 2 cos θ − 1 = 1 − 2 sin θ =
1 + tan 2 θ


2 tan θ (x) 1 + cos θ = 2 cos 2 θ
(ix) tan 2θ =
1 − tan 2 θ 2





(xi) 1 − cos θ = 2 sin 2 θ (xii) sin 3q = 3 sin q – 4 sin3 q
2




(xiii) cos 3q = 4 cos3 q – 3 cos q 3 tan θ − tan 3 θ
(xiv) tan 3θ =
1 − 3 tan 2 θ




 x+ y  x− y
(xv) tan −1  = tan −1 x + tan −1 y (xvi) tan −1  = tan −1 x − tan −1 y
 1 − xy   1 + xy 




(xvii) 2 tan −1 x = tan −1  2x 2  (xviii) 2 tan −1 x = sin −1  2x 2 
1− x  1+ x 




 1 − x2 
(xix) 2 tan −1 x = cos −1 
 1 + x 2 

(xx) sin x = cos ( )
π
2
−x




(xxi) sin x = − cos ( ) π
2
+x (xxii) cos x = sin ( )
π
2
+x




(xxiii) cos x = sin ( )
π
2
−x (xiv)
1 − tan x
1 + tan x
π
( )
= tan − x
4




(xxv)
1 + tan x
1 − tan x
π
( )
= tan + x
4


9. Logarithmic Differentiation


y = [ f (x)]g(x) ⇒ log y = g(x) log [ f (x)]








1 dy  g ( x) f ′ ( x) 
⇒ = g ′ ( x ) log  f ( x ) +  
y dx  f ( x) 



dy   g ( x) f ′ ( x) 
⇒ = y  g ′ ( x ) log { f ( x )} +  
dx   f ( x) 



10. Differentiation of Parametric Function


dy dx dy  dy/d θ  f ′ (θ )
If y = f (q) and x = g(q) then = f ′ (θ ) and = g ′ (θ ) ⇒ = =
dx  dx/d θ  g ′ (θ )

dθ dθ

134 Mathe atics–12
m
E:\AMIT_WORKS\Exam_Guru\EG_Mathematics-12_(working_02-06-2022)\EG_Mathematics-12_working\Open_Files\Chap_5\Chap_5
\ 16-Aug-2022  Amit   Proof-4 Reader’s Sign _______________________ Date __________


COMMON ERRORS
ERRORS CORRECTIONS

(i) Students commit mistake in applying Chain Rule


(i) For example, Differentiate: y = sin2 x2 and find dy Generally,


dx



they do it as: dy = 2 cos x 2 x 2
dx ( )
( )( )
in place of dy = 2 sin x 2 cos x 2 ( 2x ) .
dx


(ii) In continuity, children find LHL = RHL, but forget to prove (ii) Def. of continuity to be drilled.




that equal of f (a).

(iii) Derivative of implicit function like xy + y2 + x2 = 100, they (iii) Practice product rule in various forms.





don’t consider xy as a product of 2 functions.

(iv) While differentiating Inverse trigonometric functions, (iv) Simple way






usually students differentiate directly. They do not simplify  1 + cos x 
and differentiate, thus not obtaining the answer on the most y = tan −1 
 1 − cos x 
simplified manner.


−1  1 + cos x 
For e.g., y = tan 
 1 − cos x 
 2 cos 2 x / 2 
= tan −1  2
 2 sin x / 2 
−1 x
 = tan cot 2 ( )


dy 1 d  1 + cos x  π x
( )
= . = −
dx 1 + 1 + cos x dx  1 − cos x  2 2
1 − cos x

dy 1
=−

which becomes complicated. dx 2


Sometimes, students use this method but forget to

differentiate. They stop after just simplification. Do not
forget to differentiate.

(v) Logarithmic differentiation: (v) log(a + b) ≠ log a + log b. So, we should take log separately




If y = xsin x + cos xsin x students usually write as shown: y = xsin x + cos xsin x
Let u = xsin x; v = cos xsin x

log y = log xsin x + log cos xsin x.
 


dy du dv
⇒ y = u + v, = +
dx dx dx


Calculate du , dv separately.
dx dx

(vi) Parametric differentiation: 2nd order derivative is usually
(vi) dy = t


calculated incorrect. dx


Let y = at2, x = 2at
d 2y dt

dy dy / dt 2at = 1.
= = =t dx 2 dx
dx dx / dt 2a


d 2y
• = 1 (which is wrong)
dx 2


Usually students forget to multiply with dt/dx

d 2 y d 2 y / dt 2
• = which is also wrong
dx 2 dx 2 / dt 2


ontinuity nd i e entiability 135
C
A
D
ff
r

E:\AMIT_WORKS\Exam_Guru\EG_Mathematics-12_(working_02-06-2022)\EG_Mathematics-12_working\Open_Files\Chap_5\Chap_5
\ 16-Aug-2022  Amit   Proof-4 Reader’s Sign _______________________ Date __________


REVISION CHART

C ontinuity an d Dis continuity of a F unction at a Point


A functrion y = f(x), x ∈ D is called continuous at a point x = m ∈ D, if there is no interruption in the graph of f(x) at x = m. If
there is interruption or break in the graph or there is a gap or hole in the graph of the function at the point x = m ∈ D, then the
function y = f(x), x ∈ D is called discontinuous at x = m ∈ D.

Differentiation of a F unction at a P oint

Graphical Method Limit of the Function method

Implicit Functions Inverse Trigonometric Functions

Direct
Differentiation of Parametric Differentiation of
Logarithmic Differentiation
Functions Different ype ofT

F unction s

Higher Order Derivatives

136 Mathe atics–12


m
E:\AMIT_WORKS\Exam_Guru\EG_Mathematics-12_(working_02-06-2022)\EG_Mathematics-12_working\Open_Files\Chap_6\Chap_6
\ 17-Aug-2022  Ved_Goswami   Proof-5 Reader’s Sign _______________________ Date __________


Topics
C
overed
6 Application of Derivatives

6.1 Rate of Change of Bodies 6.2 Increasing and Decreasing Functions






6.3 Local Maximum and Local Minimum Values of a Function


6.4 Absolute Maximum and Absolute Minimum Values of a Function


C hapter map
APPLICATION OF DERIVATIVES
y = f(x)

Rate of Change of Bodies Increasing and Decreasing Functions Maximum and Minimum

Increasing if x1 < x2 implies


f(x1) ≤ f(x2) or
if f'(x) ≥ 0 for all x1, x2 in (a, b)

Decreasing if x1 < x2 implies


f(x1) ≥ f(x2) or
if f'(x) ≤ 0 for all x1, x2 in (a, b)

First Derivative Test Second Derivative Test

Local Maximum and Local Minimum


Absolute Maximum and Word Problems on
Absolute Minimum Maxima and Minima

INTRODUCTION
In this chapter we shall learn how the derivatives can be used (i) to determine the rate of change of quantities; (ii) to find the interval
 
on which a function is increasing or decreasing. (iii) to find turning points on the graph of a function which in turn will help us to
locate at which largest or smallest value (locally) of a function occurs.

Topic 1. Rate of Change of Bodies


(i) Whenever one quantity y varies with respect to (ii) If two variables x and y are varying with respect to another




another quantity x, satisfying some rule, y = f (x), variable t, i.e., if x = f(t) and y = g(t), then we can work out

dy dy
then or f ′(x) represents the rate of change of y with the formula:
dx dx

dy dy
with respect to x and or f ′(x 0 ) represents dy dt
dx

x = x0 = dx
dx


the rate of change of y with respect to x at x = x 0. dt

137
E:\AMIT_WORKS\Exam_Guru\EG_Mathematics-12_(working_02-06-2022)\EG_Mathematics-12_working\Open_Files\Chap_6\Chap_6
\ 17-Aug-2022  Ved_Goswami   Proof-5 Reader’s Sign _______________________ Date __________


Thus, the rate of change of y with respect to x can be
Now putting dA = 12 cm 2/sec and dP = 4 cm 2/sec in the

calculated using rate of change of y and that of x both with dt dt


respect to t. dA
x
Under this heading we shall discuss the rate of change of equation: ddtP = we get
2
perimeters and areas of square and rectangle with respect to dt
another quantity. 12 x
= ⇒ x = 6 cm
Example 1. The side of a variable square is increasing at the rate 4 2




of 2 cm/sec, find at what rate the perimeter and the area of the Hence, length of the side at that time is 6 cm.


dP
square are increasing with respect to time when the length of the On putting the value of in the equation dP = 4 dx ,
side of the square is 25 cm. dt dt dt


we get
Solution. Let the length of the side of the square at any time
dx dx
be = x. Then we are given: 4= 4 ⇒ = 1 cm/sec
dt dt




 
 
dx \ The side of the variable square is increasing at that time at
= 2 cm/sec and x = 25 cm


dt the rate of 1 cm/sec.


Perimeter of the square: P = 4x
Questions Based on Variable Rectangle

The rate of change of perimeter of the square with respect to
Perimeter of the rectangle P = 2(l + b)

dP
t, will be: , which can be calculated as follows: Area of the rectangle = A = lb
dt
dP dx
P = 4x ⇒ =4 2
Diagonal of the rectangle = D = l + b
2
dt dt



dP Example 3. The length of a variable rectangle is decreasing
⇒ = 4 × 2 cm/sec = 8 cm/sec  dx = 2 cm/sec 
dt  dt  at the rate of 1 cm/sec and breadth is increasing at the rate of




Now let A be the area of the square at any time t, then 2 cm/sec. Find at what rate the perimeter and area of the rectangle

A = x2 cm2. are changing when length and breadth of the rectangle are 30 cm
 
 
 
and 25 cm respectively.
The rate of change of area A with respect to time t is: dA ,
dt Solution. If l and b be the length and breadth of the rectangle,

which can be calculated as follows:
then we are given:
A = x2 ⇒ dA = 2x dx dl
dt dt = –1 cm/sec,



dA  dt


2 2
⇒ = 2 × 25 × 2 cm /sec = 100 cm /sec
dt  x =25 cm db = 2 cm/sec, l = 30 cm and b = 25 cm



 dx = 2 cm/sec  dt


 dt  dP dl db
( )

Perimeter P = 2(l + b) ⇒ =2 +
Hence, (i) rate of change of perimeter of the square w.r.t. dt dt dt



time t when x = 25 cm is 8 cm/sec, and (ii) rate of change of area dP
⇒ = 2(–1 + 2) cm/sec = 2 cm/sec
dt

of the square with respect to time t when x = 25 cm is 100 cm2/sec.



⇒ Perimeter of the rectangle is increasing at the rate of
Example 2. At some particular time the area and perimeter of

2 cm/sec

( )
a variable square are increasing at the rate of 12 cm2/sec and 4
dA db dl

cm/sec respectively. Find what is the length of the side of the Area: A = lb ⇒ = l +b
dt dt dt




square at that time? Also find at what rate is the side of the square dA 2
⇒ = [30 × 2 + 25 (–1)] cm /sec
increasing at that time? dt



Solution. Let x is the length of the side of the variable square at = (60 – 25) cm2/sec = 35 cm2/sec


any time. P and A be the perimeter and area of the variable square ⇒Area of the variable rectangle is increasing at the rate of


at that time. Now we are given: 35 cm2/sec.
dP dA Questions Based on Variable Circle
= 4 cm/sec, = 12 cm 2/sec
dt dt

Example 4. Find the radius and rate of change of radius of a
dx
and we want to find out: x and at that time variable circle, when its area and circumference are increasing
dt

P = 4x and A = x2 at the rate of 20 cm2/sec and 5 cm/sec respectively.


dP Solution. Let r be the radius of the variable circle at some
⇒ = 4 dx and dA = 2x dx particular time, then we are given:
dt dt dt dt


dC dA
dA dx dA = 5 cm/sec and = 20 cm 2/sec and we want to find
2x x dt dt

dt dt dt
⇒ dP = 4 dx ⇒ dP = dr
2 out: r and at that time.




dt dt dt dt

138 Mathe atics–12


m
E:\AMIT_WORKS\Exam_Guru\EG_Mathematics-12_(working_02-06-2022)\EG_Mathematics-12_working\Open_Files\Chap_6\Chap_6
\ 17-Aug-2022  Ved_Goswami   Proof-5 Reader’s Sign _______________________ Date __________


Now, circumference of the variable circle dA 2πr dr

C = 2pr ⇒ dC = 2π dr ⇒ dt = dt = r ⇒
20
cm = r ⇒ r = 4 cm
dt dt dC 2π dr 5







dt dt
Area of the variable circle:
dC dr dr

Also = 2π ⇒ 5 cm/sec = 2π
A = pr2 ⇒ dA = 2πr dr dt dt dt




dt dt




dr 5
cm/sec
⇒ =
dt 2π




EXERCISE 6.1
I. Multiple Choice Questions (MCQs) seconds. Find out at what rate the perimeter and area of
Choose the correct answer from the given options. the rectangle are changing with respect to time, when 10
seconds has elapsed.
1. The rate of change of the area of a circle with respect to
3. The length x cm of a rectangle is decreasing at the rate of


its radius r at r = 6 cm is



5 cm/min and width y cm is increasing at the rate of
(a) 10p cm (b) 12p cm
4 cm/min. Find the rate of change of perimeter and area of




(c) 8p cm (d) 11p cm the rectangle, when x = 8 cm and y = 6 cm.






2. The total revenue in rupees received from the sale of x [A.I. 2009]



units of a product is given by R(x) = 3x2 + 36x + 5. The 4. A stone is dropped into a quiet lake and waves move at a



marginal revenue, when x = 15 is fixed speed of 8 cm/sec. At the instant when the radius of
(a) 116 (b) 96 (c) 90 (d) 126 the circular wave is 15 cm, how fast is the enclosed area
increasing?








3. A point on the parabola y2 = 18x at which the ordinate


increases at twice the rate of the abscissa is : III. Long Answer Type Questions
(a) (2, 4) (b) (2, – 4) 1. The length of a variable rectangle is always two times of






 9 9 9 9 its breadth. Find out the rate with which the perimeter, area
(c)  − ,  (d)  , 
 8 2 8 2 and diagonal of the rectangle are changing, (i) with respect




4. The sides of an equilateral triangle are increasing at the to its breadth, when breadth is 10 cm. (ii) with respect to


rate of 2 cm/sec. The rate at which the area increases, when its length when length is 20 cm, (iii) with respect to its
the side is 10 cm is diagonal, when diagonal is 25 cm.
(a) 3 cm 2 / sec (b) 10 cm2/sec 2. The area of an expanding rectangle is increasing at the






10 rate of 48 cm2/sec. The length of the rectangle is always
(c) 10 3 cm 2 / sec (d) cm 2 / sec
3 equal to the square of the breadth. At what rate, the length




II. Short Answer Type Questions-I of the rectangle is increasing at the instant when breadth
1. The side of a square is increasing at the rate of 2 cm/sec, is 4.5 cm?


find out at what rate the perimeter, area and diagonal of 3. The two sides of an isosceles triangle with fixed base are
the square are increasing with respect to time when length


decreasing at the rate of 3 cm/sec. How fast is the area of
of the square is 25 cm? [Delhi 2014]
the triangle decreasing when two equal sides are equal to

2. The length and breadth of a variable rectangle are : l =
the base? [A.I. 2014 (C), Delhi 2015]



(4t + 5) cm, b = (2t + 1) cm respectively. Here t is time in

Answers 6.1
I. 1. (b) Let A be the area of the circle at any time t.
 dR 

dA   = 6(15) + 36 = 126
A = pr2 ⇒ = π × 2r = 2πr cm  dx  x = 15




dr






Hence, (d) is the correct answer.
 dA 

  = 2π × 6 = 12π cm dy dx
 dr r = 6 cm 3. (b) y2 = 18x ⇒ 2y = 18




dt dt






Hence, (b) is the correct answer.
 dx  dx  dy dx 

2. (d) Total revenue ⇒ 2 y  2  = 18  dx = 2 dt (Given ) 
 dt  dt  






R(x) = 3x2 + 36x + 5


dR d dx
MR = = ( 3x 2 + 36 x + 5 ) = 6 x + 36 ⇒ (4 y − 18) = 0 ⇒ 4y – 18 = 0
dx dx dt









pplication of Derivatives 139
A

E:\AMIT_WORKS\Exam_Guru\EG_Mathematics-12_(working_02-06-2022)\EG_Mathematics-12_working\Open_Files\Chap_6\Chap_6
\ 17-Aug-2022  Ved_Goswami   Proof-5 Reader’s Sign _______________________ Date __________


18 9 Hint : Let b = x, l = 2x find D = 5x , P and A
⇒ y= =


4 2



 
Putting y =
9
in y 2 = 18x , we get (ii) dP = 3, dA = 20 cm; dD =  5 
dx dx dx  2 



2

2
9 81 9 x x2
  = 18x ⇒ x = 4 × 18 = 8 Hint: Let l = x, b = , P = 3x, A =
2 2
2
 






9 9
Thus, the required point is  ,  . and D = 5 x
8 2 2

Hence, (d) is the correct answer.


(iii) dP = 6 ; dA = 20 cm; dD = 1

4. (c) Let the side of the equilateral triangle be x cm. dx 5 dx dx





3 3 2
Area = (side) 2 = x Hint : Let D = x, then b = x and l = 2x
4 4 5 5



dA 3 dx dl = 64 cm/sec
\ = ⋅2x⋅ 2.
dt dt 9



4 dt



dx 3
Now, = 2 cm /sec and x = 10 cm Hint: Let length b 2 = l ⇒ b = l , A = l 2 .
dt




dA 3 dA
\ = ⋅ 2(10) ⋅ 2 cm 2 /sec =10 3cm 2 /sec Now find .
dt 4 dl




Hence, (c) is the correct answer. l2
3. dA = 3l cm 2/sec Hint: A = l x2 −

dP = 8 cm/ sec; dA = 100 cm 2/ sec; dt 2 4


II. 1.
dt dt

dD = 2 2 cm/sec.
dt

2. dP = 12 cm/sec; dA = 174 cm 2/sec
dt dt


3. dP = − 2 cm/min; dA = 2 cm 2/min
dt dt


4. dA = 204 π cm 2/ sec.
dt


dP = 6; dA = 40 cm and dD = 5
III. 1. (i)
dx dx dx

(A) RELATED RATES and S = 6x2 ⇒ dS = 12x dx
dt dt





Cube dV
If x is length of an edge of a cube, then: 3x 2 dx dV
dt dt x 50 x
Now, dS = dx ⇒ ddtS = ⇒ cm =
12x 4 10 4






S = 6x2, V = x3, D = 3x dt dt dt
­


Example 1. If x is the length of the edge ⇒ x = 20 cm



of a variable cube, find x and dx when On putting x = 20 cm and dS = 10 cm2/sec in dS = 12x dx ,
dt dt dt dt

we get
the volume and surface area of the cube are increasing
dx dx 1
at the rate of 50 cm 3 /sec and 10 cm 2 /sec respectively. 10 cm2/sec = 12 × 20 cm × ⇒ = cm/sec
dt dt 24



dV \ x = 20 cm and dx = 1 cm/sec
Solution. In a variable cube, we are given = 50 cm3/sec,
dt dt 24



dS = 10 cm2/sec.
dt Cuboid
If x is the length of the cube at some time then we have to find If l, b and h are the length, breadth and height of
a cuboid then:

dx
out: and x. S = 2(lb + bh + hl), V = lbh,
dt


V = x3 ⇒ dV = 3x 2 dx D= l 2 + b2 + h2
dt dt





140 Mathe atics–12
m
E:\AMIT_WORKS\Exam_Guru\EG_Mathematics-12_(working_02-06-2022)\EG_Mathematics-12_working\Open_Files\Chap_6\Chap_6
\ 17-Aug-2022  Ved_Goswami   Proof-5 Reader’s Sign _______________________ Date __________


Example 2. If height, breadth and length of a variable cuboid
are x, 2x and 3x respectively, find the rate with which its surface Solution. In a cylinder, it is given: dr = 2 cm/sec , dh = 3 cm/sec ,
h = 20 cm, r = 5 cm. dt dt
area, volume and diagonal are increasing with respect to x, when
x = 10 cm.
To find: dS and dV
Solution. Given that: h = x, b = 2x and l = 3x. dt dt


\ Surface area of the variable cuboid: S = (2prh + 2pr2)

( )




S = 2(lb + bh + hl) = 2[3x × 2x + 2x × x + x × 3x] dS  dh dr dr 
\ =  2π r +h + 2π ( 2r ) 


⇒ S = 2[6x2 + 2x2 + 3x2] = 22x2 dt  dt dt dt 







⇒ S = 22 x2 dS
⇒ = 2p[(5 × 3) + (20 × 2) + 2(5 × 2)] cm2/sec



dS dt




⇒ = 44x
dx = 2p[15 + 40 + 20] cm2/sec = 150p cm2/sec





⇒ dS  = 44 × 10 cm = 440 cm V = pr2h

­
dx  x =10 cm




dV = π  r 2 dh + h 2r dr 
Volume of the variable cuboid: V = lbh ⇒
dt  dt ( ) dt 





⇒ V = (3x)(2x)(x) = 6x3 ⇒ V = 6x3
dV




dV ⇒ = p[(5)2 × 3 + 20 × (2 × 5) × 2] cm3/sec
⇒ = 18x2 dt




dx



dV

dV 
= 18(10)2 cm2 = 1800 cm2 ⇒ = p[75 + 400]cm3/sec = 475p cm3/sec.
dx  x =10 cm dt






Diagonal of the variable cube: D = l 2 + b 2 + h 2 Cone
Volume of a variable cone increases or decreases in such

⇒ D= 9x 2 + 4x 2 + x 2 cm = 14 x a way that ratio between its radius, height and slant height



is always same. In questions based on variable cones, a
dD
⇒ = 14 relation between radius, height and slant height is given.
dx



Such questions are solved either by changing radius into height
Sphere or height into radius. For example, if a variable cone is such that
4 3 its radius is always 3 times of its height, then r = 3h.
If r is the radius of a sphere then: S = 4pr2 and V = πr .

()
3
1 2 1 2 r 1 3
Example 3. The radius of a spherical balloon is increasing at the V = 3 πr h = 3 πr 3 = 9 πr
rate of 2 cm/sec when it is being inflated. Find rates with which


its surface area and volume are increasing at the time when its
radius is 25 cm. or V=
1 2
3
1
3
( )
πr h = π 9h 2 h = 3πh3



dr dV 1 2 dr
Solution. It is given that: = 2 cm/sec, r = 25 cm dV dh
dt Then = πr = 9πh 2 or
dt 3 dt dt dt




dS dV
To find
dt
and
dt Also r = 3h ⇒ dr = 3 dh
dt dt






dS dr
S = 4pr2 ⇒ = 8πr Example 5. Falling sand forms a conical heap on the ground in
dt dt



such a way that its radius is increasing at the rate of 1 cm/sec. If
dS
⇒ = 8p × 25 × 2 cm2/sec = 400 p cm2/sec the radius of cone so formed is always 5 times of its height, find
dt



the rate with which sand is falling on the ground when the radius
4 3 of the cone is 20 cm.
V = 3 πr


dr
dV = 4πr 2 dr = 4p (25)2 × 2 cm3/sec Solution. In a variable cone: r = 5h, = 1 cm/sec, r = 20 cm.
⇒ dt
dt dt d V




To find:
= 5000 p cm3/sec dt
()

1 1 r 1


Volume of a cone: V = πr 2h ⇒ V = πr 2 = πr 3
Cylinder 3 3 5 15


If r and h are the radius and height of a cylinder, dV 1 2 dr 1
= π × ( 20) × 1 cm3/sec
2
⇒ = πr
then: dt 5 dt 5



S = 2prh + 2pr2 and V = pr2h dV
⇒ = 80 p cm3/sec
Example 4. If the radius and height of a variable dt



right circular cylinder are increasing at the rate of Example 6. An inverted conical vessel whose height is 10 m and
2 cm/sec and 3 cm/sec respectively, find at what radius 5 m is filled with water at the rate of 8 cubic metre/minute.
rates its total surface area and volume increasing

Find the rate at which the level of the water in the vessel rising
when its height is 20 cm and radius 5 cm. when the depth of the water is 4 m.

pplication of Derivatives 141


A

E:\AMIT_WORKS\Exam_Guru\EG_Mathematics-12_(working_02-06-2022)\EG_Mathematics-12_working\Open_Files\Chap_6\Chap_6
\ 22-Aug-2022  Ved_Goswami   Proof-5 Reader’s Sign _______________________ Date __________


h 10 Volume of water in the tank
Solution. In a cone, it is given: = =2


r 5
V = 1 πr 2h
3
h = 2r and dV = 8 m3/min, h = 4 m




dt
()



2
1 l  3l  1 l 2 3l
()
2 ⇒ V= π = π ×
To find
dh 1 1 h
, V = πr 2h = π
1
h = πh3 3 2  2  3 4 2




dt 3
3 2 12
1
dV 1 dh ⇒ V= πl 3 ⇒ dV = 3 πl 2 dl
⇒ = πh 2 8 3 dt 8 3 dt





dt 4 dt



1 2 2 dh dh 2 3 2 dl 48π dl
⇒ 8 m3/min = π ( 4) m ⇒ = m/min. ⇒ 1= π × ( 4) × =
4 dt dt π 8 3 dt 8 3 dt







\ The level of the water in the conical vessel is increasing at
6π dl
⇒ dl = 3 cm/sec

2 ⇒ 1=
the rate of m/min. 3 dt dt 6π
π





Example 7. Water is dripping out at the rate of 1 cm3/sec through ⇒ Slant height of water is decreasing at the rate of
a tiny hole at the vertex of an inverted conical vessel whose axis


3 cm/sec.
is vertical. When the slant height of the water in the vessel is 6π
4 cm, find the rate of decrease of (i) slant height of the water and
(ii) Let A be the area of water surface, i.e.,
(ii) the area of the water surface, given that the vertical angle of
2
the vertex is 60°. [V. Imp.]
A = πr 2 = πl  r = l 

Solution. (i) Semi vertical angle = q = 30° 4  2 



dV = 1 cm3/sec, l = 4 cm
dA 2π dl 2π 3
dt ⇒ = l = ×4×


dt 4 dt 4 6π



r
In DABC, = sin 30°
l  dl 3



l l = 4 cm, dt = 6π 
⇒ r=  

2



3
h = cm 2/ sec
= cos 30° ⇒ h = 3l 3


l 2


EXERCISE 6 (A)
I. Very Short Answer Type Questions 6. The volume V cm 3 of water in a leaking vessel is


1. If x is the side of variable cube, find at what rates its surface given in terms of time in seconds by the formula
V = 100 – 4t + t2. Find the rate at which the water is running


area, volume and diagonal are increasing with respect to
x, when x = 20 cm. out of the vessel when t = 10 sec.
II. Short Answer Type Questions-I
2. If x is the length of the side of a variable cube and
dx = 1 cm/sec, find the rates with which its surface 1. The height of a closed variable right circular cylinder is




dt always 5 times of its radius. Find the radius and rate of
area, volume and diagonal are changing w.r.t. time, when change of radius of the cylinder when volume and total
surface area of the cylinder are changing at the rate of

x = 10 cm.
400 π cm3/sec and 50 π cm2/sec respectively.


3. Find the rates with which the volume and surface area of a 2. If length, breadth and height of a cuboid are changing at




spherical balloon are increasing with respect to the radius, the rate of –1 cm/sec, 2 cm/sec and 1 cm/sec respectively,
find the rates of change of surface area and volume of the
when radius of the balloon is 20 cm.
cuboid when its length, breadth and height are 20 cm, 15
4. The radius of a spherical balloon is increasing at the rate cm and 10 cm respectively.


of 2 cm/sec when it is being inflated. Find rates with this 3. Find the radius of the variable spherical balloon at the time


when its volume and surface area are increasing at the rate
its surface area and volume are increasing at the time when
of 200 cm3/sec and 40 cm2/sec. Also find the rate with
p
p
its radius is 25 cm. which the radius of the balloon is increasing at that time.
5. The volume of a spherical balloon is increasing at the rate 4. An oil tank is being filled. The oil volume V in litres after



of 20π cm3/sec. Find the rate of change of the surface area t minutes is given by: V = 3t + t2. How fast is the volume

­
at the instant when the radius is 10 cm. increasing when there is 10 litres of oil in the tank?

142 Mathe atics–12
m
E:\AMIT_WORKS\Exam_Guru\EG_Mathematics-12_(working_02-06-2022)\EG_Mathematics-12_working\Open_Files\Chap_6\Chap_6
\ 17-Aug-2022  Ved_Goswami   Proof-5 Reader’s Sign _______________________ Date __________


Cone the height of the cone is always one-sixth of the radius of
III. Short Answer Type Questions-II the base. How fast is the height of the sand cone increasing
1. Water is leaking from a conical funnel at the rate of 5 cm3/ when the height of the cone is 4 cm? [Delhi 2011]




sec. If the radius of the base of the funnel is 5 cm and height 2. Water tank has the shape of an inverted circular cone with



10 cm, find the rate at which the water level is dropping its axis vertical and vertex is lower most. Its semi-vertical
when it is 2.5 cm from the top. angle is tan–1 (0.5). Water is poured into it at a constant rate
IV. Long Answer Type Questions of 5 m3/hr. Find the rate at which level of the water rising
1. Sand is falling from a pipe at the rate of 12 cm3/sec. The at the instant when the depth of the water in the tank is 4
metres.


falling sand forms a cone on the ground in such a way that

Answers 6 (A)
dS = 240 cm dV = 1200 cm 2 dD = 3 5. dS = 4 p cm2/sec
I. 1. ; ;
dx dx dx dt



Hint: It is a cube.  dV 

S = 6x2 Hint: It is a sphere. Use:  dt  = r
 dS  2



dS  dt 
⇒ = 12x cm = 12 × 20 cm = 240 cm.
dx



V = x3 Put dV = 20π cm3/ sec and r = 10 cm .
dt



dV
⇒ = 3x2 cm2 = 3(20)2 cm2
dx 6. d V = 16 cm3/sec.



dD dt


D = 3x ⇒ = 3
dx Hint: V = 100 – 4t + t2







dS dV = (–4 + 2t) cm3/sec
2. = 120 cm2/sec; dV = 300 cm3/sec ⇒
dt dt dt





dD = 3 cm/sec Now put t = 10 sec.

dt
II. 1. r = 12.8 cm and dr = 125 cm/sec

Hint: It is a cube dt 768

S = 6x2 Hint: It is a cylinder.
( )


dS

dx cm 2/sec
⇒ = 12x h = 5r, dV = 400 p cm3/sec
dt dt dt





V = x3 dS = 50 p cm2/sec, r = ?, dh = ?
( )


dV dx cm3/sec dt dt


⇒ = 3x 2
dt dt



V = pr2h = pr2(5r) = 5pr3


dD = 3 cm/sec dV = 15pr2 dh
D = 3x ⇒ ⇒
dt dt




dt



Now put x = 10 cm and dx S = 2prh + 2pr2 ⇒ S = 2pr(5r) + 2pr2
= 1 cm/sec
dt




⇒ S = 10pr + 2pr = 12pr2
2 2
dV
3. dS = 160 p cm,



= 1600 p cm2. 15πr 2 dr
dr dr
dt = 15 r



⇒ dS = 24pr dh ⇒ dV/dt =
Hint: It is a sphere. dt dt dS/dt 24πr dr 24






S = 4pr2 ⇒ dS = 8pr cm 400π 15 r
dt
dr




⇒ = ⇒ 15r = 24 × 8
dV 50π 24





4 πr 3
V= ⇒ = 4pr2 cm2
3 dr 64 cm dh




⇒ r= . Now find dt .
Now put r = 20 cm. 5



4. dS = 400 p cm2/sec; dV = 5000 p cm3/sec 2. dS = 140 cm2/sec; dV = 550 cm3/sec
dt dt dt dt




Hint: It is a sphere. Hint: It is a cuboid.


S = 4pr2 S = 2[lb + bh + hl]

( ) ( ) ( )




⇒ dS = 8πr dr cm 2/ sec ⇒ dS = 2 l db + b dl + b dh
dt  dt

dt dt dt dt






( ) ( ) ( )
4 πr 3 dr
V= ⇒ dV = 4πr 2 cm3/sec + h db + h dl + l dh  cm 2/sec

3 dt dt



dt dt dt 
dh
Now put r = 25 cm and dt = 2 cm/sec V = lbh



pplication of Derivatives 143
A

E:\AMIT_WORKS\Exam_Guru\EG_Mathematics-12_(working_02-06-2022)\EG_Mathematics-12_working\Open_Files\Chap_6\Chap_6
\ 17-Aug-2022  Ved_Goswami   Proof-5 Reader’s Sign _______________________ Date __________


dV =  dl (bh ) + db (lh ) + dh (lb ) cm3/ sec Also h = (10 – 2.5) = 7.5 cm




 dt dt dt  1 2 1 2
dt V = πr h = πr ( 2r )



dl 3 3



Now put l = 20 cm, b = 15 cm, h = 10 cm, = –1 cm/sec
dt 2 πr 3 dV = 2π 2 dr
( )




db dh ⇒ V= ⇒ r
= 2 cm/sec and = 1 cm/sec 3 dt dt
dt dt








dV = 5 cm3/sec
3. r = 10 cm and dr = 1 cm/sec Now put , r = 5 cm and get dh
dt 2 dt dt




Hint: It is a sphere.
IV. 1. dh = 1 cm/sec

V = 4 πr 3 ⇒ dV = 4πr 2 dr cm3/sec dt 48π
3 dt dt



Hint: It is a cone.
dS = 8πr dr cm 2/ sec


S = 4pr2 ⇒ 1
dt dt V = πr 2h, r = 6h
3






 dV   4πr 2 dr 
 dt  = 

dt  = r ⇒ V=
3 (
1 π 36h 2 h = 12πh3
)
dr  2




 dS 


 8 πr
 dt  dt  dV 2 dh
⇒ = 36πh
dt dt
Now put dV = 200 π cm3/sec




dt

dV
Now put = 12 cm3/sec, h = 4 cm and get dh
dS dt dt
= 40 p cm2/sec


dt


2. dh 5 35
= m/h = m/h
4. dV = 7 litres/min. dt 4π 88


dt


Hint: It is a cone.
Hint: V = 10, V = t2 + 3t

h r



⇒ t2 + 3t – 10 = 0 ⇒ t = 2 min. = 0.5 ⇒ h=
r 2








dV 
dt  t =2 min
= 2t + 3 = 7 litres/min.
V = 1 πr 2h = 1 π 4h 2 h ( )

3 3


dh = 16 cm/sec
III. 1. 4 πh3 dV = 4πh 2 dh
dt 45π ⇒ V= ⇒
3 dt dt





Hint: It is a cone.

h dV
= 10 ⇒ h = 2r Now put = 5 m3/hr, h = 4 m and get dh
r 5 dt dt





(B) SHADOW FORMATION, SLIDING OF LADDER AND MOTION OF A
PARTICLE ALONG A CURVE
Example 1. A man of height 2 m is running away from a lamp 5 x+ y

post of height 5 m at the rate of 1 m/sec. Find at what rate the ⇒ = ⇒ 5x = 2x + 2y
2 x





dy
shadow of the man is lengthening. Also ⇒ 3x = 2y ⇒ 3 dx = 2
dt dt





find with what rate tip of the shadow is
moving away with respect to the lamp-post. dx = 2 ×1 dx 2
⇒ 3 ⇒ = m/sec
Solution. Let AB be the lamp-post of dt dt 3




height 5 m, CD be the running man of ⇒ The length of the shadow of the man is increasing at the


height 2m. DE = x be the length of the 2
rate of m/sec .
shadow at any time, y be the distance of 3
the man from the pole. Now the rate with which tip of the shadow of the man is

dx moving away from the pole
\ = rate of lengthening the shadow of the man
dt
( )



dy dx dy 2 5
= + = + 1 m/sec = m/sec
= rate with which man is running away from the dt dt 3 3


dt


pole = 1 m/sec
Example 2. A ladder 13 m long is leaning against a vertical wall.
AB BE
DABE ~ DCDE ⇒ = The foot of the ladder is dragged away from the wall at a rate of
CD DE



[Corresponding sides of similar Ds are proportional] 3 m/sec. How fast the angle q between the ladder and the ground

2

144 Mathe atics–12


m
E:\AMIT_WORKS\Exam_Guru\EG_Mathematics-12_(working_02-06-2022)\EG_Mathematics-12_working\Open_Files\Chap_6\Chap_6
\ 17-Aug-2022  Ved_Goswami   Proof-5 Reader’s Sign _______________________ Date __________


is changing when the foot of the ladder is 12 m away from the dθ −507 −3
⇒ = = = − 0.3 radian/sec.


foot of the wall. dt 1690 10




3
Solution. Let AB be the ladder, x = 12 m Example 3. A particle moves along the curve: y = 2x + 1 . Find


be the distance of the foot of the ladder 3
the point on the curve at which y-coordinate is changing twice
and foot of the wall and y be the height
as fast as x-coordinate.


of the upper end of the ladder. Here
Solution. Let the required point be P(x, y) on the curve
dx 3
= m/sec is the speed with which 2x3
dt 2 y= +1 .
lower end of the ladder is being dragged. 3
dy Given condition: rate of change of y-coordinate = 2(rate of
is the speed with which upper end of


dt change of x-coordinate).
the ladder is coming down. dy dx
⇒ = 2
dt dt




Since ABC is right triangle

y 2x3 dy dx
\ x + y = 169 and tan q = = 5
2 2 Now y=
3
+1 ⇒
dt
= 2x 2
dt
x 12









dy dy
⇒ 2x dx + 2y
dy
= 0 ⇒
dy −18
= m/sec On putting = 2x 2 dx in dx
= 2 , we get
dt dt dt 5 dt dt dt dt








dy
y x − y dx 2x 2
dx
= 2
dx
⇒ x2 = 1 ⇒ x = ±1
dθ2 dt dt dt dt
Now tan q = ⇒ sec θ =




x dt x2 3
x = ±1, in y = 2x + 1 ,




On putting
 x dy − y dx  3



dy
dθ  dt dt  x − y dx
⇒ = = 2 dt dt y = 5
, when x = 1
dt x 2 sec 2 θ ( )
x 1 + tan 2 θ ( )  3



we get 
1

( ) ()
y = , when x = −1
12 −18 − 5 3 −216 − 15  3
dθ 5 2 = 5
( ) ( )
2
( )
⇒ = 169 5 1
dt 25 ⇒ The required points are: 1, and −1, .
( ) 144
144 1 + 144 ×



144 3 3

EXERCISE 6 (B)
I. Long Answer Type Questions road. How fast is his shadow increasing if he is walking
1. A man of height 2 metres walks at a uniform speed of 5 away from the light pole at a rate of c metres/sec?


km/hour away from a lamp post, which is 6 metres high. 6. A particle moves along a curve: y = 5x3 + 4x + 5. Find the


Find the rate at which the length of his shadow increasing. point on the curve at which the y-coordinate is changing
2. A particle moves along a curve : 6y = x3 + 2. Find the points 19 times as fast as x-coordinate.


on the curve at which the y-coordinate is changing 8 times 7. The ends of a rod AB which is 5 metres long moves along


as fast as x-coordinates. [V. Imp.] the two axes OX and OY which are at right angles. If A

1
3. At what points on the ellipse 16x2 + 9y2 = 400, does the moves at a constant speed of m/sec. What is the speed
2



ordinate decreasing at the same rate at which abscissa is of B which is 4 metres from O?
increasing? 8. A particle is moving according to the equation S = 2t3 – 9t2


2
4. Find the point on the curve y = 8x for which the abscissa + 12t + 1, where S is the displacement in cm, and t is the



and ordinate changes at the same rate. time in seconds find (i) velocity and acceleration of the
5. A point source of light along straight road is at a height of particle at t = 2 sec. (ii) The time when the particle stops



‘a’ metres. A boy ‘b’ metres in height is walking along the momentarily and (iii) the distance between the two points.

Answers 6 (B)
1. Rate of increase of shadow = dx = 2.5 km/hour 
2. (4, 11) and  − 4,
− 31

dt  3 




Hint: See the solution of Example 1.
Hint: See the solution of Example 3.


pplication of Derivatives 145
A

E:\AMIT_WORKS\Exam_Guru\EG_Mathematics-12_(working_02-06-2022)\EG_Mathematics-12_working\Open_Files\Chap_6\Chap_6
\ 17-Aug-2022  Ved_Goswami   Proof-5 Reader’s Sign _______________________ Date __________


3. ( )
3, 16 and  − 3,
3 
− 16 
3 
 \ 19
dx
dt
= 15x 2 + 4 ( )
dx
dt ( )






Hint: It is an ellipse. ⇒ 15x2 + 4 = 19




16x2 + 9y2 = 400 ⇒ 15x2 – 15 = 0

dy


⇒ 32x dx + 18y = 0 ⇒ x2 = 1
dt dt






⇒ x = ±1. Now find y.
dy dx




⇒ 18y = −32x dy − 3
dt dt 7. = m/sec



dt 8



⇒ 18y = –32 x  dy = dx 
 dt dt  dx 1 m/sec




Hint: =
⇒ y = −16 x dt 2




9



x2 + y2 = 25
Put it in 16x2 + 9y2 = 400




9 × 256 2 dy
16x 2 + x = 400 ⇒ 2x dx + 2y = 0
81 dt dt




⇒ x = ±3 and y = ± 16 ⇒
dy
x dx + y = 0
3 dt dt





( )
3, 16 and  − 3,
3 
− 16 
3 
 Put x = 3 m,
dx
dt
1
= m/sec and y = 4 m in
2



4. (2, 4) dy dy
x dx + y = 0 to get .


Hint: y = 8x 2 dt dt dt




⇒ 2y
dy
= 8
dx 8. (i) dS = 0 cm/sec;
dt



dt dt



dy dx dv = d 2s
Since = \ 2y = 8 ⇒ y=4 = 6 cm/sec2
acc. =
dt dt dt 2

dt







Now find x. Hint: S = 2t3 – 9t2 +12t + 1



dS = 6t 2 − 18t + 12

dx =  bc  ⇒Velocity =
5. m/sec dt
dt  a − b 





dS 
dy = dt  = 24 – 36 + 12 = 0 cm/sec
 t =2


Hint: = c m/sec
dt (ii) t = 1 sec, 2 sec





Now see the solution of Example 1. Hint: 6(t2 –3t + 2) = 0



6. (1, 14) and (– 1, – 4) ⇒ t = 1 sec, 2 sec




Hint: y = 5x3 + 4x + 5 ⇒ Particle stops momentarily at t = 1 sec and t = 2 sec.








dy
= 15x 2 + 4
dx
( ) (iii) distance between two points = 1 cm.


dt dt Hint: St = 1 = (2 – 9 + 12 + 1) cm = 6 cm



( )



 dy  dx St = 2 = (16 – 36 + 24 + 1) cm = 5 cm
Now   = 19


dt dt St = 2 – St = 1 = 6 – 5 = 1 cm.





Topic 2. Increasing and Decreasing Functions
Increasing Function Finding the intervals in which a function is
(i) A function y = f (x) is said to be increasing on an interval increasing or decreasing


[a, b] if f ′(x) ≥ 0 on the interval [a, b]. Working Rule

(ii) A function y = f (x) is said to be strictly increasing on an (i) Take the function y = f (x).




interval [a, b] if f ′(x) > 0 on the interval [a, b]. dy
= f ′ ( x)

(ii) Work out

dx


Decreasing Function
(iii) To find the intervals in which f (x) increases, solve
(i) A function y = f (x) is said to be decreasing on an interval


f ′(x) ≥ 0 and to find the intervals in which f (x) strictly



[a, b] if f ′(x) ≤ 0 on the interval [a, b]. increases, solve f ′(x) > 0.


(ii) A function y = f (x) is said to be strictly decreasing on the (iv) To find the intervals in which f (x) decreases, solve





interval [a, b] if f ′(x)< 0 on the interval [a, b]. f ′(x) ≤ 0 and to find the intervals in which f (x) strictly



decreases solve f ′(x) < 0.

146 Mathe atics–12
m
E:\AMIT_WORKS\Exam_Guru\EG_Mathematics-12_(working_02-06-2022)\EG_Mathematics-12_working\Open_Files\Chap_6\Chap_6
\ 17-Aug-2022  Ved_Goswami   Proof-5 Reader’s Sign _______________________ Date __________


Example 1. Find the intervals in which the function: If a > b and (x – a) (x – b) < 0 ⇒ b < x < a]

{ } { ( )}


3 2
f (x) = x – 4x + 4x + 5 is 2 2
⇒ f (x) is S↓ in the interval: < x < 2 or x ∈ , 2

(i) strictly increasing or S↑ 3 3


Third Method:


(ii) strictly decreasing or S↓


f ′(x) = (3x – 2)(x – 2).


Solution. f (x) = x3 – 4x2 + 4x + 5




⇒ x = 2 ,2


⇒ f ′(x) = 3x2 – 8x + 4 = (x – 2)(3x – 2) Take (3x – 2)(x – 2) = 0
3






To find the x-coordinates of the critical point, solve:
Plot the points x = 2 and x = 2 on a number line.

(x – 2) (3x – 2) = 0 3


⇒ x = 2, 2 Decide the sign of f ′(x) below x = 2


3 3





First Method: x = 2 , 2
3


Intervals in which f (x) is S↑ or S↓ It can be done by putting x = 0, a number just smaller




than x = 2 . f ′(0) = (–) (–) = +, so it gives + sign.
 2 2
( )


3
x < 3 or x ∈ −∞,


3

2 2
 < x < 2 or x ∈ , 2 ( ) On the number line put + sign before x = 2 put – sign between
3


3 3 2 and 2 and + sign next to 2.


 3
 x > 2 or x ∈( 2, ∞ )
⇒ f (x) is strictly increasing in −∞, 2 ∪ ( 2, ∞ ) and strictly ( )

3
( )
f (x) is S↑ in the intervals:
2


decreasing in ,2 .
{ }
x<
2 ∪ {x > 2}
3
or { ( x ∈ − ∝,
2
3 )}∪ {x ∈ (2, ∞)}
3

The monotonicity of a function on a given values
and f (x) is S↓ in the interval:
2
3
2
< x < 2 or x ∈ , 2
3 { } { ( )} internal of x

Example 2. For all real values of x, the function: f (x) = x3 + 3x


Decision Making Table + 5 is
Intervals x f ′(x) = (3x – 2) Sign Result (a) strictly decreasing (b) strictly increasing





(x –2) (c) increasing (d) decreasing

( )




2 2 0 (–) (–) + S↑ Solution. f (x) = x + 3x + 5 3
x < or x ∈ −∞,



3 3 ⇒ f ′(x) = 3x2 + 3 = 3(x2 + 1)




2 2
( ) 1 (+) (–) – S↓ Since f ′(x) = 3(x2 + 1) > 0 for all real values of x,

< x < 2 or x ∈ , 2



3 3 \ f (x) is strictly increasing function for all values of x.


x > 2 or x ∈ (2, ∞) 3 (+) (+) + S↑ Hence, option (b) is the correct answer.
Second Method: f (x) = x – 4x + 4x + 5 3 2 Example 3. For all real values of x, the function: f (x) = 3x3 +





⇒ f ′(x) = 3x2 – 8x + 4 = (x – 2) (3x – 2) 6x2 + 4x + 8




For S↑ intervals, solve: (a) increases (b) strictly increasing

( )





2 (c) decreases (d) strictly increasing
(x – 2) (3x – 2) > 0 ⇒ 3 ( x − 2) x − > 0




3


3 2
2 Solution. f (x) = 3x + 6x + 4x + 8

⇒ x > 2 or x <


3 ⇒ f ′(x) = 9x2 + 12x + 4 = (3x + 2)2






[If a > b and (x – a) (x – b) > 0 ⇒ x > a or x < b] Since (3x + 2)2 ≥ 0 for all real values of x,

⇒ f (x) is S↑ in the intervals

\ f (x) increases for all real values of x.

{ } { ( )}


2 2
x< ∪{x > 2} or x ∈ − ∞, ∪ { x ∈ (2, ∞)} Hence, option (a) is the correct answer.
3 3


Example 4. For all real values of x, the function: f (x) = 3x + sin x is

For S↓ intervals, solve
(a) strictly increasing (b) strictly decreasing

⇒ 2<x<2




(3x – 2) (x – 2) < 0 (c) increasing (d) decreasing
3






pplication of Derivatives 147
A

E:\AMIT_WORKS\Exam_Guru\EG_Mathematics-12_(working_02-06-2022)\EG_Mathematics-12_working\Open_Files\Chap_6\Chap_6
\ 17-Aug-2022  Ved_Goswami   Proof-5 Reader’s Sign _______________________ Date __________


Solution. f (x) = 3x + sin x ⇒ f ′(x) = 3 + cos x Solution. f (x) = x3 + 18x2 + 144x + 5










Since (3 + cos x) > 0 for all real values of x, ⇒ f ′(x) = 3x2 + 36x + 144






\f (x) is a strictly increasing function for all real values of x. ⇒ f ′(x) = 3[x2 + 12x + 48]






Hence, option (a) is the correct answer. ⇒ f ′(x) = 3[(x + 6)2 + 12]


Example 5. For all real values of x, the function: f (x) = x3 + 18x2




+ 144x + 5 is ⇒ f ′(x) = 3(x + 6)2 + 36 > 0 for all real values of x





(a) strictly increasing (b) decreasing \ f (x) is a strictly increasing function.







(c) increasing (d) strictly decreasing Hence, option (a) is the correct answer.




EXERCISE 6.2
I. Multiple Choice Questions (MCQs) 4. Find the intervals in which the function: f (x) = (x – 1)3




Choose the correct answer from the given options. (x – 2)2 is
1. The function given by f(x) = 3x + 17 is (i) increasing (ii) decreasing







(a) increasing on R (b) decreasing on R [A.I. 2011 (C)]






3 2
(c) strictly increasing on R (d) strictly decreasing on R 5. Show that the function: f (x) = x – 3x + 4x, x ∈ R, strictly




increasing (S↑) on R. [A.I. 2011 (C)]




2. The intervals in which the function f(x) = ex is strictly


6. Find the intervals in which the function: f (x) = x4 – 8x3 +


increasing are




22x2 – 24x + 21
(a) Only ∀ x ∈ R (b) Only ∀ x ∈ N
(i) strictly increases (S↑) (ii) strictly decreases (S↓)




(c) Only ∀ x ∈ Q (d) None of these



[A.I. 2012 (C), Delhi 2014]




2 –x
3. The function y = x e is decreasing in the interval
7. Find the intervals in which the function:


(a) (0, 2) (b) (2, ∞)


3x 4 4 x 3 36x
f ( x) =




(c) (– ∞, 0) (d) (– ∞, 0) ∪ (2, ∞) − − 3x 2 + + 11
10 5 5




II. Short Answer Type Questions-I (i) strictly increases (S↑) (ii) strictly decreases (S↓)




1. Show that the function f (x) = x3 – 6x2 + 12x + 8 increases [Delhi 2014 (C)]



(↑) for all real values of x. 8. 2
Prove that the function : f (x) = x – x + 1 is neither increases


2. Find the intervals in which the function is given by: f (x) = or decreases in (– 1, 1). Hence, find the intervals in which



x2 – 4x + 6 is (a) strictly increasing, (b) strictly decreasing. f (x):

(i) strictly increases (S↑) (ii) strictly decreases (S↓)
3. Find the intervals in which the function is given by f (x) =





[Delhi 2014 (C)]


2x2 – 3x is (a) strictly increasing, (b) strictly decreasing.

9. Find the intervals in which the function:

4. Find the intervals for each of the following functions (a)


1
f ( x ) = x3 + 3 , x ≠ 0


strictly increasing or (b) strictly decreasing.
x
(i) f (x) = xex (ii) f (x) = xe–x (i) strictly increases (S↑) (ii) strictly decreases (S↓)










(iii) f (x) = x2ex (iv) f (x) = x2e–x [Delhi 2014]







10. Find the intervals in which the function: f (x) = (x – 1)

III. Short Answer Type Questions-II


(x – 2)3
1. Find the intervals in which the function: f (x) = 2x3 – 8x2
(i) strictly increases (S↑) (ii) strictly decreases (S↓)



+10x + 5 is




[A.I. 2014 (C)]
(i) strictly increasing or S↑

11. Find the intervals in which the function: f (x) = (x + 2)e–x



(ii) strictly decreasing or S↓ [A.I. 2011 (C)]


(i) strictly increases (S↑) (ii) strictly decreases (S↓)



2. Find the intervals in which the function: f (x) = (x – 1)




[Delhi 2016 (C)]



(x – 2)2 is

x 2
(i) increasing or ↑ (ii) decreasing or ↓ 12. Find the intervals in which the function: f ( x ) = + , x ≠ 0
2 x






3. Find the intervals in which the function f (x) = 2x3 + 9x2 + (i) strictly increases (S↑) (ii) strictly decreases (S↓)







12x + 20 is 13. Find the intervals in which the function:

( )


(i) strictly increasing or S↑ π
on [ 0, π ]
f ( x ) = sin x +


(ii) strictly decreasing or S↓ 4


[Delhi 2011 (C), A.I. 2011 (C)] (i) strictly increases (S↑) (ii) strictly decreases (S↓)





148 Mathe atics–12
m
E:\AMIT_WORKS\Exam_Guru\EG_Mathematics-12_(working_02-06-2022)\EG_Mathematics-12_working\Open_Files\Chap_6\Chap_6
\ 17-Aug-2022  Ved_Goswami   Proof-5 Reader’s Sign _______________________ Date __________


14. Find the intervals in which the function: f (x) = sin x 2. Find the intervals in which the function: f(x) = 20 – 9x +






– cos x on [0, 2p] 6x2 – x3.
(i) strictly increases (S↑) (ii) strictly decreases (S↓) (i) strictly increasing or S↑







15. Find the intervals in which the function: f (x) = sin x (ii) strictly decreasing or S↓ [A.I. 2010]







+ cos x on [0, 2p] 3. Show that the function f given by: f (x) = x – 3x2 + 4x, 3




(i) strictly increases (S↑) (ii) strictly decreases (S↓) x ∈ R is strictly increasing on R.




16. Find the intervals in which the function: 4. Prove that the function given by: f (x) = x3 – 3x2 + 3x – 100






π is increasing in R.
f ( x ) = sin 4 x + cos 4 x on 0, 
 2  5. Show that the function: f (x) = x3 – 6x2 + 12x –18 is an




(i) strictly increases (S↑) (ii) strictly decreases (S↓) increasing function on R.




 4 sin x  6. Show that each of the following functions is an increasing
17. Prove that the function: f ( x ) =  − x  is an



 2 + cos x  function on R.


increasing function of x in 0, π . ( ) (i) f (x) = 4x3 – 18x2 + 27x + 5


[A.I. 2011]



2 (ii) f (x) = 3x3 + 6x2 + 4x + 8




3 2
18. Show that the function f (x) = 3x + 6x + 4x + 8 is 7. Show that each of the following functions, is strictly




increasing function for all real values of x. increasing function on R.
19. Show that the function f (x) = 3x + sin x is strictly increasing (i) f (x) = 4x3 – 30x2 + 87x + 15






function for all real values of x. (ii) f (x) = x3 – 6x2 + 15x + 3


( )


20. Show that the function: f ( x ) = log ( x + 1) − 2x (iii) f (x) = log x, x > 0
, x > −1


x+2




(iv) f (x) = x + cot–1 x

strictly increases (S↑) for all real values of x.


(v) f (x) = x + tan–1 x

IV. Long Answer Type Questions


1. Find the intervals in which f (x) = [x(x – 2)]2 8. Show that each of the following functions, is an increasing


function on R.



(i) increases (↑) (ii) decreases (↓).
(i) f (x) = x + sin x




Also find the points on the curve f (x) = [x(x – 2)]2 where




(ii) f (x) = x – sin x

the tangent is parallel to x-axis. [Delhi 2010, AI 2014]




Answers 6.2
I. Multiple Choice Questions (MCQs) (b) f(x) S↓ in the interval f ′(x) < 0



1. (c) Strictly increasing on R ⇒ 2x – 4 < 0 ⇒ x<2







Hint: f ′(x) = 3 > 0 ∀ x ∈ R, or x ∈ (–∞, 2)

( )




\ f (x) is S↑ ∀ x ∈ R. 3. (a) f (x) S ↓ in −∞, 3


4




2. (a) ∀ x ∈ R
Reason: f (x) = 2x2 – 3x




Hint: f (x) = ex ⇒ f ′(x) = ex > 0 ∀ x ∈ R





⇒ f ′(x) = 4x – 3




\ f (x) is S↑ ∀ x ∈ R.




f (x) S↓ in the interval f ′(x) < 0



3. (d)

3


⇒ 4x – 3 < 0 ⇒ x<
II. 1. Hint: f (x) = x3 – 6x2 + 12x + 8 4






( )



f ′(x) = 3x2 – 12x + 12
or x∈ −∞, 3



⇒ f ′(x) = 3[x2 – 4x + 4] 4




( )



= 3(x – 2)2 ≥ 0 (b) f (x) S ↑ in 3 ,∞



4


\ f ′(x) is ↑ ∀ x ∈ R

Reason: f (x) S↑ in the interval f ′(x) > 0


2. f (x) is S↑ in (2, ∞,)




⇒ x> 3


f (x) is S↓ in (–∞, 2) ⇒ 4x – 3 > 0

4






Reason: f (x) = x2 – 4x + 6 ⇒ f ′(x) = 2x – 4
x ∈ 3 ,∞ ( )


or




(a) f(x) S↑ in the interval f ′(x) > 0 4






⇒ 2x – 4 > 0 ⇒ x>2 4. (i) f (x) S ↑ in (–1, ∞) and S ↓ in (–∞, –1)









or x ∈ (2, ∞) Reason: f (x) = xex ⇒ f ′(x) = (x + 1)ex










pplication of Derivatives 149
A

E:\AMIT_WORKS\Exam_Guru\EG_Mathematics-12_(working_02-06-2022)\EG_Mathematics-12_working\Open_Files\Chap_6\Chap_6
\ 17-Aug-2022  Ved_Goswami   Proof-5 Reader’s Sign _______________________ Date __________


(a) f (x) S ↑ in the interval f ′(x) > 0 ⇒ (x + 1)ex > 0
x ∈ (–∞, 1) ∪ 5 ,∞ ( )






⇒ x > –1 or x ∈ (–1, ∞) 3









(b) f (x) S ↓ in the interval f ′(x) < 0

f (x) is S↓ in the interval: f ′(x) < 0







⇒ (x + 1)ex < 0 ⇒ x < –1 ⇒ 2(x – 1)(3x – 5) < 0







⇒ x ∈ (–∞, –1)
5
or x ∈ 1, 5 ( )



(ii) f (x) S ↑ in (–∞, 1) and S ↓ in (1, ∞) ⇒ 1<x<
3 3







–x –x
Reason: f (x) = xe ⇒ f ′(x) = (–x + 1)e
{ }


2. f (x) is ↑ in the intervals: x ≤ 4 ∪ { x ≥ 2}





(a) f (x) S ↑ in the interval f ′(x) > 0 3







–x

{ ( }
⇒ (1 – x)e > 0 ⇒ 1–x>0
x ∈ −∞, 4  ∪ { x ∈[ 2, ∞ )}





⇒ x< 1 or x ∈ (–∞, 1) or
3 








(b) f (x) S ↓ in the interval f ′(x) < 0


f (x) is ↓ in the interval


⇒ (1 – x)e–x < 0 ⇒ 1–x<0


{ } { }





4 ≤ x ≤ 2 or x ∈  4 , 2
⇒ x–1> 0 ⇒ x>1
3  3 





⇒ x ∈ (1, ∞)


Hint: f (x) = (x – 1)(x – 2)2




(iii) f (x) S ↑ in (–∞, –2) ∪ (0, ∞) and S ↓ in (–2, 0)





⇒ f ′(x) = (x – 2)(3x – 4)


Reason: f (x) = x2ex ⇒ f ′(x) = x(x + 2)ex












(a) f (x) S ↑ in the interval f ′(x) > 0 (a) f (x) ↑ in the interval f ′(x) ≥ 0










or x(x + 2)ex > 0 ⇒ x(x + 2) > 0 ⇒ (x – 2)(3x – 4) ≥ 0







⇒ x < –2, x > 0 ⇒ x ∈ (–∞, –2) ∪ (0, ∞) 4
⇒ x≥2 or x ≤
3









{ ( )}
(b) f (x) S ↓ in the interval f ′(x) < 0



x ∈ −∞, 4 ∪ { x ∈( 2, ∞ )}


x(x + 2)ex < 0 ⇒ –2 < x < 0 or
3






⇒ x ∈ (–2, 0)
(b) f (x) is ↓ in the interval: f ′(x) ≤ 0




(iv) f (x) S ↑ in (0, 2); S ↓ in (–∞, 0) ∪ (2, ∞)



⇒ (x – 2)(3x – 4) ≤ 0


Reason: f (x) = x2e–x ⇒ f ′(x) = –x(x – 2)e–x


{ }







(a) f (x) S ↑ in the interval: f ′(x) > 0 ⇒ 4≤x≤2 or x ∈  4 , 2
 3 



3






⇒ – x(x – 2)e–x > 0 ⇒ x(x – 2) < 0
3. f (x) is S↑ in the intervals: {x < –2} ∪ {x > –1}






⇒ 0<x<2





⇒ x ∈ (0, 2) or {x ∈ (–∞, –2)} ∪ {x ∈ (–1, ∞)}





(b) f (x) S ↓ in the interval f ′(x) < 0 f (x) is S↓ in the interval: {–2 < x < –1}







⇒ –x(x – 2)e–x < 0 ⇒ x(x – 2) > 0 or {x ∈ (–2, –1)}







⇒ x>2 or x < 0 Hint: f (x) = 2x3 + 9x2 + 12x + 20









⇒ x ∈ (–∞, 0) ∪ (2, ∞) ⇒ f ′(x) = 6x2 + 18x + 12 = 6(x + 1)(x + 2)

{ }






III. 1. f (x) S↑ in the interval: x > 5 ∪ { x < 1} (a) f (x) is S↑ in the interval: f ′(x) > 0



3

⇒ 6(x + 1)(x + 2) > 0 ⇒ {x > –1} ∪ {x < –2}
5
{ ( )}




or { x ∈ (−∞,1)} ∪ x ∈ , ∞ or {x ∈ (–∞, –2)} ∪ {x ∈ (–1, ∞)}
3




f (x) is S↓ in the interval: (b) f (x) is S↓ in the interval: f '(x) < 0



} { ( )}

{1 < x < 5 or x ∈ 1, 5
⇒ 6(x + 1)(x + 2) < 0 ⇒ {–2 < x < – 1}




3 3 or {x ∈ (–2, –1)}



Reason: f (x) = 2x3 – 8x2 + 10x + 5 4. f (x) is ↑ in the intervals:



{ } {



⇒ f ′(x) = 6x2 – 16x + 10 x ≤ 8 ∪ x ≥ 2}

5



⇒ f ′(x) = 2[3x2 –8x + 5]

{ ( }





⇒ f ′(x) = 2(x – 1)(3x – 5) or x ∈ − ∞, 8  ∪ { x ∈[2, ∞)}
5 






f (x) is S↓ in the interval: f ′(x) > 0
f (x) is ↓ in the interval:



} { }
⇒ 2(x – 1)(3x – 5) > 0
{ 8 ≤ x ≤ 2 or x ∈  8 , 2


5 5  5 
⇒ x<1 or x >
3





150 Mathe atics–12
m
E:\AMIT_WORKS\Exam_Guru\EG_Mathematics-12_(working_02-06-2022)\EG_Mathematics-12_working\Open_Files\Chap_6\Chap_6
\ 17-Aug-2022  Ved_Goswami   Proof-5 Reader’s Sign _______________________ Date __________


Hint: f (x) = (x – 1)3 (x – 2)2 1 6x3 − 12x 2 − 30x + 36


⇒ f ′(x) =



­
5 


2
⇒ f ′(x) = (x – 2)(x – 1) (5x – 8)








f (x) is ↑ in the interval: f ′(x) ≥ 0 6  x3 − 2x 2 − 5x + 6
⇒ f ′(x) =


5 






⇒ (x – 2)(x – 1)2(5x – 8) ≥ 0
6 x −1 x + 2 x − 3


8
{ } ⇒ f ′(x) = ( )( )( )


⇒ {x ≥ 2} ∪ x ≤ 5




5


Solve: (x – 1)(x + 2)(x – 3) = 0
{ ( }
x ∈ −∞, 8  ∪ { x ∈[ 2, ∞ )}


or ⇒ x = –2, 1, 3
5 





f (x) is ↓ in the interval: f ′(x) ≤ 0  

⇒ (x – 2)(x – 1)2(5x – 8) ≤ 0 Put x = –3 in f ′(x). It gives negative sign. Therefore, in between




} { }


{ 8 ≤ x ≤ 2 or x ∈  8 , 2 –2 and 1, f ′(x) is positive. In between 1 and 3, f ′(x) is negative.




5  5  For x > 3, f ′(x) is positive.



5. Hint: f (x) = x3 – 3x2 + 4x, x ∈ R \ f (x) S↑ in the interval f ′(x) > 0










⇒ f ′(x) = 3x – 6x 4 2 ⇒ {–2 < x < 1} ∪ {x > 3}






= [3(x – 1) + 1] > 0 ∀ x ∈ R 2 or {x ∈ (–2, 1)} ∪ {x ∈ (3, ∞)}





\ f (x) is S ↑ ∀ x ∈ R and f (x) S↓ in the interval f ′(x) < 0







6. f (x) is S↑ in the intervals: {1 < x < 2} ∪ {x > 3} ⇒ {x < –2} ∪ {1 < x < 3}




or {x ∈ (1, 2)} ∪ {x ∈ (3, ∞)} or {x ∈ (–∞, –2)} ∪ {x ∈ (1, 3)}

{ } { ( )}




f (x) is S↓ in the intervals: {x < 1} ∪ {2 < x < 3}
8. f (x) is S↑ in the interval: x > 1 or x ∈ 1 , ∞


2 2


or {x ∈ (–∞, 1)} ∪ {x ∈ (2, 3)}


Hint: f (x) = x4 – 8x3 + 22x2 – 24x + 21 and f (x) is S↓ in the interval:


{ } { ( )}



⇒ f ′(x) = 4x3 – 24x2 + 44x – 24 x < 1 or x ∈ − ∞, 1




⇒ f ′(x) = 4(x – 1)(x – 2)(x – 3) 2 2





f (x) is S↑ in the interval: f ′(x) > 0 Since the behaviour of the function f (x) in the interval (– 1, 1)



and f (x) S↓ in the interval: f ′(x) < 0 is not same.




f ′(x) = 4(x – 1)(x – 2)(x – 3) = 0 f (x) neither strictly increases nor strictly decreases in the interval






(– 1, 1).
gives x = 1, 2, 3



Hint: f (x) = x2 – x + 1 in (–1, 1)




⇒ f ′(x) = 2x – 1




If you put x = 0 in f ′(x) = 4(x – 1)(x – 2)(x – 3), we get a negative f (x) is S↑ in the interval: f ′(x) > 0




{ }
number. So, for x < 1, f ′(x) is negative, for 1 < x < 2, f ′(x) is
x> 1


positive, for 2 < x < 3, f ′(x) is negative and for x > 3, f ′(x) is ⇒ 2x – 1 > 0 ⇒
2







{ ( )}
positive.
(a) f (x) is S↑ in the interval: {1 < x < 2} ∪ {x > 3}
or x ∈ 1 ,∞
2




or {x ∈ (1, 2)} ∪ {x ∈ (3, ∞)} f (x) is S↓ in the interval: f ′(x) < 0




{ }
(b) f (x) is S↓ in the interval: {x < 1} or {2 < x < 3}
x< 1


⇒ 2x – 1 < 0 ⇒
or {x ∈ (–∞, 1)} ∪ {x ∈ (2, 3)} 2







7. f (x) is S↑ in the intervals: {–2 < x < 1} ∪ {x > 3}
or { ( )} x ∈ −∞, 1



or {x ∈ (–2, 1)} ∪ {x ∈ (3, ∞)} 2




f (x) is S↓ in the intervals: {x < –2} ∪ {1 < x < 3} 9. f (x) is S↑ in the intervals: {x < –1} ∪ {x > 1}




or {x ∈ (–∞, –2)} ∪ {x ∈ (1, 3)} or {x ∈ (–∞, –1)} ∪ {x ∈ (1, ∞)}




3x 4 4x3 2 36x and f (x) is S↓ in the interval:
Hint: f (x) = 10 − 5 − 3x + 5 + 11


{–1 < x < 1} or {x ∈ (–1, 1)}






3
6x − 12x − 6x + 36 2 1
⇒ f ′(x) = Hint: f (x) = x3 + 3 , x ≠ 0

x

5 5 5






pplication of Derivatives 151
A

E:\AMIT_WORKS\Exam_Guru\EG_Mathematics-12_(working_02-06-2022)\EG_Mathematics-12_working\Open_Files\Chap_6\Chap_6
\ 17-Aug-2022  Ved_Goswami   Proof-5 Reader’s Sign _______________________ Date __________


1 f (x) is S↑ in the interval f ′(x) > 0


f ′(x) = 3x2 – 3x–4 = 3  x 2 − 4 


⇒ 1 x+2 x−2 >0


 x  ⇒ ( )( )



2x 2



⇒ f ′(x) = 34  x 6 − 1
x  

⇒ {x > 2} ∪ {x < –2}





or {x ∈ (2, ∞)} ∪ {x ∈ (–∞, –2)}
= ( )(
3 x − 1 x + 1 × x2 + x + 1 x2 − x + 1
( )( ) )



4 f (x) is S↓ in the interval: f ′(x) < 0





(a) f (x) S ↑ in the interval f ′(x) > 0 1 x+2 x−2 <0
⇒ ( )( )




2x 2



3
⇒ (x – 1)(x + 1)(x2 + x + 1)(x2 – x + 1) > 0
4 ⇒ {–2 < x < 2} or {x ∈ (–2, 2)}






⇒ {x > 1} ∪ {x < – 1} 13. f (x) is S↑ in the interval:




{ } { ( )}
or {x ∈ (1, ∞)} ∪ {x ∈ (–∞, –1)}
0 < x < π or x ∈ 0, π


f (x) S ↓ in the interval f ′(x) < 0 4 4




⇒ 3(x – 1)(x + 1)(x2 + x + 1)(x2 – x + 1) < 0
f (x) is S↓ in the interval:



⇒ {–1 < x < 1}
} { ( )}
or{x ∈ (–1, 1)}
{ π < x < π or x ∈ π , π




10. f (x) is S↑ in the interval: 4 4


{ } { ( )}

x ≥ 5 or x ∈ 5 , ∞
4 4 Hint: f (x) = sin x +
π
( )
on [0, p]


4




( )
f (x) is S↓ in the intervals: π
f ′(x) = cos x +
{ } { ( )}


4


x < 5 or x ∈ − ∞, 5



( )
4 4
π
Solve: cos x + =0

Hint: f (x) = (x – 1)(x – 2) 3 4





f ′(x) = (x – 2)2(4x – 5) π 3π π , 5π
⇒ ⇒ x+ π = , ⇒ x=

2 2



4 4 4





f (x) is S↑ in the interval f ′(x) > 0


⇒ (x – 2)2(4x – 5) > 0 π π  5π > π 
Intervals: 0 < x < and < x < p Since 
4 4  4


{ ( )}

⇒ { } x> 5
4
or x ∈ 5 ,∞
4 Decision Making Table


f (x) is S↓ in the interval f ′(x) < 0 Intervals x ( )
f ′ ( x ) = cos x + π sign Result


4
⇒ { } x< 5
4
or { ( )}
x ∈ −∞, 5
4
0< x< π


30° cos (75°) + S↑
4
11. f (x) is S↑ in the interval:


{x < –1} or {x ∈ (–∞, –1)} π <x<π
90° cos (135°) – S↓
4



f (x) is S↓ in the interval:

{ } { ( )}

{x > –1} or {x ∈ (–1, ∞)} π or x ∈ 0, π
\ f (x) is S↑ in 0 < x <
4 4





12. f (x) is S↑ in the intervals:

{ } { ( )}


{x < –2} ∪ {x > 2} π < x < π or x ∈ π , π
f (x) is S↓ in
4 4


or {x ∈ (–∞, –2)} ∪ {x ∈ (2, ∞)}, x ≠ 0
14. f (x) is S↑ in the intervals:


f (x) is S↓ in the interval:


{ } {
0 < x < 3π ∪ 7π < x < 2π or }

{–2 < x < 2} or {x ∈ (–2, 2)} 4 4

1  x2 
{ ( )} { ( )}
1 2

Hint: f ′(x) = 2 − 2 = 2  2 − 2
x x  x ∈ 0, 3π ∪ x ∈ 7π , 2 π





4 4
1  x 2 − 4

⇒ f ′(x) = f (x) is S↓ in the interval:
2x 2  




} { ( )}

⇒ f ′(x) =
1 x+2 x−2
2x 2
( )( ) { 3π < x < 7π or x ∈ 3π , 7π
4 4 4 4





152 Mathe atics–12
m
E:\AMIT_WORKS\Exam_Guru\EG_Mathematics-12_(working_02-06-2022)\EG_Mathematics-12_working\Open_Files\Chap_6\Chap_6
\ 17-Aug-2022  Ved_Goswami   Proof-5 Reader’s Sign _______________________ Date __________


Hint: f (x) = sin x – cos x on [0, 2p] π π π


( )
Intervals 0 < x < , <x<



f ′(x) = cos x + sin x = 2 sin x + π 4 4 2


4



Now make the decision table and decide the intervals in which
Now solve: cos x + sin x = 0


f (x) is S↑ or S↓.



3π , 7π
⇒ tan x = –1 ⇒ x= 17. f (x) is S↑ in the interval:
4 4
} { ( )}







Intervals: 0 < x <

4
, and

4
<x<

4
, { 0 < x < π or x ∈ 0, π
2 2


7π < x < 2π Hint: f ′ ( x ) = cos x (4 − cos2 x)
4 (2 + cos x)




Decision Making Table
( )
In first quadrant 0, π , cos x, (4 – cos x) and (2 + cos x)2 are positive:

( )
2


π
Intervals x f ′ ( x ) = 2 sin x +
4
sign Result \ f ′ ( x ) > 0 in 0, π
2 ( )



0< x<

4
π
2 ( )
2 sin

4
+ S↑
{ } { ( )}
\ f ( x ) is S ↑ in 0 < x < π or x ∈ 0, π
2 2




<x<

p 2 sin ( 5π ) – S↓ 18. Hint: f ′(x) = (9x2 + 12x + 4)
4 4 4




⇒ f ′(x) = (3x + 2)2 ≥ 0



2 sin (375°)


< x < 2π 330° + S↑ \ f ′(x) is ↑ ∀ x ∈ R
4




19. Hint: f ′(x) = (3 + cos x) > 0 ∀ x ∈ R


\ f (x) is S↑ in the interval



\ f (x) is S ↑ ∀ x ∈ R


{ } { }



0 < x < 3π ∪ 7π < x < 2π 20. Hint : f ′ ( x ) = x2
4 4
(x + 1) (x + 2) 2



{ ( )} { (
x ∈ 0, 3π ∪ x ∈ 7π , 2π )} \ f ′(x) > 0 ∀ x ∈ R except x > – 1

or


4 4 \ f (x) is S ↑ ∀ x ∈ R except x > –1.





f (x) is S↓ in the interval IV. 1. f (x) is ↑ in the intervals:

{ ( )}

{ }

3π < x < 7π {0 ≤ x ≤ 1} ∪ {2 ≤ x < ∞}
or x ∈ 3π , 7π

4 4 4 4 or {x ∈ [0, 1]} ∪ {x ∈ [2, ∞)}



15. f (x) is S↑ in the intervals: f (x) is ↓ in the intervals:

{ } { }


{x ≤ 0} ∪ {1 ≤ x ≤ 2}
0 < x < π ∪ 5π < x < 2π

4 4 or {x ∈ (–∞, 0]} ∪ {x ∈ [1, 2]}


{ ( )} { ( )}

Points at which tangents are parallel to x-axis are: A(0, 0),
x ∈ 0, π ∪ x ∈ 5π , 2π

or and B(1, 1), C(2, 0)
4 4


2. f (x) is S↑ in the interval:
f (x) is S↓ in the interval:


} { ( )}
{1 < x < 3}
{

π < x < 5π or x ∈ π , 5π

4 4 4 4 or {x ∈ (1, 3)} and


f (x) is S↓ in the intervals:

Hint: See the Solution of Q.No. 14.

{x < 1} ∪ {x > 3}

16. f (x) is S↓ in the interval:

or {x ∈ (–∞, 1)} ∪ {x ∈ (3, ∞)}
{ } { ( )}


0 < x < π or x ∈ 0, π


and 3. Hint: f (x) = x3 – 3x2 + 4x

4 4


⇒ f ′(x) = 3x2 – 6x + 4 = 3[(x – 1)2 – 1] + 4




f (x) is S↑ in the interval: = 3(x – 1)2 + 1 > 0 ∀ x ∈ R

{ } { ( )}


π < x < π or x ∈ π , π \ f (x) is S ↑ ∀ x ∈ R.

4 2 4 2


4. Hint: f ′(x) = 3[x2 – 2x + 1] = 3(x – 1)2 ≥ 0




 π \ f (x) is ↑ ∀ x ∈ R.
Hint: f (x) = sin4 x + cos4 x on 0, 



 2

5. Hint: f ′(x) = 3(x2 – 4x + 4) = 3(x – 2)2 ≥ 0



⇒ f ′(x) = –sin 4x







Solve: sin 4x = 0 ⇒ 4x = 0, p, 2p \ f (x) is ↑ ∀ x ∈ R.



6. (i) Hint: f ′(x) = 3[4x2 – 12x + 9] = 3[(2x – 3)2] ≥ 0



π π

⇒ x = 0, ,



4 2 \ f (x) is ↑ ∀ x ∈ R.






pplication of Derivatives 153
A

E:\AMIT_WORKS\Exam_Guru\EG_Mathematics-12_(working_02-06-2022)\EG_Mathematics-12_working\Open_Files\Chap_6\Chap_6
\ 19-Aug-2022  Ved_Goswami   Proof-5 Reader’s Sign _______________________ Date __________


(ii) Hint: f ′(x) = 9x2 + 12x + 4 = (3x + 2)2 ≥ 0
x2




\ f (x) is ↑ ∀ x ∈ R. (iv) Hint: f ′(x) =>0

1+ x 2







7. (i) Hint: f ′(x) = 12x2 – 60x + 87 \ f (x) is S ↑ ∀ x ∈ R.






( )



2
= 12 x − 5 + 12 > 0 x2 + 2
2 (v) Hint: f ′(x) = >0
x2 + 1






\ f (x) is S ↑ ∀ x ∈ R. \ f (x) is S ↑ ∀ x ∈ R.



( )





(ii) Hint: f ′(x) = 3x2 – 12x + 15 = 3(x – 2)2 + 3 > 0 8. (i) Hint: f ′(x) = 1 + cos x = 2 cos 2 x ≥ 0







2




\ f (x) is S ↑ ∀ x ∈ R.

\ f (x) is ↑ ∀ x ∈ R.



1



(iii) Hint: f ′(x) =
x
>0 Q x>0 (ii) Hint: f ′(x) = 1 – cos x = 2 sin 2 x ≥ 0








2



\ f (x) is S ↑ ∀ x ∈ R. \ f (x) is ↑ ∀ x ∈ R.







Topic 3. Local Maximum and Local Minimum Values of a Function
Take a function y = x3 – 3x and draw its graph. y = x3 – 3x at points A and B are zero. So to get coordinates of
In the graph, observe the point A(x1, y1) we find that at the point A and B, solve:
point A, the function y = x3 – 3x has the maximum value in its dy
= 3x2 – 3 = 0 ⇒ 3(x + 1)(x – 1) = 0 ⇒ x = –1, 1
dx



neighbourhood. If we move to left or right side of the point A
on the graph, the value of the function reduces. Therefore the Now we want to decide which value of x; – 1 or 1, maximises
3 or minimises the value of the function y = x3 –3x. For this we use
function y = x – 3x has the maximum value at A and this value
the second derivative test.
of the function is called local maximum value of the function
2
y = x3 – 3x and value of x1 at A is called point of local maxima. So, find d y2 = 6x.
dx
d 2y
Now put the values of x in = 6x, which we have got
dx 2
dy
after solving = 0 (x = – 1, 1).
dx
d 2y
The value of x out of x = – 1, 1, makes negative will
dx 2
3
maximise the value of y = x – 3x and value of x out of x = – 1,
d 2y
1 makes positive, will minimise the value of the function
dx 2
y = x3 – 3x.
At point A(x1, y1), x1 is the point of local maxima and y1 is
d 2y 
the local maximum value of the function y = x3 – 3x.  = 6x ]x = −1 = −6 ( negative )
dx 2  x =1


Now observe the point B on the graph of the function y =
x = – 1 is the point of local maxima and y] x = –1
x3 – 3x. If we move to the left or right side of the point B on the

3
= [x – 3x]x = –1 = 2 is the local maximum value of the given
graph, the value of the graph increases, therefore the value of the
function.
function y = x3 – 3x at point B is minimum in its neighbourhood
d 2y 
and hence this value of the function is called local minimum  = [ 6x ]x =1 = 6 ( positive )
dx 2  x =1


value of the function y = x3 – 3x and value of x2 at B(x2, y2), is
x = 1 is the point of local minima and y] x = –1
called point of local minima. On point B(x2, y2), x2 is the point

= [x3 – 3x]x = 1 = –2 is the local minimum value of the given
of local minima and y2 is the local minimum value of the function function y = x3 – 3x.
y = x3 – 3x. Hence,
How to get points A and B? (i) Point of local maxima : x = – 1


3
If we observe the graph of the function y = x – 3x, the points A and B (ii) Local maximum value : y = 2


are located at such position that the tangent through them are parallel (iii) Point of local minima : x = 1


to x-axis. Hence the slopes of the tangents on the curve of the function (iv) Local minimum value : y = – 2


154 Mathe atics–12
m
E:\AMIT_WORKS\Exam_Guru\EG_Mathematics-12_(working_02-06-2022)\EG_Mathematics-12_working\Open_Files\Chap_6\Chap_6
\ 19-Aug-2022  Ved_Goswami   Proof-5 Reader’s Sign _______________________ Date __________


Local Maximum and Minimum Write these numbers as shown in the above figure.


y Now put x = – 2, 0 and 3 turn by turn in


f ′(x) = 6(x + 1)(x – 2)


Local maximum



Decision Making Table

x f ′(x) = Sign Result


6(x + 1)(x – 2)
f(x) –2 (–)(–) + When x changes from –2 to 0,
f ′(x) changes (+ to –) \ x =


–1 is the local maxima.
Local minimum 0 (+)(–) – When x changes from 0 to 3,
x f ′(x) changes (– to +) \ x = 2


is the point of local minima.
3 (+)(+) +
First Derivative Test
Local f ' (a) = 0 f ' (a–) > 0 f ' (a+) < 0 Point of local maxima : x = – 1



Maximum Local maximum value : f (– 1) = 11



Local f ' (b) = 0 f ' (b–) < 0 f ' (b+) > 0
Point of local minima : x = 2
Minimum

Saddle f ' (c) = 0 f ' (c–) > 0 and f ' (c+) > 0 Local minimum value : f (2) = – 16



Point or f ' (c) < 0 and f ' (c) < 0 Example 2. If f (x) = (x – 4)3 (x + 1)2 find a point of local maxima,


Second Derivative Test point of local minima and the point of inflexion.
Solution. f (x) = (x – 4)3 (x + 1)2
Local f ' (a) = 0 f ' ' (a) < 0



Maximum ⇒ f ′(x) = 3(x – 4)2(x + 1)2 + 2(x + 1)(x – 4)3




Local f ' (b) = 0 f " (b) > 0 ⇒ f ′(x) = (x – 4)2(x + 1)[3(x + 1) + 2(x – 4)]




Minimum
⇒ f ′(x) = 5(x – 4)2 (x + 1)(x – 1)

Saddle f ' (c) = 0 f " (c) = 0



Point To get critical points or turning points, solve

Example 1. If f (x) = 2x3 – 3x2 – 12x + 4, find the point of local 5(x – 4)2 (x + 1)(x – 1) = 0 ⇒ x = –1, 1, 4.


maxima, point of local minima, local maximum value and local
minimum value of the function. Use first derivative test. x=–1 x=1 x=4
Solution. f (x) = 2x3 – 3x2 – 12x + 4



⇒ f ′(x) = 6x2 – 6x – 12 = 6(x – 2)(x + 1) –2 0 2 5




To find turning points, solve f ′(x) = 0

Decision Making Table

\ 6(x + 1)(x – 2) = 0 ⇒ x = –1, 2


For making decision of the point of local maxima or point of x f ′(x) = 5 Sign Result


local minima, we are going to use first derivative test. (x – 4)2
f ′(x) = 6(x + 1)(x – 2) and x = –1, 2 (x + 1)

(x – 1)


x=–1 x=2 –2 (+)(–)(–) + (Changes + ⇒ x = –1 is
to –) the point of
–2 0 3 L. maxima.
0 (+)(+)(–) – (Changes – ⇒ x = 1 is
We have two points x = – 1, 2. First take x = – 1 and take a to +) the point of
number just less than – 1. (We have taken it x = – 2). Also take L. minima.
a number just greater than – 1 but less than 2. (We have taken 2 (+)(+)(+) + (No change ⇒ x = 4 is
it x = 0).
in sign) the point of
Now take x = 2 and choose a number just less than 2 but inflexion.
greater than – 1, (We have taken it x = 0) and choose a number
5 (+)(+)(+) +
just greater than 2. (We have taken it 3).

Applic tion of Deriv tives 155


a
a

E:\AMIT_WORKS\Exam_Guru\EG_Mathematics-12_(working_02-06-2022)\EG_Mathematics-12_working\Open_Files\Chap_6\Chap_6
\ 19-Aug-2022  Ved_Goswami   Proof-5 Reader’s Sign _______________________ Date __________


y Gradient = 0
Point of horizontal inflection
Positive Negative
Positive f ' (c–) > 0 f ' (c+) > 0
Gradient Gradient Positive
– + Gradient Gradient
f ' (a ) > 0 f ' (a ) < 0
Saddle
Point
gradient = 0
gradient = 0
Local minimum
Local maximum

y = f (x)
Negative Gradient Positive Gradient

f ' (b ) < 0 f ' (b+) > 0
x
x=a x=b x=c

Local Maximum and Local Minimum Value of Functions Not Having Local Maximum and Local
Modulus Function Minimum Values
Example 1. Find the local maximum or local minimum value, if
Example 1. Show that the following functions do not have local
any, of the following functions:
maxima or local minima points and values.
(i) f (x) = | x – 3 | + 5 (ii) f (x) = 7 – | x + 4 |










Solution. (i) f (x) = | x – 3 | + 5 (i) f (x) = ex






f (x) = – (x – 3) + 5 when x < 3 (ii) f (x) = x3 + 18x2 + 144x + 5







⇒ f (x) = – x + 8 when x < 3
Solution. (i) f (x) = ex






⇒ f ′(x) = –1 < 0 when x < 3




Now f (x) = x + 2 when x > 3 ⇒ f ′x) = 1 > 0 ⇒ f ′(x) = ex. To find points of local maxima or local minima,








when x > 3 solve: f ′(x) = 0



Therefore, sign of f ′(x) changes negative to positive as x ⇒ ex = 0. No solution.

changes from x < 3 to x > 3.



\ x = 3 is the point of local minima. \ Here f (x) = ex is a differentiable function and f ′(x) = 0 has




Thus, local minimum value: f (3) = | 3 – 3 | + 5 = 5 no solution.




Hence, there is no local maxima. \ f (x) = ex has no local maximum or local minimum values.



(ii) f (x) = 7 – | x + 4 |
(ii) f (x) = x3 + 18x2 + 144x + 5







f (x) = 7 – [– (x + 4)] for x + 4 < 0





f ′(x) = 3x2 + 36x + 144 = 3[x2 + 12x + 48]





⇒ f (x) = x + 11 for x < –4







⇒ f ′(x) = 1 > 0 for x < – 4 To find the points of local maxima or local minima,






Now f (x) = 7 – (x + 4) for x + 4 > 0 solve f ′(x) = 0.






⇒ f (x) = 3 – x for x > – 4 \ 3(x2 + 12x + 48) = 0






⇒ f ′(x) = –1 < 0 for x > – 4 ⇒ x2 + 12x + 48 = 0






\ f ′(x) changes sign from +ve to –ve when x changes from
Here: D = b2 – 4ac = (12)2 – 4 × 1 × 48


x < – 4 to x > – 4.





⇒ D = 144 – 192 = – 48 < 0
Thus, x = –4 is the point of local maxima.




Hence, local maximum value: f (–4) = 7 – | – 4 + 4 | = 7. \ f ′(x) = 0 has no solution.








156 Mathe atics–12
m
E:\AMIT_WORKS\Exam_Guru\EG_Mathematics-12_(working_02-06-2022)\EG_Mathematics-12_working\Open_Files\Chap_6\Chap_6
\ 17-Aug-2022  Ved_Goswami   Proof-5 Reader’s Sign _______________________ Date __________


EXERCISE 6.3
I. Multiple Choice Questions (MCQs) 6. Find the point of local maxima, point of local minima, local



Choose the correct answer from the given options. maximum value and local minimum value of the function:
1 f (x) = sin x + cos x in (0, 2p)


1. The maximum value of [ x ( x − 1) + 1 ]3 , 0 ≤ x ≤ 1 is 7. Find the point of local maxima, point of local minima, local




1 maximum value and local minimum value of the function:
 1 3 1 f (x) = sin x – cos x in (0, 2p)
(a)   (b) (c) 1 (d) 0


3 2








8. Find the point of local maxima; point of local minima, local



1 − x + x2 maximum value and local minimum value of the function:
2. For all real values of x, the minimum value of is
f (x) = 2 cos 2x – cos 4x in (0, p)
( )
2


1 + x + x2


1 9. Find the point of local maxima, point of local minima, local



(a) 0 (b) 1 (c) 3 (d) maximum value and local minimum value of the function:
3








f (x) = – x + 2 sin x in (0, 2p)


log x
3. The maximum value of is 10. Find the point of local maxima, point of local minima, local
x




maximum value and local minimum value of the function:
2 1
(a) 1 (c) e (b) (d) π
e e f ( x ) = sin 4 x + cos 4 x on 0,  .








 2 

   
II. Short Answer Type Questions-II
1. It is given that at x = 1, function f (x) = x 4 – 62x 2
Local Maximum and Local Minimum Value of
Modulus Function


+ ax + 9 attains the maximum value at the interval
[0, 2], find the value of a. IV. Very Short Answer Type Questions
2. The function f (x) = a log x + bx2 + x has extreme value at 1. Find the local minimum value of the function: f (x) = 3 +





| x |, x ∈ R.


x = 1 and x = 2, find the values of a and b. [Delhi 2009]  




2. Find the local maximum or local minimum value of the
III. Long Answer Type Questions


function: f (x) = 5 – | 2x – 7 |, if any.
1. If f (x) = x4 – 62 x2 + 120x + 9, find the points of local





maxima, local minima, local maximum value and local Functions Not Having Local Maximum and Local
minimum value of the function. Minimum Values
x 2
2. If f ( x ) = + , x ≠ 0 , find the point of local maxima, Prove that the following functions do not have local maxima or
2 x local minima.


point of local minima, local maximum value and local
minimum value of f (x). 3. g(x) = log x 4. f (x) = 4x + 5






5. f (x) = 3 – 5x 6. f (x) = x3 + 3x + 5
3. If f ( x ) = − 3 x 4 − 8x3 − 45 x 2 + 105 , find the point of local






4 2 7. f (x) = e1/x 8. f (x) = (x + 1)2/3




maxima, point of local minima, local maximum value and




V. Short Answer Type Questions
local minimum value of the function.
Prove that the following functions do not have local maxima or
4. If f (x) = (x – 1)(x + 2)2, find point of local maxima, point local minima.



of local minima, local maximum value and local minimum
1. h(x) = x3 + x2 + x + 1 2. f (x) = x3 – 6x2 + 15x – 8
value of the function f (x). Use first derivative test.





5. If f (x) = (x – 3)3, find the point of local maxima, point of 3. f (x) = 4x3 – 18x2 + 30x + 5 4. f (x) = x3 + 18x2 + 144x + 5








local minima and point of inflexion. 5. f (x) = x3 – 6x2 + 15x + 3



Answers 6.3
1 \ The maximum value of f is 1.


I. 1. (c) y = [ x ( x − 1) +1]3 Hence (c) is the correct answer.



1
dy 1 − x + x2
= 1 [ x ( x − 1) +1]3 × d ( x 2 − x +1)
−1
\ 2. (d) Let y =
dx 3 dx 1 + x + x2







2
1 − 2x −1
= [ x ( x − 1) + 1] 3 × (2 x − 1) = dy (1 + x + x 2 )(−1 + 2 x) − (1 − x + x 2 )(1 + 2 x)
3 2 =



dx (1 + x + x 2 ) 2


3[ x( x − 1) + 1] 3
dy 1
Now, for max. or min., = 0 ⇒ 2 x − 1= 0 ⇒ x = dy 2 ( x 2 −1)
dx 2 ⇒ =

dx (1 + x + x 2 ) 2



1
 3 3
Now f (0) = 1, f  1  =   , f (1) = 1 For max. or min.,
dy
=0
2 4

dx

pplication of Derivatives 157
A

E:\AMIT_WORKS\Exam_Guru\EG_Mathematics-12_(working_02-06-2022)\EG_Mathematics-12_working\Open_Files\Chap_6\Chap_6
\ 17-Aug-2022  Ved_Goswami   Proof-5 Reader’s Sign _______________________ Date __________


Local Max. Value = f (x = 1) = 68
2 ( x 2 −1)
= 0 ⇒ x2 − 1 = 0 ⇒ x = ± 1


⇒ Point of Local Min : x = 5 and
2 2
(1 + x + x )

   

Local Min. Value = f (x = 5) = – 316
dy


When x is slightly < 1, < 0 and when x is slightly > 1, 2. Point of Local Max. : x = – 2 and
dx





dy Local maximum Value = f (– 2) = – 2
> 0.


dx Point of local minima : x = 2


dy Local Min. Value = f (x = 2) = 2
\ changes sign from – ve to + ve.


dx 3. Point of Local Max. : x = – 5 and




Hence, y is minimum at x = 1. 295
Local Max. Value = f (x = – 5) =


4
1 − x + x2


1 − 1 + (1) 2 1
Minimum value of 2 = = Point of Local Min. : x = – 3 and
1+ x + x 1 + 1 + (1) 2 3


231
Hence, (d) is the correct answer. Local Min. Value = f (x = – 3) = .
4



log x Point of Local Max. : x = 0 and
3. (d) f (x) =


x Local Max. Value = f (x = 0) = 105.





1 4. Point of Local Max. : x = – 2 and
× x − (log x) × 1



1 − log x Local Max. Value = f (x = – 2) = 0
\ f ′(x) = x =


x2 x2



Point of Local Min. : x = 0 and


For max. or min., f ′(x) = 0 Local Min. Value = f (x = 0) = – 4




1 − log x 5. No point of local max. and No point of local min.


\ = 0 ⇒ 1 – log x = 0 There is only point of inflexion at x = 3.
x2






⇒ loge x = 1 = loge e ⇒ x = e ∈ (0, ∞) π
6. Point of Local Max. : x = and
4







()
1 2
x  −  − (1 − log x ) 2 x Local Max. Value = f π = 2.
 x 4
f ′′(x) =

x4 5π


Point of Local Min. : x = and
− 3 x + 2 x log x − 3 + 2 log x 4

( )
= =
x4 x3 Local Min. Value = f 5π = − 2 .


4

−3 + 2 log e −3 + 2(1) − 1
Now, = f ′′(e) =
= 3 <0 7. Point of Local Max. : x = 3π and
e3 e3 e



4


Thus, f (x) is maximum at x = e and maximum value
Local Max. Value = f ( )3 π = 2

log e 1 4

= =
e e Point of Local Min. : x = 7π and


4

Hence, (d) is correct answer.
( )
7 π

II. 1. a = 120 Local Min. Value = f =− 2
4


Hint: f (x) = x4 – 62x2 + ax + 9 π

8. Point of Local Max. : x = and



⇒ f ′(x) = 4x3 – 124x + a 6


()




f (x) maximises at x = 1 Local Max. Value = f π = 3


6 2

\ f ′(x = 1) = 0

Point of Local Min. : x = 0 and



⇒ 4 – 124 + a = 0 ⇒ a = 120

Local Min. Value f (0) = 1





− 2 − 1

2. a = ;b = .

3 6 5π


Point of Local Max. : x = and
Hint: f (x) = a log x + bx2 + x 6


( )



5π = 3
⇒ f ′(x) = a + 2bx + 1 Local Max. Value = f

x 6 2




f(x) maximises at x = 1 and x = 2 π
9. Point of Local Max. : x = and

\ f ′(1) = 0 and f ′(2) = 0 3


()( )





π = 3−π
⇒ a + 2b = –1 and a + 4b = −1 Local Max. Value = f
3 3
2




Solve for a and b. 5π
Point of Local Min. : x = and

III. 1. Point of Local Min. : x = – 6 and 3

Local Min. Value = f (x = – 6) = – 1647
( )
3

 3

Local Min. Value = f 5π = −  5π + 3 
2 


Point of Local Max. : x = 1 and

158 Mathe atics–12
m
E:\AMIT_WORKS\Exam_Guru\EG_Mathematics-12_(working_02-06-2022)\EG_Mathematics-12_working\Open_Files\Chap_6\Chap_6
\ 17-Aug-2022  Ved_Goswami   Proof-5 Reader’s Sign _______________________ Date __________


10. Point of Local Max. : x = 0 and
⇒ f ′(x) = –2 < 0 if x > 7



2



Local Max. Value = f (0) = 1
Hence, sign of f ′(x) changes from (+) to (–) as x changes from



π
Point of Local Min. : x =
4
and x < 7 to x > 7
2 2

Local Min. Value = f
π =1
4 2 () 7
\ x = is the point of local maxima and
2



() ( )
π
Point of Local Max : x = and local maximum value: f 7 = 5 − 2 × 7 − 7 = 5
2

2 2


Local Max. Value = f π = 1.
2 () 1
3. Hint: f ′ ( x ) = . Here f (x) is differentiable and


x



IV. 1. Local minimum value = f (0) = 3. f ′(x) ≠ 0.




Hint: f (x) = 3 + | x |, ⇒ f (x) = 3 –x for x < 0 \ f (x) = log x has no local maxima or local minima.








⇒ f ′(x) = –1 < 0 for x < 0 and f (x) = 3 + x for x > 0



4. Hint: f ′(x) = 4 ≠ 0. Hence f (x) is differentiable but f ′(x) ≠ 0.







⇒ f ′(x) = 1 > 0 for x > 0. So sign of f ′(x) changes from (–) to





5. Hint: f ′(x) = –5 ≠ 0.


(+) when x changes from x < 0 to x > 0.






\ x = 0 is the point of local minima and local minimum value: 6. f ′(x) = 3x2 + 3 ⇒ (x2 + 1) = 0 has no solution.








f (0) = 3 + | x | = 3 + 0 = 3. 1/ x
7. f ′ ( x ) = −e 2 = 0 has no solution.



   
2. (i) Point of local maxima: x = 7 x



2



(ii) No point of local minima. 8. Hint: f ′ ( x ) = 2 ( x + 1)−1/ 3 = 2
2 =0
has no solution.
3
()


3( x + 1) 3


(iii) Local maximum value: f 7 = 5
2 V. 1. Hint: f ′(x) = 3x2 + 2x + 1 = 0 has no solution because




(iv) No local minimum value. D = b2 – 4ac < 0.


Hint: f (x) = 5 – [– (2x –7)] if x < 7 Hence f (x) is differentiable and f ′(x) = 0 has no solution.


2



⇒ f (x) = 5 + (2x – 7) if x < 7 2. Hint: f ′(x) = 3x2 – 12x + 15 = 3(x2 – 4x – 5) = 0 has no solution





2 because D < 0. Hence f (x) is differentiable but f ′(x) = 0 has no




⇒ f (x) = 2x – 2 if x < 7 solution.
2



3. f ′(x) = 12x2 – 36x + 30 = 6(2x2 – 6x + 5) = 0 has no solution

7


⇒ f ′(x) = 2 > 0 if x < because D < 0.

2


4. f ′(x) = 3(x2 + 12x + 48) = 0 has no solution.

f (x) = 5 – (2x – 7) if x > 7 = 12 – 2x if x > 7


2 2 5. f ′(x) = 3(x2 – 4x + 5) = 0 has no solution.







Topic 4. Absolute Maximum and Absolute Minimum Values of a Function
Absolute maximum and absolute minimum values of a function: of the function on f (x) from A to G (including A and G) will be
y = f (x) on [a, b] are the maximum and minimum values of the the absolute maximum value of f (x) and any minimum value of

given function within the values x = a and x = b, including x = a the function f (x) from A to G (including A and G) will be absolute


and x = b. minimum value.
Note: Sometimes, absolute maximum value and the local
maximum value coincide.
This discussion shows that the absolute maximum and
absolute minimum values depend upto some extent on the values
of a and b.
If the values of a and b are changed then values of absolute
maxima and absolute minima may also change.

How to work out Absolute Maximum and Absolute


Minimum Values of a Function.
(i) Take the given function: y = f (x) on [a, b].


To understand the concept of absolute maximum and absolute dy
minimum, draw the graph of y = f (x) and x = a and x = b. (ii) Work out = f ′ ( x ) and solve f ′(x) = 0. Assume that it

dx


Now take the whole curve ABCDEFG. Any maximum value gives x = c, d.

pplication of Derivatives 159


A

E:\AMIT_WORKS\Exam_Guru\EG_Mathematics-12_(working_02-06-2022)\EG_Mathematics-12_working\Open_Files\Chap_6\Chap_6
\ 17-Aug-2022  Ved_Goswami   Proof-5 Reader’s Sign _______________________ Date __________


(iii) Out of c and d choose those values which lie between a To find the critical points, solve:



and b. Assume that only c lies between a and b. f ′(x) = 0 \ (x – 1)(x – 5) = 0



Take x = a, b, c




⇒ x = 1, 5 out of these two values, only x = 1 lies



(iv) Work out f (a), f (b) and f (c).




between 0 and 4.


(v) Out of these three values, choose the maximum value. That
\ Take x = 0, 1, 4 and work out f (0), f (1) and f (4).


gives absolute maximum value. Now choose the minimum






value out of the above three given values. This value is
f (0) = 8; f (1) = 31 and f ( 4) = 4


absolute minimum value. 3 3



Example 1. If f ( x ) = 1 x3 − 3x 2 + 5x + 8 on [0, 4], find the 4
3 Here is the minimum value and 31 is the maximum value.
3 3


point of absolute maxima, the point of absolute minima, absolute
maximum value and absolute minimum value of the above 4
x = 4 is the point of absolute minima and f (4) = is the
function. 3



absolute minimum value of the function.
Solution. f (x) = 1 x3 − 3x 2 + 5x + 8 on [0, 4]
x = 1 is the point of absolute maxima and f (1) = 31 is the

3


3


⇒ f ′(x) = x2 – 6x + 5 = (x – 1)(x – 5) absolute maximum value.




EXERCISE 6.4
I. Short Answer Type Questions-II 2. Find the absolute maximum and the absolute minimum



1. Find the absolute maximum and the absolute minimum values of the function:


values of the function: 4 1
f ( x) = x − 4 on [4, 29]. f ( x ) = 12x 3 − 6x 3 on [ −1, 1]


2. Find the absolute maximum and the absolute minimum 3. Find the absolute maximum and the absolute minimum




value of the function: value of the function:
f ( x ) = x 32 − x 2 on [ −4, 4]. f (x) = sin x + cos x on [0, p].


4. Find the absolute maximum and absolute minimum value

II. Long Answer Type Questions


1. Find absolute maximum and the absolute minimum value of the function:


of the function. π
f ( x ) = sin 4 x + cos 4 x on 0, 
f (x) = 2x3 – 15x2 + 36x + 1 on [1, 5]  2 



Answers 6.4
I. 1. Absolute maximum value = f (29) = 5.
3. Absolute maximum value = f π = 2 and ()


Absolute minimum value = f (4) = 0 4



2. Absolute maximum value = f (4) = 16 and


Absolute minimum value = f (– 4) = – 16 Absolute minimum value = f ( ) = – 1

π

II. 1. Absolute maximum value = f (5) = 56
4. Absolute maximum value = f ( 0) = f
π =1
()

Absolute minimum value = f (1) = 24. 2



2. f (1) = 18 is the absolute maximum value.
and absolute minimum value = f π = 1 ()


()
f 1 = −9 is the absolute minimum value. 4 2

8 4

WORD PROBLEMS ON MAXIMA AND MINIMA
I. Short Answer Type Questions-II 2. Show that the rectangle of maximum area that can be


1. Find two numbers whose sum is 12 and product is inscribed in a given circle of diameter d is a square.


maximum. [A.I. 2011, 2013]

2. Show that among the rectangles of given perimeter, the 3. Show that the rectangle of maximum perimeter that can


be inscribed in a given circle of diameter d is a square.


square has the maximum area. [V. Imp.] [Delhi, 2011]
4. A rectangle is inscribed in a semicircle of radius r with

II. Long Answer Type Questions


one of its side on the diameter of the semicircle. Find the
1. Show that among all the rectangles with given area square dimensions of the rectangle so that its area is maximum.


has the least perimeter. [Delhi 2011] Find the maximum area also.

160 Mathe atics–12
m
E:\AMIT_WORKS\Exam_Guru\EG_Mathematics-12_(working_02-06-2022)\EG_Mathematics-12_working\Open_Files\Chap_6\Chap_6
\ 17-Aug-2022  Ved_Goswami   Proof-5 Reader’s Sign _______________________ Date __________


5. Find the dimensions of a rectangle of maximum perimeter is 16 metres, find the dimensions of the window so that


that can be inscribed in a semicircle of radius r. Find the maximum amount of light may enter in the room.
maximum perimeter also. 20. The sum of the surface areas of a rectangular box



6. If the length of three sides of a trapezium other than the with sides 2x cm, x cm and x cm and a sphere of


base are equal to 10 cm, then find the maximum area of 2
the trapezium. [A.I. 2014 (C), Delhi 2013 (C)] radius r cm is given to be constant. Prove that the
sum of their volumes is minimum if 3x = 7r. Also

7. Prove that the area of a right angled triangle of given find the minimum value of the sum of the volumes.


hypotenuse is maximum when the triangle is isosceles.
21. The sum of the surfaces of a sphere and a cube is given.
[Delhi 2012 (C)]



Show that when the sum of the two volumes of the cube

8. Prove that the perimeter of a right angled triangle of given and sphere is least, the diameter of the sphere is equal to


hypotenuse is maximum when the triangle is isosceles. edge of the cube.
[V. Imp.] 22. An open rectangular box with a square base and given



9. Two sides of a triangle have lengths a cm and b cm and surface area is made in such a way that its volume is


angles between them is . What value of will maximise maximum. Show that its height will be half of its width.
θ
θ
the area of the triangle? Find the maximum area of the 23. An open tank with square base and vertical sides is to
triangle.



be constructed from a metal sheet so as to hold a given
10. If the sum of a side and the hypotenuse of a right angled quantity of water. Show that material will be least when


triangle be given. Show that the area of the triangle will the depth of the tank is half of its width.
be maximum when the angle between the given side and 24. 48 cm2 metal sheet is to be used in construction of an open



hypotenuse is 60°. [A.I. 2014, Foreign 2005] rectangular tank with square base, find the dimensions of

11. The perimeter of a triangle is 8 cm. If one of the sides is 3 the tank so that the tank has the maximum capacity.


cm, what are the other two sides for maximum area of the 25. An open rectangular tank with square base and capacity 108


triangle? cm3 is to be made out of a metal sheet. Find the dimensions
12. Let AP and BQ be two vertical poles at points A of the tank, so that minimum sheet is used.


and B respectively. If AP = 16 m, BQ = 22 m and 26. An open rectangular box with square base is to be made


AB = 20 m, then find the distance of a point R on AB from out of a given quantity of sheet of area a2. Show that the
the point A such that (RP)2 + (RQ)2 is minimum. a3
maximum volume of the box will be .
13. A wire of length l is cut into two pieces. One of the pieces 6 3


is to be bent into a square and the other into a circle. What [A.I. 2012 (C)]

should be the length of the two pieces, so that the combined 27. 3
A square tank of capacity 250 m has to be dug out. The


area of the square and the circle is minimum? cost of the land is ` 50/m2. The cost of digging increases
14. A wire of length 28 cm is to be cut into two pieces. One of with depth and digging for the whole tank is 400 (depth)2


the pieces is to be bent into a square and other pieces into rupees. Find the dimensions of the tank for least cost.
a circle. What should be the lengths of two pieces so that 28. A tank with rectangular base and rectangular sides open


the combined areas of the square and circle is minimum? at the top is to be constructed so that its depth is 2m and
15. A wire of length 36 cm is cut into two pieces. One piece volume is 8m3. If building of the tank cost ` 70/m2 for the
base and ` 45/m2 for the sides. What is the cost of least


is bent into a square and other piece into an equilateral
triangle. Find the lengths of each piece so that the sum of expensive tank? [Delhi 2015 (C)]

the areas of the square and triangle is minimum. 29. Prove that surface area of a solid cuboid with square base


16. A wire of length l cm is cut into two pieces. One piece is and given volume is minimum, when it is a cube.


bent into a circle and other piece is bent into an equilateral [Delhi 2015]

triangle. Find the lengths of the two pieces when sum of 30. A rectangular closed box with square base and given


the two areas of two figures is minimum. surface area is made in such a way that its volume is
17. Sum of the perimeters of a circle and a square is K, where maximum. Show that it will be a cube.


K is some constant. Prove that the sum of their areas is 31. A closed rectangular box with square base and given


least when the side of the square is double of the radius of volume 343 cm3 is made in such a way that its surface
the circle. [Delhi 2012, 2014 (C)] area is minimum. Find the dimensions of the box.

18. A window is in the form of a rectangle surmounted by a 32. A closed rectangular box with square base and given




semicircular opening. The total perimeter of the window surface area 726 cm2 is made in such a way that its volume
is P metres. Find the dimensions of the window when the is maximum. Find the dimension of the box.
window admits maximum light through the whole opening. 33. A metal square piece of tin of given side l cm, is to be made


19. A rectangular window is surmounted by an equilateral into a box without a top, by cutting equal square pieces


triangle. Given that the perimeter of the whole window from each corner and folding up the flaps to form an open

pplication of Derivatives 161


A

E:\AMIT_WORKS\Exam_Guru\EG_Mathematics-12_(working_02-06-2022)\EG_Mathematics-12_working\Open_Files\Chap_6\Chap_6
\ 17-Aug-2022  Ved_Goswami   Proof-5 Reader’s Sign _______________________ Date __________


rectangular box with square base. Prove that metal square i.e., with rectangular base and semi-circular ends. Show
pieces will be one-sixth of the side of the metal square that the ratio of the length of the cylinder to the diameter of
piece, when the volume of the box is maximum. its semi-circular ends is : ( + 2), when the total surface

π
π
34. A rectangular sheet of metal of length ‘a’ units and breadth area is minimum. [HOTS]




‘b’ units is to be made into a box without a top by cutting 47. Show that the semi-vertical angle of a cone of maximum
( 2 ).



equal square pieces from each corner and folding up the volume and given slant height is tan −1
flaps to form an open box. Show that when the volume of
the box is maximum, the depth of the box will be equal to [V. Imp.] [A.I. 2011, Delhi 2014]
48. Show that the right circular cone of least curved surface
(a + b) − a 2 + b 2 − ab



. [HOTS] area and given volume has an altitude equal to 2 times
6 
the radius of the base and semi-vertical angle is equal to
35. A square piece of tin of side 18 cm is to be made into a
 1 


box without a top, by cutting equal square pieces from tan −1  .
 2 
each corner and folding up the flaps to form the box, what
should be the size of the square to be cut off so that the 49. Show that the semi-vertical angle of a right circular cone



volume of the box is maximum. of given total surface area and the maximum volume is
36. A rectangular sheet of tin 45 cm long and 24 cm broad is sin −1 .
1
3() [HOTS]




to be made into a box without a top by cutting off equal
square pieces from each corner and folding up the flaps. 50. Show that semi-vertical angle of a cone of given curved



What should be the side of each square to be removed from −1  1 
surface area and maximum volume is sin   .
each corner of the sheet so that the volume of the box is  3
maximum. 51. Show that the height of a right circular cylinder of


37. An open topped box is to be constructed by removing equal maximum volume that can be inscribed in a given right
circular cone of height H is H .


square pieces from each corner of a 3 metres by 8 metres [V. Imp.]
rectangular sheet of aluminium and folding of the flaps. 3


Find the volume of largest box. 52. Show that the volume of the greatest cylinder which can


be inscribed in a cone of height H and semi-vertical angle
38. Show that the height of an open cylindrical can of given
4πΗ 3 tan 2 α


surface area and greatest volume is equal to radius of the α is .
base. 27
39. Show that the height of an open cylindrical can of given 53. Prove that the radius of a right circular cylinder of




volume and least surface area is equal to the radius of the maximum curved surface area which can be inscribed in
base. [Delhi 2011 (C)] a given cone is half of the radius of the cone.

40. Show that a closed right circular cylinder of given surface [V. Imp.] [A.I. 2012, 2013 (C)]



area and the maximum volume is such that its height is 54. Prove that the volume of the largest cone that can be inscribed


equal to diameter of the base. [Delhi 2012]
in a sphere of radius R is 8 of the volume of the sphere.

41. Show that a closed right circular cylinder of given volume 27
[V. Imp.], [A.I. 2013 (C), 2014]


is made in such a way that its surface area is minimum.

Show that its height will be equal to diameter of the base. 55. Show that the altitude of the right circular cone of


42. A closed circular cylinder has volume 2156 cm3. What will maximum volume that can be inscribed in a sphere of


be the radius of its base and height so that its total surface radius R is 4R . [Delhi 2013 (C)]
22  3


area is minimum. Take π = 7  . [Delhi 2014] 56. Show that the height of a right circular cylinder of
 



43. A closed circular cylinder has surface area 924 cm2, what maximum volume that can be inscribed in a sphere of
2r


will be its radius and height if its volume is maximum? radius r is . Also find the maximum volume.
 22  3
Find the maximum volume also.  Take π = 7  . [Delhi 2012 (C), 2013, A.I. 2012 (C), 2014]
 
44. A closed right circular cylinder of maximum volume is to 57. Show that volume of the greatest cylinder which can be




be made so that sum of its radius and height is equal to K, inscribed in a given cone of height H and semi-vertical
a constant. Prove that the ratio of its radius and height is 3
angle 30° is 4πH .
2 : 1. Find the maximum volume also. 81
45. A closed circular cylinder of maximum volume is made in 58. A cylinder is inscribed in a sphere of radius a,




such a way that the sum of its circumference and height is show that, if the volume of the cylinder is V, then
equal to K, a constant. What will be the ratio of its height V = 2πr 2 a 2 − r 2 , where r is the radius of the base of the
and radius? Find the maximum volume also. cylinder. Hence find the height of the cylinder when V is
46. A given quantity of metal is to be cast into half cylinder maximum and also find the maximum volume.


162 Mathe atics–12
m
E:\AMIT_WORKS\Exam_Guru\EG_Mathematics-12_(working_02-06-2022)\EG_Mathematics-12_working\Open_Files\Chap_6\Chap_6
\ 17-Aug-2022  Ved_Goswami   Proof-5 Reader’s Sign _______________________ Date __________


59. Find the maximum area of an isosceles triangle volumes is minimum if x is equal to three times the radius


2
y2 of the sphere. [A.I. 2015 (C)]
inscribed in the ellipse x 2 + 2 = 1 with the vertex at


a b 64. A manufacturer can sell x items at a price of ` (250 – x)



one end of the major axis. [HOTS] each. The cost of producing x items is ` (2x2 – 50x + 12).
Determine the number of items that can be sold so that he


60. Find the area of the greatest rectangle that can be inscribed
can make maximum profit. Find maximum profit also.


y2
( )
2
is an ellipse x 2 + 2 = 1 . [A.I. 2013]
a b 65. A manufacturer can sell x items at a price of ` 5 − x


( )
100



61. AB is the diameter of a circle and C is any point on the x
each. The cost price of x items is ` + 500 .
5


circle. Show that area of the ABC is maximum when it


is isosceles. [A.I. 2014 (C)] Find the number of items he should sell to earn maximum



62. A point on the hypotenuse of a right triangle is at a distance profit.
66. Find a point on the parabola y2 = 2x, which is closest to a


of a and b from the sides of the triangle. Show that the



3 point (1, 4). [Delhi 2011 (C)]
 2 2 2


minimum length of the hypotenuse is  a 3 + b 3  . 67. An apache helicopter of enemy is flying on the



  curve : y = x 2 + 7. A soldier is placed at a point
[HOTS] [Delhi 2015 (C)] (3, 7), wants to shoot down the helicopter when it is nearest
to him. Find the nearest distance.

63. The sum of the surfaces of a sphere and a cuboid with
68. Find the shortest distance of the point A(0, c) from the


x
sides , x and 2x, is constant. Show that the sum of their



3 parabola y = x2 where 0 ≤ c ≤ 5.

Answers
Hint: If a rectangle is inscribed in a circle of diameter d. The
I. 1. 6, 6

diagonal of the rectangle passes through the centre of the circle.

Hint: Let the numbers be x and (12 – x).

2 2 2 2 2 2
Then their product = P = x(12 – x) = 12x – x2 Area = x d − x ⇒ (area) = A = x (d − x )


Find dP .
dx Now maximise A and prove that: l = b = d

2

Then solve: dP = 0 ⇒ x = 6, and 12 – x = 6.
dx

d 2P = −2
Find (negative)
dx 2

x = 6 maximise P.


2. l = b = P
4


Hint: Let P be the fixed perimeter and x be one side of the

P d , P = 2  x + d 2 − x2 
rectangle. Then other side will be  − x . 3. l = b =
2  
2 


d .
P
( )
If A is the area of the rectangle A = x − x .
Now maximise P. It will give x =
2

2

P
Now Maximise A and prove that: l = b =
4

II. 1. l = b = A

Hint: Rectangle: A = fixed area, x = one side.

A
Other side = .
x



P be the perimeter of the rectangle.

( )

Then P = 2 x + A . r . A = r2
4. l = 2r and b =
x 2



Now minimise P and prove that l = b = A .

l = 2x, b = r 2 − x 2 , Area = 2x r 2 − x 2
d

2. l = b =
2 A = (area)2 = 4(x2r2 – x4)



pplication of Derivatives 163
A

E:\AMIT_WORKS\Exam_Guru\EG_Mathematics-12_(working_02-06-2022)\EG_Mathematics-12_working\Open_Files\Chap_6\Chap_6
\ 17-Aug-2022  Ved_Goswami   Proof-5 Reader’s Sign _______________________ Date __________


r 9. θ = 90°
Now maximise A. It will give x = .
2




1
Hint: ∆ = ab sin θ
2



Find d ∆ and maximise .



It will give you = 90°.


θ
5. l = 4r , b = r , Pmax = 2 5r 10. θ = 60° Hint: BC = a, AB + BC = S



5 5


⇒ AB = (S – a)



Hint : Take fig. of Q. No. 4.
⇒ AC = (S − a) 2 − a 2 = S2 − 2aS

l = 2x, b = r 2 − x 2



Area of ABC

\ P = 2  2x + r 2 − x 2  Maximise.



  = 1 (BC) (AC) = a S2 − 2aS


2 2
It will give x = 2 r .


5

2
⇒ A = (area) 2 = a (S2 − 2aS)
4



6. A max = 75 3 cm 2


S
⇒ A = (Sa 2 − 2a 3)
4


Maximise A. Find

da ( S
It gives: a = .
3 )

11. a = 3 cm, b = 2.5 cm and c = 2.5 cm


Hint: Three sides of the are a, b and c.



a = 3 cm, b + c = (8 – 3) = 5 cm.
100 − x 2 cm

Hint: AB = (2x + 10) cm, DE =
Let b = x cm . Then c = (5 – x) cm



100 − x 2 10 + (2x + 10) Let S be the half perimeter of the D.
Area = A = [ ]

2

S = 4 cm
A = 100 − x 2 [ x + 10]



Area of the = S(S − a) (S − b) (S − c)




Maximise A. It will give x = 5 cm.

h ; AC = h = 4 × 1 × (4 − x) (x − 1) = 4(x − 1) (4 − x)
7. BC = x =
   
2 2


(Area)2 = A = 4(– x2 + 5x – 4). Maximise A.

x h2 − x2 It gives: x = 2.5 cm
Hint: Area =
2



2
12. AR = 10 metres
⇒ (Area) 2 = A = x (h 2 − x 2)


4


1
⇒ A = (h 2 x 2 − x 4).
4


Now maximise A.

It gives x = h .

2
h and AC = h
8. BC = x =
2 2


Hint: S = (RP)2 + (RQ)2
Hint: BC = x

⇒ S = (x2 + 256) + (20 – x)2 + 484


2 2


AC = h −x
⇒ S = 2x2 – 40x + 1140



⇒ P = h + x + h2 − x2 Minimise S. It gives x = 10 m.



Maximise P.
13. x = πl ; (l − x) = 4l

It gives x = h . π+4 π+4


2

164 Mathe atics–12
m
E:\AMIT_WORKS\Exam_Guru\EG_Mathematics-12_(working_02-06-2022)\EG_Mathematics-12_working\Open_Files\Chap_6\Chap_6
\ 17-Aug-2022  Ved_Goswami   Proof-5 Reader’s Sign _______________________ Date __________


Hint: 2
S = x + 3 (36 − x) 2


16 36


Minimise S : it will give you x = 144 3 .
9+4 3


 3πl 
16. Length of first piece = x =  cm
 3π + 9 



2πr = x ⇒ r = x Length of second piece = l − x =
 9l 
cm
2π  3π + 9 



l − x
4 (side) = (l – x) ⇒ side = 
 4  Hint:



2
S = x + 1 (l − x) 2
4π 16

Minimise S. It gives: x = πl .
π+4

14. Length of first piece = 28π cm
π+4


Length of second piece = 112 cm
π+4

2
S = x + 3 (l − x )
2
Hint: 4π 36



Maximise S and on solving dS = 0 ,
dx

 
You will get x =  3πl  cm
 3π + 9 

17. x =  K  ; r = K ∴ x = 2r
 π + 4 2 (π + 4)


2πr = x ⇒ r = x cm, side =  28 − x  cm
2π  4 


 2 
⇒ S =  x + 1 ( 28 − x )2  cm2
4 π 16


 
Now minimise S : It gives x = 28π cm
π+4 ( )

 144 3 
15. Length of first piece =
 9 +4 3  cm


Hint: 4x + 2 r = K ⇒ r = K − 4x
 324  2π


π
Length of second piece =
 9 + 4 3  cm

If length of the wire is l cm, then length of first piece = S = x 2 + 1 (K − 4x) 2


 
 9l   
and length of second piece =  4 3l  cm

 9 + 4 3  cm  9 + 4 3 dS = 0
Now solve to minimise S.
dx

Hint:


K
It gives x = K and r =
π+4 2(π + 4)

2P metres, b = y =  P  metres and
18. l = x =  π + 4 
π+4


r= x =
P metres
2 π+4

pplication of Derivatives 165


A

E:\AMIT_WORKS\Exam_Guru\EG_Mathematics-12_(working_02-06-2022)\EG_Mathematics-12_working\Open_Files\Chap_6\Chap_6
\ 17-Aug-2022  Ved_Goswami   Proof-5 Reader’s Sign _______________________ Date __________


πx 1
Hint: P = 2y + x + 2 2
2 Hint: S = 6x2 + 4pr2 ⇒ r =  S − 6x 

 4π 





⇒ y = P − x − πx 2 2
3
2 2 4 
V = x + 4 πr 3 ⇒ V = x3 + 4 π  S − 6x 


3
2 2 3 3  4π 
A = Px − x − πx


2 2 8 Solve: dV = 0 (for min. value of V)
dx
  


For Max. of A, solve
2

dA = 0. 22. Hint: S = 4xh + x2 ⇒ h = S− x
dx 4x






 S − x2 
V = x 2h = x 2 
It gives x = 2P metres  4x 
π+4

 16 
=
4
(
1 Sx x 3
− )
l=x= m,


 6 − 3  Solve:
dV = 0
19. dx
( )


8 3− 3  (for max. value of V)


b= y= m


 6 − 3  It gives S = 3x2


23. Hint: V = x 2h ⇒ h = V2
x



Hint: 2y + 3x = 16


 V
S = x2 + 4xh = x 2 + 4x  2 
⇒ y = 16 − 3x and A = xy + 3 (x) 2 x 
  
2 4


⇒ S= x +2 4 V

(
A = x 16 − 3x + 3 x 2 ) x


2 4 Now minimise S
  

dA = 0 3
For maximum value of A, solve Solve dS = 0 ⇒ V = x
dx dx 2


 16  24. x = 4 cm, h = 2 cm
It gives: x =   metres.
6− 3



2
Hint: x2 + 4xh = 48 ⇒ h =  48 − x 
20. Hint: Surface area of the rectangular box = 7x2  4x 




V = x2h


Surface area of the sphere = 4 r2
  
 2 3

π
S = 7x2 + 4 r2 = x 2  48 − x  = 48x − x
 4x  4

π
Maximise V, solve dV = 0 ⇒ x = 4 cm

   
dx

48 − x 2
and put x = 4 in h =
4x

25. x = 6 cm, h = 3 cm


Hint: x2h = 108 cm3
3


 S − 7x 2  2 ⇒ h = 108 cm
V = x3 + 4 πr 3 = x3 + 4 π  x2


3 3  4π 
S = x 2 + 4x  108 
  
Solve: dV = 0 (for min. value of V)  x 2 
dx


= x 2 + 432 Minimise S.
  
It gives S = (
343 + 36π x 2
) x
   
49

Solve: dS = 0 ⇒ x = 6 cm and h = 108
( )
= 3 cm
343 + 36π x 2 = x 2 + πr 2 ⇒ 3x = 7r dx x2

⇒ 7 4 2 2
49 26. Hint: x + 4xh = a




21. a2 − x2
⇒ h=


4x


 2 2
V = x2h = x 2  a − x 
 4x 

  
= 1  a 2 x − x3 
4
  
166 Mathe atics–12
m
E:\AMIT_WORKS\Exam_Guru\EG_Mathematics-12_(working_02-06-2022)\EG_Mathematics-12_working\Open_Files\Chap_6\Chap_6
\ 17-Aug-2022  Ved_Goswami   Proof-5 Reader’s Sign _______________________ Date __________


For maximum volume, It gives: V = x3


2 3
Solve:
dV = 0 ⇒ x = a ; V = a x − x Put it in h = V2 ⇒ h = x.
dx 4 x


3

a a3 30. Hint: S = 2x2 + 4 xh
Put x = in it ⇒ Vmax =



3 6 3 2
h = S − 2x


4x



Height of the tank = x = 5 metre,
27. 2 V = x 2h = 1 Sx − 2x3 
4


Length of the square base = 10 metre

  
Hint: Height = x metres, Volume = 250 m3 Solve dV = 0 ⇒ S = 6x 2
dx


250
base area =
x S − 2x 2


Put it in h = = x.
4x
Cost of land = ` 250 × 50 = ` 12500


x x x = 7 cm and h = 7 cm

31.
Cost of digging = ` 400x2



Hint: V = 343 cm3.

Total cost


x2h = 343

C = ` 12500 + 400x 2 


 x  ⇒ h = 343 cm

x2



Minimise C:
S = 2x2 + 4xh

Solve: dC = 0 ⇒ x = 5 m


dx 2
= 2x 2 + 1372 .

( )
250 m 2 = 100 m 2 x
  
x
Now solve dS = 0 ⇒ x = 7 cm.

⇒ Side of the square tank = 10 m. dx




28. Cost = ` 1000 32. x = 11 cm and h = 11 cm




Hint: Height = 2m,
Hint: S = 726

Volume = 8m3.

⇒ 2x2 + 4xh = 726

Length of the base



2
= x metre ⇒ h = 726 − 2x
4x

4


Breadth = metres
x

3
Four walls area V = 726x − 2x
4
( )

=4 x+ 4
  
x Solve: dV = 0
dx


Cost of base construction = ` 4 × 70 = ` 280
⇒ x = 11 cm

Cost of construction of four walls


( ) ( )

726 − 2x 2
= ` 4 x + 4 × 45 = 180 x + 4 Put it in h = ⇒ h = 11 cm.
x x 4x


( )
Total cost = C = 280 + 180 x + 4
x
l
33. x = cm



6


dC = 0
Minimise C and solve ⇒ x = 2 metres.
dx

Put x = 2 in C = 280 + 180 x + 4
x ( )

V
29. Hint: V = x 2h ⇒ h = 2
x


2
S = 2x + 4xh

= 2x 2 + 4V
x

dS
Find
dx

dS = 0 l = (l – 2x) cm, b = (l – 2x) cm,
and solve
dx




h = x cm, V = lbh


pplication of Derivatives 167
A

E:\AMIT_WORKS\Exam_Guru\EG_Mathematics-12_(working_02-06-2022)\EG_Mathematics-12_working\Open_Files\Chap_6\Chap_6
\ 17-Aug-2022  Ved_Goswami   Proof-5 Reader’s Sign _______________________ Date __________


V = (l – 2x)2x
( )

37. Vmax = 200 m3



⇒ V = 4x3 – 4lx2 + l2x 27






dV = 0 ⇒ x = l and l
Solve
dx 2 6


34.


Hint: l = 8 – 2x = 2(4 – x) cm, b = (3 – 2x) cm





h = x cm, V = 4x3 – 22x2 + 24x




dV dV = 0 ⇒ x = 3 cm, 2 cm
Find . Solve
dx dx 3


3 2 200 m3
Vmax = [4x − 22x + 24x] 2 =
x= 27



3
Hint: l = (a – 2x) cm, b = (b – 2x) cm
38. Hint: Radius = r




h = x cm, V = lbh = (a – 2x) (b – 2x) x



Height = h




⇒ V = 4x3 – 2(a + b)x2 + abx S = pr2 + 2prh



  
2
2
dV = 0 ⇒ x = (a + b) ± (a + b) − 3ab ⇒ h = S − πr
Solve 2πr


dx 6


V = pr2h
  
35. x = 3 cm  2
= πr 2  S − πr  = 1 [Sr − πr 3]


Hint: l = 2(9 – x) cm, b = 2(9 – x) cm  2πr  2
   



dV dV = 0 ⇒ S = 3πr 2
Find and solve
dr dr

S − πr 2
Now find h using h =
2πr

39. Hint: Radius = r, Height = h


V = πr 2h ⇒ h = V2
πr

S = pr2 + 2prh

S = πr 2 + 2πr ×  V2 
 πr 
h = x cm, V = lbh



⇒ V = 4[x3 – 18x2 + 81x] S = πr 2 + 2V
r




dV = 0
Solve
dx
⇒ x = 9 cm and 3 cm. Now find dS and solve dS = 0 ⇒ V = πr 3
dr dr




36. x = 5 cm V
Find h using h =


πr 2

Hint: l = (45 – 2x) cm, b = (24 – 2x) cm, h = x cm
40. Hint: Height = h, Radius = r





S = 2pr2 + 2prh

2
⇒ h = S − 2πr
2πr


V = pr2h

 S − 2πr 2 
= πr 2  
V = lbh = 2[2x3 – 69x2 + 540x]  2πr 



Find dV ; Solve dV = 0 ⇒ x = 18 cm, 5 cm = 1 [Sr − 2πr 3]
dx dx 2


168 Mathe atics–12
m
E:\AMIT_WORKS\Exam_Guru\EG_Mathematics-12_(working_02-06-2022)\EG_Mathematics-12_working\Open_Files\Chap_6\Chap_6
\ 17-Aug-2022  Ved_Goswami   Proof-5 Reader’s Sign _______________________ Date __________


dV
Now find and solve dV = 0 ⇒ S = 6 r2 ⇒ r = 0; r = 2K ,
dr dr 3

π

  
Find h using h = K – r
S − 2πr 2


Find h using h =
2πr Find Vmax and show that r = 2 : 1 .

h


41. Hint: Radius = r 3
45. r = K ; h = K ; h = π and Vmax = K


Height = h 3π 3 r 1 27π




V = πr 2h ⇒ h = V2 Hint: Height = h cm
πr



Radius = r cm




Now S = 2pr2 + 2prh and h + 2 r = K



π

⇒ h= K–2 r

= 2πr 2 + 2V




π
r V = r2h = (Kr2 – 2 r3)



π
π
π
Find dV and

Solve dS = 0 ⇒ V = 2πr 3. dr


dr
Solve dV = 0

V = 2r. dr


Now find h using h =
πr 2
⇒ r = 0, K . Find h using h = K – 2pr

42. r = 7 cm, h = 14 cm 3π





Hint: Height = h cm ⇒ h = K . Find h and Vmax.
3 r





Radius = r cm 46. Hint: Let r, h, V and S be the radius, length, volume (fixed) and

V = 2156 cm3 (given)


surface area of the half cylinder.

r2h = 2156
V = 1 πr 2h ⇒ h = 2V2



π
⇒ h = 2156 2 πr
πr 2



S = πr 2 + 2rh + πrh
S = 2πr 2 + 2πrh = 2πr 2 + 2 × 2156( )

2(π + 2) V
r = πr 2 +
πr

dS dS = 0 ⇒ r = 7 cm

Now find and solve dS
dr dr Find and solve

dr

2156 , use π = 22
Find h using h = dS = 0 ⇒ V =  π 2r 3 
πr 2 7  π + 2 

dr
3
43. r = 7 cm, h = 14 cm, Vmax = 2156 cm

2V ⇒ h =  π 


Hint: Height = h cm, Radius = r cm Now find h using h =
πr 2 2r  π + 2 



S = 924 cm2 = 2 r2 + 2 rh = 924
47. Hint: Let l, x and V be the slant height, height and volume of
  
π
π
2
h = 924 − 2πr
( )


⇒ 2
1
2πr the cone. Then radius = l 2 − x 2 and V = π l 2 − x 2 x


3
V = πr h = 1 [924r − 2πr 3]
2
2 = 1 π (l 2 x − x3)
3


Find dV and

dr Find dV and solve

dx

dV = 0 ⇒ r = 7 cm
Solve dV = 0 ⇒ = l
dr x

dx 3
924 − 2πr 2
, use π = 22

Find h using h = 2πr 7 If is the semi-vertical angle, then


θ
3 l 2 − x2
44. r = 2K ; h = K ; Vmax = 4πK ; r = 2 tan
x
== 2
3 3 27 h 1

θ


Hint: Height = h cm, 48. Let h, x and l be the height, radius and slant height of the cone,
so that l2 = h2 + x2.



Radius = r cm
Let V be the fixed volume.


r+h= K⇒h=K–r

V = 1 πx 2h ⇒ h = 3V2


V = r2h = (r2K – r3)
3 πx


π
π
Find dV and

dr (Curved surface area)2


2
dV = 0
solve S = (πxl) 2 =  πx h 2 + x 2 
dr  


pplication of Derivatives 169
A

E:\AMIT_WORKS\Exam_Guru\EG_Mathematics-12_(working_02-06-2022)\EG_Mathematics-12_working\Open_Files\Chap_6\Chap_6
\ 17-Aug-2022  Ved_Goswami   Proof-5 Reader’s Sign _______________________ Date __________


2 AM = ΜC
⇒ S = π 2 x 4 + 9V2 ⇒
EF FC
x





dS dS = 0 ⇒ 3V = 2 πx3 ⇒ H R
Find and solved =
h R−x



dx dx

3 ⇒ h = H (R − x)
Now h = 3V2 = 2π2x = 2 x;tan θ = x = 1 R



πx πx h 2 Let V be the volume of the cylinder.


49. Hint: Let h, r and l be the height, radius and slant height of the
cone, such that l2 = h2 + r2.
\ V = πx 2h = πH Rx 2 − x3( )


R



Let S be the total surface area (fixed) dV dV = 0 ⇒ x = 2R
Find and solve

\ S = prl + pr2 = pr(l + r) dx dx 3


( ) ( )


H H 2R
⇒ l= S −r Now h = (R − x) = R− = H.
R R 3 3


πr


(Volume of the cone)2 52. Hint: Same fig. as in Q. No. 51.



2R

2 We have proved x =
= V =  1 πr 2h  = 1 π 2r 4h 2 3


 3  9 R
Also = tan α ⇒ R = H tan α.

( )
H

2 4  2 


= 1 π 2r 4(l 2 − r 2) = π r  S − r − r2 2H tan α
9 9  πr  \ x= and h = H .
3 3



Put these values in V = x2h.

⇒ π 2r 4  S2 2 2Sr 2
V=  π 2r 2 + r − πr − r 


π
9 4πH3 tan 2 α


  It gives V = .
27

⇒ V = S Sr 2 − 2π r 4  . 53. Hint: Let R and H be the fixed radius and height of the cone.
9




i.e., AM = H and MC = R.
dV = 0 ⇒ S = 4πr 2
Find dV and solve Let h and x be the height and
dr dr


radius of the cylinder i.e., DG
  = PM = EF = h and MF = x.
r  r  πr 2 DAMC ~ DEFC
sin θ = =  2  =
l S − πr 2

S − πr H= R
  ⇒
πr h R−x



Now put S = 4 r2. h = H (R − x)
R

π
50. Let h, r and l be the height, radius and slant height of the cone,
  
Let C be the curved surface
such that : l2 = h2 + r2



area of the cylinder
Let S be the fixed curved surface area.
\ C = 2πxh = 2πx H (R − x) = 2πH (Rx − x 2)

2 R R
S = πrl ⇒ l 2 = S2 2


\
πr dC dC = 0 ⇒ x = R


Find and solve
1 2 4 2 dx dx 2

V = (Volume)2 = π r h 54. Hint: Two dimensional cone ABC has been inscribed in a fixed
9
  


sphere of radius R and centre O. AO = BO = CO = R and OM = x.
2 4

 
 
⇒ V = π r (l 2 − r 2) ⇒ Radius of the variable cone = R 2 − x 2.
9




If V is the volume of the cone, then,
2 4  2 
V = π r  S2 2 − r 2 


9 π r ( )
V = 1 π R 2 − x2 (R + x)


 3

⇒ V = 1 S2r 2 − π 2r 6  .
9


dV dV = 0.
Find and solve
dr dr


S= 3πr 2
  
Now sin θ = r = r = 1
l S 3

πr
51. Hint: ABC is fixed cone in two dimensions. DEFG is the variable


cylinder inscribed in the fixed cone. Let R and H be the radius
dV = 0 ⇒ x = R .
and height of the fixed cone. AM = H and MC = R. Now find dV and solve
dx dx 3

Let h and x be the height and radius of the variable cylinder. i.e.
Now find volume of the sphere and cone and find

DG = PM = EF = h and MF = MG = x.

Volume of cone
= 8


DAMC ~ DEFC
Volume of sphere 27


170 Mathe atics–12
m
E:\AMIT_WORKS\Exam_Guru\EG_Mathematics-12_(working_02-06-2022)\EG_Mathematics-12_working\Open_Files\Chap_6\Chap_6
\ 17-Aug-2022  Ved_Goswami   Proof-5 Reader’s Sign _______________________ Date __________


55. Hint: Fig. same is in Q. No. 54.


R R 4R
Prove x = , Now h = R + x = R + = .
3 3 3

56. Hint: AC = 2r and BC = x


Radius of the cylinder

= 1 4r 2 − x 2
2
   
Volume of the cylinder

2
1 4 2
=V=π r − x 2  x.
 2  Hint: An isosceles ABC has been inscribed in an ellipse. The




   
vertices of ABC are:
⇒ V = π (4r 2 x − x3)


4 A (a, 0), B (a cos , b sin ) and



θ
θ
dV 2r C (a cos , – b sin ).
dV
and solve dx = 0 ⇒ x = .


θ
θ
Find OM = a cos and MB = b sin .
dx 3



θ
θ
MA = OA – OM = (a – a cos )





θ
3 3
= a (1 – cos ) and BC = b sin – (– b sin )
Vmax = π [4r 2 x − x3] = 4πr ⇒ Vmax = 4πr



θ

θ

θ
4 3 3 3 3 = 2b sin



θ
∆ = 1 BC × AM = ab sin θ (1 − cos θ )

57. Hint: Same as Q. No. 52. 2

    
D = ab(sin q – sin q cos q)


4πH3 tan 2 α


( )


We have proved V = 1
27 = ab sin θ − sin 2 θ


2


4πH3 d ∆ = ab [cos θ − cos 2θ]. d∆ = 0
Now put α = 30°. It gives V = ⇒ Solved:
81 dθ dθ



3 θ θ
58. Height of the cylinder = 2a ⇒ cos θ − cos 2θ = 0 ⇒ 2 sin sin = 0
2 2


3


3θ = 0 or sin θ = 0
3 ⇒ sin
Vmax of the cylinder = 4πa 2 2


3 3 ⇒ 3 θ θ
= 0, π or sin = 0, π
2 2


Hint: ABCD is a variable cylinder with radius r and height =
π , θ = 0, 2π ⇒ θ = 2π .
⇒ θ = 0, θ = 2

2 a2 − r 2 . 3 3


2
Find d ∆2 and it becomes negative for θ = 2π .
dθ 3

Max. ∆ = ab [sin θ − sin θ cos θ ]θ = 2π

3
3 3 ab
= ab sin 2π − sin 2π cos 2π  =
 3 3 3  4

60. Area of the greatest rectangle = 2ab


Here a is the radius of the sphere.

V = (volume of the cylinder)2

= 4p2r4(a2 – r2) ⇒ V = 4p2(a2r4 – r6)



dV dV = 0 2a
Now find and solve ⇒ r=
dr dr 3

Height of the cylinder = 2 a 2 − r 2

2 2
2a 2a Hint: GAEDHCFBG is the ellipse x 2 + y2 = 1 and ABCD is a
Put r = in it. It gives height = . rectangle with vertices shown in theafig. bl and b are length and


3 3

breadth of the rectangle.
2a
Vmax = 2πr 2 (a 2 − r 2). Put r = . l = DA = 2a cos , b = AB = 2b sin
3




θ
θ

A = lb = 2ab sin 2 .
4πa 3 .


θ
It gives Vmax = dA
3 3 Find and solve dA = 0

dθ dθ

3 3 ab π
59. A max = ⇒ θ = and A max = 2ab.
4 4



    
pplication of Derivatives 171
A

E:\AMIT_WORKS\Exam_Guru\EG_Mathematics-12_(working_02-06-2022)\EG_Mathematics-12_working\Open_Files\Chap_6\Chap_6
\ 17-Aug-2022  Ved_Goswami   Proof-5 Reader’s Sign _______________________ Date __________


61. Let ∠A = ; AB = 2r 2
P = −x + 24x − 500


θ
Now AC = 2r cos and BC = 2r sin ⇒
100 5




θ
θ
Area = A = r2 sin 2
Find dP and solve dP = 0 ⇒ x = 240.

θ
dA = 0
Find dA and solve dx dx


dθ dθ

⇒θ= . π 66. P = (2, 2)



4

Now find AC and BC. We get AC = BC = 2r.

62. Hint: Let AM = x, PL = a and PM = b


CPL ~ PAM



PL = CL
\
AM PM


⇒ CL = PL × PM = ab
AM x


Now AB = x + a

and BC = b + CL
Hint: PA = (x − 1) 2 + (y − 4) 2

⇒ BC = b + ab



x ⇒ D = (PA)2 = (x – 1)2 + (y – 4)2


( )


2
⇒ l = (x + a) 2 + b + ab
2 2
 y2 
D =  − 1 + ( y − 4)
2
x ⇒


 2 


1 2
dl dl
Find and solve = 0 ⇒ x = a 3b 3
dx dx dD dD = 0

Find and solve ⇒y=2
Find l using x = dy dy


3 Now find x.
( )  2
2 2 2

(x + a) + b + ab
2
=  a3 + b3 67. D = 5 units
x  



63.


1
 2 2
Hint: S = 4πr + 6x ⇒ r = S − 6x
2 2
 4π 


3
V = Vcuboid + Vsphere = 2x + 4 πr 3
3 3
PA = (x − 3) 2 + (y − 7) 2
  
dV dV = 0
Find and solve
dx dx


⇒ 9S = x2 (4p + 54). Put the value of S in D = PA2 = (x –3)2 + (y –7)2



1 = (x – 3)2 + (x2 + 7 – 7)2 = (x – 3)2 + x4
 S − 6x 2  2

dD = 0 ⇒ x = 1.
r=  ⇒ x = 3r Find dD and solve
 4π  dx dx

   
64. x = 50 and Pmax. = ` 7, 488 . Find distance when x = 1



Hint: P = x(250 – x) – (2x2 – 50x + 12) Distance = (x − 3) 2 + x 4 = 5.



⇒ P = –3x2 + 300x – 12


dP dP = 0 4c − 1
Find and solve 68. Shortest distance =
dx dx 2



⇒ x = 50. Pmax. = ` 7488. Hint: AP = (x − 0) 2 + (y − c) 2



65. x = 240


= x 2 + (x 2 − c) 2
( )
 2 
Hint: P =  5x − x  − x + 500

 100  5 (Distance)2 = D = x2 + (x2 – c)2



172 Mathe atics–12
m
E:\AMIT_WORKS\Exam_Guru\EG_Mathematics-12_(working_02-06-2022)\EG_Mathematics-12_working\Open_Files\Chap_6\Chap_6
\ 17-Aug-2022  Ved_Goswami   Proof-5 Reader’s Sign _______________________ Date __________


dD
Find and
dx

dD = 0
Solve
dx

2c − 1
⇒ x=0 or x =
2




⇒ c≥ 1
2


( )
2
2c − 1 + 2c − 1 − c
Distance =
2 2

= 4c − 1
2

Case Based Questions
1. The relation between the height of the plant (y in cm) with respect 2. P(x) = –5x2 + 125x + 37500 is the total profit function of




to exposure to sunlight is governed by the following equation a company, where x is the production of the company.
1 2
y = 4x – x where x is the number of days exposed to
2
sunlight.

(i) What will be the production when the profit is




maximum?
(i) The rate of growth of the plant with respect to sunlight
(a) 37500 (b) 12.5 (c) –12.5 (d) –37500


is ______.








1 2 (ii) What will be the maximum profit?


(a) 4 x − x (b) 4 – x (a) ` 38,28,125 (b) ` 38281.25
2








1 2 (c) ` 39,000 (d) None
(c) x – 4 x (d) x −




2 (iii) In which interval the profit is strictly increasing?







(ii) What is the number of days it will take for the plant to (a) (12.5, ∞)


grow to the maximum height?


(b) for all real numbers


(a) 4 (b) 6 (c) 7 (d) 10 (c) for all positive real numbers










(iii) What is the maximum height of the plant? (d) (0, 12.5)




(a) 12 cm (b) 10 cm (iv) When the production is 2 units, what will be the profit






(c) 8 cm (d) 6 cm of the company?




(iv) What will be the height of the plant after 2 days? (a) ` 37500 (b) ` 37,730






(c) ` 37,770 (d) None
(a) 4 cm (b) 6 cm




(v) What will be the production of the company when the




(c) 8 cm (d) 10 cm


profit is ` 38250?




(v) If the height of the plant is 7/2 cm, the number of days (a) 15


it has been exposed to the sunlight is ______.


(b) 30


(a) 2 (b) 3 (c) 2






(c) 4 (d) 1 (d) data is not sufficient to find






Ans. (i) (b) 4 – x (ii) (a) 4 (iii) (c) 8 cm Ans. (i) (b) 12.5 (ii) (b) ` 38281.24











(iv) (b) 6 cm (v) (d) 1 (iii) (d) (0 , 12.5) (iv) (b) ` 37,730 (v) (a) 15











pplication of Derivatives 173
A

E:\AMIT_WORKS\Exam_Guru\EG_Mathematics-12_(working_02-06-2022)\EG_Mathematics-12_working\Open_Files\Chap_6\Chap_6
\ 17-Aug-2022  Ved_Goswami   Proof-5 Reader’s Sign _______________________ Date __________


3. Rohan, a student of class XII, visited his uncles, flat with (i) Let the side of the square tank be x m, then the cost of




his father. He observed that the window of the house was land is:
in the form of a rectangle surmounted by a semicircular (a) ` 400 xxx (b) ` 100 x2
opening having perimeter 10 m as shown in the figure.






(c) ` 50 x2 (d) ` 200 x2
Based on the above information, give the answer of the





following questions. (ii) Total cost of land and digging is:



(a) ` (100 x2 + 3200 h2) (b) ` (100 x2 + 3200)





(c) ` (x2 + 3200 h2) (d) None of these





(iii) Height (h) of tank in terms of x and volume is:



x 4x
(a) 1000 x (b) 1000





1000 x2
y (c)2 (d)
x 500





(iv) Village Panchayat wants minimum cost for making



tank, then h should be:
(a) 5 mx (b) 2.5 m (c) 3 m (d) 5.5 m
x









(v) For minimum cost, x should be :
(i) If x and y represent the length and breadth of the



(a) 10 mx z(b) 15 m (c) 20 m (d) 30 m


rectangular region, then relation between x and y can









be represented as: Ans. (i) (b) `100x2 (ii) (a) `(100 x2 + 3200h2)






1000
(a) x + y + π = 10 (b) x + 2y + πx = 10 (iii) (c)
x2
(iv) (b) 2.5 m
2




2




π (v) (c) 20 m
(c) 2x + 2y = 10 (d) x + 2y + = 10



2 5. Shreya got a rectangular parallelopiped shaped box and






(ii) The area (A) of the window can be given by: spherical ball inside it as return gift. Sides of the box are


3 2
x 2 πx 2 x, 2x and x/3, while radius of the ball is r.
(a) x − x − x (b) 5x − −
8 2 2 8




πx3 3x 2 x 2 πx 2
(c) x + − (d) 5x + −
8 8 2 8




(iii) Rohan is interested in maximising the area of the whole


window, for this the value of x should be:
10 20 20 10
(a) 2 − π (b) 4 − π (c) 4 + π (d) 2 + π








(iv) Maximum area of the window is: Based on the given information, answer the following


30 50

30 50 questions:
(a) (b) 4 + π (c) 4 − π (d)
4−π 4+π (i) If S represents the sum of volume of parallelopiped and










(v) For maximum value of A, the breadth of rectangular sphere, then S can be written as:


part of the window is: 4x3 2 2
+ πr x 2x 2 4 2
10 10 20 20 (a) 3 2 (b) + πr
(a) (b) (c) (d) 3 3




4+π 4−π 4+π 4−π








2x3 4 3 2 4
x + πr
x (c) + πr (d)
Ans. (i) (b) x + 2y + π = 10 3 3 3 3




2



2 2 (ii) If sum of the surface area of box and ball are given to
(ii) (b) A = 5x − x − πx


2 8 be constant k2, then, x is equal to:



20 50 (a) k 2 − 4πr 2 (b) k 2 − 4πr
(iii) (c) (iv) (d)
4+π 4+π 6 6










10
(v) (a) k 2 − 4π
(c) (d) None of these
4+π







4. During rainy season, a lot of water is wasted. To prevent 6
(iii) The radius of the ball, when S is minimum, is:


it, village Panchayat decides to dug out a square tank of


capacity 1000 cubic metres. The cost of land is `100 per k2 k2
m2. The cost of digging increases with the depth and for (a) 54 + π (b) 54 + 4π




the whole tank, it is ` (32000 × h2), where h metres is the
depth of the tank. Based on the above information, answer k2 k2
the following questions: (c) 64 + 3π (d) 4π + 3




174 Mathe atics–12
m
E:\AMIT_WORKS\Exam_Guru\EG_Mathematics-12_(working_02-06-2022)\EG_Mathematics-12_working\Open_Files\Chap_6\Chap_6
\ 19-Aug-2022  Ved_Goswami   Proof-5 Reader’s Sign _______________________ Date __________


(iv) Relation between length of the box and radius of the 7. As we know good planning can save energy, time, and




ball can be represented as: money. A farmer wants to construct a circular well and
r 3r square garden in his field. He wants to keep the sum of
(a) x = 2r (b) x = (c) x = (d) x = 3r
2 2 their perimeters 600 m:








(v) Minimum value of S is:


(a) k2 (b) k2
2/3
2(3π + 54) (3π + 54)3/ 2




3
(c) k (d) None of these
3(4π + 54)1/ 2




3
Ans. (i) (c) 2x + 4 πr 3 (ii) (a) k 2 − 4πr 2
3 3 6





(iii) (b) k2 (iv) (d) x = 3r


54 + 4π





(i) If the radius of the circular garden be r m and the side
k3



(v) (c) of the square garden be x m then sum of area S is:
3 ( 4π + 54)
1/ 2



2
600 + 2πr 
6. A gardener wants to construct a rectangular bed of garden (a) S = πr 2 +  
 4




in a circular patch of land. He takes the maximum perimeter
of the rectangular region as possible. (See the images given 300 + πr 
2

here for calculations) (b) S = πr 2 +  


 4



2
600 + 2πr 
D C (c) S = 2πr 2 +  
 4


a
2
y
O  600 + πr 
a (d) S = πr +  
 4


A B (ii) Radius of circular well is:


600 300
(a) r = (b) r =

π+4 π+4




(i) The perimeter (P) of rectangle is:


(a) 4 x + 4 a 2 − x 2 (b) x + a 2 − x 2 300 150
(c) r = (d) r =




2π + 4 π+2




(c) 4 x + a 2 − x 2 (d) x + 4 a 2 − x 2




(ii) To find critical points put (iii) For the given condition



dP dP d 2S d 2S
(a) >0 (b) <0 (a) =0 (b) <0
dx dx dr 2 dr 2








dP d 2S
(c) =0 (d) None of these (c) >0 (d) None of these
dx




dr 2




(iii) Value of y is: (iv) The relationship between the side of the square garden




a a and the radius of the circular garden
(a) (b) (c) 2a (d) 2a
2 2 1








(iv) P is maximum when the rectangle is: (a) a = r (b) 2a = r2 (c) a = r (d) a = 2r
2










(a) Square (b) Parallelogram
(v) The number which exceeds its square by the greatest




(c) Rectangle (d) Trapezium


possible number:




(v) If a rectangle of the maximum perimeter which can be
1


inscribed in a circle of radius 10 cm is a square then (a) (b) 2 (c) 1 (d) 0
the sides of the region: 2








2
600 + 2πr 
(a) 10 8 cm (b) 2 10 cm Ans. (i) (a) S = πr 2 +  
 4







(c) 20 2 cm (d) 10 2 cm 300
(ii) (b) r =




dP π+4



Ans. (i) (a) 4 x + 4 a 2 − x 2 (ii) (c) =0
dx d 2S





a (iii) (c) >0 (iv) (d) a = 2r
dr 2





(iii) (b) (iv) (a) Square
2





1
(v) (d) 10 2 (v) (a)
2






Applic tion of Deriv tives 175
a
a

E:\AMIT_WORKS\Exam_Guru\EG_Mathematics-12_(working_02-06-2022)\EG_Mathematics-12_working\Open_Files\Chap_6\Chap_6
\ 17-Aug-2022  Ved_Goswami   Proof-5 Reader’s Sign _______________________ Date __________


Author’s Comments
Questions based on following types are very important for Exams. So, students are advised to revise them thoroughly.
1. Questions based on increasing and decreasing or strictly increasing/decreasing function.


2. Questions based on tangents and normals. (Most Important)


3. Word Problems on Maxima and Minima.


IMPORTANT FORMULAE
1. Rectangle: If P is the perimeter, A is the area, D is the diagonal, l is the length and b is the breadth of a rectangle, then


P = 2(l + b) cm


A = lb cm2


D = l 2 + b 2 cm


2. Square: If x is the length of the side of the square, P is the perimeter, A is the area and D is the diagonal of the square, then


P = 4x cm


A = x2 cm2


D = 2x cm


3. Circle: C is the circumference, A is the area and r is the radius of the variable circle, then


C = 2pr cm


A = pr2 cm2


4. Cuboid: V = Volume, S = surface area, D = diagonal and l, b, h are length, breadth and height of a cuboid, then


V = lbh cm3


S = 2[lb + bh + hl] cm2


D = l 2 + b 2 + h 2 cm



5. Cube: V = Volume, S = Surface area and x is the side of the cube.


V = x3 cm3


S = 6x2 cm2


D = 3x cm


6. Sphere: V = Volume, S = Surface area and r is the radius of the sphere


V = 4 πr 3 cm3
3


S = 4pr2 cm2


dy
7. (a) For the values of x for which = f ′ ( x ) ≥ 0, f (x) increases.
dx




(b) For the values of x for which dy = f ′ ( x ) ≤ 0, f (x) decreases.

dx


dy
(c) For the values of x for which = f ′ ( x ) > 0, f (x) strictly increases.
dx



dy
(d) For the values of x for which = f ′ ( x ) < 0, f (x) strictly decreases.
dx



COMMON ERRORS
ERRORS CORRECTIONS
(i) Ignore –ve sign while finding increasing or decreasing (i) Students have to check increasing/decreasing using (1) and




interval. not (2).
E.g y = –2x3 – 3x2 + 36x – 1

dy
dx (
= 0 ⇒ − 6 x2 + x − 6 = 0 ) ...(1)


⇒ x2 + x – 6 = 0 ...(2)


While checking increasing/decreasing, students ignore –ve

sign as in (ii)

176 Mathe atics–12


m
E:\AMIT_WORKS\Exam_Guru\EG_Mathematics-12_(working_02-06-2022)\EG_Mathematics-12_working\Open_Files\Chap_6\Chap_6
\ 17-Aug-2022  Ved_Goswami   Proof-5 Reader’s Sign _______________________ Date __________


REVISION CHART

I ncreasing and D ecreasing


F unctions

(a) The function y = f (x) is



called increasing function


for all x values which satisfy A pp ication of
l

f ′(x) ≥ 0.

D eri ati es
v v

(b) The function y = f (x) is



The two main


called decreasing function applications are ate of hange of Bodies
for all x values which satisfy
R C

using derivatives This point reminds us about the application/


f ′(x) ≤ 0. to determine interpretation of derivatives.

(c) The function y = f (x) is called



information about
strictly increasing function graphs of functions
for all x values which satisfy and optimization
f ′(x) > 0.

problems.
(d) The function y = f (x) is called

strictly decreasing function


for all x values which satisfy
f ′(x) < 0.

Loca Maxi
l m u m and Loca l Mini u
m m Va ues of a
l F unction

(A) Based on second derivative test: Let f (x) be a function



(B) Based on first derivative test to decide points of local


defined on an interval I and x = c ∈ I. Let f (x) be twice  

maxima or points of local minima:


differentiable. Then, (i) Let f ′(x) = 0 at x = c. If f ′(x) > 0 at every point

   

(i) x = c is the point of local maxima if f ′(x) = 0 and



sufficiently close to and to the left of x = c and


f ″(x) < 0. Then value f (c) is local maximum value
   

f ′(x) < 0 at every point sufficiently close to and


of f (x).  

to the right of x = c, then x = c is the point of local


(ii) x = c is the point of local minima if f ′(x) = 0 and

maxima.
f ″(x) > 0. In this case f (c) is local minimum value
   

(ii) Let f ′(x) = 0 at x = c. If f ′(x) < 0 at every point



   

of f (x).  

sufficiently close to and to the left of x = c and


(iii) This second derivative test fails when f ′(x) = 0

f ′(x) > 0 at every point sufficiently close to and


and f ″(x) = 0.  

to the right of x = c, then x = c is point of local


minima.
(iii) If f ′(x) does not change sign as x increases

through c, then c is neither a point of local


maxima nor a point of local minima. In fact such
a point is called point of inflexion or inflection.

pplication of Derivatives 177


A

E:\AMIT_WORKS\Exam_Guru\EG_Mathematics-12_(working_02-06-2022)\EG_Mathematics-12_working\Open_Files\Chap_7\Chap_7
\ 17-Aug-2022  Amit   Proof 5 Reader’s Sign _______________________ Date __________


7 Integrals
Topics overed
C
I. INDEFINITE INTEGRALS
7.1 Integration of Simple Algebraic Functions and Simple 7.2 Integration of Simple Trigonometric Functions




Exponential Functions

7.3 Integration by Substitution 7.4 Integration Using Standard Formulae




7.5 Directly using Formulae & Special Types of Integrals 7.6 Integration Using Partial Fractions




7.7 Integration By Parts 7.8 Repeating After Twice Integration




7.9 Integrals of Exponential Forms 7.10 Three More Formulae for Integration




II. DEFINITE INTEGRALS
7 .11 Direct Evaluation of Definite Integrals 7.12 Properties of Definite Integrals
7.13 Odd and Even Functions


C hapter map
I. INDEFINITE INTEGRALS

Integration of Simple Integration by Substitution Integration of Simple


Algebraic Functions Exponential Functions

Integration of Simple
Integration using Integration using Trigonometric Functions
Standard Formulae Partial Fractions

Integration Three More


by Parts Formulae for Integration

II. DEFINITE INTEGRALS

Direct Evaluation of Properties of Definite


Definite Integrals Integrals

I. INDEFINITE INTEGRALS
Indefinite integral is the inverse-process of differentiation. If derivative of a function F(x) is f (x), then integral of function f (x) is F(x).


For example derivative of x5 is 5x4 then integral of 5x4 will be x5. Let us introduce a new symbol ∫ f ( x ) dx . This symbol represents
integration of f (x) with respect to x. If derivative of the function F(x) is another function f (x) then symbolically we write it as:


∫ f ( x ) dx = F(x), F(x) is also called anti-derivative of f (x).



178
E:\AMIT_WORKS\Exam_Guru\EG_Mathematics-12_(working_02-06-2022)\EG_Mathematics-12_working\Open_Files\Chap_7\Chap_7
\ 17-Aug-2022  Amit   Proof-5 Reader’s Sign _______________________ Date __________


Constant of Integration
From the above definition of indefinite integral, we can write:
3 4
( )
unique. Actually ∫ 4x dx = x + (some unknown constant)
= x4 + C. It is true for all functions. Hence, when we integrate
(i)
d 4
dx
x = 4x 3 ( ) ⇒ ∫ 4x dx = x
3 4
( ) a function, we must put ‘+ C’ with its integral. Here C is any


arbitrary constant called constant of integration.




(ii)
d 4
dx
(
x + 7 = 4x 3 ) ⇒ ∫ ( 4x ) dx = x
3 4
+7 In ∫f ( x ) dx = F ( x ) + C
the function f (x) to be integrated is called integrand.




(i)




(
(iii) d x 4 + k = 4x3 ) ∫ ( 4x ) dx = x (ii) F(x) is called indefinite integral of f (x).
3 4
⇒ +k




dx (iii) C is called constant of integration and






From the above examples, we conclude that integral of 4x3 is not (iv) x is called variable of integration.



Topic 1. Integration of Simple Algebraic Functions and Simple Exponential Function
Simple Algebraic Functions
∫ 7x
5
Example 3. Evaluate: dx
Formulae
 x5+1 
∫ 7x dx = 7  5 + 1 + C
5
Solution. I=
 x n +1  d  x n +1  



(i) ∫ x dx =  n + 1 + C, n ≠ − 1
n
Q    + C = x n
dx  n + 1  ⇒ I = 7 x6 + C


6




x3
Example 1. ∫ x + 1 dx is equal to
(iii)
∫  f ( x ) + g(x) + φ (x) dx
= ∫ f ( x ) dx + ∫ g(x) dx + ∫ φ(x) dx + C


x 2 x3
(a) x + + − log 1 − x + C
2 3


 1

∫  x
5
x 2 x3 Example 4. Evaluate: + 3x 2 + 5x 2  dx
(b) x + − − log 1 − x + C 
2 3


 5 1
I = ∫  x + 3x + 5x 2  dx
2
x 2 x3 Solution.
(c) x − − − log 1 + x + C  


2 3


1

∫ x dx + ∫ 3x dx + ∫ 5x 2 dx
5 2
x 2 x3 =
(d) x − + − log 1 + x + C


2 3  3


x6  x3  x2
x 3
I= + 3  + 5  + C
Solution. Let I = ∫ x + 1 dx 6  3  3


 2


1 6 10 3/ 2
\ I=

∫  x
2
− x +1−
1 
dx =x +x +
3
x +C ( )
x + 1 6 3





x3 x 2
= −
3 2
+ x − log x + 1 + C (iv) ∫ k dx = kx + C, Here k is constant




x 2 x3 d
=x− + − log x + 1 + C Q
dx
[ kx + C] = k
2 3



Hence, the correct option is (d). Example 5. Evaluate: ∫ 4 dx

∫ 4 dx
3
Solution. I= = 4x + C
Example 2. Evaluate: ∫ x 2 dx


 3 +1 
x2  2
5 (ax + b) n + 1
∫ (ax + b) dx =
n
Solution. I = ∫ x dx =  3/ 2
+ C = x2 + C (v) + C, n ≠ − 1
 3 + 1 5 a(n + 1)




2 
d  ( ax + b )
n +1 
+ C = ( ax + b )
n
Q 
 xn + 1  dx  a ( n + 1) 

∫ ax dx = a  n + 1 + C, n ≠ − 1
n
(ii) 


∫ (3x + 5)
7
Example 6. Evaluate: dx
d   x n +1  
Q  a  + C = ax n ( 3x + 5)
7 +1

∫ (3x + 5) dx = 3
7
dx   n + 1  Solution. I= +C


(7 + 1)


ntegrals 179
I

E:\AMIT_WORKS\Exam_Guru\EG_Mathematics-12_(working_02-06-2022)\EG_Mathematics-12_working\Open_Files\Chap_7\Chap_7
\ 17-Aug-2022  Amit   Proof 5 Reader’s Sign _______________________ Date __________


(3x + 5) + C = 1 (3x + 5)8 + C ∫ ( x )
8 66 
I=  2
+ 10x + 27 + dx
= x − 4 



24 24


dx 3
Example 7. Evaluate: ∫ =x + 5x 2 + 27x + 66 log x − 4 + C
8x + 3 + 8x + 1 3



dx
Solution. I= ∫
8x + 3 + 8x + 1 dx −1
∫ x n = (n − 1) x n − 1 + C, n ≠ 1


(vi)
Rationalise denominators




( 8x + 3 − 8x + 1 dx )  
I= ∫ Q
d  −1
+ C  = 1n
8x + 3 − 8x − 1
( )
dx  ( n − 1) x n −1


 x


( 8x + 3 − 8x + 1 dx )  
= ∫ 2 1


 3/ 2 
Example 11. Evaluate: ∫ x 4 dx
⇒ I= 1  ( 8x + 3)
3/ 2

( 8x + 1) 
+C
2  3 ×8 3 ×8   1 −1 −1
Solution. ∫  x 4  dx = ( 4 − 1) x 4−1 + C = 3x3 + C


I=
 2 2 



1
= (8x + 3) − (8x + 1)  + C
3/ 2 3/ 2
dx −1
24 (vii) ∫ (ax + b)n = a (n − 1) (ax + b)n − 1 + C, n ≠ 1


( )


Example 8. Evaluate: ∫ x 2 − 1  x + 13  dx
x  x     
Q d  −1 1
Solution. I= ∫ ( 2 1
x 
1
x  )
x −  x + 3  dx
dx   a ( n − 1) ( ax + b ) 

n −1  + C =
  ( ax + b )
n


 1 1 dx
⇒ I= ∫  x
3
+ − 1 − 4  dx Example 12. Evaluate: ∫ ( 4x + 3)5
x x 



4
dx −1
=x + log x − x + 1 3 + C Solution. I= ∫ ( 4x + 3)5 = 4 (5 − 1) ( 4x + 3)5−1 + C
4 3x




(x 2
)
+ 2 (3x + 5)  −1   −1 
Example 9. Evaluate: ∫ x 2 dx I=  4 +C =  4 +C
 4 × 4 ( 4x + 3)  16 ( 4x + 3) 


(x 2
)
+ 2 (3x + 5)
Solution. I= ∫ x2
dx
(viii) ∫
dx
= log | x | + C Q d log x + C = 1


x dx x




 3x3 + 5x 2 + 6x + 10 
⇒ I= ∫  x2  dx Example 13. Evaluate: ∫
dx



x
 6 + 10  dx
=∫  3x + 5 +
 x x2 
 Solution. I= ∫
dx
= log x + C


x


2
⇒ I = 3x + 5x + 6 log x − 10 + C  dx 
2 x 1
(ix) ∫  ax + b  = a log | ax + b | + C





 ( x − 3) ( x + 2) ( x + 7 ) 
Example 10. Evaluate: ∫  ( x − 4)  dx
 Q
d 1
log ax + b + C =
1
dx  a  ax + b

 ( x − 3) ( x + 2) ( x + 7 ) 
Solution. I= ∫  ( x − 4)  dx dx
∫ (7x + 9)


 Example 14. Evaluate:
 ( x − 3) x 2 + 9x + 14  ( ) dx 1
⇒ I=∫ 
x−4
 dx
 Solution. I= ∫ (7x + 9) = 7 log 7x + 9 + C





 
 x3 + 9x 2 + 14x − 3x 2 − 27x − 42  1
=∫  x−4  dx = log 7x + 9 + C
7




 3 2  (3x − 7)2 dx
I = ∫  x + 6x − 13x − 42  dx Example 15. Evaluate: ∫ 3x + 2
( x − 4)


 
3 2
(3x − 7)2 dx
\ x + 6x − 13x − 42 = x 2 + 10x + 27 + 66
x−4 x−4
( ) Solution. I= ∫ 3x + 2



180 Mathe atics–12
m
E:\AMIT_WORKS\Exam_Guru\EG_Mathematics-12_(working_02-06-2022)\EG_Mathematics-12_working\Open_Files\Chap_7\Chap_7
\ 17-Aug-2022  Amit   Proof-5 Reader’s Sign _______________________ Date __________


 9x 2 − 42x + 49  Example 18. Evaluate: ∫ 5 xdx
⇒ I= ∫  3x + 2  dx



5x
∫ 5 dx = log 5 + C
x
2 Solution. I=
9x − 42x + 49 81
= (3x − 16) +



\
3x + 2 3x + 2

5x

( )
⇒ I= +C
 81  log 5
∫ (3x − 16) +




\ I=  dx
3x + 2



a mx + n d  a mx + n 
3x 2 81 (iv) ∫ a dx =
mx + n
+C Q + C = a mx + n
= − 16x + log 3x + 2 + C m log a dx  m log a 





2 3


2 ( 2x + 5)dx
⇒ I = 3x − 16x + 27 log 3x + 2 + C Example 19. Evaluate: ∫7
2



( 2x + 5)
( 2x + 5)dx = 7
 dx  2 ax + b
(x) ∫ 
Solution. I= ∫7 2 log 7
+C
= +C



 ax + b  a


( 2x+ 5)
⇒ I= 7 +C
  2 log 7
Q d  2 ax + b + C = 1



dx  a  ax + b  5 x + 23x 

Example 20. Evaluate: ∫  dx
 7 2x 
dx
Example 16. Evaluate: ∫  5 x + 23x   5 x 23x 
5x − 3 Solution. I= ∫  7 2x  dx = ∫  2x + 2x  dx
 7 7 



dx 2 5x − 3
Solution. I= ∫ 5x − 3
=
5
+C  ( 5) x (8)x  dx


I=∫  ( 49)x ( 49)x 
 +


Simple Exponential Functions  
 5 x
( ) ( )
x
8
(i) ∫e
x
dx = e x + C Q d e x + C = e x =∫  49 + 49  dx
dx  




( ) ( )
x x
1 5 8
(ii) ∫ e mx + n
dx = e mx + n + C 49 49
m
( ) ( )
= + +C


log 5 log 8


Q d  1 e mx + n + C = e mx + n 49 49
dx  m 
∫ (2 )

x 2
Example 21. Evaluate: + 3x dx
∫e
4x + 7
Example 17. Evaluate: dx

() ∫ (2 )
x 2
1 4x + 7 Solution. I= + 3x dx
∫ e dx =
4x + 7
Solution. I= e +C


4


()
( x) ( x)
∫ ( 2) + (3) + 2 × 2 × 3  dx
2x 2x
1 4x + 7 =
⇒ I= e +C


4



∫ 4 + 9 + 2 × 6  dx
x x x
=


ax  x 
∫ a dx = log a + C Q d  a + C = a x
x
(iii) 4x 9x 2.6 x
dx  log a  = + + +C




log 4 log 9 log 6


EXERCISE 7.1
I. Multiple Choice Questions (MCQs)  1 
3. The antiderivative of  x +
1.
d
dx ∫
f ( x)dx = ( )   equals
x




1 1 1 1
2 2 1
(a) f ′ (x) (b) (f(x))2 (a) x 2 + 2 x 2 + C (b) x 3 + x 2 + C
2 3 3 2








(c) f (x) (d) one of these 2 32 1
3 2 1
(c)
x + 2x 2 + C (d) x 3 + x 2 + C




2. One value of ∫ f ′(x) dx is 3 2 2






1 4. ∫ (1 + 5x + 10x2 + 10x3 + 5x4 + x5) dx =
(a) f ′ (x) (b) (f(x))2


2 (a) 5 + 20x + 30x2 + 20x3 + 5x4






1 5 x 2 10 x 2 10 x 4
(c) (d) f(x) (b) x + + + + x5 + 5 x6
f ( x)




2 3 4


ntegrals 181
I

E:\AMIT_WORKS\Exam_Guru\EG_Mathematics-12_(working_02-06-2022)\EG_Mathematics-12_working\Open_Files\Chap_7\Chap_7
\ 17-Aug-2022  Amit   Proof 5 Reader’s Sign _______________________ Date __________


(1 + x)6 dx
(c) 2. ∫ 4x + 3 + 4x − 5



3!


(d) None of these dx
3. ∫


8 − 5x − 7 − 5x



d 3
5. If f ( x) = 4 x 3 − 4 such that f(2) = 0, then f (x) is
dx x
( ) 1  2 1 


4. ∫ x+
x 
x +  dx
x



1 129 1 129
(a) x 4 + (b) x 3 + +
( )

x3 8 x4 8  1  1




5. ∫  x + 
x
x+
x
dx



1 129 4 1 129 3
(c) x + 3 + (d) x + 4 +
x 8 x 8
( ) 1  2 1




6. ∫ x+  x − 3  dx
x  x



II. Very Short Answer Type Questions
Evaluate each of the following Integrals IV. Long Answer Type Questions
 3  1. ∫ (x + 1) (x + 2) (x + 3) dx
∫ (ax ∫ 5x 2 + x + 7 dx
2



1. + bx + c) dx 2.

( )




   1 − x 2  x3 + 1 dx
∫x
2
2.  2 
3. ∫ (7x + 5) dx
5
4. ∫ dx7 x x



x




dx dx (x + 1) (3x 2 − 1)
5. ∫ (4x + 7)5 6. ∫ 7x − 3
3. ∫ x2
dx






dx  (3x + 2) (2x − 3) 2 
∫ 9x + 5 ∫ (3x
3
7. 8. + 5) 2 dx 4. ∫   dx




x2


2

2  4 1   1 
9. ∫ x  x + x + x + 3 dx 10. ∫  x − x  dx 5.
 (2x + 1) 2 
∫  3x + 2  dx






∫e ∫7
(4x + 3) (2x − 5)
11. dx 12. dx
 (x + 1) (x + 2) (x + 3) 




6. ∫  (x + 4)  dx



∫ (3 ∫ (5
x
13. × 2 x) dx 14. x
× 32x) dx




 x 4 + x 2 + 1
8 + 3x x a +b x x
7. ∫  x 2 + x + 1  dx


15. ∫  5 x 
dx 16. ∫  a xb x 
dx
 x4 + 7 




8. ∫  x + 1 
dx
∫ (e )
3 log x
+ e 2 log x dx


17.


( )  dx
 3 x + 5 2x 2
III. Short Answer Type Questions-I 9. ∫ 
 22x 


1. ∫ 5 x + 7x3 x + x x  dx  


Answers 7.1
I. 1. (c) 2. (d) 3 x4 x− 3

∫  4 x



3
f (x) = 4
dx = 4 ⋅ − 3 ⋅
− +C
 1  1 1
x  4 −3



3. (c) ∫  x +  dx = ∫ x dx + ∫ x dx
2 2
1 1
 x


⇒ f (x) = x 4 + 3 + C \ f (2) = x 4 + 3 + C
x x




3 1
x 2
x 2 3 21
1 129
= + = x 2 + 2x 2 + C Now, f (2) = 0 ⇒ 16 + + C = 0 ⇒ C = −
3 1 3 8 8




2 2 4 1 129
Thus, f (x) = x + 3 −
4. (c) I = ∫ (1 + 5 x + 10 x 2 + 10 x3 + 5 x 4 + x5 ) dx x 8



5


ax3 + bx 2 + cx + D x 2 + 7x + C
(1 + x)6 (1 + x)6 (1 + x)6 II. 1. 2. 2x 2 +
= ∫ (1 + x)5 dx = = = 3 2 2



6 3 ⋅ 2 ⋅1 3! (7 x + 5) 6
−1
d 3 3. +C 4. +C
5. (a) We have 3
f ( x) = 4 x − 4 42 6x 6




dx x


−1 2 7x − 3
5. +C 6. +C
Integrating both sides, we get 16(4x + 7) 4 7





182 Mathe atics–12
m
E:\AMIT_WORKS\Exam_Guru\EG_Mathematics-12_(working_02-06-2022)\EG_Mathematics-12_working\Open_Files\Chap_7\Chap_7
\ 17-Aug-2022  Amit   Proof-5 Reader’s Sign _______________________ Date __________


7. 1 log | 9x + 5 | + C
4 2
9 IV. 1. x + 2x3 + 11x + 6x + C


4 2


9x 7 + 15x 4 + 25x + C Hint: First multiply all the 3 factors and then integrate.
8.


7 2


8
x
Hint: Expand (3x2 + 5)2 and integrate. 2. log | x | − + C
8




5 2
9. x + x + log | x | + 3x + C Hint: First multiply 3 factors and then integrate.
5 2




3x 2 + 3x − log | x | + 1 + C
Hint: First multiply and then integrate. 3.
2 x




x 2 − 2x + log | x | + C
10. ( x + 1)(3x 2 − 1)
2 = 3x + 3 − 1 − 12


2 Hint: Now integrate.
Hint: Expand  x − 1  and then integrate. x2 x x



 x 4. 6x 2 − 28x + 3 log | x | − 18 + C

x



(2x − 5)
11. 1 e(4x + 3)  + C 12. 7 +C Hint: Expand Nr and divide by x2, then integrate.
4 2 log 7





x 2x 2 + 4x + 1 log | 3x + 2 | + C
13. 6 + C . Hint: 3x × 2x = 6x. 5.
3 9 27



log 6


x ( 2x + 1)2 4x 2 + 4x + 1
14. (45) + C . Hint: 5x × 32x = 5x × 9x = (45)x. Hint:
3x + 2
=
3x + 2




log (45)


4 4 1
( ) + ( ) +C
x x x+ +
8 3 =
3 9 9 (3x + 2)


15. 5 5
log ( 8 ) log ( 3 )
x3 + x 2 + 3x − 6 log | x + 4 | + C
6.


5 5 3


( x + 1)( x + 2)( x + 3) = x3 + 6x 2 + 11x + 6
= ( ) +( )
x x x x
8 +3 8 3 Hint:
Hint: ( x + 4) ( x + 4)

x5 5
5



Now integrate. = x + 2x + 3 − 6
2
( x + 4)



16. −1 1 x3 − x 2 + x + C
− +C 7.
(a x) log a (b x) log b 3 2




Hint:
ax + bx 1
= x+ x
1
Now integrate. Hint:
x4 + x2 + 1
=
x2 + x + 1 x2 − x + 1 ( )( )
a xb x b a x2 + x + 1 x2 + x + 1 ( )








4 3
17. x + x + C;
 m log x
Q e =e
log x m 
= x m 
( ) = x2 – x + 1.



4 3 Now integrate.



log x3 log x 2 4 3 2
Hint: e3 log x + e2 log x = e +e 8. x − x + x − x + 8 log | x + 1 | + C
4 3 2





3 9 5
III. 1. 10 x 2 + 14 x 2 + 2 x 2 + C
3 9 5 x4 + 7 x4 − 1 + 8 ( )

 3 3 Hint: =
2. 1 (4x + 3) 2 − (4x − 5) 2  + C x +1 x +1



48  ( x + 1)( x − 1)( x 2 + 1)



dx = + 8
x +1 x +1
∫ 4x + 3 + 4x − 5
Hint:


( )

= ( x − 1) x 2 + 1 + 8
Rationalise Dr and integrate. x +1



 3 3 3 2 8
3. − 2 (8 − 5x) 2 + (7 − 5x) 2  + C = x − x + x −1+
x +1


15 


 Now integrate.
Hint: Rationalise the Dr and then integrate.

( 32 ) ( ) 2 ( 75 )
2x 2x x

3 −1 25
x 4 + 2 x 2 + x 2 − 2x 2 + C
4. 9. 2 4
2 log ( ) 2 log ( ) log ( 75 )
4 3 2 + + +C


3 25


Hint: Multiply two factors and then integrate. 2 2 4

5 −1
2 x 2 + 2x − 2x 2 + C
(3 )
x 2
5. + 52x 32x + 54x + 2 ( 75)
x
5 Hint: =


2x
Hint: Multiply two factors and then integrate. 2 22x


() ( ) ( )

4 2 2x 2x x
6. x + 1 + x + 1 3 + C = 3 + 25 + 2 75
4 x 2 3x 2 2 4




Hint: First multiply two factors and then integrate. Now integrate.


ntegrals 183
I

E:\AMIT_WORKS\Exam_Guru\EG_Mathematics-12_(working_02-06-2022)\EG_Mathematics-12_working\Open_Files\Chap_7\Chap_7
\ 17-Aug-2022  Amit   Proof 5 Reader’s Sign _______________________ Date __________


Topic 2. Integration of Simple Trigonometric Functions
Formulae 12. cos 4x = 8 sin4 x – 8 sin2 x + 1



cos 2 x − cos 2θ
1. ∫ sin x dx = − cos x + C Example 1. ∫ cos x − cos θ
dx is equal to


d (a) 2(sin x + x cos q) + C
Q ( − cos x ) + C = sin x



dx (b) 2(sin x – x cos q) + C



(c) 2(sin x + 2x cos q) + C
2. ∫ cos x dx = sin x + C





(d) 2(sin x – 2x cos q) + C
d



Q (sin x + C) = cos x cos 2 x − cos 2θ
dx Sol. Let I = ∫ dx

cos x − cos θ




(2 cos x − 1) − (2 cos 2 θ − 1)
2
3. ∫ tan x dx = log | sec x | + C = ∫
cos x − cos θ
dx



2 2
d 2 cos x − 1 − 2 cos θ + 1
Q log sec x + C = tan x = ∫ dx
dx  cos x − cos θ



2 cos 2 x − 2 cos 2 θ
= ∫
4. ∫ cot x dx = log | sin x | + C cos x − cos θ
dx




cos 2 x − cos 2 θ
d
log sin x + C = cot x = 2∫ dx
Q
dx  cos x − cos θ



(cos x + cos θ) (cos x − cos θ)
= 2∫ dx
5. ∫ sec x dx = log | sec x + tan x | + C
(cos x − cos θ)
= 2 ∫ (cos x + cos θ) dx



d \ I = 2(sin x + cos q . x) + C.
Q log sec x + tan x + C = sec x
dx 


Hence, correct option is (a).


dx
Example 2. ∫ is equal to
6.
∫ cosec x dx = log | cosec x − cot x | + C sin ( x − a ).sin ( x − b)
= − log | cosec x + cot x | + C sin( x − b)


(a) sin (b − a) log +C
sin ( x − a)
Q d log ( cosec x − cot x ) + C = cosec x


dx sin( x − a )

(b) cosec (b − a) log +C
or d  − log cosec x + cot x + C = cosec x sin ( x − b)


dx

sin( x − b)
(c) cosec (b − a) log +C
∫ sec x dx = tan x + C
2
7. sin ( x − a )




d sin( x − a )
Q
dx
[ tan x + C] = sec2 x (d) sin (b − a) log
sin ( x − b)
+C



dx
∫ cosec x dx = − cot x + C
2
8. Sol. Let I = ∫ sin ( x − a).sin ( x − b)




d
Q
dx
[ − cot x + C] = cosec2 x Multiplying and dividing by sin (b – a) we get,


1 sin (b − a )
I= ∫
sin (b − a ) sin ( x − a ).sin ( x − b)
dx
∫ (sec x tan x) dx = sec x + C

9.


1 sin ( x + b − x − a )
sin (b − a ) ∫ sin ( x − a ).sin ( x − b)
= dx
d
Q (sec x ) + C = sec x tan x

dx

1 sin [( x − a ) − ( x − b)]
10. ∫ (cosec x cot x) dx = − cosec x + C
= ∫
sin (b − a ) sin ( x − a ).sin ( x − b)
dx



d sin ( x − a )cos ( x − b)
Q [ −cosec x ] + C = cosec x cot x 1 − cos ( x − a )sin ( x − b)
dx = ∫ sin ( x − a).sin ( x − b) dx

sin (b − a )

Note: If angle of T-ratio is (ax + b) then divide the integral of
1  sin ( x − a ).cos ( x − b)
T-ratio by a.
sin (b − a ) ∫  sin ( x − a ).sin ( x − b)
= 

For example:
cos ( x − a ).sin ( x − b) 
−1
−  dx
sin ( x − a ).sin ( x − b) 
∫ sin (ax + b) dx = a
cos (ax + b) + C
1  cos ( x − b) cos ( x − a ) 
= ∫ − dx

11. cos 4x = 8 cos x – 8 cos x + 1
4 2 sin (b − a )  sin ( x − b) sin ( x − a ) 



184 Mathe atics–12
m
E:\AMIT_WORKS\Exam_Guru\EG_Mathematics-12_(working_02-06-2022)\EG_Mathematics-12_working\Open_Files\Chap_7\Chap_7
\ 17-Aug-2022  Amit   Proof-5 Reader’s Sign _______________________ Date __________


1 I=
∫ ( 4 cos )
2
[cot ( x − b) − cot ( x − a)] dx x − 3 dx
sin (b − a ) ∫
=




=
1
sin (b − a )
[log sin ( x − b) − log sin ( x − a)] + C =
 1 + cos 2x
∫ 4 2 − 3 dx

( )





1 sin ( x − b)
=
sin (b − a )
.log
sin ( x − a )
+C =∫ [ 2 cos 2x − 1] dx = sin 2x − x + C



sin ( x − b)  sin 3x 
I = cosec (b − a ).log
sin ( x − a )
+C Example 4. Evaluate: ∫  sin 3 x  dx . [HOTS]


Hence, the correct option is (c).  3 sin x − 4 sin 3 x 
 sin 3x 
( ) ∫  sin 3 x  dx = ∫ 

Example 3. Evaluate: ∫
cos 3x
dx . [HOTS]
Solution. I=
sin 3 x  dx



cos x 
( ) ∫ 3 cosec x − 4 dx = −3 cot x − 4x + C
2
cos 3x  4 cos3 x − 3 cos x  =
Solution. I = ∫ cos x dx = ∫   dx



cos x


EXERCISE 7.2
I. Multiple Choice Questions (MCQs) Evaluate each of the following Integrals
cos 4 x x II. Very Short Answer Type Questions
1. If ∫ 2
dx = A cot x + B sin 2 x + C + D, then
sin x 2


∫ (tan 3x) dx ∫ (sin 3x) dx
2 2
1. 2.
1




∫ (cos 5x) dx
2
∫ [tan (5x + 3)] dx
2
(a) A = –2 (b) B = 3. 4.
4








 (cos 2x)  dx
(c) C = 3 (d) A = –1, B = −
1
, C = –3
5. ∫  sin 2 x cos2 x  dx 6. ∫ (sin 2 x cos2 x)




4




tan x dx dx
cos 8 x + 1 7. ∫ sec x + tan x 8. ∫ 1 + cos 2x
2. ∫ tan 2 x − cot 2 x dx = a cos 8 x + C, then






 sin 2 x − cos 2 x 
(a) a = −
1
(b) a =
1 9. ∫  sin x cos x  dx [A.I. 2017]



16 16




 sin 2x   sin 2x 
1 1 10. ∫  sin 3 x  dx 11. ∫  cos3 x  dx




(c) a = − (d) a =
8 18
( ) ( )




sin 2x cos 2x
sin 8 x − cos8 x 12. ∫ sin 4x
dx 13. ∫ cos x
dx
3. Evaluate: ∫




dx
∫( )
1 − 2 sin 2 x cos 2 x


cos 2x  cos 2x 
1
14. sin x
dx 15. ∫  cos2 x  dx




(a) Sin 2x + C (b) − sin 2x + C
2  cos 2x 
( cos 2x
)




1
16. ∫  sin 2 x  dx 17. ∫ sin 4x
dx




(c) sin 2x + C (d) –sin 2x + C
2




 sin 2 x   sin 2 x 
cos 2 x + 2 sin 2 x 18. ∫  cos x  dx 19. ∫  sin 2x  dx
4. ∫




dx =
cos 2 x


 cos 2 x   cos 2 x 
(a) 2 sec x + C (b) 2 tan x + C 20. ∫  sin x 
dx 21. ∫  sin 2x  dx








(c) tan x + C (d) None of these
III. Short Answer Type Questions-I




cos 2 x − 1
5. ∫ dx is equal to 1. ∫ (sin
2
x cos 2 x) dx 2. ∫ (sin
3
x cos3 x) dx
cos 2 x + 1






(a) tan x – x + C (b) x + tan x + C 1 + sin x
3. ∫ dx 4. ∫ 1 + cos 2x dx




(c) x – tan x + C (d) – x – cot x + C 1 − sin x








6. ∫ (sin
−1
x + cos −1 x) dx = 5. ∫ 1 + sin 2x dx




1
(a)
2
πx + C (b) x(sin–1 x – cos–1 x) + C 6. ∫ 1 + 2 tan x (sec x + tan x) dx






π  cos 2x − cos 2α   cos 2x − cos 2α 
(c)
2
+x+C (d) None of these 7. ∫  cos x + cos α 
dx 8. ∫  sin x + sin α 
dx








ntegrals 185
I

E:\AMIT_WORKS\Exam_Guru\EG_Mathematics-12_(working_02-06-2022)\EG_Mathematics-12_working\Open_Files\Chap_7\Chap_7
\ 17-Aug-2022  Amit   Proof 5 Reader’s Sign _______________________ Date __________


 cos 2x − cos 2α   cos 2x + cos 2α  dx
9. ∫  dx 10. ∫  dx 9. ∫ sin ( x + a )sin ( x − b) [HOTS]
sin (x + α)  cos (x − α) 








dx
cos 2x + cos 2α cos 2x + cos 2α 10. ∫ cos ( x − a ) cos ( x + b) [HOTS]
11. ∫ cos x − sin α dx 12. ∫ sin x + cos α dx








dx
11. ∫ sin ( x + a ) cos ( x − b) [HOTS]
∫ [sin 3x cos 2x ] dx 14. ∫ sin 7x 1 − cos 4x  dx




13.




dx
12. ∫ cos ( x − a )sin ( x + b) [HOTS]
( )cos 4x




 cos 3x  dx
15. ∫ cos 2x
dx 16. ∫ 
 cos 2 x 
∫ sin
4
13. x cos 4 x dx [HOTS]







∫( )sin 3x  cos 3x 
17.
sin x
dx 18. ∫  cos3 x  dx 14. dx
∫ 15. 1 − sin 2x dx

1 + sin 2x




1 − sin 2x





sin x dx dx
IV. Long Answer Type Questions 16. ∫ 17. ∫
1+ sin x cos 3x − cos x





∫ (sin x) dx
4
1. [A.I. 2007 (C)] 18. ∫ ( tan 3x tan 2x tan x ) dx





∫ cos 3x  dx
( )
4
2. [Delhi 2008 (C)] sin 5x + sin 3x + sin x
∫ dx



19. cos 5x + cos 3x + cos x
dx dx
3. ∫ sin (x + a) sin (x + b) 4. ∫ sin (x + a) cos (x + b)



( )




 cos 4x  sin 3x dx
5. dx
∫ cos (x − a) cos (x − b)
20. ∫  cos2 x  dx 21. ∫ sin 2x






6. ∫ dx [Delhi 2000]

22. ∫ ( cos 3x dx
sin 2x ) 23.
 cos 4x 
∫  cos2 x  dx



sin (x − a) cos x



dx  cos 4x 
7. ∫
sin (x − a) sin (x + b)
8. ∫ [sin 4x cos 3x sin 2x ] dx 24. ∫  sin 2 x  dx






Answers 7.2
cos 4 x (1 − sin 2 x) 2 1 + sin 4 x − 2 sin 2 x [(sin 2 x + cos 2 x) 2 − 2 sin 2 x cos 2 x] (sin 4 x − cos 4 x)
I. 1. (d) I = ∫ dx = ∫ sin 2 x dx = ∫ dx = ∫ dx
sin 2 x sin 2 x (sin 2 x + cos 2 x) 2 − 2 sin 2 x cos 2 x



= ∫ (sin 4 x − cos 4 x) dx
= ∫ cosec x dx + ∫ sin 2 x dx − 2 ∫ dx
2




= ∫ (sin x + cos x)(sin x − cos x) dx
2 2 2 2

 1 − cos 2 x 


= − cot x + ∫   dx − 2 x
 2 sin 2 x


x sin 2 x
= ∫ (sin 2 x − cos 2 x) dx = ∫ − cos 2 x dx = − 2
+C



= – cot x + − 2 x + D (constant) 4. (c) tan x + C
2 4




1 3 5. (c) x – tan x + C


= − cot x − sin 2 x − x + D 1
4 2 6. (a) πx + C


1 2


1
\ A = –1, B = − and C = –3. II. 1. 3 tan 3x − x + C;
4




Hint: Use: tan2 3x = sec2 3x – 1
cos 8 x + 1 2 cos 2 4 x

2. (b) I = ∫ tan 2 x − cot 2 x ∫ sin 2 x cos 2 x dx
dx = 2. x sin 6x C;
− +
− 2 12






cos 2 x sin 2 x 1 − cos 6x
2
2 cos 4 x Hint: Use : sin 2 3x =
=∫ 2 ⋅ sin 2 x cos 2 x dx 2

sin 2 x − cos 2 2 x 3. x sin 10x C;


+ +
2 20


2
cos 4 x
= ∫ − (cos 2
2 x − sin 2 2 x)
⋅ ( 2 sin 2 x cos 2 x) dx Hint : Use: cos 2 5x =
1 + cos 10x


2

tan (5x + 3)
cos 2 4 x 1 4. − x + C;
= ∫ − cos 4 x .sin 4 x dx = − 2 ∫ 2 cos 4 x sin 4 x dx 5


Hint: Use: tan2 (5x + 3) = sec2 (5x + 3) – 1



1 1  cos 8 x  1 5. – (tan x + cot x) + C;
2∫
=− sin 8 x dx = −  −  + C = cos 8 x + C



2 8  16 cos 2x = cos 2 x − sin 2 x


Hint: = cosec2 x – sec2 x
sin 2 x cos 2 x sin 2 x cos 2 x
4 4 4 4
(sin x + cos x)(sin x − cos x)


3. (b) I = ∫ (sin 2
x + cos 2 x) 2 − 2 sin 2 x cos 2 x
dx
Now integrate.





186 Mathe atics–12
m
E:\AMIT_WORKS\Exam_Guru\EG_Mathematics-12_(working_02-06-2022)\EG_Mathematics-12_working\Open_Files\Chap_7\Chap_7
\ 17-Aug-2022  Amit   Proof-5 Reader’s Sign _______________________ Date __________


6. (tan x – cot x) + C; − 3 cos 2x cos 6x
2. + +C;


Hint: 1 sin 2 x + cos 2 x = sec2 x + cosec2 x 64 192
2 =



2
sin x cos x sin 2 x cos 2 x


Hint: sin 3 x cos3 x = 1 sin 3 2x
OR Ans. –2 cot 2x + C 8





Hint : 2 1 2 = 42 = 4 cosec22x = 1 (3 sin 2x − sin 6x )
sin x cos x sin 2x 32


3. log| sec x + tan x | + log| sec x | + C
Now integrate.








7. [sec x – tan x + x] + C; 1 + sin x = 1 + sin x = sec x + tan x
Hint:
1 − sin x cos x



tan x
Hint: = tan x (sec x − tan x ) Now integrate.
sec x + tan x


= sec x tan x – tan2 x 4. Ans. 2 sin x + C




= sec x tan x – sec2 x + 1 Hint: 1 + cos 2x = 2 cos 2 x = 2 cos x


Now integrate.


Now integrate.

1 tan x + C;


8. 5. (sin x – cos x) + C;
2




1 1 1 Hint: 1 + sin 2x = sin 2 x + cos 2 x + 2 sin x cos x
Hint : = = sec2 x
1 + cos 2x 2 cos 2 x 2


=(sin x + cos x ) = sin x + cos x
2

Now integrate.


Now integrate.

9. –log | sin x cos x | + C.




6. log | sec x + tan x | + log| sec x | + C


10. – 2 cosec x + C







1 + 2 tan x (sec x + tan x )



sin 2x = 2 sin x cos x = 2 cosec x cot x Hint:
Hint:

sin 3 x sin 3 x

11. 2 sec x + C. = 1 + 2 sec x tan x + 2 tan 2 x



Hint:
sin 2x = 2 sin x cos x = 2 sec x tan x
cos3 x cos3 x
= (1 + tan x) + 2 sec x tan x + tan
2 2
x


12. 1 log | sec 2x + tan 2x | + C. = sec2 x + 2 sec x tan x + tan 2 x


4



Hint:
sin 2x = 2 sin x cos x = 1 (sec 2x ) = (sec x + tan x) 2 = sec x + tan x

sin 4x 4 sin x cos x cos 2x 2

Now integrate.
13. 2 sin x – log | sec x + tan x | + C






cos 2x = 2 cos 2 x − 1 = 2 cos − sec 7. 2[sin x – x cos a] + C
x x


Hint: cos 2x − cos 2α
cos x cos x Hint:

14. 2 cos x + log | cosec x – cot x | + C. cos x + cos α







15. 2x – tan x + C (2 cos 2 x − 1) − (2 cos 2 α − 1)
=


16. –[2x + cot x] + C cos x + cos α


1 log cosec 2x − cot 2x + C

17. = 2(cos x – cos a)
4



18. log | sec x + tan x | – sin x + C 8. 2[cos x + x sin a] + C;





− 2 sin 2 x − sin 2 α 


Hint:
sin 2 x = 1 − cos 2 x = sec − cos
x x Hint: cos 2x − cos 2α =
cos x cos x sin x + sin α sin x + sin α


1 log sec x + C = –2[sin x – sin a]
19.

2 Now integrate.


sin 2 x = sin 2 x − 1 tan x

Hint: 9. 2 cos(x – a) + C;
sin 2x 2 sin x cos x 2



20. log | cosec x – cot x | + cos x + C Hint: cos 2x − cos 2α = −2 sin (x + α) sin (x − α)
sin (x + α) sin (x + α)






cos 2 x = 1 − sin 2 x = cosec − sin = –2sin (x – a)
Hint: x x
sin x sin x


Now integrate.
21. 1 log sin x + C

10. 2 sin (x + ) + C;
2




α
Hint: cos 2x + cos 2α = 2 cos (x + α) cos (x − α)
2 2
Hint: cos x = cos x = 1 cot x cos (x − α) cos (x − α)

sin 2x 2 sin x cos x 2

1 sin 4x  = 2 cos (x + a)
III. 1.  x − +C

8 4  Now integrate.


11. 2[sin x + x cos a] + C
Hint: sin 2 x cos 2 x = 1 (sin 2 2x) = 1 [1 − cos 4x]


4 8 cos 2x + cos 2α

Hint:
Now integrate. cos x − sin α


ntegrals 187
I

E:\AMIT_WORKS\Exam_Guru\EG_Mathematics-12_(working_02-06-2022)\EG_Mathematics-12_working\Open_Files\Chap_7\Chap_7
\ 19-Aug-2022  Amit   Proof 5 Reader’s Sign _______________________ Date __________


=
(2 cos 2
) (
x − 1 + 1 − 2 sin 2 α ) =
1  sin {(x + a) − (x + b)} 
 sin (x + a) sin (x + b) 
cos x − sin α sin (a − b)  



=
(
2 cos 2 x − sin 2 α )  sin ( x + a ) cos ( x + b ) −  
 
cos x − sin α 1   cos ( x + a ) sin ( x + b )  


=
= 2(cos x + sin a) sin ( a − b )  sin ( x + a ) sin ( x + b ) 
 


Now integrate.  


12. 2[cos x + x cos a] + C 1
= cot ( x + b ) − cot ( x + a )


Hint: See solution of Q.No.11 sin ( a − b ) 


cos 5x cos x 
13. Ans. −  + + C; Now integrate.
 10 2 




 1   sin (x + a) 
1 4.   log  cos (x + b)  + C
Hint: sin 3x cos 2x = [sin 5x + sin x ]  cos (a − b )   



2

14.
sin 5x sin 9x
− +C; Hint:  1 
 sin (x + a) cos (x + b) 


5 2 9 2


Hint: (sin 7x) 1 − cos 4x = 2 sin 7x sin 2x 1  cos {(x + a) − (x + b)} 
=

cos (a − b)  sin (x + a)cos (x + b) 
= 1 [ cos 5x − cos 9x ]

2
 cos ( x + a ) cos ( x + b ) + 

Now integrate.  
1  sin ( x + a ) sin ( x + b ) 

=
15. sin 2x − 1 log sec 2x + tan 2x + C cos ( a − b )  sin ( x + a ) cos ( x + b ) 
2  


 
cos 4x = 2 cos 2 2x − 1 = 2 cos 2 − sec 2
Hint: x x

cos 2x cos 2x

 1  cos (x − b)
16. 4 sin x – 3 log | sec x + tan x | + C 5. 
sin (b − a )  log cos (x − a) + C
 







3
Hint: cos 23x = 4 cos x −2 3 cos x 1  sin {(x − a) − (x − b)} 
cos x cos x Hint:

sin (b − a)  cos (x − a) cos (x − b) 
 

= 4 cos x – 3 sec x


17. x + sin 2x + C. 1 log sin (x − a) + C
6.


3 cos a cos x


Hint: sin 3x = 3 sin x − 4 sin x
sin x sin x

4 (1 − cos 2x ) Hint :  1 
= 3 – 4 sin2 x = 3 −  sin (x − a) cos x 

 
­
­
2


= 3 – 2 + 2 cos 2x = 2 cos 2x + 1    cos {x − (x − a)} 
= 1 


18. 4x – 3 tan x + C  cos a   sin (x − a) cos x 
 


1 12x − 8 sin 2x + sin 4x + C

IV. 1.
32
[ ] sin (x − a)
1

7. log +C
Hint : sin 4 x = 1 [3 − 4 cos 2x + cos 4x ] sin (a + b) sin (x + b)


8
sin 5x sin 9x sin x sin 3x

Now integrate. 8. − + − +C
20 36 4 12



1 3x + 2 sin 6x + sin 12x  + C
2. Hint: sin 4x cos 3x sin 2x
8  3 12 



= 1 [cos 5x − cos 9x + cos x − cos 3x]
Hint: cos 4 3x = 1 [3 + 4 cos 6x + cos 12x ] 4
8


Now integrate. 1 sin ( x − b )
9. log +C

sin ( a + b ) sin ( x + a )


 1   sin (x + b) 
3.   log  sin (x + a)  + C ;
 sin (a − b)   


 −1  log cos ( x + b ) + C
1 10.  
Hint:  (a + b) 
sin cos ( x − a )


sin (x + a) sin (x + b)

188 Mathe atics–12
m
E:\AMIT_WORKS\Exam_Guru\EG_Mathematics-12_(working_02-06-2022)\EG_Mathematics-12_working\Open_Files\Chap_7\Chap_7
\ 17-Aug-2022  Amit   Proof-5 Reader’s Sign _______________________ Date __________


20. tan x + 2 sin 2x – 4x + C
1 sin ( x + a )



11. log +C
cos ( a + b ) cos ( x − b ) cos 4x = 2 cos 2 2x − 1


Hint:
cos 2 x cos 2 x


1  sin ( x + b ) 
12. log  
2
cos ( a + b )  cos ( x − a )  2  2 cos 2 x − 1 − 1


=
cos 2 x



13. 3x − sin 4x + sin 8x + C
128 128 1024 2  4 cos 4 x − 4 cos 2 x + 1 − 1


1 = 
Hint: sin4 x cos4 x = sin4 2x cos 2 x



16

4 2
= 1 [3 − 4 cos 4x + cos 8x ] = 8 cos x − 82cos x + 1
cos x



128

14. ( ) ( )
−1 log cosec π − x − cot π − x 
 + C
= 8 cos2 x – 8 + sin2 x



2  4 4 1 + cos 2x  8
= 8


− + sec 2 x
 2 



Hint: 1 1
=
1 − sin 2x
(
) = 4 + 4 cos 2x –8 + sec2 x

1 − cos π − 2x



2 = sec2 x + 4 cos 2x –4
= 1 cosec ( π − x )



1
4 21. − log sec x + tan x + 2 sin x + C
2 2




1 log 1 + sin 2x + C
15. sin 3x 3
2 Hint: = 3 sin x − 4 sin x


sin 2x 2 sin x cos x



1 − sin 2x = 1 − sin 2x
Hint:  2 
1 + sin 2x cos 2x = 3 sec x − 2  1 − cos x 

2  cos x 


= sec 2x – tan 2x


( )
x x π+x 3
16. 2 sin − 2 cos − 2 log cosec = sec x − 2 sec x + 2 cos x
2


2 2 4 2


sin x = (1 + sin x ) − 1 = − 1 sec x + 2 cos x
Hint: 2


1 + sin x 1 + sin x

1 log cosec x − cot x + 2 cos x + C
1 22.
= 1 + sin x − 2


1 + sin x


cos 3x 4 cos3 x − 3 cos x
Hint: =
= cos x + sin x − 1 sin 2x 2 sin x cos x



2 2
( )


1 − cos π + x 2 
2 
= 2  1 − sin x  − 3 cosec x
 sin x  2
= cos x + sin x − 1 cosec ( π + x )


2 2 2 4 2 3
= 2 cosec x − 2 sin x − cosec x


2


17. − 1 log sec x + tan x − cosec x  + C
4 = 1 cosec x − 2 sin x


2


2 2
= sin x + cos x
Hint: 1
cos 3x − cos x −2 sin 2x sin x 23. 2 sin 2x + tan x – 4x + C



cos 4x 4 2
18.
1 log sec 3x − 1 log sec 2x
– log | sec x | + C Hint: = 8 cos x − 82cos x + 1
3 2 cos 2 x cos x








= 8 cos2 x –8 + sec2 x
Hint: tan 3x = tan(2x + x) = tan 2x + tan x


1 − tan 2x tan x = 4(1 + cos 2x) – 8 + sec2 x



⇒ tan 3x tan 2x tan x = tan 3x – tan 2x – tan x. = 4 cos 2x + sec2 x – 4



19. 1 log sec 3x + C 24. –cot x – 2 sin 2x – 4x + C
3




cos 4x 8 sin 4 x − 8 sin 2 x + 1
Hint: sin 5x + sin 3x + sin x Hint: 2
sin x
=
sin 2 x
cos 5x + cos 3x + cos x




(sin 5x + sin x ) + sin 3x = 4(1 – cos 2x) – 8 + cosec2 x
=


(cos 5x + cos x ) + cos 3x = –4 cos 2x + cosec2 x – 4




ntegrals 189
I

E:\AMIT_WORKS\Exam_Guru\EG_Mathematics-12_(working_02-06-2022)\EG_Mathematics-12_working\Open_Files\Chap_7\Chap_7
\ 17-Aug-2022  Amit   Proof 5 Reader’s Sign _______________________ Date __________


Topic 3. Integration by Substitution
x9 dx
Example 1. ∫ (4 x 2 + 1)6 dx is equal to Example 3. Evaluate: ∫x+ x

dx dx
1  1
−5
1 1
−5
Solution. I= ∫x+ x
=∫
(
x x +1 )
(a)  4 + 2  +C (b)  4 + 2  +C




5x  x 5 x




 1  dx
(c)
1
(1 + 4)− 5 + C (d)
1 1 
+ 4
−5
+C
⇒ I= ∫  x + 1 x
.





10 x 10  x 2 




Substitute: x = z dx ⇒
= 2dz
x 9 x9 x





Solution. Let I = ∫ (4 x 2 + 1)6 dx = ∫  1
6
dx
1  dx  dz

x12  4 + 2 
 x 
\ I= ∫( x +1

x) = 2∫ (z + 1)




1
= ∫ 6
dx

 1
x3  4 + 2  = 2log|z+1|+C⇒ I = 2 log x + 1 + C.
 x 



 1 −2 dx 1 Integration of Trigonometric Functions by
Put  4 + 2  = t  3 dx = dt  3 = − dt
 x  x x 2


Substitution
1 dt 1 1
\ I = − ∫ 6 = − × − t −5 + C
2 t 2 5 Example 4. Evaluate:
( ) ∫
sin 3x
dx


−5 cos x
1 −5 1  1
= t +C=  4 + 2  +C
∫( ) ∫
10 10  x sin 3x 3 sin x − 4 sin 3 x

Solution. I = dx = dx
cos x cos x



x3
Example 2. If ∫ 1 + x2
dx = a (1 + x 2 )3/ 2 + b 1 + x 2 + C, then
 sin 3 x 
⇒ I = 3∫ tan x dx − 4∫  dx
 cos x 



1 1
(a) a = , b = 1 (b) a = − , b = 1
3 3
(1 − cos x) sin x dx




2

1 1 ⇒ I = 3 log sec x − 4∫
(c) a = − , b = − 1 (d) a = , b = − 1 cos x



3 3




⇒ I = 3 log | sec x | – 4I1



x3



Solution. Let I = ∫ 1 + x2
dx
 1 − cos 2 x 

dt
⇒ I1 = ∫  cos x  sin x dx



Put 1 + x2 = t  2x dx = dt  x dx =
2

1 (t − 1) Put cos x = z ⇒ sin x dx = – dz  
2∫
\ I= dt





t


 1 − z2   z 2 − 1
I1 = ∫   ( −dz ) = ∫  dz
1 t
= ∫
1 1
dt − ∫ dt z   z 


2 t 2 t

=
1
2∫
1
t dt − ∫ t −1/ 2 dt
I1 = ∫ ( )z−
1
z
dz =
z2
2
− log z + C


2

1 2 3/ 2 1 cos 2 x
= × (t ) − 2 t + C ⇒ I1 = − log cos x + C
2 3 2 2




1 I = 3 log|sec x| – 4I1
= (1 + x 2 )3/ 2 − 1 + x 2 + C


3

But I = a (1 + x 2 )3/ 2 + b 1 + x 2 + C  cos 2 x 
⇒ I = 3 log sec x − 4  − log cos x  + C

 2 



1
Comparing the like terms we get, a = and b = – 1
3

Hence, the correct option is (d).
⇒ I = 3 log |sec x| – 2 cos2 x + 4 log|cos x| + C




190 Mathe atics–12
m
E:\AMIT_WORKS\Exam_Guru\EG_Mathematics-12_(working_02-06-2022)\EG_Mathematics-12_working\Open_Files\Chap_7\Chap_7
\ 17-Aug-2022  Amit   Proof-5 Reader’s Sign _______________________ Date __________


EXERCISE 7.3
I. Multiple Choice Questions (MCQs) III. Short Answer Type Questions-I
10 x 9 + 10 x log e 10
(sin x)
−1 2
1. ∫ dx = dx
10 x + x10 1. ∫ 2. ∫ dx


sin x cos3 x 1 − x2





1 1
(a) − (b) log (10x + x10) + C
x
2 (10 + x10 ) 2
+C
 tan −1 x   sin −1 x 2 
( )




3. ∫  e 2  dx
4. ∫   dx
1 1  2 x 1− x 
 1+ x 





(c) x
+C (d) None of these [NCERT]  
2 (10 + x10 ) 2





(3x + 4) dx
5. ∫ 6. ∫ x x + 2 dx
1 − cos 2 x 2x + 3





2. ∫ tan
−1
dx =
1 + cos 2 x


dx  e 2x − 1
(a) 2x + C
2
(b) x + C 2 7. ∫ x + x log x
8. ∫  e2x + 1 dx









2
x  tan x   (x + 1) (x + log x) 2 
(c) 2x + C (d) +C
2 9. ∫  sin x cos x  dx 10. ∫   dx




x 





e x (1 + x )
3. ∫ cos dx is equal to 10x9 + 10 x log 10   (x + 1) e x
( e x x) 11. ∫   dx 12. ∫  2 x  dx
2


(x10 + 10 x)   cos (xe ) 





(a) –cot (ex x) + C (b) tan (xex) + C
 xe − 1 + e x − 1 
∫ (sin )




3
(c) tan (ex) + C (d) cot (ex) + C 13. x cos x dx 14. ∫  e x  dx
 x +e 








sin x
4. The value of
2∫
 π
sin  x − 
dx is
15. ∫ ( cos 3x
sin x )
dx 16. ∫  cos23x  dx
 sin x 






 4
 sin 3x   sin 3x 
 π
(a) x − log cos  x −  + C
17. ∫  cos2 x  dx 18. ∫  cos3 x  dx




 4


 sin 3x   sin 4x 
 π
19. ∫  cos4 x  dx 20. ∫  cos2 x  dx




(b) x + log cos  x −  + C
 4  sin 4x   sin 4x 


21. ∫  cos3 x  dx 22. ∫  sin 4 x  dx




 π
(c) x − log sin  x −  + C
 4  sin 4x   sin 4x 


23. ∫  sin 4 x  dx 24. ∫  cos4 x  dx




 π
(d) x + log sin  x −  + C
 4 IV. Short Answer Type Questions-II


log ( x + 1 ) − log x  sin x dx
5. ∫ dx is equal to 1. ∫  x 
dx 2. ∫ sin x cos3 x




x ( x + 1)


{
 x + cos −1 3x 2 
}
2
1   x + 1   x sin −1 x 2  ( ) 
(a) −  log   +C 3. ∫   dx 4. ∫  dx
2   x    


 1 − x 4  2




 1 − 9 x 
(b) C – [{log (x + 1)}2 – (log x)2]
( cos 4x
) cos 4x
( )


5. ∫ cos x
dx [HOTS] 6. ∫
sin x
dx [HOTS]





  x + 1 
(c) − log  log  +C
 x    cos 4x   cos 4x 
∫  cos2 x  dx ∫  cos4 x  dx


 7. [HOTS] 8. [HOTS]





x + 1
(d) − log  +C
 x   cos 4x 
9. ∫  sin 4 x  dx [HOTS]





Evaluate each of the following Integrals
V. Long Answer Type Questions
II. Very Short Answer Type Questions
 1 
   1 + sin 2x  dx x2 
1. ∫  1+ cot x  dx 2. ∫  
1. Evaluate: ∫ 1 1 dx
x + log ( sin x )  x − cos 2 x   2 






   x − x3 
1
 ( cos 2x )  x2
3. ∫  2  dx 4. ∫ sec
4
x tan x dx 2. Evaluate: ∫ 3 dx
 (sin x + cos x ) 






x4 +1

ntegrals 191
I

E:\AMIT_WORKS\Exam_Guru\EG_Mathematics-12_(working_02-06-2022)\EG_Mathematics-12_working\Open_Files\Chap_7\Chap_7
\ 17-Aug-2022  Amit   Proof 5 Reader’s Sign _______________________ Date __________


Answers 7.3
( )
2. 1 sin −1 x + C. Hint: Put sin–1 x = q.
10 x + 10 log e 10 9 x 3
I. 1. (b)
10 x + x10
dx I= ∫ 3






Put 10x + x10 = t, then (10x loge 10 + 10x9) dx = dt −1
3. e tan x + C . Hint: Put tan–1 x = q.




dt
(
4. 1 sin −1 x + C . Hint: Put sin −1 x = θ .)
3
\ I = ∫ = log | t | + C = log | 10 x + x10 | + C
t 3





1 − cos 2 x 2 sin 2 x  3 
2. (d) I = ∫ tan −1 dx = ∫ tan −1 dx 5. 1 (2x + 3) 2 − 2x + 3] + C
1 + cos 2 x 2 cos 2 x




2




x2
= ∫ tan −1 tan x dx = ∫ x dx = +C  x  + 4 1 
2 Hint: 3x + 4 = 3 


2x + 3  2x + 3   
2x + 3 




e x (1 + x )
3. (b) I= 2
( e x x)
dx ∫ cos 3  ( 2x + 3) − 3  − 4  1 




=  
Put exx = t, then {(ex)x + ex (1)} dx = dt 2  2x + 3  2x + 3 




ex (x + 1)dx = dt ⇒ = 3 2x + 3 − 9
+ 4



2 2 2x + 3 2x + 3


dt
I= ∫
cos 2 t ∫
\ = sec 2 t dt = tant + C = tan (xex) + C
3 x+ − 1



= 2 3
π 2 2 2x + 3


4. (d) Put x − = t, so that dx = dt
4



Now integrate.
π 
sin  + t  dt 5 3
4  6. 2 (x + 2) 2 − 4 (x + 2) 2 + C
\ I = 2∫ 5 3


sin t



π π Hint: substitute: x + 2 = z
sin cos t + cos sin t

4 4 7. log |1 + log x| + C; Hint: Substitute: log x = z
= 2∫ dt


sin t 8. log | ex + e–x | + C







 1 1  Hint: Divide Nr and Dr by ex and substitute:
= 2∫  cot t + dt = ∫ (cot t + 1) dt

 2 2  ex + e–x = z




= log |sin t| + t + C 9. 2 tan x + C .




 π  π Hint: Divide Nr and Dr by cos2x.
= log sin  x −  +  x −  + C

 4   4 1 (x + log x)3 + C;
10.


3


 π
= x + log sin  x −  + C Hint: Substitute: (x + log x) = z
 4



11. log | (x)10 + (10)x | + C;
log ( x + 1 ) − log x






5. (a) I= dx Hint: Substitute: (x10 + 10x) = z
x ( x + 1)





12. tan (xex) + C; Hint: Substitute: xex = z


 1 1 −2 3 7
Put log (x + 1) – log x = t, then  − dx = dt 13. cos 2 x + 2 cos 2 x + C;
 x + 1 x 



3 7


dx dx Hint: Write:


⇒ = dt ⇒ = – dt
x ( x + 1) x ( x + 1) sin 3 x cos x = (1 − cos 2 x) cos x sin x




1 and substitute: cos x = z
\ I = ∫ t ( − dt ) = − t 2 + C

2 14. 1 log x e + e x + C; Hint: Substitute: xe + ex = z



e


2
1   x + 1 
= − log    +C 15. [log | sin x | – 2 sin2 x] + C.
2   x 







cos 3x 3
II. 1. log | x + log (sin x) | Hint: = 4 cos x − 3 cos x




sin x sin x




2. log | x – cos2 x |



 1 − 4 sin 2 x 


3. log | sin x + cos x | + C =   cos x
sin x 







1 Put sin x = z
4. sec4 x + C

4 16. –[cosec x + 4 sin x] + C


III. 1. log | tan x | + 1 tan2 x + C


17. –[sec x + 4 cos x] + C



2
( )


1 sec4 x 1 + tan 2 x sec 2 x sin 3x 3 sin x − 4 sin 3 x
Hint: =
Hint: 3 = tan x = 2
cos x cos 2 x
sin x cos x tan x




192 Mathe atics–12
m
E:\AMIT_WORKS\Exam_Guru\EG_Mathematics-12_(working_02-06-2022)\EG_Mathematics-12_working\Open_Files\Chap_7\Chap_7
\ 17-Aug-2022  Amit   Proof-5 Reader’s Sign _______________________ Date __________


=
(4 cos 2
x −1 ) sin x 5.  − 8 sin 3 x + log
 3
sec x + tan x  + C




2
cos x


Put cos x = z cos 4x 8 cos x − 8 cos 2 x + 1
4
Hint: =
cos x cos x




18.  − 1 sec2 x − 4 log cos x  + C = 8(cos2 x – 1)cos x + sec x
 2 




= –8 sin2 x cos x + sec x
19.  − 1 sec3 x + 4 sec x  + C



 3  6.  8 cos3 x + log cosec x − cot x  + C


 3 



20. [4 log |cos x| – 4 cos2 x] + C
cos 4x 8 sin 4 x − 8 sin 2 x + 1


sin 4x  4 sin x cos x cos 2x  Hint: =
Hint: =  sin x sin x




cos 2 x  cos 2 x 



= 8 sin3 x –8 sin x + cosec x



 2 cos x − 1 2
= 8(sin2 x – 1) sin x + cosec x
= 4  sin x
 cos x 





= –8 cos2 x sin x + cosec x



Put cos x = z. 7. [–4x + 2 sin 2x + tan x] + C



21. [8 cos x + 4 sec x] + C.
cos 4x 8 cos 4 x − 8 cos 2 x + 1


22. –[4 cosec x + 8 sin x] + C Hint: 2 =
cos x cos 2 x






sin 4x 4 cos 2x cos x
Hint: = = 8 cos2 x – 8 + sec2 x
sin 3 x sin 2 x





2 
= –8 sin2 x + sec2 x
 x cos x
= 4 1 − 2 sin


 1 − cos 2x  + sec2 x
2
 sin x  = −8 


 2 


Put sin x = z.
8. 8x − 7 tan x + 1 tan 3 x  + C

23. –[ 2 cosec2 x + 8 log |sin x | ] + C
 



3




24. [–2 sec2 x – 8 log | cos x |] + C


8 cos 4 x − 8 cos 2 x + 1


sin 4x  cos 2x sin x  cos 4x
Hint: = 4 Hint: =
4
cos x cos 4 x
 cos3 x 



cos 4 x



2 = 8 – 8 sec2 x + sec4 x
 
= 4  2 cos 3x − 1 sin x


= 8 – 7 sec2 x – sec2 x + sec4 x
 cos x 




Put cos x = z. = 8 – 7 sec2 x + sec2 x (–1 + sec2 x)



IV. 1. − 2 cos x + C; = 8 – 7 sec2 x + sec2 x tan2 x


Hint: Substitute: x=z 9. 8x − 1 cot 3 x + 7 cot x  + C
 

3


2
tan x + log | tan x | + C
2. ;
2 cos 4x 8 sin 4 x − 8 sin 2 x + 1


Hint: Divide Nr and Dr of 1 4 Hint: =
3 by cos x sin 4 x
4
sin x



sin x cos x

It gives: = 8 – 8 cosec2 x + cosec4 x

2 2


1 sec4 x (1 + tan x) sec x = 8 + (cot2 x – 7)cosec2 x
3 = tan x = tan x
sin x cos x


Put cot x = z.


Then, substitute tan x = z 5 2 1 1 1 1
V. 1. x + 6 x 6 + 3 x 3 + 2x 2 + 3x 3 + 6x 6 + 6 log x 6 − 1 + C;

3. 1 (sin − 1 x 2) 2 + C; 5 2
4


Hint: LCM of 2, 2, 3 is 6. Substitute x = z6.
Hint: Substitute: sin–1 x2 = z


(
2. 4  x3/ 4 − log x3/ 4 + 1  . )

4. −2 1 − 9x 2 + 1 cos −1 3x + C ( )
3
3 


9 9


Topic 4. Integration Using Standard Formulae
sin x dt 1 dt
Example 1. ∫ 3 + 4 cos2 x dx = . \ I= −∫
3 + 4t 2
= − ∫3
4 + t2


sin x 4
1 dt
Solution. Let I = ∫ 3 + 4 cos2 x dx =− ∫
4  3 2

    
Put cos x = t   + t
2
2

\ – sin x dx = dt  sin x dx = – dt




ntegrals 193
I

E:\AMIT_WORKS\Exam_Guru\EG_Mathematics-12_(working_02-06-2022)\EG_Mathematics-12_working\Open_Files\Chap_7\Chap_7
\ 17-Aug-2022  Amit   Proof 5 Reader’s Sign _______________________ Date __________


1 1  t 
= − × tan − 1  +C  1  1 a+ x
4 3 2  3 2  (v) ∫  a 2 − x 2  dx = 2a log +C

a−x



1  2t 
= − tan − 1   + C
2 3  3

dx
= −
1 −1 2 cos x 
Example 6. Evaluate: ∫ 9 − x2
tan   +C
2 3 3 

Hence, dx dx
Solution. ∫ = ∫ (3)2 − (x)2

1 −1 2  9 − x2



I= − tan  cos x + C .
2 3 3 

f ′ ( x ) dx 1 3+ x 1 3+ x
= log + C = log +C
(i) ∫ = log f ( x ) + C 2×3 3− x 6 3− x
f ( x)






Example 2. Evaluate:
 3 + cos x 
∫  3x + sin x  dx
(vi)
dx
∫ a2 + x2 =
1
a
tan −1
x
a
+C ()



(3 + cos x) dx (3x + sin x )′ dx
Solution. I= ∫
3x + sin x
=∫
(3x + sin x) dx
∫ 9 + x2


Example 7. Evaluate:
⇒ I = log| 3x + sin x | + C





(ii) ∫
f ′ ( x ) dx
f ( x)
= 2 f ( x) + C Solution. I=
dx dx
∫ 9 + x 2 = ∫ (3)2 + x 2 = 3 tan
1 −1
()
x
3
+C





dx
(14x + e ) dx x (vii) ∫ 2
x +a 2
= log x + x2 + a2 + C



Example 3. Evaluate:
7x 2 + e x

Solution.
(14x + e ) dx = (7x x 2
)′
+ e x dx
Example 8. Evaluate: ∫
dx
I=
∫ ∫ x2 + 9


2 x 2 x
7x + e 7x + e

⇒ I = 2 7x 2 + e x + C dx
Solution. I= ∫ = log x + x2 + 9 + C



2


x +9
n +1
 f ( x )
f ′ ( x ) dx = 
n
(iii) ∫  f ( x ) n +1
+ C , n ≠ −1
(viii) ∫
dx
= log x + x2 − a2 + C


x − a2
2


∫ (x ) ( x ) dx
3 5 2
Example 4. Evaluate: +4
dx
1 Example 9. Evaluate: ∫
∫ (x + 4) (x ) dx = 3∫
3 5 2
Solution. I= (x3 + 4)5 (3x 2) dx x2 − 9


dx
1
( ) ( x + 4)′ dx = 181 ( x + 4) + C
5 6
⇒ I=
3∫
x3 + 4 3 3 Solution. I= ∫ 2
x −9
= log x + x2 − 9 + C





(iv)
 1 
∫  x 2 − a 2  dx = 2a log
1 x−a
x+a
+C (ix) ∫
dx
2
a −x 2
= sin −1 () x
a
+C




 1 
Example 5. Evaluate: ∫  x 2 − 5  dx Example 10. Evaluate: ∫
dx
9 − x2
 
 1  1
Solution. I = ∫ 2 dx = ∫  
2 dx
dx dx

 x − 5 x − 5
2
( )  Solution. I= ∫ =∫


2
  9− x (3) 2 − x 2


()
1 x− 5
= log +C x
2 5 x+ 5 =sin −1 +C


3


194 Mathe atics–12
m
E:\AMIT_WORKS\Exam_Guru\EG_Mathematics-12_(working_02-06-2022)\EG_Mathematics-12_working\Open_Files\Chap_7\Chap_7
\ 17-Aug-2022  Amit   Proof-5 Reader’s Sign _______________________ Date __________


EXERCISE 7.4
I. Multiple Choice Questions (MCQs) 1  1 
dx (c) tan −1  cot 2 x + C
1. ∫ 2 equals 2  2 



x + 2x + 2


(d) None of these
(a) x tan–1 (x + 1) + C (b) tan–1 (x + 1) + C







(c) (x + 1)tan–1 x + C (d) tan–1 x + C [NCERT]





dx Evaluate each of the following Integrals
2. ∫ equals
9x − 4x2 II. Very Short Answer Type Questions


1  9x − 8  1 − 1  8x − 9  dx (x 4 + 5)dx
(a) sin −1   + C (b) sin   +C
9  8 2 9 
1. ∫ 2. ∫ x2 − 5




x (x − 9)





(c)
1  9x − 8 1  9x − 8  sec 2 x e x dx
sin − 1  + C (d) sin −1   +C
3  8  2  9  3. ∫ 7 − 3 tan 2 x dx 4. ∫ 9 − 16e2x








[NCERT]
x x dx sec x tan x dx

3. ∫ 2
sin x
dx equals
5. ∫ 7x5 − 9 6. ∫ 8 + 3 sec 2 x





a + b 2 cos 2 x


dx 3x dx
(a) log (a + b cos x) + C (b)
2 1  a cos x 
tan −1 
2 2
+C
7. ∫ e x + e−x 8.
∫ 5 + 32x




ab  b 




III. Short Answer Type Questions-I
1  b cos x  1  a cos x
(c) cot −1   + C (d) cot −1   + C
ab  a  ab  b x3 + x dx
∫ ∫




1. dx 2.
x4 − 4 x. (3x + 4)




sin x
4. ∫ 3 + 4 cos
x
dx equals 2
7 x dx dx


(a) log (3 + 4 cos2 x) + C
3. ∫ 3 + 2 × (49) x 4. ∫3 x + 5x5 / 2






−1  cos x  dx
(b)
2 3
tan −1 
 3 
+C 5. ∫ x 2 + 4x + 8 [Delhi 2017]





−1  2 cos x  (5 sin 2x − cos x ) dx
(c) tan −1 
 3 
 +C
6. ∫ 9 sin 2 x + 4


2 3


1  2 cos x  x dx
(d) tan −1   +C
7. ∫ 5x3 − 3


2 3  3 


(sin 2x) dx dx
dx 8. ∫ 9 − 16 cos4 x 9. ∫
5. ∫ sin 4 x + cos4 x is equal to [CBSE 2014] x 5x + 7







x
(a)
1  1
tan −1 

tan 2 x + C
10. ∫ a − x3
3
dx


2  2 


(b)
 1  11. (x 3
+ x 2 + 1 dx )
2 tan −1 
 2
sin 2 x + C
 ∫ 2




x −1

Answers 7.4
dx dx dx dx
I. 1. (b) I= ∫x 2
+ 2x + 2
=∫ 2
( x + 2 x + 1) + 1
= ∫ ( x + 1) 2
+1
= ∫ 81  2 81 





−  4 x − 9 x + 
Put x + 1 = t, then dx = dt 16 16
dx


dt 1 t = ∫ 2 2
I = ∫ 2 2 = tan − 1   + C = tan −1 (t ) + C  9  9


\
t +1 1  1   −  2 x − 



4 4
9 1
Hence I = tan–1 (x + 1) + C Put 2x − = t, then 2dx = dt ⇒ dx = dt
4 2



1 dt



dx dx I= ∫
∫ =∫ 2 2


2. (b) I=  9 2
  − t
2
9x − 4x − ( 4x2 − 9x )




4

ntegrals 195
I

E:\AMIT_WORKS\Exam_Guru\EG_Mathematics-12_(working_02-06-2022)\EG_Mathematics-12_working\Open_Files\Chap_7\Chap_7
\ 17-Aug-2022  Amit   Proof 5 Reader’s Sign _______________________ Date __________


  x −3
 Form ∫
dx
= sin − 1
x
+ C II. 1. 1 log + C;
 2
a −x 2 a  3 x +3

Hint: Put x=z
 


1 −1  t  1  4t  3
\ I = sin  + C = sin − 1   + C 2. x + 5x + 3 5 log x − 5 + C;
2 9  2  9  3 x+ 5





 
4  4 4
 9 Hint: x + 5 = (x − 25) + 30
x2 − 5 x2 − 5


4 2x − 
1 − 1  4
= sin +C = x + 5 + 230
2
2 9 x −5



1  8x − 9  1 log 7 + 3 tan x + C
sin − 1 
= +C 3.
2  9  2 21 7 − 3 tan x





sin x 1 log 3 − 4e x + C
3. (c) I= ∫ 2 dx 4.
a + b 2 cos 2 x 24 3 + 4e x







1
Put b cos x = t, then – b sin x dx = dt ⇒ sin x dx = − dt 1 log 7 x5/ 2 − 3 + C
b 5.

1 15 7 7 x5/ 2 + 3




1 − dt
I = ∫ 2 b 2 dt = ∫ 2 2 x x dx x3/ 2 dx
a +t b a +t Hint: =


7x5 − 9 7 ( x )5 − 9


1 t 1  b cos x 
= cot −1   = cot −1
  +C Now put x5/2 = z.
ab  a  ab a 



sin x 6. 1 tan −1  3 sec x  + C
 
4. (c) I = ∫ 3 + 4 cos dx 24 8 


2
x




Put cos x = t, so that sin x dx = –dt Hint: Put sec x = z.


− dt −1 dt 7. tan–1(ex) + C
\ I= ∫
4 ∫
=


2
3 + 4t 2  3 1 ex



−x =
2
t +  x
 2  e +e 1 + e2x


  Now put ex = z.
−1  t  1 2t

= tan −1   +C = − tan −1 +C  −1  3x  
3 3 2 3 3 8. 1



4×   tan    + C
 2  5 (log 3) 5 


2 
−1  2 cos x  Hint: Put 3x = z.
= tan −1   +C

2 3  3 


2
III. 1. 1 log x 4 − 4 + 1 log x − 2 + C;
4 8 2
dx x +2
5. (a) I = ∫ sin 4
x + cos 4 x




2. 1 tan −1  3x 
dx  2  + C
= ∫ 3


(sin 2 x + cos 2 x) 2 − 2 sin 2 x cos 2 x


Hint: Put x = z.
2dx

= ∫ 2 − (2 sinx cos x)2  x
tan −1  2 × 7  + C
1


3.
6 ( log 7 )  3 


2 2 sec 2 2 x
⇒ I= ∫ 2 − sin 2
2x
dx = ∫
2 sec 2 2 x − tan 2 2 x
dx
2 tan −1  5 x 



4.   +C
2 sec 2 x 2 2
2 sec 2 x 15 3


= ∫ 2 + 2 tan dx = ∫ 2 + tan dx

( )
2
2 x − tan 2 2 x 2
2x


1 tan −1 x + 2 + C
5.
Put tan 2x = 3t, so that 2 sec2 2x dx = dt 2 2



( )
dt dt 5 log sin 2 x + − 1 tan −1 3 sin x + C
⇒ I= ∫
2 + t2 ∫ 2 2 + t2
= 6. 9 4
( ) 9 6 2





1  t  1  1  Hint:
5 sin 2x − cos x = 5 sin 2x − cos x
= tan −1   + C = tan −1  tan 2 x + C
2  2 2  2  9 sin 2 x + 4 9 sin 2 x + 4 9 sin 2 x + 4



196 Mathe atics–12
m
E:\AMIT_WORKS\Exam_Guru\EG_Mathematics-12_(working_02-06-2022)\EG_Mathematics-12_working\Open_Files\Chap_7\Chap_7
\ 17-Aug-2022  Amit   Proof-5 Reader’s Sign _______________________ Date __________


3
9. 2 log
1 log 5x 2 − 3 + C 5x + 5x + 7 + C
7. 5



3
3 15


5x 2 + 3 Hint: Put x = z.


()
Hint: Put x3/2 = z. 3/ 2
2 sin −1 x
10. +C

3 a



2
8. 1 log 3 + 4 cos x + C
24 3 − 4 cos 2 x Hint: Put x3/2 = z.



x2 x 1 x
Hint: Substitute cos2 x = z. 11. + + log x 2 − 1 + log − 1 + C
2 2 x +1



Topic 5. Directly using Formulae & Special Types of Integrals
( ) 1  x + 1
 2
 2 
How to change ax2 + bx + c into a  x + b −  b − 42ac   ? = tan −1 
 2 
+C
2a  4a   2




We have
()
ax2 + bx + c = a  x 2 + b x + c 
 a a 
Clearly, A is true and R is the correct explanation for A.
Hence, the correct option is (a)


( )  2
b2 c  dx
= a x+
b
− 2+  ∫ Example 1. Evaluate:
x 2 + 4x + 1
 2a 4a a


dx dx
∫ ∫ Solution. I= =
( ) ( 3)
 2
 b 2 − 4ac   x 2 + 4x + 1 2
= a x+ b



2
−  (x + 2) −
2a  4a 2  



b 1 (x + 2) − 3
= a [(x + B)2 – A], where B = ⇒ I= log +C
2a 2 3 (x + 2) + 3





 2 
and A =  b − 42ac  dx 1 A + (x ± B)
 4a  (ii) ∫ A 2 − (x ± B)2 = log +C



2A A − (x ± B)


Use the following formulae directly
dx
dx 1 (x ± B) − A
Example 2. Evaluate: ∫ 4 − 2x −x 2
(i) ∫ ( x ± B)2 − A 2 = 2a log (x ± B) + A
+C


dx dx
Solution. I= ∫ 4 − 2x − x 2 = ∫ − [x 2 + 2x − 4]


Assertion-Reasoning Type Questions
dx dx
⇒ I= ∫ − [(x + 1)2 − 5] = ∫
Direction: In the following questions, a statement of
( 5)
2
− (x + 1) 2



Assertion (A) is followed by a statement of Reason (R).
1 5 + (x + 1)
Mark the correct choice as: = log +C
2 5 5 − (x + 1)


(a) Both A and R are true and R is the correct explanation


of A. dx 1  x ± B + C
∫ (x ± B)2 + A 2 = A tan
−1
(iii)  
A 


(b) Both A and R are true but R is NOT the correct explanation


of A.
dx
(c) A is true but R is false. Example 3. Evaluate: ∫ x 2 − 4x + 10


(d) A is false and R is True.
dx dx


1. Assertion (A): ∫ 2
dx
=
1  x + 1
tan −1  +C
Solution. I= ∫ x 2 − 4x + 10 = ∫ (x − 2)2 + 6


x + 2x + 3 2  2 
dx 1  x 1 tan −1  x − 2  + C
Reason (R): ∫ 2 = tan −1   + C ⇒ I=  
2 a  a 6 6 



x +a
dx 1 x −1 
Sol. ∫ x 2 + a 2 = a tan   + C
a (iv) ∫
dx
= log (x ± B) + (x ± B) 2 − A 2 + C

2 2
(x ± B) − A


This is a standard integral and hence R is true.

dx dx dx
∫ x2 + 2 x + 3 = ∫ Example 4. Evaluate: ∫
( x + 1) + ( 2 )
2 2 2
x − 6x + 1

ntegrals 197
I

E:\AMIT_WORKS\Exam_Guru\EG_Mathematics-12_(working_02-06-2022)\EG_Mathematics-12_working\Open_Files\Chap_7\Chap_7
\ 17-Aug-2022  Amit   Proof 5 Reader’s Sign _______________________ Date __________


dx dx ⇒ A = 7 ; Β = − 12
Solution. I= ∫ 2
=∫
2
⇒ 7x + 2 = A(2x + 4) +B
2







x − 6x + 1 (x − 3) − 8
 7 (2x + 4) − 12 
=log (x − 3) + (x − 3) 2 − 8 + C \ I = ∫  2 2  dx
x + 4x + 8 





 
⇒ I = log (x − 3) + x 2 − 6x + 1 + C
7 (2x + 4) dx dx
I = ∫ 2 − 12 ∫ 2



2 x + 4x + 8 x + 4x + 8



dx
(v) ∫ (x ± B) + A 2 2
= log (x ± B) + (x ± B) 2 + A 2 + C 7
= log x 2 + 4x + 8 − 6 tan − 1
x+2
+C ( )


2 2



dx
Example 5. Evaluate: ∫ (Linear polynomial in x) dx
2
x − 8x + 20 Type II: ∫ Quadratic polynomial in x


dx dx
Solution. I= ∫ =∫
2
(x − 4) 2 + 4 ( px + q )


x − 8x + 20
How to evaluate ∫ dx ?
⇒ I = log (x − 4) + (x − 4) 2 + 4 + C ax 2 + bx + c



( px + q )
I = log (x − 4) + x 2 − 8x + 20 + C We have I= dx ∫
ax 2 + bx + c





Split px + q as given below
dx x ± B
= sin −1 

(vi) ∫ 2  2A 
 +C

px + q = A(ax2 + bx + c)′ + B


A − (x ± B)


⇒ px + q = A(2ax + b) + B



dx Solve for A and B
Example 6. Evaluate: ∫

5 + 4x − x 2 A ( 2ax + b ) B
\ I= ∫ dx + ∫ dx + C
2 2



dx dx ax + bx + c ax + bx + c
Solution. I= ∫ 5 + 4x − x 2
=∫
2
− [x − 4x − 5]


(5x + 3) dx
dx x − 2
Example 8. Evaluate: ∫
⇒ I= ∫ = sin −1   +C x 2 + 8x + 20
9 − (x − 2) 2  3 



(5x + 3) dx
Solution. I= ∫ x 2 + 8x + 20


Some Special types of Integrals
Take: 5x + 3 = A(x2 + 8x + 20)′ + B
(Linear polynomial in x) dx



Type I. ∫ ⇒ 5x + 3 = A(2x + 8) + B ⇒ A = 5 ; B = −17
Quadratic polynomial in x

2




 5 (2x + 8) − 17 
 px + q 
How to evaluate ∫  ax 2 + bx + c  dx ? 
\ I= ∫2
2
 dx




 x + 8x + 20 
 px + q 
We have I= ∫  ax 2 + bx + c  dx 5
 (2x + 8)  dx


= ∫  dx –17 ∫ 2
Split px + q as given below 2  x + 8x + 20 
2 x + 8x + 20


 

px + q = A(ax2 + bx + c)′ + B


⇒ px + q = A(2ax + b) + B (x 2 + 8x + 20)′ dx dx
I= 5∫ − 17 ∫



Now find the values of A and B 2 2
(x + 4) 2 + 4


x + 8x + 20

 A 2ax + b 
( ) B f ′ ( x)
\ I= ∫  ( ax 2 + bx + c )  dx + ∫ ax 2 + bx + c dx Q∫ dx = 2 f ( x )
f ( x)





 
(7x + 2) dx I = 5 × 2 x 2 + 8x + 20
Example 7. Evaluate: ∫ x 2 + 4x + 8 ⇒
2



– 17 log (x + 4) + (x + 4) 2 + 4 + C
(7x + 2) dx

Solution. I= ∫ x 2 + 4x + 8


Take: ⇒ I = 5 x 2 + 8x + 20




7x + 2 = A(x2 + 4x + 8)′ + B –17 log (x + 4) + x 2 + 8x + 20 + C



198 Mathe atics–12
m
E:\AMIT_WORKS\Exam_Guru\EG_Mathematics-12_(working_02-06-2022)\EG_Mathematics-12_working\Open_Files\Chap_7\Chap_7
\ 17-Aug-2022  Amit   Proof-5 Reader’s Sign _______________________ Date __________


Integration of Trigonometric Functions Reducible 1 1 + 2z
⇒ I= log +C
px + q 2×2 1 − 2z




 
to the form: ∫  ax 2 + bx + c  dx ⇒ I=
1
log
1 + 2 cos x
+C
4 1 − 2 cos x




 sin 2 x 
Example 9. Evaluate: ∫  sin 3x  dx Example 10. Evaluate: ∫
sin x
dx
cos 2x
 sin 2 x  sin 2 x dx sin x sin x
Solution. I= ∫  sin 3x  dx = ∫ 3 sin x − 4 sin 3 x Solution. I= ∫ cos 2x
dx = ∫ dx
2 cos 2 x − 1





Put cos x = z ⇒ sin x dx = – dz
sin x dx sin x dx
⇒ I= ∫ 3 − 4 sin 2 x = ∫ 4 cos2 x − 1





sin x dx −dz
∫ =∫



I=



2
2 cos x − 1 2z 2 − 1
Put cos x = z
= − 1 log



⇒ sin x dx = – dz 2 z + 2z 2 − 1  + C
2  






sin x dx dz
⇒ I= ∫ 4 cos2 x − 1 = ∫ 1 − 4z 2 =−
1 
log 2 cos x + 2 cos 2 x − 1  + C


2  



EXERCISE 7.5
I. Multiple Choice Questions (MCQs) x
1. ∫ 2
2x − 1
dx equals
4. ∫x 2
+ 2x + 2
dx is equal to


2x + 2x + 1


1
1 (a)
log ( x 2 + 2 x + 2) − tan −1 ( x + 1) + C
2


(a) log (2 x 2 + 2 x + 1) − 2 tan −1 (2 x + 1) + C
2


1
(b) log ( x 2 − 2 x − 2) + tan −1 ( x + 1) + C
1 2


(b) log (2 x 2 + 2 x + 1) − 2 sin −1 (2 x + 1) + C
2 1


(c) log ( x 2 − 2 x − 2) − tan −1 ( x + 1) + C
1 2


(c) log (2 x 2 + 2 x + 1) − 2 cos −1 (2 x + 1) + C
2 (d) None of these




(d) None of these 2x − 1
5. ∫ 2 dx is equal to


x2 + x + 1 x + 2x + 3


2. ∫ x2 − x + 1
dx equals
3 x +1


(a) log ( x 2 + 2 x + 3) − +C tan −1
2 2


2  2 x − 1
(a) x + log ( x − x + 1) + 2
tan  −1
+C
3  3  2 x +1


(b) (b) log ( x 2 − 2 x − 3) + tan −1 +C
3 2


2  2 x − 1
(b) x + log ( x 2 + x + 1) + tan −1  +C
3  3  2 x +1


(c) log ( x 2 + 2 x + 3) − tan −1 +C
3 2


2  2 x + 1
(c) x + log ( x 2 − x + 1) + tan −1  +C (d) None of these
3  3 




Evaluate each of the following Integrals
(d) None of these
II. Short Answer Type Questions-I


x
3. ∫ 2
x + 4x + 5
dx is equal to
1. ∫( sin 2x
dx ) 2.
 cos 2 x 
∫  cos 3x  dx


cos 3x




∫( ) ( )
1
(a) log ( x 2 − 4 x + 5) + 2 tan −1 ( x + 2) + C sin x cos x
2 3. dx 4. ∫ dx


cos 2x cos 2x




1  sin 2 x 
(b) log ( x 2 + 4 x + 5) + 2 tan −1 ( x + 2) + C sin x
2 5. ∫ dx 6. ∫  sin 3x  dx


5 − sin 2 x




1 III. Long Answer Type Questions
(c) log ( x 2 + 4 x + 5) − 2 tan −1 ( x + 2) + C
2


1. x dx
(d) None of these ∫ x4 + x2 + 3 [Delhi 2005]





ntegrals 199
I

E:\AMIT_WORKS\Exam_Guru\EG_Mathematics-12_(working_02-06-2022)\EG_Mathematics-12_working\Open_Files\Chap_7\Chap_7
\ 17-Aug-2022  Amit   Proof 5 Reader’s Sign _______________________ Date __________


dx (3 sin x − 2) cos x dx
2. ∫ 9x 2 + 12x + 7 [A.I. 2009 (C)] 10. ∫ 5 − cos 2 x − 4 sin x
[Delhi 2013 (C)]






dx (5x + 3) dx
3. ∫ x (log x)2 + 2 (log x) + 2 11. ∫ x 2 + 4x + 20
[Delhi 2009 (C), S.P. 2014]






 
dx (2x + 5) dx
4. ∫ [A.I. 2009] 12. ∫ 7 − 6x − x 2
[Delhi 2009 (C), S.P. 2014]




9 + 8x − x 2



cos x dx dx
5. ∫ 13. ∫ [A.I. 2009]




16 − 2x − 2x 2


2 sin 2 x − 6 sin x + 5
5 x dx
6. ∫ 2 dx 14. ∫
3x + 13x − 10



52x − 2 × 5 x − 3


7. ∫
e x dx 15. ∫( cos 3x
cos 2x )
dx [HOTS] 16. ∫( )sin 3x
cos 2x
dx [HOTS]







5 − 4 e x − e 2x


∫( ) ∫( )
(5x − 2) dx sin 4x sin 4x
17. dx 18. dx
8. ∫ 1 + 2x + 3x 2 [Delhi 2013, 2014 (C)]
cos 3x sin 3x








(2 sin 2θ − cos θ) d θ  sin 2x 
9.
∫ 6 − cos 2 θ − 4 sin θ
[V. Imp.] [A.I. 2006] 19. ∫   dx
cos 2x 





Answers 7.5
2x − 1 2  2 x − 1
I. 1. (a) I = ∫ 2x 2
+ 2x + 1
dx 2
= x + log ( x − x + 1) +
3
tan −1 
 3 
+C




1 1
( 4 x + 2) − 1 − 1 3. (c) log ( x 2 + 4 x + 5) − 2 tan −1 ( x + 2) + C
=∫2 2 dx Q d (2 x 2 + 2 x + 1) = 4 x + 2 2


2x + 2x + 1  dx 



  1
1 4x + 2 dx 4. (a) log ( x 2 + 2 x + 2) − tan −1 ( x + 1) + C
= ∫ 2 dx − 2 ∫ 2 2


2 2x + 2x + 1 2x + 2x + 1


3 ( x + 1)
1 dx 5. (a) log ( x 2 + 2 x + 3) − tan −1 +C
= log (2 x + 2 x + 1) − ∫ 2 2
2


2 1


2
x +x+
1
2 II. 1. −1 log 2 cos x − 3 + C
= log (2 x 2 + 2 x + 1) − I1 2 3 2 cos x + 3
2


sin 2x 2 sin x cos x = 2 sin x
 1 Hint: =
x+ cos 3x 4 cos3 x − 3 cos x 4 cos 2 x − 3



dx dx 1 −1  2 Now put cos x = z.
=∫ 1 ∫
= 2
= tan 
1 1 

2
x +x+ 1  1


 x +  +   1 1 + 2 sin x
2 2 4 2 2  2. log +C
2 2 1 − 2 sin x


= 2 tan–1(2x + 1)
cos 2 x cos 2 x cos x


1 Hint: = =
2 −1
Hence, from (1), I = log (2 x + 2 x + 1) − 2 tan (2 x + 1) + C cos 3x 4 cos x − 3 cos x 4 cos 2 x − 3
3



2

x2 + x + 1 =  cos x 2 
2. (a) I = ∫ x 2 − x + 1 dx  1 − 4 sin x 






Put sin x = z.

Dividing numerator by denominator, we get 2 cos x − 1 + C
3. − 1 log

 2x   (2 x − 1) + 1 2 2 2 cos x + 1


I = ∫ 1 + 2 dx = x + ∫  2 dx
 x − x + 1  x − x + 1 


Hint: sin x = sin x
dx cos 2x 2 cos 2 x − 1



= x + log ( x 2 − x + 1) + ∫ 2 Put cos x = z.
 1 3



 x −  +
2 4
4. 1 log 1 + 2 sin x + C
1 2 2 1 − 2 sin x


2 x−
2 +C
Hint: cos x = cos x
2 −1
= x + log ( x − x + 1) + tan
3 3 cos 2x 1 − 2 sin 2 x





2 Put sin x = z.

200 Mathe atics–12
m
E:\AMIT_WORKS\Exam_Guru\EG_Mathematics-12_(working_02-06-2022)\EG_Mathematics-12_working\Open_Files\Chap_7\Chap_7
\ 17-Aug-2022  Amit   Proof-5 Reader’s Sign _______________________ Date __________


5. − cos x + 4 + cos 2 x  + C
 
12. Ans. −2 7 − 6x − x 2 − sin −1 x + 3 + C
4 ( )





sin x sin x Hint: 2x + 5 = – (– 2x – 6) – 1
Hint: =


2 2



5 − sin x cos x + 4 and 7 − 6x − x 2 = 16 − (x + 3) 2
Put cos x = z.


1 sin −1  2x + 1 + C

1 2 cos x − 1 + C 13.  
6. − log 2 33 



4 2 cos x + 1


Hint:
sin 2 x
sin 3x
=
sin 2 x
=
sin x 14.
log 5 ( )
1 5 x − 1 + 5 2x − 2 × 5 x − 3 + C
3 sin x − 4 sin 3 x 3 − 4 sin 2 x






sin x 15. 1 log 2 sin x − 1 + 2 sin x + C
=
4 cos 2 x − 1 2 2 2 sin x + 1





Put cos x = z. 4 cos3 x − 3 cos x
Hint: cos 3x =

1 tan −1  2x + 1
2 cos 2x 1 − 2 sin 2 x




III. 1. Ans.
 11  + C;
11
=
(1 − 4 sin x) cos x
2

Hint: Substitute: x2 = z 1 − 2 sin 2 x




2. Ans. 1 tan −1  3x + 2  + C; Put sin x = z.
 


3 3 3 


16. −2 cos x − 1 log 2 cos x − 1 + C
( )
 2 
Hint: 9x + 12x + 7 = 9  x + 2 2 2 2 cos x + 1
+ 1


2
3 3 sin 3x 3
= 3 sin x −2 4 sin x

 Hint:
cos 2x 2 cos x − 1



3. Ans. tan–1[1+ (log x)] + C
(
4 cos 2 x − 1 sin x )


Hint: Substitute: log x = z =
2 cos 2 x − 1



 x −4  + C;.
4. Ans. sin −1  Put cos x = z.
 5 



17. −2 cos x − 1 log 2 cos x − 3 + C
Hint: 9 + 8x − x 2 = 25 − (x − 4) 2 2 3 2 cos x + 3



5.
2
( 2 )
1 log sin x − 3 + sin 2 x − 3 sin x + 5 + C
3
Hint:
sin 4x
cos 3x
=
4 sin x cos x cos 2x
4 cos3 x − 3 cos x





1 log 3x − 2 4 sin x cos 2x
6. +C =
17 3x + 15 4 cos 2 x − 3




 x 
7. Ans. sin −1 e + 2 + C; 4  2 cos 2 x − 1 sin x
= 
 3 


4 cos 2 x − 3


Hint: Substitute: ex = z Put cos x = z.


5 11 tan −1  3x + 1 + ;
8. Ans. log | 1 + 2x + 3x 2 | −   C 18. 2 sin x + 1 log 2 sin x − 3 + C
6 3 2 2 


2 3 2 sin x + 3


5 11 sin 4x 4 sin x cos x cos 2x
Hint: 5x − 2 = (6x + 2) −
6 3 Hint: =
3 sin x − 4 sin 3 x

sin 3x



9. Ans. 2 log | sin2 q – 4 sin q + 5 | + 7 tan–1 (sin q – 2) + C
( )





2
2 sin 2θ − cos θ 4 cos 2x cos x = 4 1 − 2 sin x cos x
Hint: =
6 − cos 2 θ − 4 sin θ 3 − 4 sin 2 x 3 − 4 sin 2 x



Put sin x = z.
(4 sin θ − 1) cos θ

= 19. − 2 cos 2 x − 1 + C
sin 2 θ − 4 sin θ + 5


sin 2x

10. Ans. 3 log | sin x −2 | − 4
+C Hint: = 2 sin x cos x
sin x − 2 cos 2x 2 cos 2 x − 1





Put cos x = z.
Hint: (3 sin x2 − 2) cos x = (3 sin x − 2) cos x

5 − cos x − 4 sin x sin 2 x − 4 sin x + 4

OR 1 − 2 sin 2 x + C

sin 2x
11. Ans. 5 x 2 + 4x + 20 −7 log (x + 2) + x 2 + 4x + 20 + C Hint: = 2 sin x cos x


cos 2x 1 − 2 sin 2 x



Hint: 5x + 3 = 5 (2x + 4) − 7
2 Put sin x = z.


ntegrals 201
I

E:\AMIT_WORKS\Exam_Guru\EG_Mathematics-12_(working_02-06-2022)\EG_Mathematics-12_working\Open_Files\Chap_7\Chap_7
\ 17-Aug-2022  Amit   Proof 5 Reader’s Sign _______________________ Date __________


Topic 6. Integration Using Partial Fractions
f ( x) Putting the value of B in eqn. (ii) we have
Rational Function: The fraction of the form where f (x)


g ( x)


2(2C – 1) + C = 0  4C – 2 + C = 0


3 2 2
and g(x) are polynomials and g(x) ≠ 0. x2 + 1 , x 3 + 1 are rational 5C = 2 \ C =
5



x + 2 x +1  2 1 1
functions. \ B = 2   − 1 = − and A =
 5 5 5
f ( x)



Proper Rational Function: is called proper rational 1
g ( x) \ ∫ dx
( x + 2)( x 2 + 1)



function if degree of the numerator f (x) is smaller than the degree 1 1 2

of the denominator g(x), e.g., x+2 ,
( x − 3) . 5
− x+
5 5
= ∫ dx + ∫ 2
( x + 1) ( x + 2) x 2 + 1 ( x + 4) ( ) ( x + 2) ( x + 1)
dx


f ( x)
Improper Rational Function: is called improper rational 1 1 1 x−2
g ( x)
5 ∫ ( x + 2)
= dx − ∫ 2 dx
5 ( x + 1)


function if degree of the numerator f (x) is greater than the degree

1 1 1 x 2 1
x3 + 4 x 4 + 3x + 1 .
5 ∫ ( x + 2)
= dx − ∫ 2 dx + ∫ 2 dx
of the denominator g(x), e.g., , 2 5 ( x + 1) 5 x +1
( x + 1) ( x + 2) x + 4x + 7


1 1 1 2x 2 1
f ( x)
5∫ x + 2
= dx − ∫ 2 dx + ∫ 2 dx
Partial Fractions: Suppose is a proper rational function 10 x + 1 5 x +1
g ( x)


1 1 2
and g(x) is factorised into linear, quadratic, cubic polynomials, \ I = log x + 2 − log x 2 + 1 + tan − 1 x + C
f ( x)

5 10 5
then we can express into sum or difference of simpler
g ( x) Putting the given value of I

proper fractions. These simpler fraction are called partial fractions. 1
\ a log 1 + x 2 + b tan − 1 x + log x + 2 + C
Partial fractions can be divided into four types: 5

1 1 2
= log x + 2 − log x 2 + 1 + tan − 1 x + C
5 10 5

Type I: When the denominator of the proper partial fraction
consists of linear factors without repetition. 1 2
\ a= − and b =
10 5



ax 2 ± bx ± c Hence, the correct option is (c).

(px + q) (mx + n) (dx + e) Example 2. Change into partial fractions:
dx 3x 2 + 4x + 5
Example 1. If ∫ ( x + 2)( x 2 + 1) (x − 1) (x + 2) (x − 3)
1 3 x 2 + 4x + 5
= a log 1 + x 2 + b tan − 1 x + log x + 2 + C then Solution. Take:
5 (x − 1) (x + 2) (x − 3)

1 −2 1 −2
(a) a = − ,b= (b) a = , b = A B C
10 5 10 5 = + + ...(1)

x −1 x + 2 x − 3



1 2 1 2
(c) a = − , b = (d) a = , b =
10 5 10 5 ⇒ 3x2 + 4x + 5 =A(x + 2)(x – 3) + B(x – 1)(x – 3)




dx + C(x – 1)(x + 2) (2)
Solution. Let I = ∫ ( x + 2)( x 2 + 1)

Put x = 1 in equation (2),
Let us resolve the given integrand into partial fractions

3 + 4 + 5 = A(3) (–2) ⇒ A=−2

1 A Bx + C




Put = + 2
2
( x + 2)( x + 1) ( x + 2) ( x + 1) Put x = – 2 in equation (2),



1 = A(x2 + 1) + (x + 2) (Bx + C) 3
12 – 8 + 5 = B(–3) (–5) ⇒ B=

1 = Ax2 + A + Bx2 + Cx + 2Bx + 2C 5



Put x = 3 in equation (2),

1 = (A + B)x2 + (C + 2B)x + (A + 2C)


Comparing the like terms, we have 22
27 + 12 + 5 = C(2) (5) ⇒ C=
5





A+B=0 ...(i)


2B + C = 0 ...(ii) On putting the values of A, B and C in equation (1), we get

( ) ( ) ( )


A + 2C = 1 ...(iii) 3x 2 + 4x + 5 −2 3 1 22 1
= + +


Subtracting (i) from (iii) we get (x − 1) (x + 2) (x − 3) x −1 5 x + 2 5 x−3


2C – B = 1 \ B = 2C – 1 These partial fractions can be integrated easily.



202 Mathe atics–12
m
E:\AMIT_WORKS\Exam_Guru\EG_Mathematics-12_(working_02-06-2022)\EG_Mathematics-12_working\Open_Files\Chap_7\Chap_7
\ 17-Aug-2022  Amit   Proof-5 Reader’s Sign _______________________ Date __________


Type II: When the denominator of the fraction consists of 41
⇒ C= −
repeated and non-repeated linear factors. 6




ax 2 ± bx ± c On putting the values of A, B and C in equation (1), we get

( )


(px ± q) (mx ± n) 2 11x + − 41

3x 2 − 4x + 8 7 1 
=  + 62 6
2
2x + 5x + 7 (x − 1) (x 2 + 4x + 1) 6  x −1 x + 4x + 1



Example 3. Change into partial fractions:
(x − 2) (x − 3) 2
7  1  1  11x − 41 
2x 2 + 5x + 7 = +
Solution. Take: 6  x − 1 6  x 2 + 4x + 1



(x − 2) (x − 3) 2

A B C These partial fractions can be integrated easily.
= + + ...(1)


x − 2 x − 3 ( x − 3)2 Type IV: When the numerator and denominator of the proper



⇒ 2x2 + 5x + 7 = A(x – 3)2 + B(x – 2) (x – 3) + C(x – 2)...(2) fraction consists of purely quadratic fractions.


Put x = 2 in equation (2),
ax 2 + b

8 + 10 + 7 = A ⇒ A = 25
( )(
cx + d ex 2 + f
2
)




Put x = 3 in equation (2),

18 + 15 + 7 = C ⇒ C = 40 3x 2 + 5
Example 5. Change into partial fractions:



Equate coefficients of x on both sides of equation (2) and
2
(x 2 + 4) (x 2 + 1)

substitute the value of A, Solution. Put x2 = y in the given fraction,
2= A+B ⇒ 2 = 25 + B
3y + 5




⇒ B = – 23 we get:

(y + 4) (y + 1)




On putting the values of A, B and C in the equation (1), 3y + 5

we get Take:
(y + 4) (y + 1)



2x 2 + 5x + 7 25 23 40
2 = x−2
− + A B
(x −2) (x −3) x − 3 (x − 3) 2 = + ...(1)


y + 4 y +1



These partial fractions can be integrated easily.
⇒ 3y + 5 = A(y + 1) + B(y + 4) ...(2)

Type III: When the denominator of the proper fraction consists




of linear factors and quadratic factors. Put y = – 4 in equation (2)

ax 2 ± bx ± c –12 + 5 = A(–3) ⇒ A= 7
3
( )




( px ± q ) mx 2 ± nx ± r Put y = – 1 in equation (2),

2
– 3 + 5 = B(3) ⇒ B=
Example 4. Change into partial fractions: 3x 2 − 4x + 8 3




(x − 1) (x 2 + 4x + 1)
On putting the values of A and B in equation (1), we get

3x 2 − 4x + 8 A
+ 2
Bx + C 3y + 5 7 1  2 1 
Solution. Take: = ...(1) =  +
2 x − 1
(x − 1) (x + 4x + 1) x + 4x + 1 3  y + 4  3  y + 1

(y + 4) (y + 1)

⇒ 3x2 – 4x + 8 = A(x2 + 4x + 1) + Bx(x – 1) + C(x – 1) ...(2)
3x 2 + 5 7 1  2 1 


Put x = 1 in equation (2), or =  2 +
2 2
(x + 4) (x + 1) 3  x + 4  3  x 2 + 1


7
3 – 4 + 8 = A(1 + 4 + 1) ⇒ A= These partial fractions can be integrated easily.
6





Equate coefficient of x2 on both sides of equation (2) and Note:

substitute the value of A 1. If degree of numerator of the fraction is equal to or greater


7 11 than the degree of the denominator, divide the numerator
3= A+B ⇒ B = 3− = by the denominator and reduce the degree of the numerator
6 6




11 less than the degree of the denominator.
⇒ B= 2. Some writers say that the method given in previous
6




examples for finding values of the constants A, B and C
Equate coefficients of x on both sides of equation (2) and
is wrong. In example:

substitute the values of A and B,
3x + 4 A Β A ( x + 4) + B ( x − 1)
– 4 = 4A – B + C = + =
( x − 1) ( x + 4) x − 1 x + 4 ( x − 1) ( x + 4)



7 11 28 11
⇒ –4 = 4 × − + C ⇒ −4 = − +C
6 6 6 6 ⇒ 3x + 4 = A(x + 4) + B(x – 1).







ntegrals 203
I

E:\AMIT_WORKS\Exam_Guru\EG_Mathematics-12_(working_02-06-2022)\EG_Mathematics-12_working\Open_Files\Chap_7\Chap_7
\ 17-Aug-2022  Amit   Proof 5 Reader’s Sign _______________________ Date __________


They say that x = 1 and x = – 4 cannot be put in the above Here there is no restriction. Teachers are requested to send


A B me an example where the above substitution method gives
equation because and are not defined, we are wrong results. Then I shall change the method.
x −1 x+4
A B 3. Finally, once during board paper evaluation, it was decided
not putting x = 1 and x = – 4 in and but we are
x −1 x+4



that the values of constants A, B, C etc. can be evaluated
putting x = 1 and x = – 4 in 3x + 4 = A(x + 4) + B(x – 1). by any method, will be accepted.

EXERCISE 7.6
I. Multiple Choice Questions (MCQs)
 2 + sin x 
x (b) log  +C
1. ∫ dx equals  1 + sin x 



( x − 1) ( x − 2 )


 1 + sin x 
( x − 1)2 ( x − 2 )2 (c) log  +C
(a) log +C (b) log +C  2 + sin x 



x−2 x −1




2
(d) None of these



 x −1 
(c) log  +C (d) log |(x – 1) (x – 2)| + C ( 2x2 + 1 )
 x − 2  7. If ∫ ( x 2 − 4 ) ( x 2 − 1 ) dx






[NCERT]

dx   x + 1  a  x − 2 b 
2. ∫ x ( x 2 + 1) equals = log       + C
  x − 1  x + 2  



1 then, the values of a and b are respectively
(a) log | x | − log| x 2 + 1 | + C

2


1 3 3 3 −1 3
1 (a) , (b) −1, (c) 1, (d) ,
(b) log | x | + log | x 2 + 1 | + C 2 4 2 2 2 4








2


1 Evaluate each of the following Integrals
(c) − log | x | + log | x 2 + 1 | + C
2 II. Short Answer Type Questions-I


1 ( x + 1) dx
(d) log | x | + log| x 2 + 1| + C [NCERT] (x 2 + 3) dx
2 1. ∫ (x + 2) (x − 3) 2. ∫ (x − 1) (x + 2) (x + 3)







x −1
3. ∫ dx is equal to dx
( x + 2 )( x + 3 ) 3. ∫ (x + 2) x 2




( x + 3 )4 ( x + 2 )4
(a) log +C (b) log +C (x 2 + 5) dx
( x + 2 )3 ( x + 3 )3 4. ∫ (x − 1) (x + 2)2 [Delhi 2006, 2013 (C)]







3
(c) log ( x + 2 ) + C (d) None of these (x 2 + 5) dx dx
( x + 3 )4 5. ∫ (x + 1) (x 2 + 4) 6.
∫ (x − 4) (x 2 + 4x + 7)








x2 + 1
4. ∫ x( x dx is equal to
7. (1 − x 2) dx [Delhi 2010]

2
− 1)


x (1 − 2x)



x2 − 1 x2 − 1
(a) log +C (b) − log +C (3x + 5) dx
x x 8. ∫ x3 − x 2 [Delhi 2013, 2014 (C)]




− x +1



x x
(c) log 2 +C (d) − log 2 +C (x 2 + x + 1) dx
x +1 x +1 9. ∫ (x + 2) (x 2 + 1) [A.I. 2013 (C), 2015, Delhi 2013 (C)]







4x
5. ∫ 2 dx equals
( x + 1 ) ( x 2 + 3) (x 2 + x + 1) dx


10. ∫ (x − 1)3 [A.I. 2000]



 x2 + 1  x 2 + 3
(a) log  2 +C (b) log  2 +C
 x + 3   x + 1  cos x dx




11. ∫ (1 − sin x) (2 − sin x) [Delhi 2007]



(c) tan (x + 1) (x + 3) + C
–1 2 2


(d) 2log (x2 + 1) (x2 + 3) + C dx
12. ∫ sin x + sin 2x [A.I. (C), Delhi 2015]


cos x



6. ∫ dx equals
(1 + sin x) (2 + sin x)


2e x dx dx
(a) log (1 + sin x) (2 + sin x) + C
13.
∫ e3x − 6e2x + 11e x − 6 14. ∫ sin x (3 + 2 cos x )






204 Mathe atics–12
m
E:\AMIT_WORKS\Exam_Guru\EG_Mathematics-12_(working_02-06-2022)\EG_Mathematics-12_working\Open_Files\Chap_7\Chap_7
\ 17-Aug-2022  Amit   Proof-5 Reader’s Sign _______________________ Date __________


dx (3 sin θ − 1) cos θ
15. ∫ x 6(log x)2 + 7 (log x) + 2 2. ∫ 5 − cos2 θ − 4 sin θ d θ




 
dx dx
16. ∫ x 17. ∫ e x (e x + 1) ( 2x + 1)
e −1 3. ∫ ( x 2 + 1) ( x 2 + 4) dx [A.I. 2015]







dx
18. ∫ 2e2x + 3e x + 1  x3 − x − 2 


4. ∫  x 2 − 1 
dx



dx
19. ∫ x ( x 4 − 1) x 4 dx


5. ∫ (x − 1) (x 2 + 1) [S.P. 2014]




dx
20. ∫ x(x n + 1)


x3 dx
3
6. ∫ x 4 + 3x 2 + 2 [A.I. 2008, 2014 (C)]




21. ∫ (tan φ +3 tan φ) d φ [Delhi 2009 (C)]
tan φ + 1



x2 + x + 1
dx
7. ∫ ( x − 1)3
dx



22. ∫ (x 2 + 1) (x 2 + 2) [Delhi 2010 (C)]



2x dx
III. Long Answer Type Questions
8. ∫ ( x 2 + 1) ( x 2 + 4) [Delhi 2017]




1. (x 2 + 1) dx
∫ (x 2 + 4) (x 2 + 25) 9.
cos θ
∫ ( 4 + sin 2 θ) (5 − 4 sin 2 θ) d θ [A.I. 2017]





[Delhi 2013, 2014 (C), A.I. 2013 (C)]

Answers 7.6
x when t = 0, 1 = A(0 + 1) ⇒ A = 1
∫ ( x − 1)( x − 2 ) dx






I. 1. (b) I = when t = –1, 1 = B(–1) ⇒ B = –1








  
x A B 1 dt 1 dt 1 1
Let = + \ I= ∫ − ∫ = log | t | − log| t + 1| + C
2 t 2 t +1 2 2



( x − 1)( x − 2 ) x −1 x − 2

⇒ x = A(x – 2) + B(x – 1) ...(1) 1 2 1 2
= log | x | − log | x + 1| + C


2 2


Putting x = 1, 2 in (1)



when x = 1, 1 = A(1 – 2) ⇒ A = –1 1 1
= × 2 log | x | − log| x 2 + 1| + C






when x = 2, 2 = B(2 – 1) ⇒ B = 2 2 2








x −1 2 1
\ = + = log | x | − log| x 2 + 1| + C
( x − 1)( x − 2 ) x − 1 x − 2 2




x −1
x 1 1 3. (a) I = ∫ dx
\ I= ∫ ( x − 1)( x − 2 ) dx = − ∫ x − 1 dx + 2 ∫ x − 2 dx ( x + 2 )( x + 3 )







x −1 A B
= − log | x − 1 | + 2 log | x − 2 | + C Let = +
( x + 2 )( x + 3 ) x + 2 x + 3



= − log | x − 1| + log | ( x − 2 ) 2 | + C
Multiplying both sides by (x + 2)(x + 3), we get



( x − 2 )2 x – 1 = A(x + 3) + B(x + 2) ...(1)
= log
+C



x −1


Put x = –2, –3 in (1)

dx x when x = –2, –2–1 = A(– 2 + 3) ⇒ A = –3
2. (a) I = ∫
x( x 2 + 1) ∫ x 2 ( x 2 + 1)
= dx










when x = –3, –3–1 = B(–3 + 2) ⇒ B = 4






1 x −1 −3 4
Put x2 = t, then 2x dx = dt ⇒ x dx = dt \ = +
2 ( x + 2 )( x + 3 ) x + 2 x + 3


1 dt
\ ∫
I=
2 t ( t + 1) \ I=
x −1
∫ ( x + 2 )( x + 3 ) dx = − 3∫ x + 2 + 4 ∫ x + 3
dx dx






1 A B
Let = + = –3log | x + 2 | + 4 log | x + 3 | + C
t ( t + 1) t t +1






Multiplying both sides by t(t + 1), we get = − log| ( x + 2 )3 | + log | ( x + 3 ) 4 | + C



1 = A(t + 1) + B(t) ...(1) ( x + 3) 4
= log +C


Put t = 0, – 1 in (1) ( x + 2 )3



ntegrals 205
I

E:\AMIT_WORKS\Exam_Guru\EG_Mathematics-12_(working_02-06-2022)\EG_Mathematics-12_working\Open_Files\Chap_7\Chap_7
\ 17-Aug-2022  Amit   Proof 5 Reader’s Sign _______________________ Date __________


x2 + 1 x2 + 1 (2 x 2 + 1)
4. (a) I = ∫ x( x 2
− 1)
dx = ∫
x ( x − 1) ( x + 1)
dx 7. (a) I= ∫ (x 2
− 4) ( x 2 − 1)
dx








x2 + 1 A B C Let x2 = t
Let = + +




x ( x − 1)( x + 1) x x − 1 x + 1 ( 2 x 2 + 1) 2t + 1

\ =
2 2
( x − 4 )( x − 1) ( t − 4 )( t − 1)




Multiplying both sides by x(x – 1)(x + 1), we get

\ x2 + 1 = A(x – 1) (x + 1) + B(x)(x + 1) + Cx(x – 1) ...(1) 2t + 1 A B
Let = +
( t − 4 )( t − 1) t − 4 t −1






Put x = 0, –1, 1 in (1)

Multiplying both sides by (t – 4)(t – 1), we get 2t + 1 = A(t – 1)
when x = 0, 0 + 1 = A(0 – 1)(0 + 1) ⇒ A = –1


+ B(t – 4)






when x = –1, 1 + 1 = C(–1)(–1 –1) ⇒ 2C = 2 ⇒ C = 1 Putting t = 4, we get 2(4) + 1 = A(4 – 1) ⇒ 3A = 9 ⇒ A = 3






when x = 1, 1 + 1 = B(1)(1 + 1) ⇒ B = 1 Putting t = 1, we get






x2 + 1 1 1 1 2(1) + 1 = B(1 – 4) ⇒ –3B = 3 ⇒ B = –1
\ =− + +



x ( x − 1)( x + 1) x x −1 x +1 2t + 1 3 1 2x2 + 1

\ = − ⇒ 2
( t − 4 )( t − 1) t − 4 t−1 ( x − 4 )( x 2 −1)




x2 + 1 dx dx dx
I= ∫ x ( x − 1)( x + 1) dx = − ∫ x ∫ x −1 ∫ x +1
+ +
3 1


= −
x2 − 4 x2 − 1



= − log | x | + log | x − 1| + log | x + 1| + C


2x2 + 1
= log
( x − 1)( x + 1)
+ C = log
x2 − 1
+C
\ I= ∫ (x 2
− 4 )( x 2 −1)
dx



x x


dx dx
= 3∫
x 2 − 22 ∫ x 2 − 1

4x 2x
5. (a) I = ∫ 2 dx = 2 ∫ 2


dx
( x + 1)( x 2 + 3) ( x + 1)( x 2 + 3 )




3  x −2 1  x −1
= log   − log  +C
Put x2 = t, then 2x dx = dt 4  x + 2  2  x + 1 



dt 1  1 1  3 1
I = 2∫ = 2 × ∫ − dt  x − 24  x +1 2
( t + 1)( t + 3 ) 2  t + 1 t + 3  = log  + log  +C


 x+ 2  x − 1 


 1 1 
= ∫ − dt = log | t + 1| − log| t + 3| + C  1 3

 t + 1 t + 3    x + 1  2  x − 2 4
+C


= log 
  x − 1  x + 2  


t +1 x2 + 1  
= log + C = log 2 +C 1 3
t+3 x +3 \ a = and b =


2 4



cos x
6. (c) I = ∫ (1 + sin x) (2 + sin x) dx II. 1. Ans.
1 log | x + 2 | + 4 log | x − 3 | + C;




5 5
Put sin x = t, so that cos x dx = dt
Hint: x +1
=
1 1 +4 1
( ) ( )

(x + 2) (x − 3) 5 x + 2 5 x − 3

dt
\ I= ∫
(1 + t )(2 + t ) 2. Ans. 1 log | x − 1 | − 7 log | x + 2 | + 3 log | x + 3 | + C





3 3



( ) ( ) ( )
1 A B 2
(x + 3)
Let = + Hint: = 1 1 − 7 1 +3 1
(1 + t )(2 + t ) 1+ t 2 + t (x − 1) (x + 2) (x + 3) 3 x −1 3 x + 2 x+3





1 1
3. Ans. log | x + 2 | − log | x | − 1
\ I = A(2 + t) + B(1 + t) + C;
4 4 2x





( ) ()
Put t = –2, we get I = B(–2 + 1) ⇒ B = –1
1 = 1 1 − 1 1 + 1  1 



Hint:
Put t = –1, we get I = A(2 – 1) ⇒ A = 1 (x + 2) x 2 4 x + 2 4 x 2  x 2 





dt dt 4. Ans. 2 log | x − 1 | + 1 log | x + 2 | + 3 + C;
\ I= ∫1+ t −∫ 2 + t 3 3 x+2





Hint: x2 + 5
= log| 1 + t | − log | 2 + t | + C (x − 1) (x + 2) 2



= log
1+ t
2+t
+ C = log
1 + sin x
2 + sin x
+ C. ( ) ( )
3 x −1 3 x + 2

= 2 1 + 1 1 − 3  1 2

 (x + 2) 



206 Mathe atics–12
m
E:\AMIT_WORKS\Exam_Guru\EG_Mathematics-12_(working_02-06-2022)\EG_Mathematics-12_working\Open_Files\Chap_7\Chap_7
\ 17-Aug-2022  Amit   Proof-5 Reader’s Sign _______________________ Date __________


5. Ans. 6 log | x +1| − 1 log(x 2+ 4) + 1 tan −1 x + C;
5 10 10 2 () 13. Ans. log
(e x − 1) (e x − 3)
+ C;


(e x − 2) 2



 − x + 1
( )
x2 + 5 Hint : Put ex = z and
Hint: = 6 1 +  5 5


(x + 1) (x 2 + 4) 5 x + 1  x 2 + 4  2  2 

  3 =
2 
z − 6z + 11z − 6  (z − 1 ) (z − 2 ) (z − 3) 


6. Ans. 1 log | x − 4 | − 1 log x 2 + 4x + 7
39 78 = 1 −2 1 + 1 ( )


z −1 z−2 z −3
 x + 2
− 8 tan −1  + C;
 3 


39 3 14. 1 log 1 − cos x − 1 log 1 + cos x + 2 log 3 + 2 cos x
10 2 5



Hint: 1
Hint: 1 sin x
(x − 4) (x 2 + 4x+ 7) =

sin x (3 + 2 cos x ) (1 + cos x )(1 − cos x )(3 + 2 cos x )


 −x− 8  Put cos x = z.
1 1
( )
+  39 39 


= 1 + 2 log x
39 x − 4  x 2 + 4x + 7  15. Ans. log +C ;
  2 + 3 log x



Hint: Put log x = z and

x + log | x | − 3 log | 2x −1 | + C;
7. Ans.
( ) ( )

2 4 1 1
= 2 1 −3 1


2 =
1 − x2  x−2  6z + 7z + 2 (2z + 1) (3z + 2) 2z + 1 2 + 3z
Hint: =1+1
x (1 − 2x) 2 2  x (2x − 1) 


x

16. Ans. log e −x 1 + C;
= 1 + 1 2 − 3  e


2 2  x 2x − 1
Hint: Put ex = z
1 = 1 = 1 −1

8. Ans. 1 log x + 1 − 4 + C; ⇒ x
e − 1 z(z − 1) z − 1 z

2 x −1 x −1


Hint: 3 3 x +5 = 3x + 5 ex + 1 1
17. Ans. log − x + C; Hint : Put e x = z
x − x 2 − x + 1 (x + 1) (x − 1) 2 ex

e


( ) ( )
=1 1 −1 1 +
2 x + 1 2 x − 1 (x − 1) 2
4 \ e dx = dz ⇒z dx = dz ⇒ dx =
x dz
z





1 1
⇒ ∫ x x dx = ∫ dz

3 log | x + 2 | + 1 log | x 2 + 1 | 1 e (e + 1) (z + 1) z 2


9. Ans. + tan −1 x + C;
5 5 5


 1 − 1 + 1  dz
 2x + 1  =∫
 z + 1 z z 2 
( )

2
x + x + 1 3 1
Hint: = +  5 5 18. log(ex) + log(ex + 1) – 2 log(2ex + 1) + C
(x + 2) (x 2 + 1) 5 x + 2  x 2 + 1 



  Hint: Put ex = z.

10. Ans. log | x − 1 | − 3 − 3 4
x − 1 2(x − 1) 2
+ C; 19. 1 log x − 1 +C


4 x4


2
Hint: x + x +3 1 = 1 + 3 + 3 n
(x −1) x − 1 (x − 1) 2 (x − 1)3 20. Ans. 1 log nx + C; Hint : Put x n = z

n x +1


2 − sin x
11. Ans. log + C; x n −1 = 1  1  = 1  1 − 1 
1 − sin x ⇒


x (x n + 1) n  z (z + 1)  n  z z + 1
n

1
Hint: Put sin x = z. It gives: = 1 − 1
(1 − z )( 2 − z ) 1− z 2 − z −1
log | tan φ + 1| + 1 log tan 2 φ − tan φ + 1

21. Ans.
12. Ans. 1 log | 1 + cos x | + 1 log | 1 − cos x | 3 6


2 6  2 tan φ − 1
+ 1 tan −1 


 + C;
− 2 log | 1 + 2 cos x | + C; 3  3
3


Hint: 1 1 (tan 3 φ + tan φ) d φ tan φ sec2 φ
= Hint:
sin x + sin 2x sin x (1 + 2 cos x) ∫ =∫ dφ

3
tan φ + 1 tan 3 φ + 1

sin x
= Put tan φ = z \ sec2 d = dz
(1 + cos x) (1 − cos x) (1 + 2 cos x)



f
f

2  
\ ∫ tan φ3 sec φ d φ = ∫ 3 z dz = ∫  z

Put cos x = z and 1 2  dz
tan φ + 1 z +1  (z + 1) (z − z + 1) 


(1 + z) (1 − z) (1 + 2z)

( ) ( ) ( )
= −1 1 + 1 1 + 4 1
2 1 + z 6 1 − z 3 1 + 2z 3 z +1 ( )
= −1 ∫ 1 dz + 1 ∫  2 z + 1  dz
3  z − z + 1


ntegrals 207
I

E:\AMIT_WORKS\Exam_Guru\EG_Mathematics-12_(working_02-06-2022)\EG_Mathematics-12_working\Open_Files\Chap_7\Chap_7
\ 17-Aug-2022  Amit   Proof 5 Reader’s Sign _______________________ Date __________


  x4 (x 4 − 1) + 1
22. Ans. tan −1 x − 1 tan −1  x  + C; Hint: =
2  2 (x − 1) (x + 1) (x − 1) (x 2 + 1)
2




Hint: 1 = 1 x4 − 1 1
(x + 1) (x 2 + 2) (y + 1) (y + 2)
2 = +

(x − 1) (x 2 + 1) (x − 1) (x 2 + 1)


= 1 − 1 = 21 − 2 1 1
y +1 y + 2 x +1 x + 2 = (x + 1) +
(x − 1) (x 2 + 1)

− 1 −1 x
()
+ 8 tan −1 x + C ()


III. 1. Ans. tan − x 1
14 2 35 5 1  1   2 − 2
= (x + 1) + +
x2 + 1 y +1 2  x −1  x 2 + 1 
Hint: =  
(x + 4) (x 2 + 25) (y + 4) (y + 25)
2


    (
6. Ans. log (x 2 + 2) − 1 log x 2 + 1 + C; )
= −1  1  + 8  1  2



7  y + 4  7  y + 25 
x3 \x dx = dz
Hint: ∫ x 4 + 3x 2 + 2 dx . Put x = z.
2

2



2. Ans. 3 log sin θ − 2 − 5 + C;
(sin θ − 2) It gives



1 z  1  −1 2 
Hint: 3z − 12 = 3 + 5
∫ 2  (z + 1) (z + 2)  dz = 2 ∫  z + 1 + z + 2  dz
( z − 2) z − 2 ( z − 2)2


() 7. log x − 1 − 3 −
2 3
3. Ans. 1 log x2 + 1 + 1  tan −1 ( x ) − 1 tan −1 x  + C
 
x − 1 2 ( x − 1)2
+C


3 x +4 3 2 2 


4.
x2
2
x
+ log + 1 + C
x −1 ( ) ( )
 2
8. 1 log x 2 + 1 − 1 log x 2 + 4 + 1 tan −1  x  + C
5 10 10  2




5. Ans.
x 2 + x + 1 log | x − 1) − 1 log | x 2 + 1 |
2 2 4
− 1 tan −1 x + C;
2
1
9. − tan −1
30 2 15( )
sin θ + 2 tan −1 2 sin θ + C
( )




Topic 7. Integration By Parts
When we have to integrate a product of two functions like ∫ f ( x ) g ( x ) dx, we use the method of integration by parts.
Here first step is to decide that out of the two given functions which function will be taken as first function and which function
will be taken as a second function. First function must be easily differentiable and second function must be easily integrable. After
taking this decision, write selected first function at first place and selected second function at second place.
The selection of first and second function is done by ILATE rule. This rule tells us the order in which first and second function
are taken.

ILATE Rule
I L A T E





1. Inverse Trigonometric function


2. Logarithmic function


3. Algebraic function


4. Trigonometric function


5. Exponential function


ILATE rule tells us the order in which we take two functions as first and second.
Note: This rule is not a hard and fast rule.

Formula for Integration by Parts


∫ f ( x ) g ( x ) dx = f ( x ) ∫ g ( x ) dx − ∫  f ′ ( x ) ∫ g ( x ) dx  dx

or ∫ (first function )(second function ) dx = [first function × integral of second function]



– Integral of [derivative of first function × integral of second function]

208 Mathe atics–12


m
E:\AMIT_WORKS\Exam_Guru\EG_Mathematics-12_(working_02-06-2022)\EG_Mathematics-12_working\Open_Files\Chap_7\Chap_7
\ 17-Aug-2022  Amit   Proof-5 Reader’s Sign _______________________ Date __________


Example 1. Evaluate: ∫ (9x + 11) e
5x
dx ⇒ I = (9x + 11)
e5x
5 ∫ ( )
1
− 9 × e5x dx




∫ (9x + 11) e dx
5x
Solution. I= 5x
I = (9x + 11) e − 9 e5x + C


⇒ I= (9x + 11) ∫ e5xdx − ∫ (9x + 11)′ ∫ e5xdx  dx ⇒
5 25







EXERCISE 7.7

∫ (sin x ) dx
I. Multiple Choice Questions (MCQs) −1
5. [Delhi 2004, A.I. 2006 (C)]
∫ x e dx equals
2
3 x
1.




∫ (sec x) dx
−1


6. [Delhi 2005]
1 2




2 2
(a) ( x + 1) e x + C (b) (b) ( x 2 + 1) e x + C
2
∫ ( x cot x ) dx
2 −1




7. [A.I. 2007 (C)]




1 2
∫ (sec x ) dx
2
(d) ( x 2 − 1) e x + C
2
(c) ( x − 1) e x + C 8. −1
2






2. ∫x ∫ ( tan x ) dx
3 2
( log x ) dx = 9. −1




1 4
∫ x sec x dx
2
(a) x [ 8 (log x) 2 + 4 log x − 1 ] + C 10.
32





1 4 III. Long Answer Type Questions
(b) x [8 ( log x ) 2 + 4 log x + 1 ] + C
32
∫ (x + 1) e x dx ∫x a
2 2 x
dx


1. 2.




1 4
(c) x [ 8 ( log x ) 2 − 4 log x − 1 ] + C x dx
∫ (3x + 7) sin
2
32 3. ∫ 1+ sin x 4. x dx






(d) None of these
( xn ) dx ( )


∫ (x) e ∫
2n − 1
3. If I = ∫ e x sin 2 x dx, the for what value of K, 5. 6. x sin x dx






KI = ex (sin 3x – cos 2x) + C? (log x) dx
7. ∫ [A.I. 2015]

(a) 1 (b) 3 (c) 5 (d) 7 (x + 1) 2











4. ∫ tan
−1
x dx is equal to [NCERT Exemplar]
∫ log (x + 1) dx
2
8. [V. Imp.] [Delhi 2006, A.I. 2006]






(a) ( x + 1) tan −1 x − x + C
∫ log  x + x 2 + a 2  dx


9. 
(b) x tan −1 x − x+C




∫ (x + 1) e
x
(c) x − x tan −1 x + C 10. log (xe x) dx





(d) x − ( x + 1) tan −1 x + C  cos x + sin x 
11. ∫ (cos 2x) log  cos x − sin x  dx




 1 
5. ∫ log (log x) + (log x)  dx equals
∫  x (sin x) dx
2 −1
12. [V. Imp.] [A.I. 2009]






x
∫ (sin x) dx
−1 2
(a) x log (log x) + +C 13.
log x




∫  x (sec x) dx
−1
x 14.
(b) x log (log x) − +C


log x
∫ ( x cos x ) dx
2 −1
15.


[Foreign 2009]



log x
(c) x log (log x) + ∫  x (tan x)  dx
+C −1 2
16.
x




log x  sin −1 x 
(d) x log (log x) −
x
+C 17. ∫  x 2 
dx [V. Imp.]





Evaluate each of the following Integrals (tan −1 x) dx
II. Short Answer Type Questions
18. ∫ (1 + x 2) 2
[V. Imp.]



1. ∫ log x dx 2. ∫x
3
(log x) dx  1− x
∫  tan
−1
19. dx [V. Imp.]




1+ x 
∫( )



3. tan −1 x dx [Delhi 2008 (C)]
 x 



∫  sin
−1

∫( ) 20. dx [V. Imp.]


4. x tan −1 x dx [Delhi 2014 (C), A.I. 2006] x + a 






ntegrals 209
I

E:\AMIT_WORKS\Exam_Guru\EG_Mathematics-12_(working_02-06-2022)\EG_Mathematics-12_working\Open_Files\Chap_7\Chap_7
\ 17-Aug-2022  Amit   Proof 5 Reader’s Sign _______________________ Date __________


Answers 7.7
1 1 1
∫x
2
I. 1. (c) I =
3
e x dx = x log (log x) − ⋅x−∫ dx + ∫ dx



log x (log x) 2 (log x) 2



Put x2 = t, then 2x dx = dt


1 1 1 x2 2 x
\ I = ∫ te dt = [t e − e ] + C = e ( x − 1) + C
t t t = x log (log x) − +C
log x



2 2 2



II. 1. x log | x | – x + C;
2. (d) I = ∫x
3
(log x ) 2 dx



Hint: log x = (log x) × 1. Now integrate by parts.






1 4 1  1 4 4
= x ⋅ (log x ) 2 − ∫ ⋅ x 4 ⋅ 2 log x ⋅   dx 2. x (log x) − x + C;
4 4  x 4 16





x4 1 x 4 dx
∫ (log x ) x dx = (log x ) 4 − ∫ x ×
1 4 1 3
Hint:
= ⋅ x (log x ) 2 − ∫ x3 log x dx 4


4 2


4 4
1 11 1  1  = x ( log x ) − 1 × x + C
= x 4 (log x ) 2 −  x 4 log x − ∫ x 4 ⋅   dx  4 4 4



4 2 4 4  x 


1
3. x(tan −1 x) − log (1 + x 2 ) + C;
1 4 1 1 2



x (log x ) 2 − x 4 log x + ∫ x 3dx
=
4 8 8 Hint: I = tan −1 x × 1 dx
∫ ( )





=
1 4 1
x (log x) 2 − x 4 log x +
1 4
x +C
= tan x x − ∫
−1
( 1 .x dx
1 + x2
)


4 8 32


1 2x dx
= x tan x − ∫
−1
1 4 2 1 + x2
= x [ 8 (log x) 2 − 4 log x + 1] + C


32
( )


= x tan −1 x − 1 log 1 + x 2 + C
2
∫e


x
3. (c) I = sin 2 x dx
2
 x + 1




4.  (tan −1 x) − x + C;
Integrating by part, taking sin 2x as the Ist function, we get  2  2



I = sin 2 x ( e x ) − ∫ 2 cos 2 x ( e x ) dx
x 2 dx
∫ ( tan ) ( )
2
Hint: I = x x dx = tan −1 x x − 1 ∫ 2


−1

(
= e sin 2 x − 2 cos 2 x ⋅ e − ∫ − 2 sin 2 x ⋅ e dx ) 2 2 x +1



x x x

( )


 x 2 + 1 − 1
( )
2
= e sin 2 x − 2 e cos 2 x − 4∫ e sin 2 x dx = tan −1 x x − 1 ∫ 
x x x
 dx
2 2  x2 + 1 




= ex sin 2x – 2ex cos 2x – 4I  


⇒ 5I = ex (sin 2x – 2cos 2x) + C x 2 tan −1 x − 1  − 1  dx
2 ∫  1 + x2 
= 1 
2





Comparing with KI = ex (sin 2x – 2cos 2x) + C, we get K = 5.

5. x sin −1 x + 1 − x 2 + C;
4. (a) I = ∫ tan
−1
x dx


∫ (sin ) ( ) x dx




Put x = tan2 q, then dx = 2 tan q sec2 q dq Hint: I = −1
x .1dx = sin −1 x x − ∫
1 − x2




I = 2∫ θ tan θ sec θ d θ
2
1 −2x dx
= x sin x + ∫
−1


Now, let tan q = z, then sec2 q dq = dz 2 1 − x2



 z2 1 z2 
I = 2∫ z tan −1 z dz = 2  tan −1 z − ∫ dz  = x sin −1 x + 1 × 2 1 − x 2 + C
 2 2 1 + z 2
 2




 z2 1 z2 + 1 − 1  6. x sec −1 x − log x + x 2 − 1 + C;
= 2  tan −1 z − ∫ dz 


2 2 1 + z2 


∫ (sec )
−1
Hint: I = x . 1 dx
= z2 tan–1 z – z + tan–1 z = (z2 + 1) tan–1 z – z





= q(1 + tan2 q) – tan q
(
I = sec −1 x x − ∫ ) x dx




⇒ I = tan −1 x (1 + x) − x + C x x2 − 1



= x sec −1 x − ∫ dx
1 = x sec −1 x − log x + x 2 − 1 + C
5. (b) I = ∫ log (log x) dx + ∫ (log x) dx x2 − 1



2




x3 cot −1 x x 2 1
Integrating first integral by parts, we get 7. + − log (x 2 + 1) + C;
3 6 6



1 1
I = x log (log x) − ∫ x dx + ∫
∫ (cot )
dx −1
x log x (log x) 2 Hint: I = x x 2 dx





210 Mathe atics–12
m
E:\AMIT_WORKS\Exam_Guru\EG_Mathematics-12_(working_02-06-2022)\EG_Mathematics-12_working\Open_Files\Chap_7\Chap_7
\ 17-Aug-2022  Amit   Proof-5 Reader’s Sign _______________________ Date __________


( )
2 x x x
x3 + 1 x3 dx = x a − 2xa 2 + ∫ 2a 2 dx
3 3 ∫ x2 + 1
= cot −1 x
log a ( log a ) (log a )





3
 
= x cot −1 x + 1 ∫  x − 1  22x   dx
2 x x x
= x a − 2xa 2 + 2a 3 + C
3 3  2  x + 1  log a ( log a ) ( log a )




3  2  3. x(tan x – sec x) – log | sec x | + log |sec x + tan x | + C
= x cot −1 x + 1  x − 1 log x 2 + 1  + C









3 3 2 2  x x (1 − sin x )


Hint: =
1+ sin x cos 2 x




3 2
= x cot −1 x + x − 1 log x 2 + 1 + C = x sec2 x – x sec x tan x
3 6 6




( )
8. x sec −1 x − x − 1 + C;
Now integrate two functions by parts.


2


(3x + 7) sin 2x 3 cos 2x
4. 3x + 7x − − + C;
∫ (sec ) 4 2 4 8
−1
Hint: I = x .1 dx






(3x + 7) (1 − cos 2x)
(
= sec −1 x x − ∫ ) x dx
Hint: (3x + 7) sin 2 x =
2


2 x x x −1


1
= (3x + 7) − 12 (3x + 7)(cos 2x )
(
= x sec −1 x − 1 ∫ dx ) 2



2 x −1 Now integrate two functions.



1  e xn  [x n − 1] + C
(
= x sec −1 x − 1 × 2 x − 1 + C
2 ) 5.
n 





2n −1 ( xy ) dx =
9. (x + 1) tan −1 x − x + C; Hint: ∫ ( x) e ∫x
n  xn 
e ( x ) dx
n −1


 
∫ ( tan )



−1
Hint: I = x dx Now substitute xn = z






dz
Put tan −1 x = θ ⇒ x = tan2 q ⇒ xn – 1 dx =
n


­




⇒ dx = 2 tan q sec q dq
2 1
I = ∫ z e z dz



n


(
I = 2∫ θ × tan θ sec 2 θ d θ ) Now integrate by parts.



6. − 2x cos x + 4 x sin x + 4 cos x + C;
tan 2 θ − 2 tan 2 θ d θ


= 2θ
2 ∫ 2 Hint: I = ∫ x sin x dx





= θ tan 2 θ − ∫ sec 2 θ − 1 d θ ( ) Put x =z ⇒ dx = 2 x dz = 2z dz







= q tan q – tan q + q + C
2
I= ∫ z sin z × 2z dz


( )


= tan −1 x x − x + tan −1 x + C I = 2∫ z 2 sin z dz




= ( x + 1) tan
−1
x − x +C = 2  −z 2 cos z + ∫ 2z cos z dz 




10. x tan x + log | cos x | + C
= 2  −z 2 cos z + 2 z sin z − ∫ sin z dz  { }




 
∫ x sec
2
Hint: I = x dx


= 2[–z2 cos z + 2z sin z + 2 cos z] + C





= x tan x − ∫ tan x dx = 2  −x cos x + 2 x sin x + 2 cos x  + C




= x tan x + log | cos x | + C


= −2x cos x + 4 x sin x + 4 cos x + C



III. I. (x2 + 1)ex + C


− log | x |

7. x
∫ ( x + 1)
2 x + log + C;
Hint: I = e dx x +1 x +1





I = ( x + 1) e − 2∫ ( x + 1) e dx
x 2 x
⇒ log x  1 
Hint: I = ∫ ( x + 1)2 dx = ∫ (log x )  ( x + 1)2  dx






I = ( x + 1) e x − 2 ( x + 1) e x − ∫ e x dx 
2
⇒  



Now integrate by parts
⇒ I = (x + 1)2ex – 2(x + 1)ex + 2ex + C
( )∫(

I = − ( log x 1 + dx
)



­
⇒ I = [x2 + 2x + 1 – 2x – 2 + 2]ex + C
x +1 x x + 1)





⇒ I = [x2 + 1]ex + C

) ∫( )



2 x
2x a x log x 1 − 1 dx
2. x a − ax
+ 2 3 +C; ⇒ I= −
(x +1
+
x x +1



log a (log a) 2
(log a)


log x
x 2a x 2xa x ⇒ I= − + log x + C
∫ x a dx = log a − ∫ log a dx
2 x
Hint: I = x +1 x +1






ntegrals 211
I

E:\AMIT_WORKS\Exam_Guru\EG_Mathematics-12_(working_02-06-2022)\EG_Mathematics-12_working\Open_Files\Chap_7\Chap_7
\ 17-Aug-2022  Amit   Proof 5 Reader’s Sign _______________________ Date __________


8. x log(x2 + 1) + 2 tan–1 x – 2x + C; 1  −θ cos 2θ + sin 2θ  C
= +


Hint: I = (
∫ log x + 1 × 1 dx
2
) 2  2 4 



( )



 −θ 1 − 2 sin 2 θ 2 
= 1 + 2 sin θ 1 − sin θ  + C
( 2x 2
I = log x + 1 x − ∫ 2 dx 2
) 2 2 4 



x +1


(
I = x log x 2 + 1 − 2∫ ) ( x + 1) − 1 dx
2

= 
 2
(
1 2 sin θ − 1 θ + sin θ 1 − sin 2 θ  )
2
2 2 +C
x +1 2




 
I = x log(x2 + 1) – 2[x – tan–1 x] + C
1
( )
=  2x 2 − 1 sin −1 x + x 1 − x 2  + C


2 2 2 2
9. x log x + x + a − x + a + C; 4 





Hint: I = (
∫ log x + x + a × 1 dx
2 2
) 13. x(sin −1 x) 2 + 2 (sin −1 x) 1 − x 2 − 2x + C;



∫ (sin )
2



−1
Hint: I = x dx . Put sin–1 x = q
( 1 2x
)





⇒ I = log x + x + a ( x ) − ∫ dx 2 2
2 ⇒ x = sin q ⇒ dx = cos q dq
x + a2
2








( ) ∫θ cos θ d θ = θ 2 sin θ − ∫ 2θ sin θ d θ
2
⇒ I = x log x + x 2 + a 2 − x 2 + a 2 + C I=






10. xex[log(xex) – 1] + C; I = θ 2 sin θ − 2  −θ sin θ + ∫ cos θ d θ 
 





Hint: I = ∫ ( x + 1) e
x
log xe x dx ( ) ⇒ I = q2 sin q + 2q cos q – 2 sin q + C




( )
2



Put xex = z ⇒ I = sin −1 x x + 2 sin −1 x 1 − x 2 − 2x + C






⇒ (xex + ex)dx = dz
x 2 (sec −1 x) − 1



⇒ (x + 1)ex dx = dz 14. x 2 −1 + C;
2 2





⇒ I= ∫ (log z ) × 1 dz Hint: I = ∫ (sec
−1
)
x x dx






Integrate by parts
(x2 −
)
2

1
I = ( log z ) z − ∫ × z dz ⇒
−1
I = sec x ∫
1 × x dx
z 2 x x2 − 1 2





I = xex log(xex) – (xex) + C
( )
2
⇒ I = x sec −1 x − 1 ∫ 2x dx


 sin 2x  log  cosx + sin x  − 1 log sec 2x + C 2 4 x2 − 1



11. 
 2   cos x − sin x  2


(
x 2 sec −1 x − 1  x 2 − 
)
( )
⇒ I= 2 1 +C
cos x + sin x
(cos 2x ) dx 2 4  
∫ log



Hint: I =
cos x − sin x



2
I = x (sec −1 x ) − 1 x 2 − 1 + C
= log (
cos x + sin x sin 2x
)( ) 2 2


cos x − sin x 2


3
x3 cos −1 x 2
− 1 − x + 1 (1 − x 2) 2 + C;
( 
−∫ cos x − sin x  2
)(
 sin 2x
dx ( ) 15.
3 3 9


cos x + sin x  cos x − sin x )2  2
∫ (cos )
  −1
x x 2 dx
Hint: I =

( )



 cos x + sin x   sin 2x  Substitute cos x = q –1
⇒ x = cos q
= log
 cos x − sin x   2 





⇒ dx = – sin q dq
sin 2x



−∫   dx
∫ θ cos θ ( − sin θ) d θ
2
 cos 2 x − sin 2 x  I=


( )
3 3
sin 2x log cos x + sin x − tan 2x dx = θ cos θ − ∫ cos θ d θ
cos x − sin x ∫
= θ 3


2


1 ( x 2 − ) sin −1 x + x − x 2  + C;
3
= θ cos θ −
1 − sin 2 θ cos θ ( )
12.
4 
2 1 1
 3 ∫ 3





∫ (sin )
−1 Put sin q = z ⇒ cos q dq = dz
Hint: I = x x dx





θ cos3 θ − 1 1 − z 2 dz
( )



3∫
Put sin x = q –1 =
3



⇒ x = sin q ⇒ dx = cos q dq 3 3

= θ cos θ − 1  z − z  + C





1
I = ∫ θ sin θ cos θ d θ = 2 ∫ θ sin 2θ d θ 3 3 3




3
=
1  θ cos 2θ
− +∫
cos 2θ d θ  = θ cos θ − sin θ + 1 sin 3 θ + C
2  2 2  3 3 9




212 Mathe atics–12
m
E:\AMIT_WORKS\Exam_Guru\EG_Mathematics-12_(working_02-06-2022)\EG_Mathematics-12_working\Open_Files\Chap_7\Chap_7
\ 17-Aug-2022  Amit   Proof-5 Reader’s Sign _______________________ Date __________


3 Put tan–1 x = q ⇒ x = tan q
( )
3
1
I = θ cos θ − 1 1 − cos 2 θ + 1 − cos 2 θ




2 +C ⇒ dx = sec2 q dq
3 3 9





θ
I= ∫ × sec 2 θ d θ
x3 cos −1 x − 1 − x 2 + 1 1 − x 2
( ) (1 + tan θ)
3/ 2 2
2



= +C
3 3 9


(x 2 + 1) (tan −1 x) 2 θ sec2 θ
16. − x tan −1 x + 1 log (x 2 + 1) + C; I= ∫ dθ
2 2 sec4 θ




∫ ( tan )
2
−1 1 θ 1 + cos 2θ d θ
Hint: I = x x dx = ∫ θ cos 2 θ d θ =
2∫
( )





Substitute tan–1 x = q ⇒ x = tan q 2
= θ + 1 ∫ θ cos 2θ d θ



⇒ dx = sec2 q dq
4 2






⇒ I=
∫θ
2
( tan θ sec θ) d θ 2
θ 2 1  θ sin 2θ



sin 2θ d θ 
= + −∫
2 2 4 2  2 2 
= θ 2 tan θ − ∫ 2θ tan θ d θ



2 2


2
= θ + θ sin 2θ + cos 2θ + C
( )
2 2
= θ tan θ − ∫ θ sec 2 θ − 1 d θ 4 4 8



2


2  2 
2 2
θ tan θ − θ sec2 θ d θ + θ d θ I = θ + θ tan θ2 + 1  1 − tan 2 θ 
4 2 1 + tan θ 8  1 + tan θ 
∫ ∫


=
2


( tan x) −1 2
θ 2 tan 2 θ − θ tan θ − tan θ d θ  + θ 2 + C −1 1  1 − x2  + C
= ∫ I= + tan x  x 2  + 
2   2 4 2  1 + x  8  1 + x 2 




2 2 2
= θ tan θ − θ tan θ + log (sec θ ) + θ + C 19.
1  x cos −1 x − 1 − x 2  + C
2 2 2  




( ) ( )
2
= x tan −1 x − tan −1 x x
2
1 − x dx
∫ tan
−1
2 Hint: I =
1+ x


) ( )



2
tan −1 x
(
+ 1 log 1 + x 2 + +C Put x = cos q ⇒ dx = –sin q dq





2 2  
I = −∫  tan −1 1 − cos θ  (sin θ d θ )
 2 
( 1
)
=  x + 1 tan −1 x − x tan −1 x + log x 2 + 1 + C
2
( )  1 + cos θ 


 2  2


1 θ sin θ d θ
2∫
=−
−1 2


17. − sin x + log 1 − 1 − x + C;
x x = − 1 θ ( − cos θ ) + ∫ cos θ d θ 


2 


Hint: I =
 sin −1 x  −1  1 
∫  x 2  dx = ∫ sin x  x 2  dx ( ) = − 1 [ −θ cos θ + sin θ ] + C
2





1
Put sin–1 x = q ⇒ x = sin q = −  −θ cos θ + 1 − cos 2 θ  + C
2 





⇒ dx = cos q dq



1
 1  = −  −x cos −1 x + 1 − x 2  + C
I= ∫ (θ)  sin 2 θ  cos θ d θ 2 




−1
= ∫ θ ( cosec θ cot θ ) d θ = x cos x − 1 1 − x 2 + C
2 2




= θ ( − cosec θ ) + ∫ cosec θ d θ 
( )
  
20. a  x + a  tan −1 x  − x  + C;


a  a  a


 
= −θ + log cosec θ − cot θ + C
sin θ x dx


∫ sin
−1
Hint: I =
x+a



−1 2
= − sin x + log 1 − 1 − x + C Substitute: x = a tan2 θ
x x



⇒ dx = 2a tan θ sec2 d θ



θ
−1 2 −1 2
18. (tan x) − x tan 2x + (1 − x )2 + C; (
a tan 2 θ 2a tan θ sec 2 θ d θ
)
∫ sin
−1
4 I=
2 (1 + x ) 8 (1 + x ) a + a tan 2 θ




tan −1 x dx
∫ ( )
2
Hint: I = = ∫ sin −1 tan 2 θ 2a tan θ sec 2 θ d θ
( )
2



1 + x2 sec θ


ntegrals 213
I

E:\AMIT_WORKS\Exam_Guru\EG_Mathematics-12_(working_02-06-2022)\EG_Mathematics-12_working\Open_Files\Chap_7\Chap_7
\ 17-Aug-2022  Amit   Proof 5 Reader’s Sign _______________________ Date __________


(
= ∫ sin −1 (sin θ ) 2a tan θ sec2 θ d θ )  2 
I = 2a  θ tan θ − 1 tan θ + 1 θ  + C


 2 2 2 



(
= 2a∫ θ tan θ sec 2 θ d θ ) = a[q tan2 q – tan q + q] + C




 2 2   
I = 2a θ tan θ − ∫ tan θ d θ  = a  x tan −1 x − x + tan −1 x  + C
2 2  a a a a 



 


( ) 
( ) 
 2 
I = 2a  θ tan θ − 1 ∫ sec 2 θ − 1 d θ  + C = a  x + a tan −1 x − x  + C
 2 2   a a a





Topic 8. Repeating After Twice Integration
Example 1. Evaluate: ∫ (sin 4x) e
5x
dx (sin 4x) e5x
⇒ I=
5




∫ (sin 4x) e
5x
Solution. I = dx
4 e5x  e5x 
(cos 4x) + ∫ (4 sin 4x) dx


− 
5 5  5 


⇒ I = (sin 4x) ∫ e5x dx − ∫ (sin 4x)′ ∫ e5x dx  dx


(sin 4x) e5x 4  e5x 4 
5x 5x ⇒ I= −  (cos 4x) + I
⇒ I = (sin 4x) e − ∫ 4(cos 4x) e dx 5 5  5 5 




5 5


(sin 4x) e5x 4 16
5x
⇒ I= − (cos 4x) e5x − I
⇒ I = (sin 4x) e − 4 ∫ (cos 4x) e5x dx 5 25 25



5 5


5x
⇒ I=
(sin 4x) e5x ⇒ 41 I = e [5 (sin 4x) − 4 (cos 4x) ] + C
25 25



5


4 e5x
− (cos 4x) ∫ e5xdx − ∫ (cos 4x)′ ∫ e5x dx  dx  ⇒ I = [5(sin 4x) − 4(cos 4x)] + C
5  41


EXERCISE 7.8
I. Multiple Choice Questions (MCQs)
3. ∫e
2x
⋅ sin x cos x dx =


1. ∫ e sin bx dx is equal to
ax
2x
(a) e (sin 2 x + cos 2 x) + C


8


e ax
(a) (a sin bx − b cos bx) + C 2x
a + b2
2 e (cos 2 x − sin 2 x)


(b) +C
8


e ax
(b) (a sin bx − b cos bx) + C e 2 x (sin 2 x − cos 2 x)
a − b2
2
(c) +C


8


e ax (d) None of these
(c) (a sin bx − b cos bx) + C


b2 − a 2


Evaluate each of the following Integrals
e ax
(d) (a sin bx + b cos bx) + C
a + b2
2 II. Long Answer Type Questions


∫ (sin mx) e dx
nx
1.
∫e
ax
2. cos (bx + c) dx equals




∫ (cos mx) e dx
nx
2.
e ax [a cos (bx + c) − b sin (bx + c)]


(a) + C1
∫ sec x dx
3
(a 2 + b 2 ) 3. [V. Imp.]





ax
e [a cos (bx + c) + b sin (bx + c)]
(b)
(a 2 + b 2 )
+ C1 4. ∫ sin (log x) dx




(cos 2x) dx
(c)
e ax [a cos (bx + c) + b sin (bx + c)]
+ C1
5. ∫ sin 3 x


(a 2 − b 2 )


6. ∫ e .sin zx dx [CBSE 2022]
x
(d) None of these





214 Mathe atics–12
m
E:\AMIT_WORKS\Exam_Guru\EG_Mathematics-12_(working_02-06-2022)\EG_Mathematics-12_working\Open_Files\Chap_7\Chap_7
\ 17-Aug-2022  Amit   Proof-5 Reader’s Sign _______________________ Date __________


Answers 7.8
1 2x
∫e
ax
I. 1. (a) I= sin bx dx
∫e 2∫
2x
3. (c) I= ⋅ sin x cos x dx = e (2 sin x cos x) dx







Integrating by parts, we get
1 2x 1

2∫
 − cos bx   cos bx  ⇒ I= e sin 2 x dx = I1
 b  ∫
I = e ax  − ae ax ⋅  −  dx 2




 b 


∫e
2x
Let I1 = sin 2 x dx
1 a




= − e ax cos bx + ∫ e ax cos bx dx
b b


2 x  − cos 2 x  2 x  − cos 2 x 

2  ∫
1 a sin bx sin bx  =e   − 2e ⋅   dx
 2 
− ∫ ae ax ⋅



= − e ax cos bx + e ax ⋅ dx 
b b b b 


e 2 x cos 2 x
1 a a 2 =− + ∫ e 2 x .cos 2 x dx
= − e ax cos bx + 2 e ax sin bx − 2 ∫ e ax sin bx dx 2



b b b


e 2 x ⋅ cos 2 x 2 x  sin 2 x  sin 2 x
 2  ∫
1 a2 =− +e  − 2e 2 x ⋅ dx
2 2



= 2 e ax ( a sin bx − b cos bx) − 2 I
b b


e 2 x cos 2 x 2 x sin 2 x
a 1 ax 2 =− +e ⋅ − ∫ e 2 x sin 2 x dx
⇒ I = 2 e ( a sin bx − b cos bx) 2 2



I+
b2 b



e 2 x (sin 2 x − cos 2 x)
 a 2 + b2  1 = − I1
⇒ I = 2 e ax (a sin bx − b cos bx) 2



 b 2 
 b



e 2 x (sin 2 x − cos 2 x)
ax \ I1 = − I1
e
2



\ I= (a sin bx − b cos bx) + C
a 2 + b2



e 2 x (sin 2 x − cos 2 x)
⇒ 2I1 =
∫e
ax
2. (b) I = cos(bx + c) dx 2







e 2 x (sin 2 x − cos 2 x)
Integrating by parts, we get ⇒ I1 =
4




sin (bx + c) sin (bx + c)
I = e ax ⋅ − ∫ ae ax ⋅ dx 1 e 2 x (sin 2 x − cos 2 x)
b b Hence, I= I1 = +C


2 8


e ax sin (bx + c) a ax
= − ∫ e sin (bx + c) dx enx  n sin mx − m cos mx  + C;
b b II. 1. ( ) ( )


m + n2  2
e ax sin (bx + c) a ax a2
= + 2 e cos(bx + c) − 2 ∫ e ax cos(bx + c) dx
∫ (sin mx ) e
nx
b b b Hint: I = dx





 a 2  ax [b sin (bx + c) + a cos(bx + c)] e ax enx − m cos mx enx dx
= sin ( mx ) ( )n
i.e., 1 + 2  ∫ e cos(bx + c) dx =
 b  b2 n ∫


1 m
e ax [a cos(bx + c) + b sin (bx + c)] I = sin ( mx ) enx − ∫ cos ( mx ) enxdx
Thus, ∫ e ax cos(bx + c) dx = + C1 n n


(a 2 + b 2 )  nx

...(1) = 1 sin ( mx ) enx − m cos ( mx ) e


n n n



nx 
In order to express this result in a convenient form, we substitute + ∫ m sin ( mx ) e dx 
a = r cos q and b = r sin q n 


b 1 m 1
So that r= a 2 + b 2 andθ = tan −1 = sin ( mx ) enx −  cos ( mx ) enx + m I 
a n n  n n 




We have a cos (bx + c) + b sin (bx + c) 1 m m2
= sin ( mx ) enx − 2 cos ( mx ) enx − 2 I
n n n


= r [cos (bx + c) cos q + sin (bx + c) sin q]


= r cos (bx + c – q)  m2  1
1 + n 2  I = n 2  n sin ( mx ) − m cos ( mx ) e
nx




 b
= a 2 + b 2 cos (bx + c) − tan −1 
 a enx  n sin mx − m cos mx  + C
( ) ( )


I=
m + n2 
2


Hence, (1) is transformed into
b enx  n cos mx + n sin mx  + C;

e ax cos  bx + c − tan −1  2. ( ) ( )
 a m + n2 2


∫e
ax
cos (bx + c) dx = + k.
a ² + b² Hint: Solve as we have solved Q.No. 1.

ntegrals 215
I

E:\AMIT_WORKS\Exam_Guru\EG_Mathematics-12_(working_02-06-2022)\EG_Mathematics-12_working\Open_Files\Chap_7\Chap_7
\ 17-Aug-2022  Amit   Proof 5 Reader’s Sign _______________________ Date __________


3. 1 sec x tan x + log sec x + tan x  + C; 5. − 1 [ cosec x cot x + 3 log | cosec x − cot x |] + C;
2 2




Hint: I = ∫ sec
3
x dx = ∫ sec x sec2 x dx( ) Hint: I =  cos 2x 
∫  sin 3 x  dx = ∫ 
 1 − 2 sin 2 x 
sin 3 x 
dx






= sec x tan x − ∫ sec x tan x tan x dx = ∫ cosec3 x dx − 2∫ cosec x dx




⇒ I = sec x tan x − ∫ sec x sec2 x − 1 dx ( ) I = I1 – 2 log|cosec x – cot x| ...(1)




( )



∫ cosec x dx = ∫ cosec x cosec2 x dx
3
I1 =
I = sec x tan x − ∫ sec x dx + ∫ sec x dx
3






⇒ I = sec x tan x – I + log(sec x + tan x) ⇒ I1 = – cosec x cot x + ∫ ( cosec x cot x )( − cot x ) dx








⇒ I=
1 sec x tan x + log sec x + tan x  + C
2  ⇒ I1 = – cosec x cot x −∫ ( cosec x ) cosec 2 x − 1 dx ( )








4. x [sin (log x) − cos (log x) ] + C; ⇒ I1 = – cosec x cot x −∫ cosec3 x dx + ∫ cosec x dx





2


⇒ I1 = – cosec x cot x – I1 + log|cosec x – cot x|
Hint: I = ∫ sin (log x ) dx = ∫ sin x (log x ) × 1 dx









From eqn (1), we have
cos ( log x )
I = sin ( log x )( x ) − ∫


× x dx ⇒ I = I1 – 2 log | cosec x – cot x |
x








⇒ I = x sin ( log x ) − ∫ cos ( log x ).1 dx 1 1
I = − cosec x cot x + log cosec x − cot x
2 2





sin ( log x ) 
⇒ I = x sin ( log x ) − cos ( log x )( x ) + ∫ x dx  – 2 log|cosec x – cot x|
x



 ex
⇒ I = x sin (log x) – x cos (log x) – I 6. [sin 2 x − 2 cos 2 x] + C
5




x sin log x − cos log x  + C
⇒ I=
2
( ) ( ) Hint: Put n = 1 and m = 2 in sol no. 1




Topic 9. Integrals of the Exponential Form
x+3 Integrate only the first part:
Example 1. ∫ ( x + 4)2 .e x dx = .

(
I =  f ( x ) ∫ e x dx − ∫ f ′ ( x ) ∫ e x dx dx 
  )


x+3 x x + 4 −1 x
Solution. Let I = ∫ . e dx = ∫ . e dx
( x + 4) 2 ( x + 4) 2 +∫ f ′ ( x ) e xdx

 x+4 1  x
=f ( x ) e x − ∫ f ′ ( x ) e xdx + ∫ f ′ ( x ) e x dx
∫
= 2

 ( x + 4) ( x + 4) 2 
e dx




 1 1  =f (x)ex + C

∫  x + 4 − ( x + 4)2  e x dx


=
xe xdx


Example 3. Evaluate: ∫ (x + 1)2 [V. Imp.]

1 1
Put = t  − dx = dt
x+4 ( x + 4) 2 xe xdx [(x + 1) − 1] e x dx


1 1
Solution. I = ∫ (x + 1)2 ∫ (x + 1)2
=


Let f(x) = \ f (x) = −
x+4 ( x + 4) 2




 1 1 
∫  x + 1 − (x + 1)2  e dx
x
⇒ I=
Using ∫ e x [ f ( x) + f ′ ( x)] dx = ex f(x) + C



x 1
\ I= e . +C  1  ex
∫  x + 1 e dx − ∫ (x + 1)2 dx
x
x+4 ⇒ I=





ex
Hence, I= + C. Integrate the first part only:
x+4



 ′ 
 1  e x dx
∫  f ( x ) + f ′ ( x ) e dx 1
x
Example 2. Evaluate: e xdx − ∫  e x dx  dx − ∫
I=
x +1 ∫ 
  x + 1)  ∫  (x + 1) 2

 
∫  f ( x ) + f ′ ( x ) e dx
x
Solution. I=


 1  x e x dx e x dx ex
= x + 1 e + ∫ 2 −∫ 2 = (x + 1)
+C
= f ( x ) e dx +
∫ ∫ f ′ ( x) e
x x
dx (x + 1) (x + 1)




216 Mathe atics–12
m
E:\AMIT_WORKS\Exam_Guru\EG_Mathematics-12_(working_02-06-2022)\EG_Mathematics-12_working\Open_Files\Chap_7\Chap_7
\ 17-Aug-2022  Amit   Proof-5 Reader’s Sign _______________________ Date __________


EXERCISE 7.9
I. Multiple Choice Questions (MCQs)
 sin 4x − 4 
∫ 1 − cos 4x  e
x
9. dx [Delhi 2010]
1. ∫ e (1 − cot x + cot x ) dx =
x 2






(a) ex cot x + C (b) –ex cot x + C
 (2 − sin x)  2x
∫  1 − cos x  e dx




(c) ex cosec x + C (d) –ex cosec x + C 10. [V. Imp.]








( x + 3) e x
2. ∫ ( x + 4) 2
dx = [NCERT Exemplar]
11. ∫
(x − sin x) dx



1 − cos x



ex ex
(a) +C (b) +C  x + sin x 
x+4 x+3 12. ∫  1 + cos x  dx






1 ex
(c) +C (d) +C
( x + 4) 2 ( x + 4) 2  (2 − 5 sin 2x)  e5xdx
∫ 




13.
1 − cos 2x 



2
 log x − 1 
3. ∫  1 + ( log x )2  dx is equal to  1 


14. ∫ log (log x) + (log x)2  dx [V. Imp.] [Delhi 2010 (C)]




x
x xe
(a) +C (b) +C
( log x ) 2 + 1 1 + x2  1 + sin 2x 




∫  1 − cos 2x  e
−x
15. dx [V. Imp.]
x log x



(c) +C (d) +C  
x2 + 1 ( log x ) 2 + 1




1 − sin x  − x
Evaluate each of the following Integrals 16. ∫  e  dx
2 [V. Imp.] [A.I. 2013 (C)]
1 + cos x  



II. Short Answer Type Questions-I
∫ (sin x) a
x
(x − 3) e x dx 17. dx
1. ∫ (x + 1)3 [Delhi 2009]





2 x
III. Short Answer Type Questions-II
2. ∫ (x + 1) e 2 dx
(x + 1)
[Delhi 2005, 2006 (C)
( x cos x) dx −1



(x − 4) e x dx
1. ∫ 1 − x2
[A.I. 2014 (C)]



3. ∫ [Delhi 2008]
(x − 2) 2
( )



x 2 + 1 log x 2 + 1 − 2 log x  dx
(1 − x) 2 e x dx 2.  
4. ∫ ∫ x4
[A.I. 2011, 2014 (C)]



(x 2 + 1) 2


 1 − sin x  x
2 3. ∫  e dx
 x + 2 x  1 − cos x 
∫  x + 4  e dx


5.


IV. Long Answer Type Questions
(5x + 14) e5x dx
6. ∫
(x + 3) 2 (sin −1
x − cos −1 x dx )
∫ (sin


1. −1
x + cos −1
x )


7. ∫ [sin (log x) + cos (log x)] dx [V. Imp.]
x dx 2



2. ∫ [Delhi 2012]
∫ 3x tan x + x sec x  dx
2 3 2
( x sin x + cos x )2



8.


Answers 7.9
∫e ( x + 3) e x ( x + 4 − 1) x
x
I. 1. (b) I = (1 − cot x + cot 2 x) dx
2. (a) I = ∫ dx = ∫ e dx


( x + 4) 2 ( x + 4) 2




= ∫ e {(1+ cot x ) − cot x} dx
x 2
 1 1  x 1 x
= ∫


− 2
e dx = e +C
 x + 4 ( x + 4)  x + 4


= ∫ e { cosec x + ( − cot x )}dx
x 2


Q e x [ f ( x) + f ′ ( x)] dx = e x f ( x) 
 ∫ 
{
= − ∫ e cot x + −cosec x dx ( )}

x 2


2
 log x − 1  (log x − 1) 2
= –e cot x + C
x Q e { f ( x) + f ′ ( x)} dx = e f ( x) 
 ∫
x x

3. (a) I = ∫  1 + (log x )2 
  dx = ∫ {1 + (log x )2 }2 dx







ntegrals 217
I

E:\AMIT_WORKS\Exam_Guru\EG_Mathematics-12_(working_02-06-2022)\EG_Mathematics-12_working\Open_Files\Chap_7\Chap_7
\ 17-Aug-2022  Amit   Proof 5 Reader’s Sign _______________________ Date __________


Put log x = t, then et = x ⇒ et dt = dx
   
=  x  ex +  4 2  ex

 1 2t   (x + 4)   (x + 4) 
\ I = ∫e  t
2
− 2 2 
dt
1 + t {1 + (t )}



 


Integrate only the first part.


1
It is of the form ∫ e [ f (t ) + f ′ (t )] dt , where f (t ) =
t
. e5x + C;
1 + t2 6. Ans.

x+3



1 x
t
= e f (t ) + C = e
t
+C= +C (5x + 14) e5x  5(x + 3) − 1 5x
1 + t2 1 + (log x ) 2 Hint: = 2 e
(x + 3) 2


 (x + 3) 


ex + C ;
II. 1. Ans.
( x − 1)2
x x
=5 ( )
1 e5x −  1  e5x
x+3  (x + 3) 2 

Hint: (x − 3) e3 = [(x − 1) − 23 ] e


Integrate only the first part.
(x − 1) (x − 1)


7. Ans. x sin (log x) + C;
 2  ex



= 1 2 −  Hint: Integrate only the first part.
 (x − 1) (x − 1)3 


8. Ans. x3 tan x + C;




 1  x  2  x Hint: Integrate only the first part.


= 2 e −  3 e 9. Ans. [cot 2x] ex + C;
 (x − 1)   ( x − 1) 



Hint:


Integrate only the first part.
 sin 4x − 4  x  2 sin 2x cos 2x − 4  x
( )

 1 − cos 4x  e =  e
2. Ans. x − 1 e x + C;
 2 sin 2 2x 
x +1


= [cot 2x]ex – [2 cosec2 2x]ex

 x 2 + 1  x  (x 2 − 1) + 2  x Integrate only the first part.
Hint:  2 e =  2 e

 ( x + 1)   (x + 1) 
( )
x
10. Ans. − 2 cot x e 2 + C;

2


   
=  x − 1  ex +  2 2  ex  x x
 (x + 1 )   (x + 1)   2 − sin x  2x  2 − 2 sin 2 cos 2  2x
Hint:  e = e
 1 − cos x   2 x 


Integrate only the first part. 2 sin
 2 

( )
 x  x x
3. Ans.  e  + C; =  − cot x  e 2 + cosec 2 x e 2
 x − 2  2  2



 x − 4  x  (x − 2) − 2  x Now integrate the first part only.
Hint:  2 e =  2 e

 (x − 2)   (x − 2)  x

11. Ans. − x cot + C ;
2


 1 2  e x. x − 2 sin x cos x
x −2 − 
(x − 2) 2 
Integrate only the first part. Hint: x − sin x = 2 2


1 − cos x 2x

2 sin
x 2
4. Ans.  2e  + C;
 x + 1 = 1 x cosec2 x − cot x


2 2 2
 (1 − x) 2  x  (x 2+ 1 − 2x)  x

Hint:  2 2
e = 2 2 e Integrate only the first part.
 (x + 1)   (x + 1) 


12. Ans. x tan x + C; Hint : x + sin x
 2    2 1 + cos x


=  x + 1 2  e x −  2x 2  e x
x + 2 sin x cos x x
( ) ( ) ( )
 2   2 
 x + 1   x + 1  = 2 2 = sec2 x + tan x
2 x 2 2 2
2 cos

 x  2
=  21  e x −  22 2  e x


 x + 1  (x + 1)  Integrate only the first part.

13. Ans. –(cot x)e5x + C;

Integrate only the first part.



   2 − 10 sin x cos x  5x
Hint:  2 − 5 sin 2x  e5x = 
x
5. Ans. xe + C; 2 sin 2 x
e
 1 − cos 2x   

x+4


( )
2
x + 2 e x =  x 2 + 4x + 4  e x = – [5 cot x]e5x + [cosec2 x]e5x
Hint:  2 


x+4  (x + 4)  Now integrate only the first part.


218 Mathe atics–12
m
E:\AMIT_WORKS\Exam_Guru\EG_Mathematics-12_(working_02-06-2022)\EG_Mathematics-12_working\Open_Files\Chap_7\Chap_7
\ 17-Aug-2022  Amit   Proof-5 Reader’s Sign _______________________ Date __________


⇒ dx = –sin q dq
 
14. Ans. [x log (log x)] −  x  + C;



 log x  (cos θ) θ ( − sin θ) d θ = (cos θ) θ ( − sin θ) d θ = − θ cos θ d θ


∫ 2 ∫ sin θ ∫
Hint: Integrate only the first part twice. 1 − cos θ



−1 Now, integrate by parts
15. Ans. (cosec x) e −x + C;


2


3
2  
 1 + sin 2x  −x  sin x + cos x  −x 2. Ans. −1 1 + 12  3 log 1 + 12  − 2 + C;
e = 9  x    x  
Hint:  e



2
 1 − cos 2x   2 sin x 

x 2 + 1 log (x 2 + 1) − 2 log x 
= 1 [ cosec x ] e −x + 1 (cosec x cot x) e − x  + C; Hint:
2 2 x4


Integrate only the first part.  2 
x 2 + 1 log  x +2 1

 x  = x 2 + 1 log  x 2 + 1 1
( )
−x
=  x 2  x3
16. Ans. − sec x e 2 + C; x 4 x2
2



 cos x − sin x  −x
 1 − sin x  −x 2 2 e 2 = 1 + 12 log 1 + 12  13
Hint:  e 2 =  x  x x
  2 cos 2 x 
 1 + cos x 


 2 
Substitute: 1 + 12 = z

( ) ( )
x

−x −x
= 1 sec x e2 − 1 sec x tan x e2
2 2 2 2 2
( )
3. − cot x e x + C;

2


Integrate only the first part.

 1 − 2 sin x cos x 
17. ax
1 + ( log a )
( log a ) sin x − cos x  + C
2  Hint: 1 −
(sin x
1 − cos x
e =
x
 ) 2
2x
2  ex




 2 sin 
2
Hint: I = ∫ (sin x ) a dx
( ) ( )
x
= 1 cosec2 x e x − cot x e x

x x 2 2 2
 
I = (sin x ) a − ∫ ( cos x )  a  dx

log a  log a 
( )
Now integrate − cot x e x by parts
2


 ax  1 cos x  a x  + sin x  a x  dx 
I = (sin x )  −  log a  ∫ ( )  log a  
 log a  log a 

 IV. 1. Ans. 2 ( 2x − 1) sin −1 x + x 1 − x  − x + C;
π 

 ax   ax  −1 −1
I = (sin x )  
 log a 
− ( cos x )  2 −
1
 ( log a )  ( log a )
( )
2 ∫ (sin x ) a dx
x Hint: sin −1 x − cos −1 x
sin x + cos x

( )

a x  log a sin x − cos x  − 1 ( I ) sin −1 x − π − sin −1 x
I= ( ) 2 = 4 sin −1 x − 1
(log a )2  
(log a )2 =
π π
2

 1  I = a x  log a sin x − cos x 

1 + 2 ( ) Now substitute: sin −1 x = θ ⇒ x = sin 2 θ
 ( log a )  (log a )2  

 −x sec x 

2. Ans.   + tan x + C;
1 + ( log a )2  a x  log a sin x − cos x   x sin x + cos x 


 2 I = ( )
 ( log a )  (log a )2  
Hint: x2 =
(x sec x) (x cos x)

[x sin x + cos x]2 (x sin x + cos x) 2

ax
I= ( log a ) sin x − cos x 
2
1 + ( log a )
 x cos x 
= ( x sec x ) 

2
III. 1. Ans. − (cos −1 x) 1 − x 2 + x  + C;  ( x sin x + cos x ) 
 

Hint: Substitute: cos–1 x = q ⇒ x = cos q Now integrate by parts.




ntegrals 219
I

E:\AMIT_WORKS\Exam_Guru\EG_Mathematics-12_(working_02-06-2022)\EG_Mathematics-12_working\Open_Files\Chap_7\Chap_7
\ 17-Aug-2022  Amit   Proof 5 Reader’s Sign _______________________ Date __________


Topic 10. Three More Formulae for Integration
First Formula
Solution. I= ∫ x 2 + 4x − 7 dx = ∫ (x + 2) 2 − 11 dx



a2 x 2 −11
1A. ∫ x 2 + a 2 dx =
2
log x + x 2 + a 2 +
2
x + a2 + C =
2
log (x + 2) + (x + 2) 2 − 11




x+2
a2 + (x + 2) 2 − 11 + C
1B. ∫ (x ± b) 2 + a 2 dx =
2
log (x ± b) + (x ± b) 2 + a 2 2


− 11
x ± b)  ⇒ I= log (x + 2) + x 2 + 4x − 7 +
+  (x ± b) 2 + a 2 + C 2




 
2  x+2
x 2 + 4x − 7 + C
2


Example 1. Evaluate: ∫ x 2 + 6x + 11 dx Third Formula

Solution.I = ∫ x 2 + 6x + 11 dx ⇒I= ∫ ( x + 3) + 2 dx
2
3A. ∫ a 2 − x 2 dx =
a2
2
sin −1 ()
x x
+
a 2
a2 − x2 + C





  
2
=log ( x + 3) +
2
( x + 3)2 + 2 +
x+3
2
( x + 3)2 + 2 + C 3B. ∫ a 2 − (x ± b) 2 dx =
a2
2
sin −1
a( )
x±b
+





=log ( x + 3) + x 2 + 6x + 11 +
x+3 2
2
x + 6x + 11 + C ( )
x±b
2
a 2 − (x ± b) 2 + C


Second Formula
Example 3. Evaluate: ∫ 11 + 6x − x 2 dx .
−a 2
2A. ∫ x 2 − a 2 dx =
2
log x + x 2 − a 2 Solution. I = ∫ 11 + 6x − x 2 dx




x
+ x2 − a2 + C −  x 2 − 6x − 11 dx = ∫ − ( x − 3) − 20 dx
2
2 ⇒ I= ∫  



2B. − a2 =∫ 20 − (x − 3) 2 dx
∫ (x ± b) 2 − a 2 dx = log


2


(x ± b) + (x ± b) 2 − a 2 + ( )
x±b
2
(x ± b) 2 − a 2 + C
2

 20 

=20 sin −1  x − 3  + x − 3 20 − ( x − 3) + C
2
2


 
=10 sin −1  x − 3  + x − 3 11 + 6x − x 2 + C
Example 2. Evaluate: ∫ x 2 + 4x − 7 dx  20  2


EXERCISE 7.10
I. Multiple Choice Questions (MCQs) 1
(b) ( x + 4 ) x2 − 8x + 7 +
1. ∫ 1 + x 2 dx is equal to 2




x 1 9 log | x + 4 + x2 − 8x + 7 | + C
(a)
1 + x 2 + log x + 1 + x 2 + C

2 2


2 3
1
(b) (1 + x 2 ) 2 + C (c) ( x − 4 ) x2 − 8x + 7 −
3 2




3
2
(c) x (1 + x 2 ) 2 + C 3 2 log x − 4 + x2 − 8x + 7 + C
3



x2 x2
(d) 1 + x2 + log | x + 1 + x 2 | + C 1
2 2 (d) ( x − 4 ) x2 − 8x + 7 −


2


2. ∫ x 2 − 8 x + 7 dx is equal to
9


1 log x − 4 + x 2 − 8 x + 7 + C [NCERT]
(a) ( x − 4 ) x 2 − 8 x + 7 + 2
2


9 log | x − 4 + x 2 − 8 x + 7 | + C 3. ∫ 1 − 4 x − x 2 dx equals



220 Mathe atics–12
m
EXAM BITES

This Pdf Is
Downloaded From
www.exambites.in

Visit www.exambites.in for


More Premium Stuffs,Latest
Books,Test Papers,Lectures etc.
jeeneetadda
jna_official
jeeneetadda

VISIT NOW !!
E:\AMIT_WORKS\Exam_Guru\EG_Mathematics-12_(working_02-06-2022)\EG_Mathematics-12_working\Open_Files\Chap_7\Chap_7
\ 17-Aug-2022  Amit   Proof-5 Reader’s Sign _______________________ Date __________


x+2 5  x + 2 log x
(a) 1 − 4 x − x 2 + sin −1 (b) 16 + (log x) 2 −
  +C 3



2 2 5 


x−2 5  x + 2 8 log log x + 16 + (log x) 2 + C
(b) 1 − 4 x − x 2 + sin −1  +C


2 2 
5 


log x
(c) 16 + (log x) 2 +
2



x +1 5  x + 2
(c) 1 − 4 x − x 2 − sin −1  +C
3 2  5  8 log log x + 16 + (log x) 2 + C



(d) None of these (d) None of these





Evaluate each of the following Integrals
16 + (log x) 2 II. Long Answer Type Questions
4. ∫ x
dx equals



1. x 2 + 4x + 7 dx



log x
(a) 16 + (log x) 2 −
2 2. ∫ x 2 + 2x − 5 dx




8 log log x + 16 + (log x) 2 + C 3. ∫ 1 + 3x − x 2 dx



Answers 7.10
3 log x + 2 +
I. 1. (a) I = ∫ 1 + x dx = ∫ 1 + x dx
2 2 2
⇒ I=
2
( ) ( x + 2)2 + 3





 
( )(
x 2 a2
 Form ∫ a + x =
2 2
a + x2 + log | x + a 2 + x 2 | + C + x+2 x + 2) + 3 + C
2
2 2

  2
x 1
\ I= ∫ 1 + x 2 dx =
2
1 + x 2 + log| x + 1 + x 2 | + C
2 3 log x + 2 + x 2 + 4x + 7
( )



⇒ I=
2



2. (d) I= ∫ x 2 − 8 x + 7 dx = ∫ ( x 2 − 8 x + 16 ) + ( 7 − 16 ) dx

( )




+ x+2 x 2 + 4x + 7 + C
2
∫ ( x − 4 ) − 9 dx = ∫ ( x − 4 ) − 3 dx
2 2 2
=


x + 1 x 2 + 2x − 5 + C
Put x – 4 = t , then dx = dt so I = ∫ t − 3 dt
2 2
2. − 3 log (x + 1) + x 2 + 2x − 5 +
2





 x x2 − a2 a2 
 Form ∫ x 2 − a 2 dx = − log| x + x 2 − a 2 | + C Hint: I = ∫ x 2 + 2x − 5 dx = ∫ ( x + 1)2 − 6 dx
2 2



 

t t 2 − 32 32
\ I= − log t + t 2 − 32 + C ⇒ I = −3 log ( x + 1) + ( x + 1)2 − 6
2 2






=
( x − 4) x 2 − 8 x + 7
2
− ( )(
+ x +1
2
2
x + 1) − 6 + C


9
log ( x − 4 ) + x2 − 8x + 7 + C  
2 3. 13 sin −1  2x − 3  + 2x − 3 1 + 3x − x 2 + C

8  13  4


x+2 5  x + 2
3. (a) 1 − 4 x − x 2 + sin −1  +C
2 2  5  Hint: I = ∫ 1 + 3x − x 2 dx = ∫ −  x 2 − 3x − 1 dx





log x
4. (c) 16 + (log x) 2 + 8 log log x + 16 + (log x) 2 + C
2
( ) ( )
 2  2


3
I = ∫ − x − − 13  dx = ∫
13 − x − 3 dx

( )
2 4 4 2


3 log x + 2 + x 2 + 4x + 7 + x + 2 
II. 1.
2
( ) 2
x 2 + 4x + 7 + C

( ) ( )
2
  2x − 3 13 − x − 3
⇒ I = 13 sin −1  2x − 3  + +C
Hint: I = ∫ x + 4x + 7 dx = ∫
2
( x + 2)
2
+ 3 dx 8  13  4 4 2






ntegrals 221
I

E:\AMIT_WORKS\Exam_Guru\EG_Mathematics-12_(working_02-06-2022)\EG_Mathematics-12_working\Open_Files\Chap_7\Chap_7
\ 17-Aug-2022  Amit   Proof 5 Reader’s Sign _______________________ Date __________


II. DEFINITE INTEGRALS

Topic 11. Direct Evaluation of Definite Integrals


b π/4
dx
For finding ∫ f ( x ) dx we proceed as follows: Example 3. ∫
−π/4 1 + cos 2x
is equal to
a

(i) Find the indefinite integral ∫ f ( x ) dx . Let this be F(x). (a) 1 (b) 2 (c) 3 (d) 4






There is no need to keep integration constant C because if π/4
dx π/4
dx
we consider F(x) + C instead of F(x), we get
Solution. Let I = ∫ = ∫
− π / 4 1 + cos 2 x
2
−π/4 2 cos x
b π/4
b 1 1
∫ f ( x ) dx =  F ( x ) + C a = [F(b) + C] – [F(a) + C] sec 2 x dx = [ tan x ]− π/4
π/4
2 − π∫/ 4
=
2



a
= F(b) – F(a).
1 π  π  1 1


Thus, the arbitrary constant disappears in evaluating the =  tan − tan  −   = [1 + 1] = × 2 = 1
2 4 4  2 2



value of the definite integral. Hence, the correct option is (a).


b
(ii) Evaluate F(b) – F(a) =  F ( x ) , which is the value of π/4

∫ ( 2 tan )
a 3
Example 4. Evaluate: x dx


b

∫ f ( x ) dx . 0

a π/4 π/4

∫ ( 2 tan x ) dx = ∫ 2 tan x tan


π/2
3 2
Example 1. ∫ cos x . esin x dx is equal to . Solution. x dx



0 0 0
π/2
Solution. Let I = ∫ cos x . esin x dx π/4
=∫ 2 tan x (sec 2 x − 1) dx
0
Put sin x = t  cos x dx = dt


0

When x = 0 then t = sin 0 = 0; π/4

π π =∫ 2 tan x sec 2 x dx
When x = then t = sin = 1


2 2

0
1 π/4
\ I= ∫ et dt = [et ]10 = (e1 − e0 ) = e − 1 − ∫ 2 tan x dx = I1 − I 2


0

0
Hence, I = e – 1.
∫ 2 tan x sec
2
Consider x dx


a
1 π

Example 2. If ∫ 1 + 4 x 2 dx = 8 then a = ..
Put tan x = t ⇒ sec2x dx = dt
0




a
1 π
Solution. Given that: ∫ 1 + 4 x 2 dx = 8  ∫ 2 tan x sec 2 x dx = tan2 x



0
π/4
a

1

1
dx =
π Now I1 = ∫ 2 tan x sec 2 x dx



4 01 2 8


0
 + x 
4  tan 2 x 
π/4 π
a = 0 = tan 2 − tan 2 0 = 1
π 1 4


 ∫  1 2  dx = 2 ⇒ I1 = 1


0  
  + x 2 



 2  π/4
2 tan x dx = 2 [ log | sec x |]0
π/4
1  −1 x  π
a I2 = ∫


  tan 1/ 2  = 2 0
1/ 2 π
{ }


0
=2 log sec − log | sec 0 |
4


π
 2[tan–1 2a – tan–1 0] =
2 =2 log 2 − log 1




π π
 tan 2a =
–1
 2a = tan 1
4 4 =2 × log 2 − 0 = log 2



1 2


 2a = 1  a = π/4

∫ ( 2 tan )
2


3
1 Hence, x dx = I1 – I2 = 1 – log 2.
Hence, the value of a = .

2 0

222 Mathe atics–12
m
E:\AMIT_WORKS\Exam_Guru\EG_Mathematics-12_(working_02-06-2022)\EG_Mathematics-12_working\Open_Files\Chap_7\Chap_7
\ 17-Aug-2022  Amit   Proof-5 Reader’s Sign _______________________ Date __________


EXERCISE 7.11
I. Multiple Choice Questions (MCQs) π/2
π/4 13. ∫ (sin 2x)  tan −1 (sin x)  dx
1. ∫ tan x dx is equal to
2



0


0
1
π π π π
∫ (sin )
−1
(a) −1 (b) 1 − (c) 1 + (d) −1 14. x dx



2 4 4 8








0
3
1 e2
 
2. ∫0 1 + x 2 dx is equal to 15. 1
∫  (log x) − (log x)2  dx
1




e
π π π π
(a) (b) (c) (d) 1
12 6 4 3
∫ (cos








−1
16. x) 2 dx
π/2



0
3. ∫ cosec x dx is equal to
2


π/4 1
dx
(a) –1 (b) 1 (c) 0 (d)
1 17. ∫ e x (e x+ 1)



0
2








1
Evaluate each of the following Integrals dx
II. Short Answer Type Questions-I
18. ∫ (x + 1) (x + 2) (x + 3) [A.I. 2003]




0
π/4
2
1. ∫ 1 − sin 2x dx [Delhi 2008] 5x 2dx
19. ∫ x 2 + 4x + 3 [A.I. 2010]






0
1
1
1− x π/2
2. ∫ 1+ x
dx [A.I. 2008]
20. ∫
 x + sin x 
 1 + cos x  dx [A.I. 2011]



0



0
a
a−x
3. ∫ dx [Delhi 2008] π
a+ x
∫ [cos ax − sin bx ]
2
21. dx [Delhi 2015]



−a



−π
1
(x − x ) 3 1/ 3
π/4
4. ∫ x4
dx 22. ∫
dx [A.I. 2015]


3
cos x 2 sin 2x



1/ 3
0
π/4 π/2
sin x cos x
5. ∫ sin 4 x + cos 4 x
dx 23. ∫ x 2 sin x dx [Delhi 2014 (C)]





0 0
π /3
(sin x + cos x ) dx
( )
π
6. ∫ sin 2x
[Delhi 2010, A.I. 2014 (C)]
24. ∫e
2x
sin x +
π
dx [Delhi 2006]



π/6 4



0
π/4
sin x + cos x
7. ∫ 9 + 16 sin 2x
dx [V. Imp.] [Delhi 2014] III. Long Answer Type Questions



0 π/2
dx
π/2 1. ∫
∫( ) a 2 cos 2 x + b 2 sin 2 x


8. tan x + cot x dx [V. Imp.] [Delhi 2008, 2012] 0



0 π/4
(sin x cos x) dx
π/2
(sin 2x) dx
2. ∫ cos 2 x + sin 4 x



9. 0
sin 4 x + cos 4 x


0
π/2
cos 2 x dx
π/2
(sin x cos x) dx 3. ∫ cos 2 x + 4 sin 2 x
10. ∫ [Delhi 2000]


0
1 + sin 4 x



0
π
a
x 4 dx dx
11. ∫
4. ∫ 3 + 2 sin x + cos x




0 a2 − x2 0

π π/2
 1 − sin x  x
12. ∫  1 − cos x  e dx 5. ∫ (cos 2x) log (sin x) dx [Delhi 2012]





π/2 π/4

ntegrals 223
I

E:\AMIT_WORKS\Exam_Guru\EG_Mathematics-12_(working_02-06-2022)\EG_Mathematics-12_working\Open_Files\Chap_7\Chap_7
\ 17-Aug-2022  Amit   Proof 5 Reader’s Sign _______________________ Date __________


Answers 7.11

( )
π/4 π/4
 π π 
∫ tan x dx = ∫ (sec )
x − 1 dx = [ tan x − x ]0 I = (sin 0 − 0) − sin − 
π/4
I. 1. 2 2 ⇒
 2 2 




0 0

 π π  π
=  tan −  − (tan 0 − 0) = 1 −  − 0 = 1 −
 4 4  4
π
4


π  π −1
= ( 0 − 0) − 1 −  =
2  2 ( ) ( )




Hence, (b) is the correct answer. 3. (πa);




a
3
1 π π a − x dx

3
Hint: I =
2. ∫0 1 + x 2 dx =  tan x  0 = tan 3 − tan 0 = 3 − 0 = 3
−1 −1 −1
a+x






−a
Hence, (d) is the correct answer. Put x= a cos q ⇒ dx = –a sin q dq







When x = –a then –a = a cos q
π/2






π π ⇒ cos q = –1 ⇒ q=p
∫ cosec x dx = [ − cot x ]
π/2
3. 2
= − cot + cot = 0 + 1 = 1






π/4
2 4


π/4 When x = a then a = a cos q






Hence, (b) is the correct answer. ⇒ cos q = 1 ⇒ q=0

( ) ( )






II. 1. 2 − 1 or 1 − 2 ; 0
 a − a cos θ 
\ I= ∫  a + a cos θ 
( −a sin θ d θ)




π/4 π

( )
Hint: I = 1 − sin 2x dx 0
I = −a∫ sin θ/2 × 2 sin θ/2 cos θ/2 d θ



0 ⇒
cos θ/2 1




π/4 π
= ∫ sin 2 x + cos 2 x − 2 sin x cos x dx
= −a∫ 2 sin 2 θ d θ
0


0 2


π
π/4
0
= ∫ (cos x − sin x ) dx I = −a∫ (1 − cos θ ) d θ = −a [θ − sin θ ]π
0


0


π
π/4
I = a [sin θ − θ ]π
0
or ∫ (sin x − cos x ) dx ⇒





0 = a[(sin 0 – 0) – (sin p – p)]


I = [sin x + cos x ] π/4 ⇒ I = a[(0 – 0) – (0 – p)] = pa.



0


4. 6;


 
=  1 + 1  − ( 0 + 1) ( )
 x − x3 1/3 
1

 2 2  Hint: I = ∫   dx



 x4 



1/3  

I =  2 − 1
1/ 3


1
 x − x3   dx 
( ) ∫  x3 
=  3 
2. π x
− 1 ; Hint: Substitute: x = cos θ


1/ 3
2


1 1/ 3
1  1  dx
 1− x 
Hint: I = ∫  1 + x  dx
I= ∫  x 2 − 1 x3


  1/3



0

Put x = cos q ⇒ dx = –sin q dq 1  −2 


Put =z ⇒  3  dx = dz
x2 x 










When x = 0 then 0 = cos q
dx
= dz





⇒ q = p/2 ⇒
x3 −2






When x = 1 then 1 = cos q 1 1 =z
When x = then





⇒ q=0 3 (1/3)2








0 ⇒ z=9
 1 − cos θ 



\ I= ∫  1 + cos θ 
( − sin θ d θ) When x = 1 then 1 =z



(1)2





π/2

( )( )
0 ⇒ z=1
= − ∫ sin θ/2 × 2 sin θ cos θ d θ



1
cos θ/2 2 2  4/3 


( )
dz = − 1  ( z − 1) 
π/2 1

∫ ( z − 1)
1/ 3
0 0 \ I=
I = − ∫ 2 sin 2 θ d θ = − ∫ (1 − cos θ ) d θ −2 2 4 



⇒ 9   9
2 3



π/2 π/2
−3 0 − 16 = 6
= − [θ − sin θ ]
0
= [sin θ − θ ]π / 2
0 =
8
[ ]


π/2


224 Mathe atics–12
m
E:\AMIT_WORKS\Exam_Guru\EG_Mathematics-12_(working_02-06-2022)\EG_Mathematics-12_working\Open_Files\Chap_7\Chap_7
\ 17-Aug-2022  Amit   Proof-5 Reader’s Sign _______________________ Date __________


5. ()
π ; When x = 0 then sin 0 – cos 0 = z






8 ⇒ z = –1





π/4
 sin x cos x  dx When x = π then sin π − cos π = z
Hint: I = ∫  4  4 4 4






sin x + cos 4 x 



0 ⇒ z=0




Divide numerator and denominator by cos4 x. 0
dz

tan x sec x dx
π/4 2 \ I= ∫ 9 + 16 (1 − z 2 )
∫ tan 4 x + 1




I= −1


0 0 0
Put tan2 x = z ⇒ 2 tan x sec x dx = dz dz dz
⇒ I= ∫ 25 − 16z 2 = ∫ (5)2 − ( 4z )2




When x = 0 then tan2 0 = z




−1 −1





⇒ z=0 0
I = 1 log 5 + 4z



When x = π then tan 2 π = z 40 5 − 4z



−1
4 4





⇒ z=1 I = 1 log 5 − log 5 − 4 



40  5 5+ 4 



1
1  dz  1 1
∫ 2  z 2 + 1 = 2  tan
−1
z 
()
I=
0
I = 1 log1 − log 1 


0
40  9 



1 1 π π
=  tan −11 − tan −1 0 =  − 0 =
= 1 0 − log (3)  = 1 log (3).
−2
2 2  4  8


40   20



  
6.  2 sin −1  3 − 1  ;
 2 
8. ( 2π);




 π/2
π /3
(sin x + cos x ) dx Hint: I = ∫( tan x + cot x dx )
Hint: I = ∫



0
sin 2x



π/6 π/2
Substitute (sin x – cos x) = z sin x + cos x dx
⇒ I= ∫



sin x cos x



⇒ (cos x + sin x)dx = dz 0



(sin x – cos x)2 = z2 π/2
sin x + cos x dx
= 2 ∫


⇒ sin 2x = 1 – z2 sin 2x





0
π
When x= then sin π − cos π = z Substitute sin x – cos x = z
6 6 6








⇒ (cos x + sin x) dx = dz
 
z = 1− 3




⇒ Also sin 2x = 1 – z2
 2 






When x = 0 then sin 0 – cos 0 = z
π π
x= π





When then sin − cos = z ⇒ z = –1
3 3 3








π
 3 − 1 When x = then sin π − cos π = z
3−1 2 2 2





⇒ =z ⇒ z=
2 2  2  ⇒ z=1








3 −1 1
3 −1 dz 1
2
dz \ I= 2∫ = 2 sin −1 z 
\ I= ∫ = sin −1
( z )1−
2
2 −1



−1 1− z



3
1− 3 1 − z2 2
2 = 2 sin −11 − sin −1 ( −1)


   
= sin −1  3 − 1 − sin −1  1 − 3 
 2   2  ⇒ I= ( )
2 π + π = 2π


2 2



−1 
3 − 1 sin −1  3 − 1
= sin  + 9.  π  ;
 2   2   2 




π/2
  sin 2x
= 2 sin −1  3 − 1 Hint: I = 
∫  sin 4 x + cos4 x  dx

 2 





0

7.  1 log 3 ; π/2
2 sin x cos x
 20  = ∫ sin 4 x + cos4 x dx




0
π/4
(sin x + cos x ) dx
Hint: I =
9 + 16 sin 2x ∫ π/2
2 tan x sec2 x dx




0 I=
tan 4 x + 1


Put sin x – cos x = z ⇒ (cos x + sin x)dx = dz 0



[Q sin 2x = 1 – z2] Put tan2 x = z and proceed as in Q.No. 5.

ntegrals 225
I

E:\AMIT_WORKS\Exam_Guru\EG_Mathematics-12_(working_02-06-2022)\EG_Mathematics-12_working\Open_Files\Chap_7\Chap_7
\ 17-Aug-2022  Amit   Proof 5 Reader’s Sign _______________________ Date __________


10.  π  ;
( ) ( )
π π
x x
 8  I = − ∫ cot e dx +
1
∫ cosec2 x e xdx


2 2 2



π/2 π/2
π/2
(sin x cos x ) dx Integrate only first integral by parts
Hint: I = ∫
{ ()}
1 + sin 4 x





 π
0 x ex
I = −  cot +
Put sin2 x = z ⇒ sin x cos x dx = dz  2



π/2
2



When x = 0 then sin2 0 = z ⇒ z = 0
( ) ( )
π  π
cosec2 x e xdx  + 1 ∫ cosec 2 x e xdx
1








When x = π then sin 2 π = z ⇒ z = 1 2
π/2
2  2 π/2 2
2 2








() ( )
π π
1  x ex  1 cosec 2 x e x dx
2  π/2 2 ∫
dz/2 = 1  tan −1 z 1 I = − cot −
\ I= ∫  2
1 + z2 2  0



π/2



0

()
π
x e x dx
⇒ I = 1  tan −11 − tan −1 0 = π +1 ∫ cosec
2
2 8 2 2



π/2

() ()
 4
11.  3πa  ; = − cot π eπ − cot π eπ / 2 
 16  2 4


 



a = – [0 – 1e ] = e .
p/2 p/2
x 4 dx




Hint: I =
13.  π − 1 ;



0 a2 − x2  2 



Substitute x = a sin q π/2

∫ (sin 2x )  tan (sin x ) dx


−1

⇒ dx = a cos q dq
Hint: I =






0
When x = 0 then 0 = a sin q π/2





⇒ q=0 I = 2 ∫ (sin x )  tan −1 (sin x ) cos x dx





When x = a then a = a sin q 0





π/2
a sin θ ( a cos θ d θ )
4 4 Put sin x = z ⇒ cos x dx = dz
⇒ q= π \I= ∫





2 2 2 2 When x = 0 then sin 0 = z




a − a sin θ





0
⇒ z=0
π/2



sin 4 θ cos θ When x = π then sin π = z

4
I= a dθ
cos θ 2 2







0
⇒ z=1
π/2



1 1
∫ sin
4 4
⇒ I= a θ dθ
\ I = 2∫ z tan z dz = 2∫ tan z z dz
−1 −1
( )



0



0 0
4 π/2
Now integrate by parts
⇒ I= a ∫ (3 − 4 cos 2θ + cos 4θ) d θ

8 1



1
( )
 2 2
I =  2 tan −1 z z  − 2∫  1 2  z dz
0
π/2
 2 
0 0 1 + z  2


4
⇒ I = a 3θ − 2 sin 2θ + sin 4θ 
8  4  0
( )



1  z 2 + 1 − 1
( )
1
=  tan −1 z z 2  − ∫   dz
a 4  3π
( 
− 0 + 0 − ( 0 − 0 + 0) )  0  z2 + 1 


⇒ I= 0 
8  2 



1
( )
1
 1 
I =  tan z z  − ∫ 1 − 2  dz
−1 2
⇒ I = 3π a 4 0  z + 1


16



0

( )
1 1
12. [ep/2] I =  tan −1 z z 2  −  z − tan −1 z 
 0  0




( )
π
1 − sin x e x dx
Hint: I = ∫ 1 − cos x I = ( tan z ) z + tan z − z 
−1 2 −1
1



 


π/2 0

I = ( z + 1) tan ( z ) − z 
1
π  1 − 2 sin x cos x  2 −1
 
∫  2 sin22 x 2  e dx


x 0
=
( )


 π 
I =  2 × − 1 − {( 0 + 1) 0 − 0}
π/2  
2
 4 


( )
π

( ) ( )
1 cosec2 x − cot x e x dx
I= ∫ 2 2 2 I=
π −1 − 0 = π −1


2 2


π/2

226 Mathe atics–12


m
E:\AMIT_WORKS\Exam_Guru\EG_Mathematics-12_(working_02-06-2022)\EG_Mathematics-12_working\Open_Files\Chap_7\Chap_7
\ 17-Aug-2022  Amit   Proof-5 Reader’s Sign _______________________ Date __________


14.  π − 1 ; { }
0
 2  I = −  −θ 2 cos θ + 2 θ sin θ − ∫ sin θ d θ 
  π/2




1 1

∫ (sin ) ( )
0
Hint: I =
−1
x dx = ∫ sin −1 x .1 dx I = −  −θ 2 cos θ + 2 {θ sin θ + cos θ}
 π/2






0 0
0
Integrate by parts = −  −θ 2 cos θ + 2θ sin θ + 2 cos θ 
π/2



1
( x dx
) ()
1
I =  sin −1 x x  − ∫  
 0 2 = − ( 0 + 0 + 2) −  0 + 2 π × 1 + 0 


0 1− x   2 



1 = –[2 – p] = p – 2
I =  x sin −1 x  + 1 ∫ −2x dx
1



 0 2 1
17. log ( e + 1) − − (log 2)  ;



2
0 1− x
 e 



1 1
I =  x sin −1 x  +  1 − x 2  1
dx
  0   0 Hint: I = ∫ e x (e x + 1)





−1 −1 0
I = 1sin 1 − 0 sin 0 +  1 − 1 − 1 − 0 
Put ex = z
⇒ ex dx = dz


( )






dz
=  π − 0  π
 + [ −1] = 2 − 1
⇒ z dx = dz ⇒ dx =
z






 2


When x = 0 then e0 = z ⇒ z = 1






15.  e ( e − 2) ; When x = 1 then e1 = z ⇒ z = e
 2 








e e
dz  1 1 1
e2 
1 1  \ I= ∫ ( z + 1) z 2 = ∫  z + 1 − z + z 2  dz



Hint: I = ∫  log x − (log x )2  dx 1 1



e   e
⇒ I = log ( z + 1) − log z − 1 
e2 e2  z 1



 1  dx
∫  log x  dx − ∫ (log x )2
{ }
⇒ I=
I =  log ( e + 1) − log e − 1 − ( log 2 − log1 − 1)



e e ⇒
 e



Integrate only first integral by parts
1
I = log ( e + 1) − 1 − − log 2 + 0 + 1

e2 e2 e2 ⇒
I=  x  + 1 x dx dx  e 



 log x  ∫ (log x )2 x − ∫ (log x )2 I = log ( e + 1) − 1 − log 2


 e e e ⇒
e



e2 e2 e2
I =  x  +
 log x  e

dx −
(log x )2
dx
∫ (log x )2 1
18.  log
8 − log 3 
2 
; () ()


2 3


e e

e2 1
   2  dx
I=  x  = e 2 − e  Hint: I = ∫ ( x + 1)( x + 2)( x + 3)



 log x  e  log e log e 


0

( )( ) ( )
2 2 1
  1 1 
I=  e − e  =e −e = ∫ − 1 +1 1  dx
 2 log e log e  2 1  2 x +1 x+2 2 x+3




0

e2 − 2e = e e − 2 Integrate and find the value.


I= ( )
() ()

2 2  5 3 − 45 log 5 


16. [ – 2] 19. 5 + log ;
 2 2 2 4 




π
1

∫ (cos )
−1 2
Hint: I = x dx 2
5x 2 dx
2
 4x + 3 
Hint: I = ∫ x 2 + 4x + 3 = ∫ 5 1 − x 2 + 4x + 3  dx



0



Put cos x = q –1
⇒ x = cos q 1 1



{ ( ) ( )}
⇒ dx = – sin q dq 2
 
= 5∫ 1 − − 1 1 + 9 1




When x = 0 then 0 = cos q dx
 2 x + 1 2 x + 3 







1
⇒ q= π
Now integrate.
2




When x = 1 then 1 = cos q 20.  π  ;
 2 







⇒ q=0
( )
π/2
x + sin x



0
I = − ∫ (θ ) sin θ d θ
2 Hint: I = ∫ 1 + cos x
dx



0


π/2
Integrate by parts π/2  x + 2 sin x cos x 
 2 2  dx
= ∫

0
 2x 
I = −  −θ 2 cos θ + ∫ 2θ cos θ d θ 


0  2 cos 
  π/2 2


ntegrals 227
I

E:\AMIT_WORKS\Exam_Guru\EG_Mathematics-12_(working_02-06-2022)\EG_Mathematics-12_working\Open_Files\Chap_7\Chap_7
\ 17-Aug-2022  Amit   Proof 5 Reader’s Sign _______________________ Date __________


π/2 π/2 23. ( – 2)
1 x sec2 x dx + tan x dx



π
= ∫ ∫ π/2

∫ (x )
2 2 2 2


0 0 Hint: I = sin x dx





Integrate only first integral by parts 0
Integrate by parts

( )( ) ( )


π/2 π/2 π/2
I =  1 x tan x 2
π/2
1 tan x 2 dx +
−∫ ∫ tan x dx I =  −x 2 cos x + ∫ 2x cos x dx 
 0
 2 2 0 2 2 2





0 0
π/2
x
π/2 π/2 π / 2. I =  −x 2 cos x + 2x sin x − ∫ 2 sin x dx 
I =  x tan  − ∫ tan x dx + ∫ tan x dx  0



 2 0 2 2


0 0 π/2
I =  −x 2 cos x + 2x sin x + 2 cos x 
 0



I =  π tan π − 0 = π
 2 4  2 I = [(0 + p × 1 + 0) – (0 + 0 + 2)]




I = (p – 2)
 sin (2aπ) sin (2bπ) 



21.  2π + − ;  2
 2a 2b 


24. 
π  5 



∫ [cos ax − sin bx ]
2
Hint: I = dx
( )
π
x + π e2xdx



−π Hint: I = ∫ sin 4




π 0
⇒ I= ∫ cos 2 ax + sin 2 bx − 2 sin bx cos ax  dx Integrate by parts
 





( ) ( )
−π 2x 2x π
 
I = sin x + π e − ∫ cos x + π e dx 
( )( )
π
 1 + cos 2ax 4 2 4 2
+ 1 − cos 2bx


 0
⇒ I= ∫  2 2
( )



−π  π e 2x
I = sin x +
 (a + b) x   (b − a ) x    4 2


− sin   − sin    dx
( ) ( )
π
 2   2   2x 2x 
− cos x + π e + ∫ sin x + π e dx  
Now integrate and find the value.  4 4 4 4 
0

( ) ( )
π
22.  6  
I = sin x +
π e2x − cos x + π e2x 
−1I
 5  4 4  0 4


 4 2


{ ( ) ( )}
π/4 π
dx 5I = 1  
Hint: I = ∫ 3
cos x 2 sin 2x 4 
2 sin x + π − cos x + π e2x 



4 4 4 0


0

{ ( ) ( )}
π/4 π
dx  
I= ∫ 3 I = 1  2 sin x + π − cos x + π e2x 
2 cos x sin x cos x


5 4 4 0


0

{ ( ) ( )} { ( ) ( )}
π/4
 
= ∫
dx I = 1  2 sin 5π − cos 5π − 2 sin π − cos π 
2 cos 4 x tan x 5 4 4 4 4 




0

π/4
sec4 x dx
π/4
( tan 2
)
x + 1 sec2 x dx
   
I = 1  2 × 1 + 1  − 2 × 1 − 1  
I= ∫ = ∫ 5  2 2  2 2 


2 tan x 2 tan x


0 0
 
Sub: tan x = z ⇒ sec x dx = dz 2 I = 1 2 + 1 − 2 + 1 
5 2 2





When x = 0 then tan 0 = z
I= 1× 2= 2





⇒ z=0
5 5
()





π
When x = π then tan =z
4 4 III. 1.  π  ;





 2ab 
⇒ z=1 π/2
dx



1
( 2
1 z + 1 dz = 1  z 3/2 + 1  dz ) 1 Hint: I = ∫ a cos x + b 2 sin 2 x
2 2



I= ∫
2 ∫ 
\  0
2 z z



0 0 π/2
sec2 x dx
1 ⇒ I= ∫ a + b 2 tan 2 x
2
I = 1  2 z 5/ 2 + 2 z 



0
2  5  0


Put tan x = z ⇒ sec2 x dx = dz

( )





I = 1  2 + 2 − ( 0) = 1 × 12 = 6 When x = 0 then tan 0 = z





2 5  2 5 5 ⇒ z=0





228 Mathe atics–12
m
E:\AMIT_WORKS\Exam_Guru\EG_Mathematics-12_(working_02-06-2022)\EG_Mathematics-12_working\Open_Files\Chap_7\Chap_7
\ 17-Aug-2022  Amit   Proof-5 Reader’s Sign _______________________ Date __________


When x = π then tan π = z
I = −1
π/2 
(4 − 3cos x) − 4  dx
2
2 2
∫ 





3  4 − 3 cos 2 x 



⇒ z=∞ 0  



( )
∞ ∞
dz 1  −1 bz  π/2
\ I= ∫ a 2 + b2z 2 = ab  tan a  0 I=
−1  4
∫ 1 − 4 − 3 cos2 x  dx




0 3



0
1  −1 ∞ − 1  π − 0
⇒ I= tan tan −1 0 = π/2
ab  ab  2  I = −1  x  + 4
π/2 sec2 x dx
∫ 1 + 4 tan 2 x



3   3



0
= π 0

( )
2ab


I = −1  π  + 4  tan −1 ( 2 tan x )
π/2
π  
3 2 3× 2   0



2.
6 3


−π + 2  tan −1 ∞ − tan −1 0
I=
π/4
(sin x cos x ) dx 6 3 



Hint: I = ∫ cos 2 x + sin 4 x −π + 2  π − 
0 = −π + π = π



0 I=
6 3  2  6 3 6



Divide numerator and denominator by cos x. 4

π/4 4. π
tan x sec 2 x dx 4



I=
tan 4 x + tan 2 x + 1 dx


0 Hint: I = ∫ 3 + 2 sin x + cos x




Put tan x = z 2
⇒ 2tan x sec x dx = dz 2

2 tan x 1 − tan 2 x



When x = 0 then tan2 0 = z
Change sin x = 2 and cos x = 2





⇒ z=0
2x 2x
1 + tan 1 + tan



2 2
When x = π then tan2 π = z
4 4
and then put tan x = z





⇒ z=1 2





1 1
I = 1 ∫ 2 dz 1 dz 5.  −π + 1 log 2 + 1  ;
2 z + z +1 2 ∫
=
( )
 8 4 4 


2
z+1 + 3


0 0
2 4 π/2

1
Hint: I = ∫ (cos 2x ) logsin x dx



  
I = 1 × 2  tan −1  2z + 1 
π/4
2 3  3  0
π/2


I= ∫ log (sin x ) cos 2x dx


 
I = 1  tan −1  3  − tan −1  1  
π/4
π/2
 3  3 
3 I = log (sin x ) sin 2x − ∫ cos x sin 2x dx 


 2 sin x 2  π / 4


 
I = 1  tan −1 3 − tan −1  1   π/2
3  3  I =  1 sin 2x log (sin x ) − ∫ cos 2 x dx 


 2  π / 4


I = 1 π − π = 1 × π = π
( )
π/2
3  3 6  3 6 6 3 1 1 + cos 2x dx 
I =  2 sin 2x log (sin x ) − ∫


 2 

()


π/4
3. π
π/2
6
I =  1 sin 2x log (sin x ) − x − sin 2x 


π/2 π/2
cos x dx 2 2
cos x dx  2 2 4  π / 4


Hint: I = ∫ cos 2 x + 4 sin 2 x
= ∫ 4 − 3 cos 2 x Evaluate the limits.



0 0

Topic 12. Properties of Definite Integrals
b b b a
1. ∫ f ( x ) dx = ∫ f ( z ) dz . This property means if the limits 2. ∫ f ( x ) dx = −∫ f ( x ) dx . This property means if the limits




a a a b
and the function of the definite integral do not change and of the definite integral are interchanged, then the value of
only variable changes, then the value of the definite integral definite integral becomes negative of what it was.
remains same. a
4 4 3. ∫ f ( x ) dx = 0 . This means if the lower and upper limits
∫( ) ( )


For example: 3x 2 + 5 dx = ∫ 3p 2 + 5 dp a
of the definite integral are same, its value is zero.

1 1

ntegrals 229
I

E:\AMIT_WORKS\Exam_Guru\EG_Mathematics-12_(working_02-06-2022)\EG_Mathematics-12_working\Open_Files\Chap_7\Chap_7
\ 17-Aug-2022  Amit   Proof 5 Reader’s Sign _______________________ Date __________


d b c d π π
2 2
f ( x ) dx = ∫ f ( x ) dx + ∫ f ( x ) dx + ∫ f ( x ) dx . sin x + cos x
4. ∫
π
⇒ 2I =
∫ dx = ∫ 1 dx = [ x ]0 2


sin x + cos x



a a b c 0 0
This property is called split property. π π
= I=

2 4



b b
f ( x ) dx = ∫ f ( a + b − x ) dx So, R is true.
5. ∫ From (ii), A is also true. R is the correct explanation for A.


a a
a a
Hence, the correct option is (a)
π/2
6. ∫ f ( x ) dx = ∫ f ( a − x ) dx sin n x
Example 2. Evaluate: ∫ sin x + cos n x
n dx


0 0 0
2a a a
π/2
7. ∫ f ( x ) dx = ∫ f ( x ) dx + ∫ f ( 2a − x ) dx Solution. Let I = ∫
sin n x
dx ...(1)


sin n x + cos n x




0 0 0
0
2a a
On applying the property
8. ∫ f ( x ) dx = 2∫ f ( x ) dx if f ( 2a − x ) = f ( x )


a a


f ( x ) dx = ∫ f ( a − x ) dx , we get
0 0
= 0 if f (2a – x) = –f (x). ∫





0 0

( )
a a
9. (i) ∫ f ( x ) dx = 2∫ f ( x ) dx , when f (x) is an even function π/2 sin n π − x dx
2

=∫



sin ( ) ( )
−a 0
π − x + cos n π − x



i.e., f (– x) = f (x). 0
n


a 2 2
(ii) ∫ f ( x ) dx = 0 , when f (x) is an odd function i.e., =∫
π/2
cos n x dx



−a
cos n x + sin n x


f (– x) = – f (x). 0


nT T π/2
cos n x dx
10. ∫ f ( x)dx = n∫ f ( x)dx , where ‘T’ is the period of the =∫
sin n x + cos n x
...(2)





0 0 0
function i.e., f(x + T) = f(x)
b + nT b
On adding (1) and (2), we get

π/2 π/2
11. ∫ f ( x)dx = ∫ f ( x)dx , where f(x) is periodic with period
2I = ∫
 sin n x + cos n x 
 sin n x + cos n x  dx = ∫ dx


a + nT a


0 0
T and nŒI
π
π π
Questions Based on the Above Properties of ⇒ 2I =  x
 =  − 0 = 2
0
 2  2



Definite Integrals ⇒ I= π
4



a a
π/2
Property: ∫ f ( x ) dx = ∫ f ( a − x ) dx sin n x dx π
\ ∫ =
0 0 sin n
x + cos n
x 4

0
π π/2
cos x π 2 tan n x
Example 1. Assertion (A):
∫ dx = Example 3. Evaluate: ∫ 1 + tan n x
dx
π 0
sin x + cos x 4 0
2
Reason (R): sin x π π/2
tan n x
π/2
sin n x
∫0 sin x + cos x dx = 4 Solution. Let I = ∫ dx = ∫ dx
1 + tan n x sin x + cos n x
n


0 0
π
2
sin x Now this is the same question as the last example.
Solution. I = ...(i)
∫ sin x + cos x dx

π/2 π/2
tan n x sin n x dx π



0 \ I= ∫ 1 + tan n x
dx = ∫ =
sin n x + cos n x 4



a a

∫ f ( x ) dx = ∫ f (a − x ) dx
0 0
Q Note:


0 0 π/2
π/2
sin n x dx π cos n x dx π
π 
1. ∫ =
sin n x + cos n x 4
2. ∫ =
sin n x + cos n x 4




sin  − x dx π
0 0
2  2
I = ∫ π/2 π/2
π  π  1 π 1 π


0
sin  − x + cos  − x
2  2 
3. ∫ 1 + tan n x
dx =
4
4.
1 + cot n x
dx =
4




0 0
cos x
I= ∫ dx ...(ii) π/2 π/2
cos x + sin x tan n x dx π cot n x dx π


5.
∫ =
1 + tan n x 4
6. ∫ =
1 + cot n x 4




Adding equations (i) and (ii), 0 0

230 Mathe atics–12
m
E:\AMIT_WORKS\Exam_Guru\EG_Mathematics-12_(working_02-06-2022)\EG_Mathematics-12_working\Open_Files\Chap_7\Chap_7
\ 17-Aug-2022  Amit   Proof-5 Reader’s Sign _______________________ Date __________


EXERCISE 7.12
I. Multiple Choice Questions (MCQs) π/2
π/2
2008 sin x 7. ∫ [ 2 log sin x − log sin 2x ] dx [V. Imp.]
1. ∫




dx = 0
2008sin x + 2008cos x


0 III. Short Answer Type Questions-II
π π π
(a) 0 (b) p (c) (d) x dx
3 4 1. ∫ 1 + sin x [Delhi 2010, A.I. 2004, 2011 (C)]











π/2
0
a
200 sin x + 100 cos x x dx
2. ∫ sin x + cos x
dx = 2. (a) Evaluate: ∫ x+ a−x




0 0
(a) 50p (b) 25p (c) 75p (d) 150p a
a − x dx
(b) Evaluate: ∫ [Delhi 2008 (C)]








x+ a−x




π/2 0
φ( x )
3. The value of the integral ∫ π
dx is π
x tan x dx
 3. ∫ sec x + tan x


0
φ( x ) + φ  − x 



2  0
[V. Imp.] [Delhi 2008, 2010 (C), A.I. 2008, 2017]
π π


(a) (b) 1
 2x − 1 
4 2
∫ tan
−1




4.  1 + x − x 2  dx



(c) p (d) None of these 0




π /3
dx
2a
f ( x) 5. ∫ 1 + tan x
[Delhi 2007, 2011 (C)]




4. The value of the integral dx is equal π/6
f ( x ) + f ( 2a −x )


to 0 π
x sin x dx
6. ∫ 1 + cos2 x [Delhi 2009 (C), A.I. 2012, 2014, 2017]



0
(a) a (b) 2a (c) 1 (d) 0
π








 4 + 3 cos x 
6
x
7. ∫ log  4 − 3 cos x  dx


5. The value of the integral ∫ 9−x + x
dx is 0
8. If f (x) = f (a – x) and g (x) + g (a – x) = 4, prove that:


3





a a
1 3
(a) (b) (c) 2 (d) 1 ∫ f ( x ) g ( x ) dx = 2∫ f ( x ) dx
2 2








0 0
π  b
 − x 
π/2
4 9. If f (a + b – x) = f (x), then ∫ x f ( x ) dx = ?
6. The value of ∫ dx is


( )
1 a
sin x + cos x


0 1
10. ∫ log − 1 dx [Delhi A.I. 2008, 2014]
x



π 2 π 0
(a) (b)
4 4 2 π/2




sin 2 x dx
(c) 0 (d) None of these
11. ∫ sin x + cos x
[Delhi A.I. 2008, 2014 (C)]



0




Evaluate each of the following Integrals π/2

II. Short Answer Type Questions-I


12. ∫ log (tan x) dx [Delhi 2004]



0
n π/2
(sin x − cos x) dx π
1. ∫ x n − x dx 2. ∫ 1 + sin x cos x 13. x dx
∫ a 2 cos2 x + b2 sin 2 x [A.I. 2006, 2009 (C)]







0 0
0
π/2 π/2
dx dx
∫ ∫
3. Or π
x tan x dx
1 + cot x 1 + tan x 14. ∫ sec x cosec x [V. Imp.] [Delhi 2007, 2008, 2011 (C)]




0 0



0
π/2 1

∫ (cos x ) dx ∫ x (1 − x )
2 n π π
4. 5. dx x dx x sin x dx
15. ∫ 1 + sin 2 x 16. ∫ 1 + sin x




0 0




π/4 0 0

6. ∫ log (1 + tan x) dx π b
f ( x ) dx
17. ∫ x log (sin x) dx 18. ∫ f (a + b − x) + f ( x)


0




[V. Imp.] [A.I. 2011, Delhi 2013 (C), (SP)] 0 a

ntegrals 231
I

E:\AMIT_WORKS\Exam_Guru\EG_Mathematics-12_(working_02-06-2022)\EG_Mathematics-12_working\Open_Files\Chap_7\Chap_7
\ 17-Aug-2022  Amit   Proof 5 Reader’s Sign _______________________ Date __________


π/2 2
x 2 dx
19. ∫ (sin 2x) (log tan x) dx [A.I. Delhi 2011] 26. ∫ 1 + 5x [A.I. (North) 2016]







0 −2
a a
dx x 2 dx
20. ∫ 27. ∫ 1 + 5x




0 x + a2 − x2 −a

1 IV. Long Answer Type Questions


log (1 + x) dx
21. ∫ 1 + x2 [A.I. 2011 (C)] π/2
− π
log (sin x) dx = 



0 1. (a) Prove: ∫  2 
(log 2)



0
π/2

∫ [2 log cos x − log sin 2x] dx


π/2
22. − π
(b) Prove: ∫ log (cos x) dx =  (log 2)
 2 


0


0
2π [V. Imp.] [A.I. 2008]
dx
23. ∫


π
1 + esin x


0 2. (a) Evaluate: ∫ log (1 + cos x) dx = −π (log 2)



π 0
ecos x dx
24. ∫ cos x
π
[Delhi 2009]
e + e − cos x (b) Evaluate: ∫ log (1 − cos x) dx = −π ( log 2) [A.I. 2008]



0


0
π π/2
x sin x dx x sin x cos x dx [A.I. 2010 (C), Delhi 2014]
25. ∫ 1 + 3 cos2 x [A.I. (East) 2016]

3. Evaluate: ∫ sin 4 x + cos 4 x





0 0

Answers 7.12
π/2 π/2
2008sin x φ( x )
I. 1. (d) I= ∫ 2008 + 2008cos x
sin x
dx ...(1)
3. (a) I= ∫ π 
dx
...(1)




0 0 φ( x ) + φ  − x





2 
π 
sin  − x
π/2 2 
2008 π 
= ∫ π  π 
dx π/2 φ  − x
2 


sin  − x cos  − x
0
2008 2 
+ 2008 2 
= ∫ π 
dx ...(2)



0 φ  − x + φ( x )
 a a
 2 
Q ∫ f ( x) dx = ∫ f (a − x) dx   a a


 0 0   ∫
Q f ( x ) dx = ∫ f (a − x) dx 
π/2  0 0 
2008cos x
⇒ I= ∫ dx ...(2)

2008cos x + 2008sin x Adding (1) and (2), we get




0

π/2 π
Adding (1) and (2), we get π  π π
2I = ∫ dx = [ x]02 =  − 0 = ⇒ I =

2  2 4


π/2
2008sin x + 2008cos x 0
2I = ∫ 2008sin x + 2008cos x
dx 2a
f ( x)


0
4. (a) I= ∫ f ( x ) + f ( 2 a −x )
dx ...(1)





π/2 π
π  π 0
⇒ I= ∫ dx = [ x] 2
0
⇒ 2I =  − 0 ⇒ Ι =
2  4
2a
f ( 2a − x )





0 ⇒ I= ∫ f ( 2a − x ) + f ( x )
dx ... (2)




π/2 π/2 0
sin x + cos x sin x
2. (c) I = 100 ∫ sin x + cos x dx + 100 ∫ sin x + cos x dx  a a

Q ∫ f ( x) dx = ∫ f (a − x) dx 




0 0

π/2
 0 0 
 π  π/2 sin x π
= 100 ∫ dx + 100   Q ∫ dx =  Adding (1) and (2), we get
 4 sin x + cos x 4




0  0 2a 2a
f ( x ) + f ( 2a − x )
2I = ∫ f ( 2a − x ) + f ( x )
dx = ∫ dx


π
 π  π π 0 0
= 100 [ x ] 2 + 100   = 100  + 
0
 4  2 4 2I = [ x]02 a = (2a − 0) = 2a ⇒ I = a






3π 3
= 100 × = 75π 5. (b)
4 2




232 Mathe atics–12
m
E:\AMIT_WORKS\Exam_Guru\EG_Mathematics-12_(working_02-06-2022)\EG_Mathematics-12_working\Open_Files\Chap_7\Chap_7
\ 17-Aug-2022  Amit   Proof-5 Reader’s Sign _______________________ Date __________


π/2
π
4
−x π/2


( ) ( π2 − x) dx
sin π − x − cos
2

( ) ( π2 − x)
6. (c) I= dx ...(1) I=
sin x + cos x 1 + sin π − x cos








0 0
a a
2
Using ∫ f ( x) dx = ∫ f ( a − x ) dx π/2
(cos x − sin x ) dx


0 0 ⇒ I= ...(2)
1 + sin x cos x





0
π π 
π/2 −  − x Add (1) and (2)
4  2 
∫ dx


I=
 π  π  (sin x − cos x + cos x − sin x ) dx


0 π/2
sin  − x + cos  − x
 2   2  2I = ∫ 1 + sin x cos x



0
π
π/2 − +x π/2
π/2
4
= ∫ cos x + sin x
dx ...(2) 2I = ∫ 0 dx = C  0 =C−C=0






0 0

Adding (1) and (2), we get 3. π


4




π/2 π/2
π π dx sin x dx
π/2
4
−x π/2 −
4
+x Hint: I = ∫ 1 + cot x
= ∫ sin x + cos x
...(1)





I+I= ∫ sin x + cos x
dx + ∫
cos x + sin x
dx 0 0


0 0 a a
Apply the property: ∫ f ( x ) dx = ∫ f ( a − x ) dx
π π


π/2 −x− +x 0 0
4 4
2I = ∫ dx
sin x + cos x
( )
sin π − x dx


0 π/2
2
I= ∫
sin ( π − x ) + cos ( π − x )
π/2
0



= dx = 0 ⇒ I = 0 0
sin x + cos x 2 2


0

 3  5 π/2
II. 1. 2n  n 2  − 2  n 2  ; cos x dx
3   5 
= ∫ cos x + sin x
...(2)



0
n
Add (1) and (2)
Hint: I = ∫x n − x dx

π/2




0 π/2
Apply the property:
2I = ∫ 1dx =  x  0 2


0

a a

∫ f ( x ) dx = ∫ f ( a − x) dx ⇒ I= π
4



0 0
π/2 π/2

dx cos x dx
∫ 1+ ∫
n or =
I= ∫ (n − x) n − ( n − x ) dx tan x cos x + sin x


0 0


0
Repeat the solution of first part.

n n
I= ∫ (n − x) (
x dx = ∫ n x − x 3/ 2
) dx 4.
π;
4




0 0 π/2

∫ cos
2
n Hint: I = x dx ...(1)
I =  2n x3/ 2 − 2 x5 / 2 




0
 3 5  0


Apply the property

( ) ()
a a
=  2n ( n ) − 2 ( n )  − [0 − 0]
3/ 2 5/ 2
∫ f ( x ) dx = ∫ f ( a − x ) dx
 3 5 



0 0

= 2n ( n ) − 2 ( n )
( )
3/ 2 5/ 2 π/2
3 5 π − x dx
∫ cos
2
I=


2


2. 0; 0


π/2
π/2
(sin x − cos x ) dx = ∫ sin
2
x dx ...(2)
Hint: I = ∫ (1 + sin x cos x ) ...(1)







0 0

On applying the property: Add (1) and (2)




π/2

∫ (sin )
a a
2
2I = x + cos 2 x dx
∫ f ( x ) dx = ∫ f ( a − x ) dx



0
0 0

ntegrals 233
I

E:\AMIT_WORKS\Exam_Guru\EG_Mathematics-12_(working_02-06-2022)\EG_Mathematics-12_working\Open_Files\Chap_7\Chap_7
\ 17-Aug-2022  Amit   Proof 5 Reader’s Sign _______________________ Date __________


π/2 π/2
π/2
= ∫ 1 dx =  x 
0
Hint: I = ∫ 2 logsin x − log (sin 2x ) dx





0 0
π
2I = π ⇒ I= π/2
 sin 2 x 
2 4 I= ∫ log  2 sin x cos x  dx






5. 1 0
;

( )
(n + 1) (n + 2) π/2


tan x dx
1
= ∫ log 2



∫ x (1 − x )
n 0
Hint: I = dx

( )



π/2
0 tan x dx
Apply the property
I= ∫ log 2
...(1)




0

a a
Apply the property:
∫ f ( x ) dx = ∫ f ( a − x ) dx


a a

0 0
∫ f ( x ) dx = ∫ f ( a − x ) dx
1


n 0 0
I= ∫ (1 − x ) 1 − (1 − x ) dx
( )  dx
 tan π − x


0 π/2
 2
1 1 I= ∫ log 
2
( )



∫ (1 − x ) x dx = ∫ x − x dx  
n n n +1
I= 0
 


0 0

( )
π/2
cot x dx

I= x − x 
n +1 n + 2 1 = ∫ log 2
...(2)



0
 n + 1 n + 2 0


Add (1) and (2)

I=  1 − 1 = 1 π/2
 n + 1 n + 2  ( n + 1)( n + 2)  tan x × cot x  dx
∫ log 


2I =
2 
( )(
2


π log ; 0
6. 2)

()
8 π/2 π/2
1 dx = −2 log 2  dx 


π/4 2I = ∫ log
4
 ∫ 


 0 
Hint: I = ∫ log (1 + tan x ) dx ...(1) 0

()




π/2 π
2I = −2 ( log 2)  x  = −2 ( log 2)
0
Apply the property 0 2



a a −π log 2
∫ f ( x ) dx = ∫ f ( a − x ) dx I=
2
( )



0 0 III. 1. p

( )
π/4 π
 π−x  x dx
I= ∫ log 1 + tan 4  dx Hint: I = ∫ 1 + sin x ...(1)






0 0
π/4 Apply the property:
1 − tan x 
∫ log 1 +

I= dx a a
 1 + tan x 
f ( x ) dx = ∫ f ( a − x ) dx


0


( )
π/4 0 0
2
I= ∫ log
1 + tan x
dx ...(2) π
( π − x ) dx = π ( π − x ) dx
∫ 1 + sin ( π − x ) ∫ 1 + sin x



0 I= ...(2)



0 0
Add (1) and (2)

π/4 Add (1) and (2)
 2 

∫ log (1 + tan x ) ×
( )
2I = dx π
( x + π − x ) dx = ππ
(1 + tan x )  1
 ∫ ∫


0 2I = dx
1 + sin x 1 + sin x


0 0
π/4
= ∫ (log 2) dx π
2I = π∫  1 − sin x  dx
 cos 2 x 


0


0

2I = ( log 2)  x  = π log 2
π/4
π
0 4 (
= π∫ sec2 x − sec x tan x dx )


()


π log 0
I=
8
( 2) π
2I = π  tan x − sec x 


0


−π (log 2);
7. 2I = p[(tan p – sec p) – (tan 0 – sec 0)]
2




234 Mathe atics–12
m
E:\AMIT_WORKS\Exam_Guru\EG_Mathematics-12_(working_02-06-2022)\EG_Mathematics-12_working\Open_Files\Chap_7\Chap_7
\ 17-Aug-2022  Amit   Proof-5 Reader’s Sign _______________________ Date __________


2I = p[{0 – (–1)} – (0 – 1)] = 2p 2I = p[(sec p – tan p + p – (sec 0 – tan 0 + 0)]





I=p 2I = p[(–1 – 0 + p) – (1 – 0 + 0)]




a 2I = p[–1 + p – 1] = p(p – 2)
2. (a) ;



2


a I = π ( π − 2)
x dx 2



Hint: I = ...(1) 4. 0;
x+ a−x






0
1
Apply the property:  2x −1 
∫ tan
−1
I=  2  dx

1 + x − x 



a a 0
∫ f ( x ) dx = ∫ f ( a − x ) dx  x − (1 − x ) 
1

∫ tan
0 0 −1
I= 1 + x (1 − x )  dx
 



a 0
a−x
\ I= ∫ a−x + a−a+x
dx 1



= ∫  tan x − tan (1 − x ) dx
0 −1 −1



a 0
=∫ a − x dx ...(2)
1 1
a−x+ x



∫ tan x dx − ∫ tan (1 − x ) dx
0 −1 −1
I=



Add (1) and (2) 0 0

a a
a a
 
2I = ∫  x + a − x  dx = ∫ dx = [ x ]0 = a
a Apply the property ∫ f ( x ) dx = ∫ f ( a − x ) dx only in second


 x + a − x 


integral. 0 0
0 0

I= a
1 1

∫ ( tan x ) dx − ∫ tan 1 − (1 − x ) dx


−1 −1
2 \ I=





0 0
a
(b)
2 1 1
( )

∫ tan x dx − ∫ tan −1 x dx = 0
−1
a I=
x dx


Hint: I = ∫ x+ a−x π;
0 0



0 5.
12


Solve as we have solved. Q.No. 2 π /3
dx
Hint: I = ∫ 1+

3. π ( π − 2) , tan x



π/6
2


π
x tan x dx π/3
I= ∫ ...(1)  cos x 
sec x + tan x I= ∫   dx ...(1)



0 cos x + sin x 



π/6
Apply the property:
Now apply the property

a a

∫ f ( x ) dx = ∫ f ( a − x ) dx b b

∫ f ( x ) dx = ∫ f ( a + b − x ) dx

0 0
π
( π − x ) tan ( π − x ) dx a a
I= ∫ sec ( π − x ) + tan ( π − x )

) (
cos π + π − x dx


0 π/3
3 6
− ( π − x ) tan x  dx
I= ∫ ...(2)
cos ( π + π − x ) + sin ( π + π − x )
π
I= ∫ 



π/6
− [sec x + tan x ] 3 6 3 6


0
π
( π − x ) tan x dx Add (1) and (2)
I= ∫ ...(2)

sec x + tan x π/3
 sin x + cos x 



Add (1) and (2)
0
2I = ∫   dx
sin x + cos x 


π/6

π
( π − x + x ) tan x dx π
tan x dx π/3
2I = ∫ sec x + tan x
= π∫
sec x + tan x = ∫ dx =  x  π/6
π/3


0 0


π/6

tan x (sec x − tan x ) dx


π
π
2I = π∫ 2I = π − π = π ⇒ I=
sec2 x − tan 2 x 3 6 6 12






0
π π2 ;
6.
(
2I = π∫ sec x tan x − sec x + 1 dx 2
) 4


π


0 x sin x dx
2I = [sec x − tan x + x ] π I= ∫ 1 + cos2 x ...(1)



0 0


ntegrals 235
I

E:\AMIT_WORKS\Exam_Guru\EG_Mathematics-12_(working_02-06-2022)\EG_Mathematics-12_working\Open_Files\Chap_7\Chap_7
\ 17-Aug-2022  Amit   Proof 5 Reader’s Sign _______________________ Date __________


Apply the property
( )∫
b
a+b

a a 9. f ( x ) dx ;
2
∫ f ( x ) dx = ∫ f ( a − x ) dx



a
b

∫ x f ( x ) dx
0 0
Hint: I =
π
( π − x ) sin ( π − x ) dx




a
I= ∫ 1 + cos 2 ( π − x ) Apply the property:


0


b b
π
( π − x ) sin x dx
I= ∫ 1 + cos 2 x
...(2) ∫ f ( x ) dx = ∫ f ( a + b − x ) dx



0 a a


Add (1) and (2) b
I= ∫ ( a + b − x ) f ( a + b − x ) dx

π
sin x dx π



2I = π∫ 2 = −π  tan ( cos x ) 0
 −1  a
1 + cos x b


0
2I = –p[tan–1 (cos p) – tan–1 (cos 0)]
I= ∫ ( a + b − x ) f ( x ) dx



a


2I = –p[–tan–1 1 – tan–1 1] [Q f (a + b –x) = f (x) (given)]





π2
2I = 2π × π ⇒ I= b b
4 4 ⇒ I= ∫ ( a + b) f ( x ) dx − ∫ x f ( x ) dx




7. 0;






a a
π b
4 + 3 cos x  dx
Hint: I = ∫ log  ...(1) ⇒ I = ( a + b ) ∫ f ( x ) dx − I
 4 − 3 cos x 







0 a
Apply the property: b

a a
⇒ 2I = ( a + b ) ∫ f ( x ) dx
∫ f ( x ) dx = ∫ f ( a − x ) dx



a

0 0

( )∫
b
 4 + 3 cos ( π − x ) 
π
a+b
I = ∫ log  ⇒ I= f ( x ) dx
 dx 2
 4 − 3 cos ( π − x ) 





0 a
10. 0;
π


( ) ( )
4 − 3 cos x  dx
I = ∫ log 
1 1
...(2) 1 1 1− x
 4 + 3 cos x  Hint: I = ∫ log − dx = ∫ log dx



0 x x



0 0
Add (1) and (2)

( )
1

1 − x dx
( ) ∫ log
π
 4 + 3 cos x 4 − 3 cos x  I= ...(1)
2I = ∫ log  4 − 3 cos x × 4 + 3 cos x  dx x



0


0
π π
Apply the property:

π
= ∫ log1 dx = ∫ 0 dx = C  = C − C = 0 a a
0
∫ f ( x ) dx = ∫ f ( a − x ) dx


0 0

a 0 0
8. Hint: I = ∫ f ( x ) g ( x ) dx 1
1 − (1 − x ) 




∫ log 
0
I= dx
Apply the property: 1 − x 


0

a a

∫ f ( x ) dx = ∫ f ( a − x ) dx
( )
1
x
I= ∫ log 1 − x dx ...(2)

0 0



a 0
I= ∫ f ( a − x ) g ( a − x ) dx Add (1) and (2)



0

( )( )
1
 1− x
× x
a 
I= ∫ f ( x ) 4 − g ( x ) dx 2I = ∫ log  x 1− x  dx




0
0
[Q f (a – x) = f (x) and g(a – x) = 4 – g(x) (given)] 1 1 1


a a ⇒ 2I = ∫ log1dx = ∫ 0 dx = C  =C−C=0
0
I = 4∫ f ( x ) dx − ∫ f ( x ) g ( x ) dx



0 0


0 0 ⇒ I=0
a



I = 4∫ f ( x ) dx − I 11.
1 log
( 2 +1 ; )


2


0
a π/2
sin 2 x dx
⇒ I = 2∫ f ( x ) dx Hint: I = ∫ sin x + cos x ...(1)







0 0

236 Mathe atics–12


m
E:\AMIT_WORKS\Exam_Guru\EG_Mathematics-12_(working_02-06-2022)\EG_Mathematics-12_working\Open_Files\Chap_7\Chap_7
\ 17-Aug-2022  Amit   Proof-5 Reader’s Sign _______________________ Date __________


Apply the property Add (1) and (2)


( )
a a π π
f ( x ) dx = f ( a − x ) dx 1 − cos 2x
∫ ∫ 2I = π∫ sin x dx = π∫
2
2
dx



0 0 0 0
π/2 π
cos 2 x dx I = π  x − sin 2x 
I= ∫ ...(2) 4  2  0



cos x + sin x



0
2
Add (1) and (2) I = π ( π − 0) − ( 0 − 0) = π
4 4




π
dx π2 ;
2I = ∫ sin x + cos x 15.
2 2




0 π
Now integrate and find the value. x dx
Hint: I = ∫ 1 + sin 2 x ...(1)





12. 0 0


π/2 Apply the property:


Hint: I = ∫ log ( tan x ) dx ...(1) a a

∫ f ( x ) dx = ∫ f ( a − x ) dx




0


Apply the property: 0 0

a a π
( π − x ) dx
∫ f ( x ) dx = ∫ f ( a − x ) dx I= ∫ 1 + sin 2 x ...(2)





0 0 0

π/2 Add (1) and (2)


∫ log ( cot x ) dx

I= ...(2) π
2I = π∫  1  dx



0
Now find (1) + (2). It gives I = 0.  1 + sin 2 x 


0

π2 ; 1 1
13. Since =
2ab 1 + sin 2 ( π − x ) 1 + sin 2 x



π
x dx
Hint: I = ∫ a 2 cos2 x + b2 sin 2 x ...(1) π/2
dx




0 \ 2I = 2π ∫
1 + sin 2 x



Apply the property: 0

a a π/2
sec2 x dx
∫ f ( x ) dx = ∫ f ( a − x ) dx ⇒ I= π∫
1 + 2 tan 2 x




0 0 0
π
( π − x ) dx 1 tan −1  2 tan x  π/2
I= ∫ a 2 cos2 x + b2 sin 2 x ...(2) ⇒ I= π
2  0






0
Add (1) and (2)
= π 1  tan −1 { 2 tan π ( )} − tan −1 { }
2 tan ( 0) 

2 2 


π
2I = π∫ 2 dx
a cos 2
x + b 2 sin 2 x = π  tan −1 ( ∞ ) − tan −1 ( 2 ×0  )


0
2 


Now it is same as Q.No. 1 (Type-III) of Exercise 7.11
2

π2 ; = π  π − 0 = π
14. 
2 2 
 2 2


4


π
Hint: I =
x tan x dx
∫ sec x cosec x 16. π ( π − 2);
2





0 π
π x sin x dx
I= ∫ x sin x dx
2
...(1)
Hint: I = ∫ 1 + sin x ...(1)




0



0
Apply the property: Apply the property


a a
a a
∫ f ( x ) dx = ∫ f ( a − x ) dx
∫ f ( x ) dx = ∫ f ( a − x ) dx


0 0
0 0
π π
( π − x ) sin ( π − x ) dx
∫ ( π − x )sin ( π − x ) dx ∫
2
I= I=
1 + sin ( π − x )




0 0
π π
( π − x ) sin x dx
∫ ( π − x )sin =∫
2
I= x dx ...(2) ...(2)
1 + sin x






0 0

ntegrals 237
I

E:\AMIT_WORKS\Exam_Guru\EG_Mathematics-12_(working_02-06-2022)\EG_Mathematics-12_working\Open_Files\Chap_7\Chap_7
\ 17-Aug-2022  Amit   Proof 5 Reader’s Sign _______________________ Date __________


Add (1) and (2) π/2 π/2
I= π logsin 2x dx − π log 2 ∫ dx


( )
π
sin x dx 2 2



2I = π∫ 0 0
1 + sin x
( )
π/2


I= π π π−
0

2 ∫ log (sin 2x ) dx − 2 log 2 2


0
 (1 + sin x ) − 1
π



0
2I = π∫  dx
1 + sin x 
0


π/2 2
I= π π
∫ logsin ( 2x ) dx − 4 (log 2)
π 2



2I = π∫ 1 − 1  0
dx dz
 1 + sin x  Put 2x = z ⇒ dx =


0
2






π
π 1 dx x=0 then 0 = z
2I = π  x  − π∫





0 1 + sin x x= π then 2 × π = z


0 ⇒ z=p
2 2







π
2I = π 2 − π∫  1 − sin x  dx π 2
 cos 2 x  \ I = π ∫ log (sin z ) dz − π ( log 2)


0 4 4




0
π
2I = π − π∫ sec x − sec x tan x  dx
2 2 π 2
I = π ∫ logsin x dx − π ( log 2)


0 4 4



Now evaluate. 0
 b b 

2 Q ∫ f ( x ) dx = ∫ f ( z ) dz 
17. −π (log 2);  a a 
3


π 2 2
I = π × I − π ( log 2) = I − π ( log 2)
Hint: I = ∫ x log (sin x ) dx ...(1) 4 π 4 4 4






0
 π/2 
Apply the property
Q I = π ∫ log (sin x ) dx 

a a  0 
∫ f ( x ) dx = ∫ f ( a − x ) dx 3I = − π 2 log 2 ⇒ I = π 2
( ) − ( log 2)

0 0
4 4 3




π
b−a;
I= ∫ ( π − x ) log sin ( π − x ) dx 18.
2




0
f ( x ) dx
b
π Hint: I = ∫ f (a + b − x) + f ( x) ...(1)
I= ∫ ( π − x ) logsin x dx ...(2)




a



0 On applying the property

Add (1) and (2) b b

π ∫ f ( x ) dx = ∫ f ( a + b − x ) dx
2I = π∫ log (sin x ) dx a a



0
f ( a + b − x ) dx
b
Since log sin(p – x) = log sin x we get I = ∫ f a + b − {a + b − x} + f ( a + b − x )




π/2 a

\ 2I = 2π ∫ log (sin x ) dx b
f ( a + b − x ) dx
I= ∫ f ( x) + f (a + b − x) ...(2)



0



π/2 a
I = π ∫ log (sin x ) dx ...(3) Add (1) and (2)




0 b
b
Apply the property 2I = ∫ dx =  x  a = b − a



a a a

∫ f ( x ) dx = ∫ f ( a − x ) dx I= b−a
2



0 0

π/2
19. 0


I = π ∫ log cos x dx ...(4) π/2
Hint: I = ∫ (sin 2x ) log ( tan x ) dx ...(1)



0




0
Add (3) and (4) Apply the property


π/2 a a
2I = π ∫ log (sin 2x ) − log 2 dx ∫ f ( x ) dx = ∫ f ( a − x ) dx



0 0 0

238 Mathe atics–12


m
E:\AMIT_WORKS\Exam_Guru\EG_Mathematics-12_(working_02-06-2022)\EG_Mathematics-12_working\Open_Files\Chap_7\Chap_7
\ 17-Aug-2022  Amit   Proof-5 Reader’s Sign _______________________ Date __________


( )
π/2 2π 2π
  dx dx
I= ∫ sin ( π − 2x ) log  tan π − x  dx I= ∫ 1 + esin(2π − x) = ∫ 1 + e− sin x
 2




0 0 0
π/2

I= ∫ sin 2x log (cot x ) dx ...(2) ⇒ I= esin xdx
∫ 1 + esin x ...(2)







0
0
π/2
2I = ∫ sin 2x log ( tan x × cot x ) dx Add (1) and (2)




0

π/2  1 esin x 
⇒ 2I = ∫ (sin 2x ) log (1) dx = 0 2I = ∫ 1 + esin x + 1 + esin x  dx



0



0
I=0 2π



20. π
2I = ∫ dx =  x  0 = 2π − 0 ⇒ I=p



  

0
4 π


a 24.
dx 2



Hint: I = ∫



0 x + a2 − x2 π
ecos x
Put x = a sin q ⇒ dx = a cos q dq
Hint: I = ∫ ecos x + e− cos x dx ...(1)





0





When x = 0 then 0 = a sin q ⇒q=0 Apply the property:







When x = a then a = a sin q ⇒ θ= π a a
2 f ( x ) dx =
∫ ∫ f ( a − x ) dx







π/2 π/2
a cos θ d θ cos θ d θ 0 0
\ I= ∫ a sin θ + a cos θ
= ∫ sin θ + cos θ



π
0 0 ecos(π − x)
Apply the property
I= ∫ ecos(π − x) + e− cos(π − x) dx


0

a a

∫ f ( x ) dx = ∫ f ( a − x ) dx
π
e − cos xdx
I= ∫ e− cos x + ecos x ...(2)

0 0



0
and add. It gives

π/2
Add (1) and (2)

2I = ∫ dx π
π
∫ dx =  x  0 = π ⇒I= π


0 2I =
2




Now evaluate. 0
2π 2

π (log 2) 25.
21. 3 2


8


π
log ( x + 1) dx x sin x dx
1
Hint: I = ∫ x2 + 1 Hint: I = ∫ 1 + 3 cos2 x ...(1)







0 0

Substitute: x = tan ⇒ dx = sec2 q dq Applying the property



θ



log (1 + tan θ ) sec2 θ d θ
π/4
a a
I= ∫ ( tan θ + 1)
2
∫ f ( x ) dx = ∫ f ( a − x ) dx


0

0 0
π/4
I= ∫ log (1 + tan θ) d θ
π
( π − x ) sin x dx
I= ∫ ...(2)


1 + 3 cos 2 x



0
0
22. − π ( log 2)
2 Add (1) and (2)



Hint: See the solution Q.No. 7 (Type-II). π
sin x dx

23. p 2I = π∫
1 + 3 cos 2 x




2π 0
dx
Hint: I = ∫ 1 + esin x ...(1) Now put cos x = z.





0
8
Apply the property 26.
3



a a 2
x 2 dx
∫ f ( x ) dx = ∫ f ( a − x ) dx Hint: I = ∫ 1 + 5x . ...(1)



0 0 −2

ntegrals 239
I

E:\AMIT_WORKS\Exam_Guru\EG_Mathematics-12_(working_02-06-2022)\EG_Mathematics-12_working\Open_Files\Chap_7\Chap_7
\ 17-Aug-2022  Amit   Proof 5 Reader’s Sign _______________________ Date __________


Apply the property 1 8+8
= [ ] = 163

3



b b

∫ f ( x ) dx = ∫ f ( a + b − x ) dx I=
8
a a 3



a3

27.
2  (2 − 2 − x) 2 2
x dx2 3



I= ∫  1 + 5(2 − 2 − x)  dx = ∫ 1 + 5−x Hint: Same solution as given in Q.No.26


−2   −2


IV. 1. Hint: See the solution as given in Q.No.17
2
x 25 x dx 2. Hint: See the solution as given in Q.No.17
I= ∫ ...(2)



1 + 5x



−2 π2 ;
3.
16



Add (1) and (2)
Hint: First apply the property:

( ) dx =


2 x2 1 + 5x 2 a a
∫ ∫x
2
2I =
(1 + 5 ) x
dx
∫ f ( x ) dx = ∫ f ( a − x ) dx



−2 −2 0 0

Add the two integral.


2I = 1  x3 
2


3 −2 Now divide numerator and denominator by cos4x and put tan2x



= z.

Topic 13. Odd and Even Functions


Even Functions: If f (–x) = f (x) then the function f (x) is called Let f (x) = x3 + sin3 x








even function. ⇒ f (–x) = (–x)3 + sin3 (–x) = –x3 – sin3 x




= –(x3 + sin3 x) = –f (x)


For example, if f (x) = 3x2 + 4


\ f (x) is an odd function.






then f (–x) = 3(–x)2 + 4 = 3x2 + 4 = f (x)


π/2



⇒ f (x) = 3x2 + 4 is an even function. \ I= ∫ (x 3
+ sin 3 x dx = 0)







−π / 2
Odd Functions: If f (–x) = –f (x) then the function f (x) is called



π/2
odd function. Example 2. Evaluate: ∫ sin 2 x dx
−π / 2
For example, f (x) = x3 + x




π/2
⇒ f (–x) = (–x)3 + (–x) = –x3 – x Solution. I= ∫ sin 2 x dx






−π / 2
⇒ f (–x) = – (x3 + x) = – f (x)







Let f (x) = sin2 x

⇒ f (x) is an odd function.






⇒ f (–x) = f (x) ⇒ f (x) is an even function







Property of Even Functions
π/2 π/2
a a \ I= ∫ sin 2 x dx = 2 ∫ sin 2 x dx
∫ (even function ) dx = 2∫ ( even function ) dx



−π / 2 0


−a 0
π/2 π/2
1 − cos 2x
Property of Odd Functions
=2∫
2
dx = ∫ (1 − cos 2x ) dx


0 0

a π/2
 x − sin 2x 
∫ (odd function ) dx = 0 =
 2  0




−a
π/2
Example 1. Evaluate: ∫ (x 3
+ sin 3 x dx ) (
=π − 1 sin π − 0 − 1 sin 0
2 2 2 )( )


−π / 2

( )
π/2
Solution. I= ∫ (x 3
+ sin 3 x dx ) π 1
= − × 0 − ( 0 − 0) =
2 2
π
2




−π / 2

240 Mathe atics–12


m
E:\AMIT_WORKS\Exam_Guru\EG_Mathematics-12_(working_02-06-2022)\EG_Mathematics-12_working\Open_Files\Chap_7\Chap_7
\ 17-Aug-2022  Amit   Proof-5 Reader’s Sign _______________________ Date __________


EXERCISE 7.13
I. Multiple Choice Questions (MCQs) π/2
π 4. The value of ∫ (cos x + x 5 sin 4 x) dx is
∫ (1 − x



1. The value of 2
) sin x cos x dx is 2
−π/2


−π (a) 0 (b) 1 (c) –2 (d) 2









π3 7 1/ 2
1+ x
(a) 0 (b) π − (c) 2p – p3 (d) − 2π 3
3 2 5. The value of ∫ cos x log dx is








1− x



−1/ 2
π/2
1
2. ∫
2 2
sin x cos x ( sin x + cos x ) dx = (a) 0 (b)


2





−π/2
1
2 4 2 8 (c) − (d) None of these
(a) (b) (c) (d) 2





15 15 5 15








1
Evaluate each of the following Integrals
sin x − x 2
3. The value of ∫−1 3 − | x | dx is II. Long Answer Type Questions


a

∫ log  x + x 2 − 1 dx
1
sin x − x 2 1. Evaluate:
(a) 0 (b) 2 ∫ dx 



3−|x| −a




0
π/2
4 + 3 sin x 
log 
1
− x2 1
(c) 2∫ dx (d) 2 ∫
sin x − x 2
dx 2. Evaluate: ∫  4 − 3 sin x 
dx



3−|x| 3−|x|




0
−π / 2
0

Answers 7.13
π
 5 − 3 2 4
∫ (1 − x ) sin x cos = 2 =2× =
2 2
I. 1. (a) Let I= x dx
 15  15 15




−π

Here, f(x) = (1 – x2) sin x cos2 x Hence, (b) is the correct answer.




1 1 1
\ f(– x) = [1 – (–x) ] sin (– x) cos (– x) 2 2 sin x − x 2 sin x x2
3. (c) I= ∫−1 3 − | x | dx = −∫1 3 − | x | dx − −∫1 3 − | x | dx







= – [1 – (–x2)] sin x cos2 x = –f (x)


\ I= 0 sin x
f(x) = is an odd function ;



3−|x|


[ Here f(– x) = – f (x) so f is an odd function]


Hence, (a) is the correct answer. x2
f(x) = is an even function

3−| x|


π/2 π/2

2. (b) I= ∫ sin 3 x cos 2 x dx + ∫ sin 2 x cos3 x dx = I1 + I2 1


x2 − x2
1




−π/2 −π/2
\ I = 0 − 2∫ dx = 2∫ dx
3−| x| 3−| x|



Since sin3x . cos2 x is an odd function 0 0

π/2
Hence, (c) is the correct answer.

\ I1 = ∫ sin 3 x cos 2 x dx = 0 π/2 π/2 π
x5 sin 4 x dx = [sin x ] 2 π + I1
4. (d) I= ∫ cos x dx + ∫



−π/2





−π/2 −π/2 2
Since sin2 x cos3 x is an even function
 π  π 

π/2 π/2 = sin − sin  −   + I1
 2  2 
∫ ∫ sin


2 3 2 3
\ I2 = sin x cos x dx = 2 x ⋅ cos x dx



−π/2 0
 π π
π/2 1 =  sin + sin  + I1 = 2 + I1
 2 2


= 2 ∫ sin x (1 − sin x) cos x dx = 2 ∫ t (1 − t ) dt
2 2 2 2


π/2
0 0

1
I1 = ∫ x5 sin 4 x dx


−π/2
= 2 ∫ ( t 2 − t 4 ) dt [Put sin x = t ⇒ cos x dx = dt]
Here, f(x) = x5 sin4 x



0



⇒ f(–x) = (–x)5 sin4 (–x) = –x5 (–sin x)4 = –x5 sin4 x = –f(x)
π



When x = 0, t = 0 and when x = ,t=1 \ f(x) is an odd function.
2




π/2

t t   1 1
3 5 1 \ ∫ x5 sin 4 x dx = 0. Thus, I = 2 + 0 = 2

\ I = 2  −  = 2 −  −π/2

 3 5 0  3 5



Hence, (d) is the correct answer.

ntegrals 241
I

E:\AMIT_WORKS\Exam_Guru\EG_Mathematics-12_(working_02-06-2022)\EG_Mathematics-12_working\Open_Files\Chap_7\Chap_7
\ 19-Aug-2022  Amit   Proof 5 Reader’s Sign _______________________ Date __________


1/ 2 Here, f(x) is an odd function of x.
1+ x


5. (a) I= ∫ cos x log
1− x
dx 1/ 2
1+ x




∫ cos x log dx = 0
−1/ 2
\
1− x


1+ x −1/ 2
f(x) = cos x log
1− x


Hence, (a) is the correct answer.


−1
1− x 1+ x
\ f(–x) = cos ( − x) log = cos x log  II. 1. 0; Hint: Prove that the function log  x + x 2 + 1 is an odd
1+ x  1 − x   



function.
1+ x
= − cos x log  = − f ( x) 2. 0; Hint: Prove that the function is an odd function.
 1 − x 




Case Based Question
1. The bridge connects two hills 100 feet apart. The arch on 50
(iii) The integrand of the integral ∫ x 2 dx is ...... function.


the bridge is in a parabolic form. The highest point on the



−50
bridge is 10 feet above the road at the middle of the bridge (a) even (b) odd
as seen in the figure.





(c) neither odd nor even (d) none





(iv) The area formed by the curve x2 = 250y, x-axis, y = 0



and y = 10 is
1000 2 4
(a) (b)
3 3




1000
(c) (d) 0
3




(v) The area formed between x2 = 250y, y-axis, y = 2 and



y = 4 is
1000
(a) (b) 0
3




1000 2

Based on the information given above, answer the (c) (d) none of these
3






following questions.

(i) The equation of the parabola designed on the bridge is Ans. (i) (b) x2 = –250y


(a) x2 = 250y (b) x2 = –250y



1000





(c) y = 250x
2
(d) y2 = 250y (ii) (a)
3







50 x2
(ii) The value of the integral ∫ −50 250
dx is (iii) (a) even





1000
1000 250 (iv) (c)
(a) (b) 3



3 3




(v) (d) none of these
(c) 1200 (d) 0







Author’s Comments
Questions based on following types are very important for Exams. So, students are advised to revise them thoroughly.
1. Questions of Integration based on (a) partial fractions, (b) by-parts


2. Questions based on Special Integrals.


3. Questions based on Definite Integrals by substitution.


4. Questions based on properties of definite integrals.


242 Mathe atics–12
m
E:\AMIT_WORKS\Exam_Guru\EG_Mathematics-12_(working_02-06-2022)\EG_Mathematics-12_working\Open_Files\Chap_7\Chap_7
\ 17-Aug-2022  Amit   Proof-5 Reader’s Sign _______________________ Date __________


IMPORTANT FORMULAE

1.  x n +1  2.  xn + 1 
∫ x dx =  n + 1 + C, n ≠ − 1 ∫ ax dx = a  n + 1 + C, n ≠ − 1
n n




3.
∫  f ( x ) + g(x) + φ (x) dx = ∫ f ( x ) dx + ∫ g(x) dx + ∫ φ(x) dx + C


(ax + b) n + 1
∫ (ax + b) dx =
n
4. ∫ k dx = kx + C 5.
a(n + 1)
+ C, n ≠ − 1




dx −1 dx −1
6. ∫ x n = (n − 1) x n − 1 + C, n ≠ 1 7. ∫ (ax + b)n = a (n − 1) (ax + b)n − 1 + C, n ≠ 1




dx  dx  1
8. ∫ x
= log | x | + C 9. ∫  ax + b  = a log | ax + b | + C




 dx  2 ax + b
∫e
x
10. ∫  = +C 11. dx = e x + C
ax + b  a




e ax + b ax
∫ a dx = log a + C
x
∫ e dx =
ax + b
12. +C 13.
a




a mx + n
∫a ∫ sin x dx = − cos x + C
mx + n
14. dx = +C 15.
m log a




16. ∫ cos x dx = sin x + C 17. ∫ tan x dx = log | sec x | + C




18. ∫ cot x dx = log | sin x | + C 19. ∫ sec x dx = log | sec x + tan x | + C




20. ∫ cosec x dx = log | cosec x − cot x | + C = − log | cosec x + cot x | + C


∫ sec ∫ cosec x dx = − cot x + C
2 2
21. x dx = tan x + C 22.




23. ∫ (sec x tan x) dx = sec x + C 24. ∫ (cosec x cot x) dx = − cosec x + C




f ′ ( x ) dx f ′ ( x ) dx
25. ∫ = log f ( x ) + C 26. ∫ = 2 f ( x) + C
f ( x) f ( x)




n +1
n  f ( x ) dx 1 x−a
27. ∫  f ( x ) f ′ ( x ) dx = n + 1 + C, n ≠ −1 28. ∫ x 2 − a 2 = 2a log x+a
+C




29.
dx
∫ a 2 − x 2 = 2a log
1 a+ x
a−x
+C 30.
dx
∫ a2 + x2 =
1
a
tan −1
x
a
+C ()




dx dx
31. ∫ x2 + a2
= log x + x2 + a2 + C 32. ∫ 2 2
= log x + x2 − a2 + C




x −a

33. ∫
dx
2 2
= sin −1 () x
a
+C 34. dx
∫ ( x ± B)2 − A 2 = 2A log
1 (x ± B) − A
(x ± B) + A
+C




a −x
dx 1 A + (x ± B) dx 1 −1 (x ± B)
35.
∫ A 2 − ( x ± B)2 = 2A log A − (x ± B)
+C 36. ∫ ( x ± B)2 + A 2 = A tan A
+C




dx dx
37. ∫ = log (x ± B) + (x ± B) 2 − A 2 + C 38. ∫ = log (x ± B) + (x ± B) 2 + A 2 + C
( x ± B) 2
−A 2
( x ± B) 2
+A 2




ntegrals 243
I

E:\AMIT_WORKS\Exam_Guru\EG_Mathematics-12_(working_02-06-2022)\EG_Mathematics-12_working\Open_Files\Chap_7\Chap_7
\ 17-Aug-2022  Amit   Proof 5 Reader’s Sign _______________________ Date __________


dx (x ± B)
39. ∫ = sin −1 +C
A − ( x ± B)
2 2 A


40. Integration by Parts


∫ f ( x ) g ( x ) dx = f ( x ) ∫ g ( x ) dx − ∫  f ′ ( x ) ∫ g ( x ) dx  dx

a2 x 2
∫  f ( x ) + f ′ ( x ) e dx = f ( x ) e + C ∫ x 2 + a 2 dx = log x + x 2 + a 2 + x + a2 + C
x x
41. 42.
2 2




43. ∫ x 2 − a 2 dx =
−a 2
2
log x + x 2 − a 2 +
x
2
x2 − a2 + C 44. ∫ a 2 − x 2 dx =
a2
2
sin −1 ()
x x
+
a 2
a2 − x2 + C




45. Properties of definite integrals


b b b a

(a) ∫ f ( x ) dx = ∫ f ( z ) dz (b) ∫ f ( x ) dx = −∫ f ( x ) dx




a a a b
a d b c d
(c) ∫ f ( x ) dx = 0 (d) ∫ f ( x ) dx = ∫ f ( x ) dx + ∫ f ( x ) dx + ∫ f ( x ) dx




a a a b c
a a b b
(e) ∫ f ( x ) dx = ∫ f ( a − x ) dx (f) ∫ f ( x ) dx = ∫ f ( a + b − x ) dx




0 0 a a
2a a a
(g) ∫ f ( x ) dx = ∫ f ( x ) dx + ∫ f ( 2a − x ) dx


0 0 0
2a a
(h) ∫ f ( x ) dx = 2∫ f ( x ) dx if f ( 2a − x ) = f ( x )


0 0
= 0 if f (2a – x) = –f (x).



a a
(i) ∫ f ( x ) dx = 2∫ f ( x ) dx , when f (x) is an even function i.e., f (– x) = f (x).





−a 0
= 0, when f (x) is an odd function i.e., f (– x) = – f (x).




COMMON ERRORS
ERRORS CORRECTIONS

(
3x + 4 )
8
( 3x + 4 )
8

∫ (3x + 4) dx = ∫ (3x + 4) dx =
7 7
(i) +C. (i) +C
8 3×8




Generally students forget to divide by coefficient of x.

(ii) Generally students write
(ii) ∫ sin x dx = − cos x + C


∫ sin x dx = cos x + C


and ∫ cos x dx = sin x + C

and ∫ cos x dx = − sin x + C


(iii) Students find it difficult to identify and use the properties (iii) Learn the properties well and practice as much as you can.




of definite integral.

244 Mathe atics–12


m
E:\AMIT_WORKS\Exam_Guru\EG_Mathematics-12_(working_02-06-2022)\EG_Mathematics-12_working\Open_Files\Chap_7\Chap_7
\ 17-Aug-2022  Amit   Proof-5 Reader’s Sign _______________________ Date __________


REVISION CHART

Integration
It is an important concept in mathematics and, together with differentiation,
is one of the two main operations in Calculus.

ype T ype III

ype
T I

T II

When denominator of the component fraction When denominator of the composite fraction
When denominator of the composite fraction
can be factorised into linear factors. has linear and quadratic factors.
has linear factors and squares of linear fraction.
x 2 + 4x + 7 4 x 2 − 7x + 8
3x 2 + 4 x + 7
( x − 1)( x + 2)( x − 3) ( x − 4)( x − 3)( x − 1)2 (
( x − 1) x 2 + 4 x + 9 )
A B C A Bx + C
= + + A B C D = +
x −1 x + 2 x − 3 = + + +
x − 4 x − 3 x − 1 ( x − 1)2 (
x − 1 x 2 + 4x + 9 )

R eso ution of
l C o posite
m F ractions into artia
P l F ractions
All the following formulae are applicable when degree of the numerator of the composite fraction
is smaller than degree of the denominator.

T ype V
I

ax 2 + b
When denominator and numerator of the composite fraction has pure quadratic factors, i.e.,
(cx 2
)(
+ d ex 2 + f )
az + b
Put x2 = z and write the above composite fractions
(cz + d )(ez + f )
Now change it into partial fraction and before integration put z = x2 and then integrate.

ntegration y
I b P arts
This method is used when the integrand is a product of two function. First decide which function of the two, is easily
differentiable and which function is easily integrable. Take the function which is easily differentiable as first function and
write it at first place and use the formulae:
∫ f ( x ) g ( x ) dx = f ( x ) ∫ g ( x ) dx − ∫ f ′ (x ) ∫ g ( x ) dx  dx
When the integral is in the form: ∫ f ( x ) + f ′ ( x ) e x dx , write it as ∫ f ( x ) e x dx + ∫ f ′ ( x ) e x dx
Evaluate only first integral, by parts, second integral will get cancelled.
Identity: ∫ e x f ( x ) + f ′ ( x ) dx = e x f ( x ) + C

ntegrals 245
I

E:\AMIT_WORKS\Exam_Guru\EG_Mathematics-12_(working_02-06-2022)\EG_Mathematics-12_working\Open_Files\Chap_8\Chap_8
\ 17-Aug-2022  Amit   Proof-4 Reader’s Sign _______________________ Date __________


Topics
C
overed
8 Area Between Curves

8.1 Area between Curves 8.2 Area between a Parabola and a Line




8.3 Area between two Parabolas 8.4 Area of Circles




8.5 Area of an Ellipse 8.6 Area between a Circle and a Line




8.7 Area between Ellipse and Line 8.8 Area between two Circles




8.9 Area between Circle and Parabola


C hapter map
AREA

Area between Curves Area between Simple Curves

Area between two circles Area between Ellipse and Line


& two parabolas

Area between a Circle and a Line


Area between circle
and parabola
Area between a Parabola and a Line

Area between Ellipse and Parabola

Topic 1. Area Between Curves


Take two curves: y = f (x) and y = g(x). Assume that these two curves are intersecting at x = a on the left hand side and x = b at right
hand side. Now draw the graph of these two curves.
Area between two curves y = f (x) and y = g(x) intersecting at x = a and x = b when integral is evaluated along x-axis.
Right point of intersection of two curves
i .e., x = b b

∫ [ y of upper curve − y of lower curve]dx or Area = ∫ [ f ( x) − g ( x)]dx


Left point of intersection of two curves a
i .e., x = a

246
If we do not want full area between two curves y = f (x) and
y = g(x), but we want to find area of a limited part between curves
y = f(x) and y = g(x) between x = c and x = d, first draw their graph. upper point of
intersection
We have drawn their graphs in all possible different positions: [ x of right side curve − x of left side curve] dy

lower point of
intersection


b
⇒ Area = ∫  f ( y ) − g ( y ) dy




a
Note: 1. Equation of x-axis is: y = 0 and equation of y-axis is




x = 0.
2. UC = Upper curve, LC = Lower curve, LPI = left point


of intersection, RPI = Right point of intersection and
RA = Required area.

Area Enclosed between Straight Lines


Example 1. The area of the region bounded by the curve y = x +
1 and the lines x = 2 and x = 3 is
7 9
(a) sq. units (b) sq. units
2 2

11 13
(c)sq. units (d) sq. units
2 2

Sol. Given equation of lines are


y = x + 1, x = 2 and x = 3


Area between these two curves y = f (x), y = g(x), x = c and

x = d is
d Required area
∫( )

yof upper curve − yof lower curve dx 3
 x2 
3

c = ∫ ( x + 1) dx =  + x 
2  2 2
d 9  4 
=  + 3 −  + 2
or Area = ∫  f ( x ) − g ( x ) dx 2  2 

c 15 7
= − 4 = sq.units
2 2

Take two curves x = f (y) and x = g(y) intersecting at y = a Hence, the correct option is (a).


on the lower side and y = b on the upper side. Then area between Example 2. The area of the region bounded by the curve x = 2y
them will be calculated as discussed below: + 3 and the lines y = 1 and y = –1 is
3
Area between curves x = f (y) and x = g(y) intersecting at y = (a) 4 sq. units (b) sq. units
2

a and y = b when integral is evaluated along y-axis. (c) 6 sq. units (d) 8 sq. units

rea Between urves 247
A
C

E:\AMIT_WORKS\Exam_Guru\EG_Mathematics-12_(working_02-06-2022)\EG_Mathematics-12_working\Open_Files\Chap_8\Chap_8
\ 17-Aug-2022  Amit   Proof-4 Reader’s Sign _______________________ Date __________


Sol. Given equations of lines are x = 2y + 3, y = 1 and y = –1


\ Required area = area of ∆BCD + area of ∆BEF




In ∆BCD: UC: y =5 – x, LC: y = 0,
Required area


LPI: x = – 2, RPI: x = 5.

1


In ∆BEF: UC: y = 0, LC: y = 5 – x,
= ∫ (2 y + 3) dy


−1 LPI: x = 5, RPI: x = 8.
1 2 1


= 2.  y  −1 + 3 [ y ] 1−1 5 8
2
= (1 − 1) + 3 (1 + 1) = 6 sq.units
R.A. = ∫ yUC dx + ∫ yLC dx



−2 5

Hence, the correct answer is (c). 5 8
R.A. =
∫ ([(5 − x) − 0] dx ) + ∫ [0 − (5 − x)] dx




−2 5
5 8

Area = ∫ (5 − x ) dx − ∫ (5 − x ) dx


−2 5
Example 3. Using integration, find the area of the region bounded
5 8
by the lines: x + y = 5 and the ordinates x = – 2, x = 8 and x-axis.
= − ∫ ( x − 5) dx + ∫ ( x − 5) dx = 29 (unit)2
Hint: The given equations are: x + y = 5, x = – 2, x = 8, and x-axis.


−2 5
Their graph is shown in the fig.

EXERCISE 8.1
I. Multiple Choice Questions- (MCQ) (c) 15 sq units (d) 21 sq units






1. The area of a triangular region whose sides have the equations II. Long Answer Type Questions




y = 2x + 1 , y = 3x + 1 and x = 4 is 1. Find the area of the region bounded by the lines:



(a) 16 sq units (b) 4 sq units y = 3x + 2; x-axis and ordinates x = – 1, x = 1.





(c) 8 sq units (d) 20 sq units (Use method of integration).




2. The area bounded by the curve y = x, the x – axis and the

2. Using integration, find the area of the region bounded


ordinates x = – 2 and x = 3 is


17 by lines: y = x + 1, the x-axis and ordinates, x = – 2 and
(a) 13 sq. units (b) sq units x = 3. [Delhi 2007 (C)]
2 2





Answers 8.1
I. 1. (c) 8 sq units B (4,13)


Hint: Here equation of sides of triangle ABC are

Y
y = 2x + 1 ...(i) +
1



y = 3x + 1 ...(ii) =
3x



C (4,9)
x= 4 ...(iii)
y



Solving (i) and (ii), we have co-ordinates of point A (0, 1) 1
2x +


Solving (ii) and (iii), we have co-ordinates of point B (4, 13) y= x=4
A(0,1)

Solving (i) and (iii), we have co-ordinates of point C (4, 9)

X X
\ Required area = (area under the line AB, x-axis and x = 0,


x = 4) – (area under the line AC, x-axis and x = 0, x = 4) O D (4,0)
4 4

= ∫ ( 3x + 1) dx − ∫ ( 2 x + 1) dx


0 0
Y
4 4
 3x   2x2  2
 3 × 4 2  
= 
2
+ x − 
2
+ x =  + 4  − 0  −  ( 4 ) + 4 − 0 
2
( )


 0  0  2  


248 Mathe atics–12
m
= 28 – 20 = 8 sq. units. −2
LPI : x = ; RPI : x = 1


2. (a) 13 sq. units 3


2


−2 / 3 1
Hint: Here the curve is y = x, which represents a straight line
R.A. = ∫ [0 − (3x + 2)] dx + ∫ [(3x + 2) − 0] dx

passing through origin and making an angle 45° with x-axis.


−1 −2 / 3
We have to find out the area of shaded region in the figure.

For –2 ≤ x ≤ 0, y < 0 and 0 ≤ x ≤ 3, y > 0 −2 / 3 1

∫ (3x + 2) dx + ∫ (3x + 2) dx

\ Required area = area of OAB + area of OCD ⇒ R.A. = −





D
D
3 0 3 0 3 0 −1 −2 / 3
 x2   x2 
= ∫ y dx + ∫ ( − y ) dx = ∫ x dx − ∫ x dx =   −  
 2  0  2  −2 2. 8.5 sq. units

0 −2 0 −2



 32 02   02 ( − 2 ) 
2
9 13 Hint : Draw the graph of the given equations
=  − − −  = +2 = sq. units.


 2 2   2 2  2 2 y = x + 1, x = – 2 and x = 3.



R.A. = area of ∆AEF + area of ∆ACD.


II. 1. 13 sq.units In ∆AEF : UC : y = 0,
3




LC : y = x + 1;
Hint: Draw the graph of the given


equation: LPI : x = – 2;

y = 3x + 2, x = – 1 and x = 1 RPI : x = – 1.


R.A. = Area of ∆ABC + area of
In ∆ACD : UC : y = x + 1;

∆AED

In ∆ABC: UC: y = 0; LC : y = 0; LPI : x = – 1, RPI : x = 3


LC : y = 3x + 2; −1 3

LPI : x = – 1, RPI : x =
−2 R.A. = ∫ [0 − (x + 1)] dx + ∫ [(x + 1) − 0] dx

3 −2 −1

−1 3
In ∆AED : UC: y = 3x + 2; = − ∫ (x + 1) dx + ∫ (x + 1) dx


−2 −1
LC: y = 0;

Topic 2. Area between a Parabola and a Line



Parabola: Parabola is the locus of a point which moves in such the equations of the parabolas.
a way that its distance from a fixed point and fixed line is same. Parabolas of the form y = quadratic polynomial in x open either
Equations of Parabolas upwards or downwards. If the coefficient of x2 is positive, the
parabola opens upwards and when the coefficient of x2 is negative,
y = quadratic polynomial in x or
parabola opens downwards.


x = quadratic polynomial in y, represent equations of
Parabola y = x2 – 9, y = x2 + x – 2, will open upwards while the


parabolas.
parabolas y = 16 – x2, y = 12 – x – x2 will open downwards.
y = x2 x = y2
Parabolas of the form x = quadratic polynomials in y open either




y = x2 – 9 x = y2 – 16
on the right hand side or on the left hand side. If the coefficient of




y = x + x – 2 and x = y2 + y – 2
2
y2 is positive then the parabola will open on the right hand side




y = 16 – 9x2 x = 25 – y2 and when the coefficient of y2 is negative then the parabola will




2
y = 12 – x – x x = 7 – y – y2 are examples of open on the left hand side.




rea Between urves 249
A
C

E:\AMIT_WORKS\Exam_Guru\EG_Mathematics-12_(working_02-06-2022)\EG_Mathematics-12_working\Open_Files\Chap_8\Chap_8
\ 17-Aug-2022  Amit   Proof-4 Reader’s Sign _______________________ Date __________


Parabolas with equations, x = y2, x = y2 + 16, x = y2 + y – 2 2 3/ 2 4 1 1 2 4
will open on right hand side because coefficients of y2 are positive [ x ]0 − . [ x ]0
=
3 2 2



while the parabolas with equations : x = 25 – y2 and x = 7 – y2 will
open on left hand side because the coefficients of y2 are negative. 2 3/ 2 1 2 2 1
= [(4) − 0] − [(4) − 0] = × 8 − × 16
3 4 3 4


How to Draw Rough Graph of a Parabola?
16 4
Take a parabola equation: y = x2 – 4x – 5. = − 4 = sq. units
3 3


(i) Here the coefficient of x2 is positive, therefore this parabola Hence, the correct answer is (a).


will open upwards.


Example 2. Draw a rough graph of the curve: y = x2 – 2x – 3
(ii) In the equation: y = x2 – 4x – 5, put y = 0. and x + y = 9 and using integration find the area between them.


⇒ x2 – 4x – 5 = 0 Solution. The given equations are: y = x2 – 2x – 3 and x + y = 9




⇒ (x + 1) (x – 5) = 0 First equation is: y = quadratic polynomial in x.


⇒ x = – 1, 5. ⇒ It is an equation of a parabola.



This means the parabola will cut x-axis at x = – 1 and Here the coefficient of x2 is positive, so the parabola will open


x = 5. upwards. It will cut x-axis at x = – 1, 3, it will cut y-axis y = – 3.
Draw the rough parabola with this information:


Now solve the two equations:


x2 – 2x – 3 = 9 – x



⇒ x2 – x – 12 = 0




⇒ (x – 4) (x + 3) = 0




⇒ x = – 3, 4



⇒ Two curves intersect at x = – 3, 4.

Now draw perpendicular upwards from x = – 3. It will cut the
parabola at D. Now draw perpendicular upwards from x = 4. This



(iii) In the equation: y = x2 – 4x – 5. perpendicular cuts parabola at E.


Put x = 0 ⇒ y = – 5.

This means the parabola will cut y-axis at (0, – 5).

(iv) Now draw the rough graph of parabola cutting x-axis at


x = – 1 and 5 and y-axis at y = – 5 and opening upwards.
This is the rough graph of the parabola.

Example 1. The area of the region bounded by parabola y2 = x

and the straight line 2y = x is
4
(a) sq. units (b) 1 sq. unit
3

2 1 Now join D and E. This line is the graph of the line x + y = 9.
(c) sq. units (d) sq. units
3 3 Now shade the region between these two curves:

Solution. Given equation of parabola is y2 = x ...(i) In the shaded region upper curve is the line x + y = 9 or
y = 9 – x and lower curve is y = x2 – 2x – 3. Left point of


and equation of straight line is 2y = x ...(ii)

intersection is x = – 3 and right point of intersection x = 4.


4

∫ (9 − x) − (x
2
Required area = − 2x − 3)  dx


−3
4 4

∫( )
2  x 2 x3 
⇒ Area = 12 + x − x dx = 12x + − 
 2 3 − 3


−3



(
A =  48 + 8 −

64
3 )(
9
− −36 + + 9 
2

 )



Solving eqns. (i) and (ii) we get


A = 83 −
64 9 
+ ( )

 x x2
2
 3 2 



  = x ⇒ = x  x2 = 4x
2 4 155  343
A = 83 −

⇒ = sq units.
 x(x – 4) = 0 \ x = 0, 4  6  6






4 4
x 343
Required area = ∫ x dx − ∫ dx
2
Required area =
6
sq. units.






0 0

250 Mathe atics–12


m
EXERCISE 8.2
I. Multiple Choice Questions- (MCQ) 3. Find the area enclosed by the parabola 4y = 3x2 and the




1. Area lying between the curves y2 = 4x and y = 2x is line 2y = 3x + 12. [AI 2009, Delhi 2010]




1 4. Find the area bounded by the curves : x2 = 4y and the line
(a) 2 sq units (b) sq units



3 3 : x = 4y – 2.




3
[Delhi 2005, 2010, 2014 (C), AI 2006, 2013, 2013 (C)]
1
(c) sq units (d) sq units 5. Find the area lying between the curves : y2 = 4x and
4 4






y = 2x. [Delhi 2013 (C), 2017, AI 2017]
2. Area bounded by parabola y2 = x and straight line 2y = x is


6. Find the area of the region bounded by the curves:


4



(a) sq units (b) 2 sq units y = x2 + 2, y = x, x = 0 and x = 3. [AI 2006 (C)]
3







7. Find the area of the region: {(x, y) : 0 ≤ y ≤ x2, 0 ≤ y ≤ x +
1



2 2, 0 ≤ x ≤ 3}. [CBSE S.P. 2014, 2015]
(c) sq units (d) sq units


3 3 8. Find the area of the region bounded by: y2 = 4x, x = 1,




II. Long Answer Type Questions



x = 4 and x-axis in the first quadrant.


1. Find the area of the region bounded by the parabola [Delhi 2006, Foreign 2007]
y = x2 and y = | x |.



9. Find the area enclosed between the parabola y2 = 4ax and
[AI 2002, 2009 (C), 2012 (C), 2011 (C), 2013]



the line: y = mx. [Delhi 2007 (C), AI 2006, 2007 (C)]
2. Find the area enclosed by the parabola : y2 = x and the line


10. Find the area of the region: {(x, y) : 0 ≤ y ≤ x2 + 1, 0 ≤ y ≤


y + x = 2 and x-axis in the first quadrant.



x + 1, 0 ≤ x ≤ 2} [Delhi 2007 (C), AI 2006, 2007 (C)]


[AI 2005, 2006, 2009]

Answers 8.2
4 Hint:
I. 1. (b) sq unit
3


Hint : We are given y2 = 4x ... (1) and y = 2x ...(2)



Solving (1) and (2) simultaneously, we get x = 0, 1

These two curves (1) and (2) intersect each other at O(0, 0) and


A (1, 2) Required area


1 1 1 1 1
Draw the graph of y = x2 and y = | x |.
= ∫ 4 x dx − ∫ 2 x dx = 2∫ x 2 dx − 2∫ x dx

Find points of intersection by solving x2 = x and x2 = – x.

0 0 0 0

It gives : x = 0, x = – 1 and x = 0, x = 1 shaded area on
1
 3 1
LHS = shaded area of RHS.
1
 x2   x2  2 × 2  32  2 1 ∴ Required area = 2 [area of the region (OPAMO)]
= 2  − 2  =  x  −  x  0


3  2 0 3  0 For region (OPAMO) : UC : y = x, LC : y = x2, LPI: x = 0 and

 

 2 0 RPI : x = 1.
1
4 3 3
 4 4 1
= (1) 2 − ( 0 ) 2  − (1 − 0 ) = (1) − 1 = − 1 = sq. units
2 (
∴ R.A. = 2∫ x − x 2 dx )


3  3 3 3 0

Hence, (b) is the correct answer. 7 sq.units
2.

6


Hint: Solve the two equations:

y2 = x and x + y = 2.

It gives : x = 1 and 4 : Draw graph of y2 = x and

x + y = 2.
Required Area = Area of (OCB) + Area of (CAB).

4
2. (a) sq. units
3


II. 1. 1 sq.units
3



Are Between Curves 251
a

E:\AMIT_WORKS\Exam_Guru\EG_Mathematics-12_(working_02-06-2022)\EG_Mathematics-12_working\Open_Files\Chap_8\Chap_8
\ 17-Aug-2022  Amit   Proof-4 Reader’s Sign _______________________ Date __________


For (OCB): UC : y = x, LC : y = 0, LPI : x = 0 and RPI : x = 1 5. ()
1 sq.units

For (CAB): UC : y = 2 – x, LC : y = 0, LPI : x = 1, RPI: x = 2. 3





1 2 Hint : Draw a graph of y2 = 4x and y = 2x.
\ R.A. = ∫ ( )
x − 0 dx + ∫ ( 2 − x ) − 0 dx


Find the points of intersection of two curves:



0 1
y2 = 4x and y = 2x ⇒ (2x)2 = 4x ⇒ x = 0, 1


1 2
= ∫ x dx + ∫ ( 2 − x ) dx R.A. = Shaded Area.


0 1

3. 27sq.units


Hint : Solve the two equations: 4y = 3x2 and 2y = 3x + 12, it


gives x = – 2 , 4. Now draw graph of the two curves.

3x + 12 In the Shaded Area: UC : y = 2 x, LC : y = 2x,


In the shaded region: UC : y =


2 LPI : x = 0, RPI : x = 1

3x 2 ,

LC : y = LPI : x = – 2, RPI : x = 4. 1
4
(
R.A. = ∫ 2 x − 2x dx )

4 2 0

\ R.A. = ∫  3x + 12 − 3x  dx

 2 4 


−2 6. 21 sq.units
2


4
R.A. = 3 ∫ (8 + 2x − x ) dx
2
4 Hint: Draw graph of: y = x2 + 2, y = x, x = 0 and x = 3.



−2

4. 9 sq.units
8


Hint: Find point of intersection of two curves:

2
y = x and y = x + 2
4 4

2
Solve: x = x + 2 ⇒ x = −1, 2
4 4

Now draw the graph.

R.A = Shaded Area: In the shaded Area ABCO

UC: y = x2 + 2; LC : y = x, LPI : x = 0, RPI : x = 3



3
R.A. = ∫ (x 2 + 2) − x  dx
0

3
(
⇒ R.A. = ∫ 2 − x + x 2 dx )


0

7. 43 sq.units
R.A. = The shaded region AOB 6



x + 2 , LC : y = x 2 , Hint: Solve : y = x2 and y = x + 2.
UC : y =

4 4

It gives: x = – 1, 2.
LPI : x = – 1 and RPI: x = 2

Now draw a graph of y = x2, y = x + 2 and x = 3.


2 2
R.A. =  x + 2 x2 
∫  −  dx = 1 ∫ 2 + x − x 2 dx ( ) Total required area = area of the region (OAB) + area of the

4 4 4

−1 −1 region (ACDB)

252 Mathe atics–12


m
In the shaded region:
For region (OAB) : UC : y = x2,



LC : y = 0, LPI : x = 0, RPI : x = 2. UC : y = 2 a x,


LC : y = mx, LPI: x = 0,

For region (ACDB) : UC : y = x + 2, LC : y = 0,


4a

LPI : x = 2, RPI : x = 3. RPI : x = 2
m



2 3 4a / m2
R. Area =
∫ x dx + ∫ ( x + 2) dx
2
R.A. = ∫  2 a x − mx  dx
 


0 2 0


8. 28 sq.units 23
3 10.


6


Hint: Draw the graph of y2 = 4x, x = 1, x = 4 and x-axis.

Solve y = x2 + 1 and y = x + 1 to find their points of intersection:

x2 + 1 = x + 1
R.A. = Shaded region.

In the shaded region. UC : y = 2 x, ⇒ x(x – 1) = 0 ⇒ x = 0, 1


LC : y = 0, LPI : x = 1, RPI : x = 4. Area = area of (EOBA) + area of (ABDC)



4 4 Region (EOBA) : UC : y = x2 + 1,
( )
\ R.A. = ∫  2 x − 0 dx = 2∫ x dx

  LC : y = 0, LPI : x = 0, RPI : x = 1.



1 1
2 Region (ABDC) : UC : y = x + 1, LC : y = 0,
9. 8a sq.units


LPI : x = 1, RPI : x = 2
3m3



1 2
Hint: first solve y2 = 4ax and y = mx.
A = ∫ [(x 2 + 1) − 0] dx + ∫ [(x + 1) − 0] dx

4a
It gives: x = 0 and x = 2 . 0 1
m


1 2
Now draw graph.
⇒ A = ∫ (x 2 + 1) dx + ∫ (x + 1) dx

R.A. = Shaded region.


0 1

Topic 3. Area between two Parabolas
Example 1. Draw the rough sketch of: y = x2 and y2 = 8x and find ⇒ (x2)2 = 8x



the area between these curves, using integration. ⇒ x4 = 8x
Solution. The given equations are: y = x2 and y2 = 8x



4
⇒ x – 8x = 0
First solve these two equations and find their points of



⇒ x(x3 – 8) = 0

intersection.



⇒ x = 0 and x = 2.
y2 = 8x and y = x2



Now draw the graph of the parabola



rea Between urves 253
A
C

E:\AMIT_WORKS\Exam_Guru\EG_Mathematics-12_(working_02-06-2022)\EG_Mathematics-12_working\Open_Files\Chap_8\Chap_8
\ 17-Aug-2022  Amit   Proof-4 Reader’s Sign _______________________ Date __________


LC : y = x2,



LPI : x = 0, RPI : x = 2.



2
\ Required Area = ∫ 2 2 x − x 2  dx

( )
3 2
 3
=  2 2 × 2 x2 − x 
3 3



 0

3 2
 3
= 4 2 x2 − x 
 3 3



y = x2 and y2 = 8x. 0


R.A. = Shaded region.  4 2 × 2 2 8  


In the shaded region: =  −  − (0) 
 3 3





UC : y2 = 8x


⇒ y = 2 2 x, = 16 − 8  = 8 sq.units
 3 3  3






EXERCISE 8.3

I. Multiple Choice Questions- (MCQ) 3. Draw a rough sketch and find the area of the region




1. The area bounded by the curve 4y = x2 and 2y = 6 – x2 is between the parabola : y2 = 4ax and x2 = 4ay. Use method
of integration. [Delhi 2008]


(a) 8 sq. units (b) 6 sq. units


4. Draw a rough sketch and find the area of the region bounded




(c) 4 sq. units (d) 10 sq. units


by the two parabolas y2 = 8x and x2 = 8y by using method




2. The area included between two curves y2 = 9x and x2 = 9y is
of integration. [AI 2004, Delhi 2012]


(a) 21 sq. units (b) 27 sq. units


5. Draw the rough graphs of the parabolas: y = 2x2 and y = x2




(c) 36 sq. units (d) 18 sq. units


+ 4 and using integration find the area enclosed between




II. Long Answer Type Questions them.


1. Find the area of the region bounded by the two parabolas 6. Draw rough sketch of the parabolas: y = 5x2 – 3 and


y = x2 and y2 = x, using integration.


y = 3x2 + 5. Using integration, find the area between them.
2. Prove that the curves : y2 = 4x and x2 = 4y, divide the area 7. Find the area enclosed between the parabolas y = x2 – 9


of the square bounded by x = 0, x = 4, y = 4 and y = 0 into


and y = 3 – 2x2. Use method of integration. [Delhi 2009]
three equal parts. Use method of integration.


8. Find the area enclosed between the parabolas: y = 2x2 – 6
[Delhi 2009, A.I. 2015]


and y = – x2 – 3. Use method of integration.

Answers 8.3
I. 1. (a) 8 sq. units
In the shaded region: UC: y = x. LC: y = x2, LPI: x = 0,


2. (b) 27 sq. units

RPI: x = 1.



1 sq.units 1
II. 1.
3 \ A=∫ ( )
x − x 2 dx





Hint: Solve the two equations: y = x2 and y2 = x to find their 0
2. Hint: Solve the two equations:

points of intersection


It comes: x = 0 and x = 1. Now draw their graphs: y2 = 4x and x2 = 4y ⇒ x = 0, x = 4, or (0, 0) and (4, 4). Now

draw the graph of y2 = 4x, x2 = 4y, x = 0, x = 4, y = 4

R.A. = Shaded region.

and y = 0.]

254 Mathe atics–12
m
In the shaded region (OBQRO):

UC: y = 4, LC: y = 2 x , LPI : x = 0, RPI: x = 4.

4
⇒ Area of the region (OBQRO) = ∫ (4 − 2 )
x dx = 16 sq.units
3

0

In the shaded region (OAQBO):



x2
UC: y = 2 x, LC : y = , LPI : x = 0, RPI: x = 4.
4


42

Area of the region (OAQBO) = ∫  2 x − x  dx = 16 sq.units R.A. = Shaded region.
 4 3




2
In the shaded area: UC: y = 2 2 x, LC : y = x ,
0

In the shaded region (OPQAO): 8


LPI: x = 0, RPI: x = 8,


x2 8
 2
UC: y =
4
, LC: y = 0, LPI: x = 0, RPI: x = 4 A = ∫  2 2 x − x  dx = 64 sq.units
8 3

0
 
4
 2 


\ Area of the region (OPQAO) = ∫  x − 0 dx = 16 sq. units 5. 32 sq.units
 4  3


3



0
2
3. 16 a Hint: Solve: y = x2 + 4 and y = 2x2.
sq.units
3




⇒ 2x2 = x2 + 4 ⇒ x = –2, 2





Hint: Find the points of intersection of the parabolas: y2 = 4ax Now draw the graph.


and x2 = 4ay
2
 2
⇒  x  = 4ax ⇒ x4 = 64a3x
 4a 




⇒ x(x3 – 64a3) = 0 ⇒ x = 0 and x = 4a




Now draw the graphs of the parabolas.

R.A. = Shaded region.

In the shaded area : UC: y = x2 + 4, LC: y = 2x2, LPI: x = – 2,

RPI: x = 2
2 2

∫ (x ) ∫ (4 − x ) dx
2
A= + 4 − 2x 2 dx = 2

−2 −2

6. 64 sq.units
3


Hint: Solve: y = 5x2 – 3 and y = 3x2 + 5

R.A. = Shaded region. ⇒ x = – 2, 2. Draw graph.


In the shaded region: UC: y = 2 a x, In the shaded region : UC: y = 3x2 + 5,


LC: y = 5x2 – 3; LPI : x = – 2, RPI: x = 2.
x2

LC: y = , LPI: x = 0, RPI: x = 4a. 2 2
4a
( )

4a
 x2  16a 2 sq. units
Area = ∫ (3x 2 + 5) − (5x 2 − 3)  dx = 2 ∫ 4 − x 2 dx
 

Required Area = ∫  2 a x −  dx = −2 −2
 4a  3

0 R.A. = Shaded region.
64

4. sq.units
3


Hint: First solve the two equations:

y2 = 8x and x2 = 8y for x:

2
 2
⇒  x  = 8x ⇒ x4 = 512x
 8




⇒ x = 0 and x = 8.


Now draw the graph.

rea Between urves 255
A
C

E:\AMIT_WORKS\Exam_Guru\EG_Mathematics-12_(working_02-06-2022)\EG_Mathematics-12_working\Open_Files\Chap_8\Chap_8
\ 17-Aug-2022  Amit   Proof-4 Reader’s Sign _______________________ Date __________


In the shaded region : UC: y = 3x2 + 5,
8. 4sq.units

LC: y = 5x2 – 3; LPI : x = – 2, RPI: x = 2.




2 Hint : Solve the equations:
∫ (3x
2
+ 5) − (5x 2 − 3)  dx


Area =
y = 2x2 – 6 and y = – x2 – 3 ⇒ x = – 1, 1

−2




2 Now draw the graph.
(
= 2 ∫ 4 − x 2 dx )


R.A. = Shaded region.
−2


In the shaded region: UC: y = – x2 – 3,

7. 32 sq.units





LC: y = 2x2 – 6, LPI : x = – 1,
Hint: Solve the equation:



y = x2 – 9 and y = 3 – 2x2 RPI: x = 1.



⇒ x = – 2, 2 1 1
A= ∫ (− x − 3) − (2x − 6) dx
2 2
( )
= 3∫ 1 − x 2 dx

Now draw the graph: −1 −1


R.A. = Shaded region.
In the shaded region:

UC: y = 3 – 2x2,
LC: y = x2 – 9, LPI: x = – 2,
RPI : x = 2.

2

∫ (3 − 2x ) − (x
2 2
R.A. = − 9)  dx
−2

2
(
= 3 ∫ 4 − x 2 dx )
−2

Topic 4. Area of Circles
The general equation of a circle is: (x – a)2 + (y – b)2 = r2. Here Take a circle (x – 3)2 + (y – 4)2 = 16.
(a, b) is the centre of the circle and r is the radius of the circle. ⇒ (x – 3)2 + (y – 4)2 = 42

Area of the circle purely depends upon its radius r, it does ⇒ centre (3, 4); radius: r = 4

not depend upon its centre (a, b). Take a point centre (3, 4).

Draw two perpendicular lines parallel to x-axis and parallel
How to draw graph of a circle?

to y-axis passing through (3, 4).
Take general equation of the circle Now draw a circle with radius 4, as shown.

(x – a)2 + (y – b)2 = r2
Here centre is (a, b) and radius r.
Draw two ⊥ lines passing through (a, b). On the circles,
the right extreme point is A[(a + r), b], left extreme point is
B[(a – r), b], upper extreme point is C[a, (b + r)] and lower
extreme point is D[a, (b – r)]
Now draw a circle passing through these four points.

The two perpendicular lines divide the circle into 4 equal



parts. We shall find the area of one part by integration and multiply
it by 4.
Example 1. The area of the region bounded by the circle x2 + y2 = 1 is
 
 
(a) 2 p sq. units (b) p sq. units

(c) 3 p sq. units (d) 4 p sq. units

Sol. Given equation of circle is


256 Mathe atics–12
m
x2 + y2 = 1  y = 1 − x 2
(
− 8 sin −1 0 +
0 
16 − 0  )
   


Since the circle is symmetrical about the axes. 2 

( )


 π 
= 4  8 × + 0 − (8 × 0 + 0) 
 2 



= 4(4p) = 16p sq. units



Check: r= 4



⇒ A = r2 = (4)2 = 16 sq. units.



π
π
π
Example 3. Draw a graph of the circle: (x – 2)2 + (y – 3)2 = 25
1 and using integration, find its area.
\ Required area = 4 × ∫ 1 − x 2 dx Solution. Given equation of the circle is: (x – 2)2 + (y – 3)2 = (5)2

0

x 1 
1 ⇒ Centre (2, 3), r = 5


=4 1 − x 2 + sin −1 x  Draw the circle as shown.
2 2 0


 1 
= 4 0 + sin −1 (1) − 0 − 0
 2 
1 π
= 4 × × = π sq. units
2 2

Hence, the correct answer is (b).

Example 2. Draw a rough graph of the circle: x2 + y2 = 16 and
using integration, find its area.
Solution. Equation of the circle: x2 + y2 = 16. Here centre of the
circle is (0, 0) and radius is 4 units.
Now draw its graphs,
Area of the circle = 4 (shaded area PAD)
we shall find the area of the

25 − ( x − 2) + 3
one part of the circle and 2
In the shaded region: UC:

multiply it by 4.
LC: y = 3, LPI: x = 2, RPI: x = 7
R.A. = Area of

( )
7


the full circle. Area of the circle = 4∫  25 − (x − 2) 2 + 3 − 3 dx
 

In the shaded region 2

7
(OAC): UC: y = 16 − x 2 , = 4∫ 25 − (x − 2) dx
2


LC: y = 0, LPI: 0 and RPI : 2
7
 25 − 1 (x − 2) x−2
x = 4.
= 4  sin + 25 − (x − 2) 2 
 2 5 2 2


( 16 − x − 0) dx
4
Area of the circle = 4∫
( )
2
 25 5
⇒ A= 4 sin −1 1 + 25 − 25


0
 2 2



()
4
16
= 4  sin
2
−1 x
4
+
x
2

16 − x 2 
0 − ( 25
sin − 1 0 +
0 
25 − 0  )


2 2 

()

4
 −1 x x  ⇒ A = 25
= 4 8 sin + 16 − x 2 
4 2


π
 0


⇒ Area of the circle = 25 sq. units
π

(
= 4  8 sin −1
4
+
4
()
16 − 16 ) Verification: r = 5.

 4 2 A = r2 = 25 sq. units





π
π
EXERCISE 8.4
I. Multiple Choice Questions- (MCQ) π π
(c) sq. units (d) sq. units


1. Area lying in the first quadrant and bounded by the circle 3 4






x2+ y2 = 4 and the line x = 0 and x = 2 is [NCERT] II. Long Answer Type Questions

π 1. Draw a rough sketch of the curve: x2 + y2 – 6x = 40 and use
(a) p sq. units (b) sq. units
integration to find its area. [Delhi 2008]


2





rea Between urves 257
A
C

E:\AMIT_WORKS\Exam_Guru\EG_Mathematics-12_(working_02-06-2022)\EG_Mathematics-12_working\Open_Files\Chap_8\Chap_8
\ 17-Aug-2022  Amit   Proof-4 Reader’s Sign _______________________ Date __________


2. Draw a rough graph of the circle : 4x2 + 4y2 = 25 and using 5. Using integration, find the area of the circle: (x – 1)2 +
integration, find its area.




(y – 3)2 = 64.
3. Using integration, find the area of the circle (x – 2)2 + 6. Find the area of the circle: x2 + y2 – 4x – 6y – 36 = 0 by
y2 = 49.





using integration.
4. Draw a graph of: x2 + (y – 3)2 = 49 and using integration,


find its area.

Answers 8.4
2 2 2
I. 1. (a) x + y =2 . It is symmetrical about x-axis and y-axis both. Draw the graph.




2 2 2 2 2
y =2 –x ⇒y =± 2 −x

In the first quadrant, y > 0

\ y = 22 − x 2



2
2
Required area = ∫ y dx = ∫ 0
22 − x 2 dx


0

 x 2 a2 x
Form =  ∫ a 2 − x 2 dx = a − x 2 + sin −1 
2 2 a


2
x 2 4 −1 x 
In the shaded region: UC: y = 25 − x 2 ,
Required area =  4 − x + sin
2 2 2  0 4



5
2  π LC: y = 0, LPI: x = 0, RPI: x = .
=  [ 0] + 2sin −1 (1)  = 2 × = π 2

 2  2


5/ 2
Area of the circle: A = 4 ∫ 25 − x 2 dx
Hence, (a) is the correct answer. 4


0
II. 1. 49πsq.units 3. 49πsq.units




Hint: given equation of the circle: Hint: Equation of the circle: (x – 2)2 + y2 = 49. Centre (2, 0),


x2 + y2 – 6x = 40 ⇒ (x – 3)2 + y2 = 49 r=7
Draw the graph.

Here centre O(3, 0) and r = 7.


Now draw, its graph. In the shaded region: UC:

49 − ( x − 2)
2
y=

LC: y = 0, LPI :
x = 2, RPI : x = 9
Area of the circle =

( )
9
4∫ 49 − (x − 2) 2 dx
2

4. 49πsq.units


49 − ( x − 3)
2
In the shaded region: UC: Hint: Equation of the circle:


LC: y = 0, LPI: x = 3, RPI: x = 10 x2 + (y – 3)2 = 49. Centre (0, 3), r = 7


10 Draw the graph.
Area of the circle = 4∫ 49 − (x − 3) 2 dx


3

2. 25π sq.units
4


Hint:

Given equation of the circle:

4x 2 + 4y 2 = 25 ⇒ x 2 + y 2 = 25
4

5.
Here centre (0, 0) and r =
2

258 Mathe atics–12
m
In the shaded area: UC: y = 49 − x 2 + 3, d 6. 49π




LC: y = 3, LPI : x = 0, RPI: x = 7. Hint: Equation of the circle is:



Area of the circle = x2 + y2 – 4x – 6y – 36 = 0


( )
7 7
⇒ (x – 2)2 + (y – 3)2 = 49
4∫  49 − x 2 + 3 − 3 dx = 4∫ 49 − x 2 dx


0
  0 Here: Centre (2, 3) or r = 7



Draw the graph:
5. 64πsq.units




In the shaded region:
Hint: Circle equation is (x – 1)2 + (y – 3)2 = 64



Centre (1, 3), r = 8 UC: y = 49 − (x − 2) 2 + 3


Draw the graph.
LC: y = 3, LPI : x = 2,


RPI: x = 9


Area of the circle =


( )
9
4∫  49 − (x − 2) 2 + 3 − 3 dx
 

9
= 4∫ 49 − (x − 2) 2 dx

In the shaded region:



UC: y = 64 − (x − 1) 2 + 3,

LC: y = 3; LPI: x = 1, RPI: x = 9

Area of the circle =

( )
9
4∫  64 − (x − 1) 2 + 3 − 3 dx
 
1

9
= 4∫ 64 − (x − 1) 2 dx
1

Topic 5. Area of an Ellipse
General equation of an ellipse is:
( x − h )2 + ( y − k )2 = 1 (x − h) 2 (y − k) 2
+ =1 ⇒ Centre (h, k)
a 2
b 2 a2 b2


Draw two lines, x = h parallel to y-axis and y = k parallel
to x-axis. These two lines intersect at a point (h, k), which is the
centre of the ellipse. On the line y = k, mark two points A(x = h
+ a) and C(x = h – a). On the line x = h, mark two points B(y =
k + b) and D(y = k – b). Now join these points A, B, C and D as

shown in the graph. This is the rough figure of the ellipse.
Note: If you know the values of a and b in the equation of the
(x − h) 2 (y − k) 2
ellipse + = 1, then the area of the ellipse is ( ab).
a2 b2
π
2
x2 y
Example 1. Draw the rough sketch of the ellipse: + =1
Here (h, k) is the centre of the ellipse. Area of the ellipse 25 16
and using integration find the area of the ellipse.
purely depends on the values of a and b. Area of the ellipse is ab.
Solution. Equation of the ellipse is:
π
How to draw graph of an ellipse? x2 y
2
Let the equation of the ellipse be + = 1
25 16


rea Between urves 259
A
C

E:\AMIT_WORKS\Exam_Guru\EG_Mathematics-12_(working_02-06-2022)\EG_Mathematics-12_working\Open_Files\Chap_8\Chap_8
\ 17-Aug-2022  Amit   Proof-4 Reader’s Sign _______________________ Date __________


x2 y2
⇒ 2 + =1
( 5 ) ( 4 )2

⇒ a = 5 and b = 4 and

centre is (0, 0).
Now let us draw the rough

sketch of the ellipse.
The two axis divide the

area of the ellipse into 4 equal
parts, we shall work out only
one fourth area of the ellipse
lying in the first quadrant and will multiply it by 4. It will give
the full area of the ellipse.
In the shaded region: In the shaded region: UC:



x2 y
2 (x − 2) 2 (y − 1) 2
UC: + = 1 + = 1
25 16 25 16






y2 x 2 25 − x 2 ( y − 1)2 ( x − 2 )2
⇒ = 1− = ⇒ = 1−
16 25 25 16 25






⇒ y2 =
16
25 (
25 − x 2 ) ( y − 1)2 25 − ( x − 2)
2



⇒ =
16 25
y= 4 25 − x 2




5 16



⇒ (y – 1)2 = [25 − (x − 2) 2]
LC: y = 0; LPI : x = 0, RPI : x = 5. 25




( )
5 4
 4  ⇒ y= 25 − (x − 2) 2 + 1
Area of the ellipse = 4∫  25 − x 2 − 0 dx 5
5



 


0
5
LC: y = 1, LPI: x = 2, RPI: x = 7

16

2

( )
A= 25 − x dx 7
5  4 
Area of the ellipse = 4∫  25 − (x − 2) 2 + 1 − 1 dx


0
5
{ () }
 


5
16  25 x x  2
= sin −1 + 25 − x 2 
5  2 5 2 0


7
16
5 ∫
25 − (x − 2) 2 dx
{ }
⇒ A=
16  25 5
()



= (sin −1 1) + 25 − 25 2
5  2 2


= 16  25 sin −1  x − 2 
{25
sin −1 0 +
0
() 
} 5  2  5 


− 25 − 0 
2 2  7
x−2
25 − (x − 2) 2 

( )
+
16  25 π  2 2
× + 0 − 0 = 20π sq. units

⇒ A=
5  2 2 



Verification: Here a = 5, b = 4 ⇒ A= {
16  25
5  2
sin − 1 1 +
5
2
25 − 25 }




\ Area of the ellipse = ab = 20 sq. units.
{ 25 0
}
25 − 0 

π
π
− × sin −1 0 +
( x − 2)2 + ( y − 1)2 = 1 2 2 

Example 2. Draw rough graph of the ellipse
25 16
and find the area using method of integration. ⇒ A=
16  25 π
5  2 2{
× + 0 − {0 + 0}
 }



Solution. Equation of the ellipse:
16 25 π
⇒ A= × × = 20 π sq.units
( x − 2)2 + ( y − 1)2 = 1 5 2 2



25 16
Area of the ellipse = 20 . Verification: a = 5, b = 4

π
Centre of the ellipse (2, 1), a = 5, b = 4.
Area of the ellipse = ab = 20 sq. units


π
π
Draw the graph of the ellipse:

260 Mathe atics–12
m
EXERCISE 8.5
I. Multiple Choice Questions- (MCQ) II. Long Answer Type Questions


x2
y 2
1. Using integration, find the area of the ellipse:
1. The area of the ellipse + = 1 is



a 2 b2 (x − 1) 2 y 2


+ =1
π a2 25 9
(a) sq. units 2 2
4 2. Find the area of the ellipse: x + (y − 1) = 1. Use method


16 4



π b2 of integration.
(b) sq. units
4


(x − 3) 2 (y − 2) 2
(c) 4pa sq. units 2 3. Find the area of the ellipse: + = 1. Use
16 9



method of integration.


(d) 4pb2 sq. units


Answers 8.5
πa 2 In the shaded area: UC:
I. 1. (a) sq. units


4



y = 1 16 − x 2 + 1 ,
II. 1. 15πsq.units 2






(x − 1) 2 y 2 LC : y = 1; LPI : x = 0; RPI : x = 4
Hint: Equation of the ellipse: + = 1,
25 9



Centre (1, 0), a = 5, b = 3. Area of the ellipse



Draw the graph.
{ }
4 4
A = 4∫  1 16 − x 2 + 1 − 1 dx = 2∫ 16 − x 2 dx

0
 2  0

3. 12πsq.units


2 2
Hint: Equation of the ellipse: (x − 3) + (y − 2) = 1
16 9

In the shaded region: Here centre (3, 2), a = 4, b = 3

3 25 − (x − 1) 2

UC: y = , Draw the graph of the ellipse:
5


LC: y = 0,

LPI: x = 1; RPI: x = 6

6
A = 4∫  3 25 − (x − 1) 2 − 0 dx
 5 
1

6
12
⇒ A = ∫ 25 − (x − 1) dx
2
5


1

2. 8πsq.units


2
x 2 + (y − 1) = 1
Hint: Equation of the ellipse: .
16 4

Here Centre (0, 1), a = 4, b = 2.
In the shaded region:

Draw the graph of the ellipse:


3 16 − (x − 3) 2 + 2,
UC: y = LC: y = 2,
4

LPI : x = 3; RPI: x = 7

7
3
Area of the ellipse = 4∫  16 − (x −3) + 2 − 2 dx
2
 4 

3

7
= 3∫ 16 − (x − 3) 2 dx

3

rea Between urves 261


A
C

E:\AMIT_WORKS\Exam_Guru\EG_Mathematics-12_(working_02-06-2022)\EG_Mathematics-12_working\Open_Files\Chap_8\Chap_8
\ 17-Aug-2022  Amit   Proof-4 Reader’s Sign _______________________ Date __________


Topic 6. Area Between a Circle and a Line
Example 1. Find the area of the region in the first quadrant Example 2. Draw a rough graph of the function: {(x, y) : x2 + y2
enclosed by x-axis, the line y = x and the circle x2 + y2 = 8. ≤ 1 ≤ x + y, x, y ∈ R} and using the method of integration, find
[A.I. 2014 (C), Delhi 2015 (C), Delhi 2017] the area between them. [Delhi 2010, 2011 (C)]


Solution. The given equations are: x2 + y2 = 8 and y = x. Solution. The given equations are: x2 + y2 = 1 and x + y = 1
First, find their point of intersection First solve these two equations to find their points of

x2 + y2 = 8, y = x ⇒ x2 + x2 = 8


intersection.




2
⇒ 2x = 8
x2 + y2 = 1, x + y = 1 ⇒ x2 + (1 – x)2 = 1


⇒ x2 = 4






⇒ x2 + 1 – 2x + x2 = 1 ⇒ 2x2 – 2x = 0


⇒ x = 2 and – 2






⇒ 2x (x – 1) = 0 ⇒ x = 0, 1


Now draw the graph of the two curves:






On putting x = 0 and x = 1 in x + y = 1 respectively, we get


y = 1 and y = 0



Circle x2 + y2 = 8 with centre (0, 0)

r = 2 2, line: y = x

⇒ The points of intersection are (0, 1) and (1, 0)
The shaded region has two parts:


Total area = area of the region (OAB) + area of the region Now draw the graph of the circle with centre (0, 0), r = 1



(ABC) Now join the points (1, 0) and (0, 1).

In the region (OAB): UC: y = x; LC: y = 0, It gives the graph of the line.


LPI: x = 0, RPI: x = 2 Now we put (0, 0) in the equation 1 ≤ x + y.


In the region (ABC) = UC: y = 8− x , 2 It gives: 1 ≤ 0 (false), hence the required area does not include


the area containing (0, 0)
LC: y = 0, LPI: x = 2, RPI: x = 2 2

So the required area is (ABCDA). In the required area
( 8 − x − 0) dx
2 2 2

Area of the shaded region = ∫ ( x − 0) dx + ∫
2
(ABCDA)

0 2
2 2 2 UC: Circle: y = 1 − x2
∫ x dx + ∫
2


⇒ R.A. = 8 − x dx



0 2 LC: Line: y = 1 – x, LPI: x = 0 and RPI: x = 1


∫( )
2 2 1
1 2 8  x  x 
⇒ R.A. =  x 2  +  sin −1  + 8 − x2  Required area = 1 − x 2 − (1 − x) dx
2 0 2  2 2  2


2



0

 1
1 
⇒ R.A. =
2
[ 4 − 0] +  4 sin −1(1) + 2 22 8 − 8

1
=  sin −1 (x) +
x x2 
1 − x2 − x + 
 2 2 2 0







 1 
−  4 sin −1 
 2

+ 4 
 ⇒
 1 π
R.A. =  × + 0 − 1 +
 2 2
1
2( 
− (0 + 0 − 0 + 0) 
 )





 2 (
R.A. = 2 +  4 × π + 0 − 4 × π + 2 
4  )( ) ⇒
 π 1
R.A. =  −
 4 2
 π 1
(
− 0 = − sq. units
 4 2 )






( )
⇒ R.A. = 2 + [2p – p – 2] = 2 + p – 2 = p sq. units π 1
− sq.units



⇒ Area of the shaded region = p sq. units ⇒ Required area =
4 2




262 Mathe atics–12
m
EXERCISE 8.6
I. Multiple Choice Questions- (MCQ) functions and using integration find the area enclosed


2 2 between them.
1. Smaller area bounded by the circle x + y = 4 and the line x


+ y = 2 is  x 2 + y 2 ≤ 16  x 2 + y 2 ≤ 1
3.  4. 
 x + y ≤ 4  x − y ≥ 1





(a) 2 (p – 2) sq. units (b) (p – 2) sq. units




(c) (2p – 1) sq. units (d) 2 (p + 2) sq. units
 x 2 + y 2 ≤ 9




5.  [A.I. 2011, 2012 (C)]
II. Long Answer Type Questions  y − x ≥ 3




1. Find the area of the region in the first quadrant enclosed
 x 2 + y 2 ≤ 25


by the x-axis, the line y = x and the circle x2 + y2 = 32. 6. 
 x + y ≤ −5



[Delhi 2014]
2. Using integration, find the area of the region: 7. Find the area bounded by the circle x2 + y2 = 16, the line
{(x, y) : | x − 1 | ≤ y ≤ }




5 − x2 [CBSE S.P. Delhi 2010] 3y = x and x-axis in the first quadrant, using integration.

[Delhi 2017]
Draw a rough graph of each of the following sets of



Answers 8.6
I. 1. (b) (p – 2) sq. units
Hint: Equation of circle is x2 + y2 = 22. Its radius = 2 units

Y

B(0,2)
M
x+
y=

A(2,0)
2

X O X

Draw the graph of line y = x and the circle with centre (0, 0) and
Y

1 r = 4 2.
Area of quadrant OAMB = (π × 22 ) Shaded region = Region (AOB) + Region (ABC)
4




1 In the region (AOB)
π × 4 = π sq. units

= UC: y = x; LC: y = 0; LPI: x = 0, RPI: x = 4
4



1 In the region (ABC)
Area of quadrant AMB = × OA × OB

2

D


UC: y = 32 − x 2; LC: y = 0; LPI: x = 4
1

= × 2 × 2 = 2sq. units RPI: x = 4 2.
2



So, required area = p – 2. 4 4 2



Hence (b) is the correct answer. Area of the shaded region = ∫ x dx + ∫ 32 − x 2 dx


2 2
= 4p sq. units 0 4

Or Required area = ∫ 4 – x 2 dx − ∫ (2 – x)dx   
2. − 5 + 5 sin −1 2 + sin −1  1  



0 0
2 2 5  5


2 2
 x 4 – x 2 4 –1 x   x2 
=  + sin  – 2 x –  5π − 5    
2 2 2   2 0 or because sin −1  1  = cos −1  2 


 4 2  5  5


0

4  4  π Hint: The given equations are:


= (0 – 0) + (sin –1 1 – sin –1 0) =  4 –  – (0 – 0)  = 2   – 2

2  2  2 5 − x2

y = |x – 1| and y =

= (p – 2) sq. units
 x − 1 when x ≥ 1

⇒ y=
II. 1. 4π  −(x − 1) when x < 1


Hint: Given equations: x2 + y2 = 32 and y = x, solve them. and y = 5 − x 2 is the upper part of the circle.


⇒x=4


rea Between urves 263
A
C

E:\AMIT_WORKS\Exam_Guru\EG_Mathematics-12_(working_02-06-2022)\EG_Mathematics-12_working\Open_Files\Chap_8\Chap_8
\ 17-Aug-2022  Amit   Proof-4 Reader’s Sign _______________________ Date __________


Solve: y = 5 − x 2 and y = x – 1 and y = – (x – 1) 4. ( π4 − 12 )sq.units



( x − 1) = 5 − x 2 ⇒ x 2 − 2x + 1 = 5 − x 2
Hint: Solve the two given equations:

⇒ 2x2 – 2x – 4 = 0 ⇒ x2 – x – 2 = 0


x2 + y2 = 1 and x – y = 1.




⇒ (x – 2) (x + 1) = 0 ⇒ x = 2, –1






Draw the graph. The point (0, 2) satisfies the two inequations. It gives: x = 1, 0 ⇒ Two points (1, 0) and (0, – 1)



Shaded area is the required area. Now draw the graphs.




In the shaded region UC: y = 5 − x2 , In the shaded region: UC: y = (x – 1);



There are two LC: y = – (x – 1) LC: y = − 1 − x 2 (– sign indicates that the curve is below x-axis);


and y = x – 1, LPI: x = – 1 and MPI: x = 1, LPI: x = 0, RPI: x = 1

RPI: x = 2,
{ }
1
Area of the shaded region = ∫ (x − 1) − − 1 − x 2  dx

Area of the shaded region  


0

( )
2 1 2 1
= ∫ 5 − x 2 dx − ∫ −(x − 1)dx − ∫ (x − 1)dx = ∫ x − 1 + 1 − x 2 dx

−1 −1 1 0

( )

2 1 2
5. 9π − 9 sq.units
= ∫ 5 − x 2 dx + ∫ (x − 1) dx − ∫ (x − 1) dx 4 2


−1 −1 1
Hint: Solve the two equations:


3. (4π − 8) sq.units x2 + y2 = 9 and y – x = 3 ⇒ x = – 3, 0



Points of intersection are (– 3, 0) and (0, 3).
Hint: Solve the two equations:

Now draw the graphs of two equations.

x2 + y2 = 16 and x + y = 4 for x. It gives: x = 0, 4.


Points of intersection are: (0, 4) and (4, 0)



Now draw the graph of the two curves.

In the shaded area: UC: y = 9 − x2 ;

LC: y = x + 3; LPI: x = – 3, RPI: x= 0

0
In the shaded region: Area of the shaded region = ∫  9 − x 2 − (x + 3)  dx




−3
UC: y = 16 − x 2 ; LC: y = (4 – x)
9π − 9 sq. units

LPI: x = 0, RPI: x = 4; =
4 2



4
Area of the shaded region = ∫  16 − x − (4 − x)  dx
2 6. ( 4 2 )
25π − 25 sq.units


 

0

264 Mathe atics–12


m
Hint: Find the point of intersection of x2 + y2 = 25 and x + y Or, Area of the region:



= – 5. These are (0, – 5) and (– 5, 0). Now draw the graph. 2
{(x, y) : x2 + y2 ≤ r2 and x + y ≥ r} = r ( π − 2) sq. units.
4



7. 4π sq.units
3



Hint: x2 + y2 = 16, 3y = x


On solving these two equations we get the points of intersection.
( ) ( )


A 2 3, 2 and B −2 3, 2

In the shaded region: UC: y = – (x + 5);



LC: y = − 25 − x 2 , LPI: x = – 5, RPI: x = 0

∫  − (x + 5) − ( − )
0
Area of the shaded region =  25 − x 2  dx


−5
0 0
= − ∫ (x + 5) dx + ∫ 25 − x 2 dx
−5 −5 2 3 4
 x 
∫   dx + ∫ 16 − x dx
2
Area =

= 25π − 25 sq. units 3


0 2 3
4 2

()
2 3
Note: Smaller area between a curve x2 + y2 = r2 and a line  2 
4
=  x  x 16 x 
+  16 − x 2 + sin −1 

x + y = r is:  2 3 0 2 2 4 2


3
2
r
4
(π − 2) sq.units . = 4π sq. units .
3



Topic 7. Area between Ellipse and Line
Example 1. Draw the rough figure and find the area represented ⇒ x = 0, 5


 2
x2 y x y \ Points of intersection are (0, 4) and (5, 0).
by: (x, y) : + ≤ 1 ≤ +  . Use method of integration.

 25 16 5 4 2
y2
Now draw the graph of the ellipse x + = 1 , centre of the
Solution. Find the points of intersection of the two given equation: 25 16

2 ellipse (0, 0), a = 5, b = 4, join the points A(0, 4) and C(5, 0).
x2 y x y
+ = 1 and + = 1
25 16 5 4 x y


It gives the line + = 1.
4 x2 y
2 5 4

Put y = (5 − x) in + = 1,
Which area is to be shaded?
5 25 16




x 2 1 16 x y
It gives: + × (5 − x) 2 = 1 Put (0, 0) in + ≥ 1. It gives 0 > 1 (false). So the area
25 16 25 5 4


⇒ x2 + (5 – x)2 = 25 including (0, 0) is not in the shade. So shade the other area as



⇒ 2x(x – 5) = 0 shown in the graph. In the shaded region:



4
UC: y = 25 − x 2 , LC: y = 4 (5 − x), LPI: x = 0,
5 5

RPI : x = 5.

Area of the shaded region

5
 4 25 − x 2 − 4 (5 − x)  dx
A= ∫  5 5 


0

()
5
4  25 −1 x x 1 
⇒ A =  sin + 25 − x 2 + (x − 5) 2 
52 5 2 2 0



rea Between urves 265
A
C

E:\AMIT_WORKS\Exam_Guru\EG_Mathematics-12_(working_02-06-2022)\EG_Mathematics-12_working\Open_Files\Chap_8\Chap_8
\ 17-Aug-2022  Amit   Proof-4 Reader’s Sign _______________________ Date __________


⇒ A=
4  25 π
(
× +0+0 − 0+0+
5  2 2
25 
2 
4  25π 25 
=A= 5  4 − 2 
  )( )



⇒ A = (5p – 10) = 5(p – 2)


⇒ Area of the shaded region = 5( – 2) sq. units

π
EXERCISE 8.7
I. Multiple Choice Questions- (MCQ) 2. Draw the rough graph of the following ellipse and line and




1. The area of the smaller region bounded by the ellipse using integration, find the smaller area enclosed between


x2 y 2 x y
+ = 1 and st. line + = 1 is x2 y
2
9 4 3 2 them: + = 1 and 5x − 4y = 20.
3 16 25
(a) 3 (p – 2) sq. units (b) (p – 2) sq. units
4 2 Draw rough figure and find the area represented by each of





(c) 3p sq. units (d) 2p sq. units the following sets of equations. Use method of integration.




II. Long Answer Type Questions  x2 y
2
x y
3. (x, y) : + ≤1≤ − + 
1. Find the area of the smaller region bounded by the ellipse:  16 49 4 7





2
x2 y x y Use method
2 + 2 = 1 and the straight line : a + b = 1.  x2 y
2
x y
a b 4. (x, y) : + ≤1≤ − − 
of integration. [A.I. 2009 (C)]
 49 4 7 2



Answers 8.7
3 2 9 π 9  2 9 π  3
I. 1. (b) (p – 2) sq. units =  ⋅ −  = ⋅  − 1 = (p – 2) sq. units
2 3 2 2 2 3 2 2  2




x2 y 2 ab (π − 2) sq.units
Hint: Equation of ellipse is + =1 II. 1.
9 4 4

y2 x2 4 Hint: find the points of intersection of two equations:
⇒ = 1 − ⇒ y 2 = (9 − x 2 )

4 9 9 2

x 2 + y = 1 and x + y = 1
2 a 2
b 2 a b
⇒ y = 9 − x2

3

Points are: (0, b) and (a, 0). Now draw the graph:
x y y x 2

Equation of line AB is + = 1 ⇒ = 1− ⇒ y = (3 − x)
3 2 2 3 3


Y

B(0, 2)

A(3, 0)
X X
O

Shade the smaller common region:



b a2 − x2 b
UC: y = , LC: y = − (x − a);
a a

LPI: x = 0; RPI: x = a.

Y
3 3
Area of the shaded region:
2 2

Required area (shaded region) = ∫ 9 − x dx − ∫ (3 − x) dx
{ }
2 a
30 30 A = ∫  b a 2 − x 2 − − b (x − a)  dx



0
a a 
3 3
x 2  x2 

2x 9 a
= 
3 2
9 − x 2 + sin −1  − 3x − 
2 3 0 3  2 0 ⇒ A = b ∫  a 2 − x 2 + (x − a)] dx
a  




0

2  9 −1  2 9 
= 0 + sin 1 − 0 − 0  − 9 − − 0 + 0  = ab (π − 2) sq. units.
3  2  3 2  4



266 Mathe atics–12
m
Note: This is the standard formula for such questions. Please Points of intersection A( – 4, 0), B (0, 7).


commit to memory. 7 16 − x 2 ,
In the shaded region: UC: y =
4


2. 5(π − 2) sq.units


2
x 2 + y = 1. LC: y = 7 (x + 4), LPI: x = – 4, RPI: x = 0.
Hint: Equations are 4



16 25

0
5x – 4y = 20, points of intersection: A= 7 ∫  16 − x 2 − (x + 4)  dx


A(4, 0), B(0, – 5). 4
−4


= 7(p – 2) sq. units


4. 7 (π − 2) sq.units
2



Hint : Equations are:


2
x 2 + y = 1 and −x − y = 1
49 4 7 2


Points of intersection: A(– 7, 0), B (0, – 2).


In the shaded region: LC: y =
−5 16 − x 2 , In the shaded region: UC: y = −2 (x + 7),
4 7



5 (x − 4), 2 49 − x 2 ,
UC: y = LPI: x = 0, RPI: x = 4. LC: y = − LPI: x = – 7, RPI: x = 0.
4 7


4 0
A = 5 ∫ (x − 4) + 16 − x 2  dx  − 2 (x + 7) + 2 49 − x 2  dx
4 
0
 A= ∫  7 7 
−7


= 5(p – 2) sq. units 0
= −2 ∫ (x + 7) − 49 − x 2  dx

3. 7 (π − 2) sq.units 7 
−7



= 7 ( π − 2) sq.units
2

2
y2 y
Hint: Equations are: x + = 1 and −x + = 1
16 49 4 7

MOST IMPORTANT TOPICS

Topic 8. Area between two Circles


Example 1. Draw a rough graph of the circle: x2 + y2 = r2 and the circle: (x – r)2 + y2 = r2. Shade the common region and find the
area of the shaded region using integration.
Solution. Equations of the circles are:

x2 + y2 = r2 and (x – r)2 + y2 = r2.


Solve these two equations:

x2 + y2 = (x – r)2 + y2


⇒ x2 = x2 – 2rx + r2



⇒ 2rx = r2



⇒ x= r
2



rea Between urves 267
A
C

E:\AMIT_WORKS\Exam_Guru\EG_Mathematics-12_(working_02-06-2022)\EG_Mathematics-12_working\Open_Files\Chap_8\Chap_8
\ 17-Aug-2022  Amit   Proof-4 Reader’s Sign _______________________ Date __________


Now draw the graph of the two circles:
LPI: x = r , RPI: x = r.

2




R. Area = area of (AOBD) = 4 area of (APD)


r r
 2 
= 4 ∫  r − x − 0 dx = 4 ∫
2
r 2− x 2 dx
 



r/2 r/2

()
r
 r2 −1 x x 2 
⇒ R.A. = 4  sin + r − x2 
 2 r 2 r




2
 r 2 π   r 2 π r 2 r 2  
⇒ R.A. = 4  × + 0 −  × + r − 
Circle x2 + y2 = r2 has centre (0, 0), radius = r  2 2   2 6 4 4 






and circle: (x – r)2 + y2 = r2 has centre (r, 0) and radius = r.  πr 2  πr 2 r 3 r
R.A. = 4  − + ×


⇒  
Shade the common region. Join the two points of intersection 4  12 4 2




 

of two circles. This line AB and x-axis divides the full required
 2πr 2 3r 2 
area into 4 equal parts. We shall evaluate only one part and ⇒ R.A. = 4  −
 12 8 




then we shall multiply it by 4. We choose an area (APD) to
work out the required area because this area is below simple  2πr 2 3r 2 
⇒ R.A. =  − sq. units.
circle x2 + y2 = r2.  3 2 



Here UC: y= r 2 − x 2 and LC: y = 0, Note: This is a standard formula for this type of questions.



EXERCISE 8.8
I. Multiple Choice Questions- (MCQ) II. Long Answer Type Questions


1. The area of the region enclosed between the two circles: 1. Find the area bounded by the curves: (x – 1)2 + y2 = 1 and


x2 + y2 = 1.


x2 + y2 = 4 and (x – 2)2 + y2 = 4. is [Delhi 2007, A.I. 2013 (C)]

[Delhi 2008, A.I. 2010 (C)] 2. Find the area of the region enclosed between the two circles


x2 + y2 = 9 and (x – 3)2 + y2 = 9. [Delhi 2009, A.I. 2009]

(a) 8π − 2 3 sq. units


3 3. Find the area of the region enclosed between the two




circles: (x – 6)2 + y2 = 36 and x2 + y2 = 36.
(b) 8π + 2 3 sq. units
3 [Delhi 2009 (C), A.I. 2013 (C)]


(c) (3p – 2) sq. units 4. Find the area of the region enclosed between the two




(d) (3p + 2) sq. units circles: x2 + y2 = 1 and x2 + (y – 1)2 = 1. [Delhi 2009]



Answers 8.8
I. 1. (a) 8π − 2 3 sq.units In the shaded region: UC: y = 4 − x 2;

3 LC: y = 0, LPI: x = 1, RPI: x = 2



Hint: Solve the two equations: R.A. = Area of (AODCA) = 4 area of (ABC)

x2 + y2 = 4 and (x – 2)2 + y2 = 4 ⇒ x = 1.

2

Required area = 4∫ 4 − x dx
2
Draw graph: x2 + y2 = 4 has centre (0, 0) and r = 2




(x – 2)2 + y2 = 4 has centre (2, 0) and r = 2. 1

II. 1. 2π − 3 sq.units
3 2

Hint: Take the equations:

x2 + y2 = 1 and (x – 1)2 + y2 = 1

Find their point of intersection, it is: x = 1 .
2

268 Mathe atics–12
m
Draw the graph with the given information: RA = Area of the region (AODCA)



[x2 + y2 = 1, centre (0, 0), r = 1 = 4 area of (ABC)

and (x – 1)2 + y2 = 1, centre (1, 0), r = 1]



In the shaded region: UC: y = 36 − x 2 ,

RA = Area of (AODCA) = 4 area of (ABC)



LC: y = 0, LPI: x = 3, RPI: x = 6
In the shaded region : UC: y = 1 − x 2 ,



6
1 Required area = 4∫ 36 − x 2 dx
LC: y = 0, LPI: y = , RPI: y = 1
2



3
1
R.A. = 4 ∫ 1 − x 2 dx  
4.  2π − 3  sq.units
1/ 2  3 2 



2. 6π − 9 3 sq.units

Hint: The two given equation:
2


x2 + y2 = 1 and x2 + (y – 1)2 = 1
Hint:

Point of intersection:

Given equations:


x2 + y2 = 9 and (x – 3)2 + y2 = 9 y=
1 , [(x 2 + y 2 = 1),
centre (0, 0), r = 1]

point of intersection: 2


x = 3 , [x 2 + y 2 = 9, centre (0, 0), r = 3], and [x2 + (y – 1)2 = 1, centre (0, 1), r = 1].
2


[(x – 3)2 + y2 = 9, centre (3, 0), r = 3]. Now draw their graphs


Draw their graphs.

RA = Area of the region (AODCA)


= 4 area of (ABC).
R.A. = Area of the region (ACDOA)




In the shaded region: UC: y = 9 − x 2; = 4 area of the region (ABC). Integration will be evaluated



3 along y-axis.
LC: y = 0, LPI: x = , RPI: x = 3
2

3 In the shaded region: UC: x = 1 − y 2,

Required area = 4 ∫ 2
9 − x dx .
1

3/ 2 LC: x = 0, LPI: y = , UPI: y = 1
2

3. 24π − 18 3 sq.units 1


2 2 2 2 \ RA = 4 ∫ 1 − y 2 dy
Hint: The two equations are: x + y = 36 and (x – 6) + y =


1/ 2
36, point of intersection: x = 3, [x2 + y2 = 36, centre (0, 0), r =

6] and [(x – 6)2 + y2 = 36, centre (6, 0), r = 6]. Note: You can make this question simpler by changing x to y

Draw their graphs. and y to x.

rea Between urves 269
A
C

E:\AMIT_WORKS\Exam_Guru\EG_Mathematics-12_(working_02-06-2022)\EG_Mathematics-12_working\Open_Files\Chap_8\Chap_8
\ 17-Aug-2022  Amit   Proof-4 Reader’s Sign _______________________ Date __________


Topic 9. Area between Circle and Parabola
Example 1. Make a rough sketch and find the area of the region Example 2. Draw a rough graph of: {4x2 + 4y2 ≤ 9 and y2 ≤ 4x}


(using integration): and using method of integration, find the area enclosed between
{(x, y) : x2 + y2 ≤ 2ax; y2 ≥ ax, x ≥ 0, y ≥ 0} [Delhi 2016] them. [Delhi 2008, A.I. 2003 (C), 2013]


Solution. The given equations are: 4x2 + 4y2 = 9 and y2 = 4x


Solution. The two equations are:
x2 + y2 = 2ax ⇒ (x – a)2 + y2 = a2 (a circle) Solve these equations to find their points of intersection.


4x2 + 4(4x) = 9 ⇒ 4x2 + 16x – 9 = 0


y2 = ax (a parabola).





2
⇒ 4x + 18x – 2x – 9 = 0 ⇒ 2x (2x + 9) – 1 (2x + 9) = 0


Solve these equations, to find their points of intersection.




(x – a)2 + y2 = a2 ⇒ (x – a)2 + ax = a2 ⇒ (2x – 1) (2x + 9) = 0


2 2 2 1 −9




⇒ x – 2ax + a + ax = a ⇒ x(x – a) = 0 ⇒ x = and x = (rejected)




⇒ x = 0, a. 2 2




1


Now (x – a)2 + y2 = a2 is a circle with centre (a, 0) and r = ⇒ x=
a and y2 = ax is a parabola which opens on a right hand side 2




and passes through (0, 0). Draw the graph. Now 4x2 + 4y2 = 9

()




2
3
⇒ x2 + y2 =
is a circle with centre (0, 0) and
2




3
r= .
2



y2 = 4x is a parabola which will open on



RHS and will pass through (0, 0).

So draw their graphs.

For shading the region put (a, 0) in the parabola equation.



y2 = ax. It gives: 0 ≥ a2 (false)

Point (a, 0) is not in the required area.


So shade the other part as shown in the graph.

In the shaded region: R.A. = Area (ODBEO).

In the region (ODBEO)

UC: y= a 2 − (x − a) 2 , We want to find the area (AODCA) = 2(OACBO)

R.A. = Area of the region (AOBA) + Area of the region



LC: y= a x,


(ABCA)



LPI: x = 0, RPI: x = a In the region (AOBA):




a UC: y = 2 x and LC: y = 0,
R.A. = ∫  a 2 − (x − a) 2 − a x  dx



 1
LPI: x = 0, RPI: x = .


0
2



 2 x − a x − a 2
R.A. =  a sin − 1  + a − (x − a) 2 In the region (ABCA): UC: y =
9
− x 2,
 2  a  2 4



a 1 3
2 a 32  LC: y = 0; LPI: x = ; RPI: x =
− x  2 2

You might also like